Download as pdf or txt
Download as pdf or txt
You are on page 1of 499

Editors

Rajesh Tampi, Deena Tampi, Juan Young, Rakin Hoq and Kyle Resnick

Absolute Geriatric Psychiatry Review


Essential Questions and Answers
1st ed. 2021
Editors
Rajesh Tampi
Department of Medicine, Cleveland Clinic Lerner College of Medicine of
Case Western Reserve University, Cleveland, OH, USA

Deena Tampi
Behavioral Health Advisory Group, Princeton, NJ, USA

Juan Young
Department of Psychiatry, Yale School of Medicine, New Haven, CT, USA

Rakin Hoq
Child and Adolescent Psychiatry, NYU Grossman School of Medicine,
New York, NY, USA

Kyle Resnick
Department of Psychiatry & Behavioral Sciences, Cleveland Clinic Akron
General, Akron, OH, USA

ISBN 978-3-030-58662-1 e-ISBN 978-3-030-58663-8


https://doi.org/10.1007/978-3-030-58663-8

© Springer Nature Switzerland AG 2021

This work is subject to copyright. All rights are reserved by the


Publisher, whether the whole or part of the material is concerned,
speci ically the rights of translation, reprinting, reuse of illustrations,
recitation, broadcasting, reproduction on micro ilms or in any other
physical way, and transmission or information storage and retrieval,
electronic adaptation, computer software, or by similar or dissimilar
methodology now known or hereafter developed.
The use of general descriptive names, registered names, trademarks,
service marks, etc. in this publication does not imply, even in the
absence of a speci ic statement, that such names are exempt from the
relevant protective laws and regulations and therefore free for general
use.

The publisher, the authors and the editors are safe to assume that the
advice and information in this book are believed to be true and accurate
at the date of publication. Neither the publisher nor the authors or the
editors give a warranty, expressed or implied, with respect to the
material contained herein or for any errors or omissions that may have
been made. The publisher remains neutral with regard to jurisdictional
claims in published maps and institutional af iliations.

This Springer imprint is published by the registered company Springer


Nature Switzerland AG
The registered company address is: Gewerbestrasse 11, 6330 Cham,
Switzerland
Preface
We are living in an aging world. For the irst time in human history,
there are more people living who are above the age of 65 years when
compared to children under the age of 5 years. It is expected that the
number of older adults in the world will double over the next three
decades to over 2.1 billon individuals. This statistic also holds true for
the United States where currently 15% of the population is over 65
years. By 2060, the population of older adults in the United States is
expected to reach 95 million or a quarter of its population. Psychiatric
disorders are not uncommon among older adults. It is estimated that
between 15% and 20% of the older adults have a diagnosable
psychiatric disorder. The common psychiatric disorders seen among
the older adults include anxiety disorders, depressive disorders,
bipolar disorders, psychotic disorders, neurocognitive disorders,
personality disorders, and substance use disorders. Evidence indicates
that the presence of psychiatric disorders among older adults increases
their morbidity and risk for mortality. Although the population of older
adults with psychiatric disorders is increasing, the total number of
trained geriatric mental health professionals has not kept pace with the
needs of the aging population. Despite a signi icant growth in the
knowledge and research data regarding common psychiatric disorders
among older adults over the last two decades, this crucial information
has not been disseminated adequately to clinicians who care for these
older adults.
In an effort to improve knowledge sharing with fellow professionals
who care for older adults with psychiatric disorders, we decided to
write a concise question and answer book that covers various
important topics in geriatric psychiatry. This book contains the latest
information on psychiatric disorders among older adults. It is written
by experienced and award-winning professionals who specialize in the
care of older adults with psychiatric disorders. This new book can be
used by anyone who wants to learn about psychiatric disorders among
older adults. Additionally, this book serves as an excellent resource for
any individual who intends to attain professional certi icates in order to
treat older adults with psychiatric disorders. This book has been
written with the full awareness that the ield of geriatric psychiatry is
growing rapidly. We intend to update the information in this book with
the latest scienti ic data every few years. It is our sincere hope that the
readers of this book will ind the information in this book helpful in
improving their knowledge about psychiatric disorders among older
adults. Last but not least, we would like to acknowledge our respective
families, colleagues, trainees, and staff for their support and thank all
the contributors for their efforts in creating this book.
Rajesh Tampi
Deena Tampi
Juan Young
Rakin Hoq
Kyle Resnick
Cleveland, OH, USA
Princeton, NJ, USA
New Haven, CT, USA
New York, NY, USA
Akron, OH, USA
Contents
Part I Developmental, Psychological and Social Aspects of Aging
1 Life Course: Developmental and Transitional Events
Rajesh Tampi and Deena Tampi
2 Demography and Epidemiology
Rajesh Tampi and Deena Tampi
3 Culture and Gender
Rajesh Tampi and Deena Tampi
4 Family and Community
Rajesh Tampi and Deena Tampi
5 Ethics and Forensics
Rajesh Tampi and Deena Tampi
6 Economics and Health Policy
Rajesh Tampi and Deena Tampi
Part II Biological Aspects of Aging
7 Neuroanatomy and Neuropathology
Rajesh Tampi and Deena Tampi
8 Biochemistry and Neuropharmacology
Rajesh Tampi and Deena Tampi
9 Anatomy and Physiology
Rajesh Tampi and Deena Tampi
Part III Psychiatric Diagnosis
10 Neurocognitive Disorders
Rajesh Tampi and Deena Tampi
11 Depressive Disorders and Bipolar and Related Disorders
Juan Young
12 Sleep-Wake Disorders
Insiya Nasrulla
13 Schizophrenia and Other Psychotic Disorders
Nery A. Diaz
14 Substance-Related and Addictive Disorders
Juan Young
15 Anxiety Disorders and Obsessive Compulsive and Related
Disorders
Pallavi Joshi
16 Sexuality, Gender Dysphoria, and Paraphilic Disorders
Insiya Nasrulla
Part IV Diagnostic Methods
17 Interviewing and History Taking
Kristina Zdanys
18 Mental Status Examination
Kristina Zdanys
19 Functional Assessment
Kristina Zdanys
20 Psychological and Neuropsychological Testing
Rabin Dahal
21 The Neurological Examination
Rosemary Szparagowski
22 Clinical Laboratory Testing
Rosemary Szparagowski
23 Structural and Functional Imaging
Kristina Zdanys
Part V Treatments
24 Pharmacokinetics and Pharmacodynamics
Arushi Kapoor
25 Cytochrome P450
Mara Storto and Meera Balasubramaniam
26 Antidepressants
Emily E. Bay
27 Mood Stabilizers
Kyle Resnick
28 Antipsychotics
Rakin Hoq
29 Anxiolytics
Mara Storto and Meera Balasubramaniam
30 Hypnotics
Rakin Hoq
31 Cognitive Enhancers
Rabin Dahal and Rajesh Tampi
32 Stimulants
Rakin Hoq
33 Drugs for Tardive Dyskinesia
Rakin Hoq
34 Adverse Effects of Drugs
Kristen Cannon
35 Interventional Psychiatry
Edward V. Singh
36 Psychotherapy
Laurel J. Bessey
37 Interdisciplinary Approaches
Rajesh Tampi and Deena Tampi
38 Setting-Speci ic Treatment Issues
Arushi Kapoor
39 Special Management Considerations
Kyle Resnick
Part VI Policy, Ethical and Legal Issues
40 Role of Geriatric Psychiatrist in Healthcare Systems
Rajesh Tampi and Deena Tampi
41 Medicolegal
Rajesh Tampi and Deena Tampi
42 Ethics
Nery A. Diaz
43 Treatment-Setting Regulations
Arushi Kapoor
Part VII Medical and Neurologic Aspects of Geriatric Psychiatry
44 Geriatric Medical Syndromes
Paroma Mitra
45 Care of Patients with Neurologic Disease
Ankit Jain
46 Psychiatric Disorders Due to General Medical Condition
Paroma Mitra
47 Psychiatric Complications of Medical Treatment
Paroma Mitra
48 Psychological Factors Affecting Medical Conditions
Paroma Mitra
Index
Contributors
Meera Balasubramaniam, MD, MPH
Department of Psychiatry, NYU Grossman School of Medicine, New
York, NY, USA

Emily E. Bay, MD
Medical University of South Carolina, Charleston, SC, USA

Laurel J. Bessey, MD
Department of Psychiatry, University of Wisconsin School of Medicine
and Public Health, Madison, WI, USA

Kristen Cannon, MD
Department of Psychiatry, Ohio State University, Columbus, OH, USA

Rabin Dahal, MD
Yale New Haven Hospital, New Haven, CT, USA

Nery A. Diaz, DO
Columbia University Irving Medical Center, New York, NY, USA

Rakin Hoq, MD
Department of Child and Adolescent Psychiatry, NYU Grossman School
of Medicine, New York, NY, USA

Ankit Jain, MD
Penn State Health Milton S. Hershey Medical Center, Hershey, PA, USA

Pallavi Joshi, DO, MA


Department of Psychiatry, Yale School of Medicine, New Haven, CT, USA

Arushi Kapoor, MD, MS


Department of Psychiatry, , Perelman School of Medicine at the
University of Pennsylvania, Philadelphia, PA, USA

Paroma Mitra, MD, MPH


Department of Psychiatry, New York University Grossman School of
Medicine, New York, NY, USA

Insiya Nasrulla, MD
Department of Psychiatry, Medstar Georgetown University
Hospital/Medstar Washington Hospital Center, Washington, DC, USA

Kyle Resnick, MD
Department of Psychiatry & Behavioral Sciences,, Cleveland Clinic
Akron General, Akron, OH, USA

Edward V. Singh, MD
Geriatric Psychiatry, Yale-New Haven Hospital, New Haven, CT, USA

Mara Storto, MD
Department of Psychiatry, NYU Grossman School of Medicine, New
York, NY, USA

Rosemary Szparagowski, MD
Department of Psychiatry, University of Pittsburgh Medical Center,
Pittsburgh, PA, USA

Deena Tampi, MSN, MBA-HCA, RN, DFAAGP


Behavioral Health Advisory Group, Princeton, NJ, USA

Rajesh Tampi, MD, MS, DFAPA, DFAAGP


Department of Medicine, Cleveland Clinic Lerner College of Medicine of
Case Western Reserve University, Cleveland, OH, USA

Juan Young, MD
Department of Psychiatry, Yale School of Medicine, New Haven, CT, USA

Kristina Zdanys, MD
University of Connecticut School of Medicine, Farmington, CT, USA
Part I
Developmental, Psychological and Social Aspects of
Aging
In this part, we review various facets of aging including developmental
issues, psychological factors and social aspects. The topics that will be
covered include developmental and transitional events, demography,
epidemiology, culture, gender, family, community, ethics, forensics,
economics and health policy.
Developmental and transitional events · Demography ·
Epidemiology · Culture · Gender · Family · Community · Ethics ·
Forensics · Economics and health policy
Rajesh R. Tampi
Deena J. Tampi
© Springer Nature Switzerland AG 2021
R. Tampi et al. (eds.), Absolute Geriatric Psychiatry Review
https://doi.org/10.1007/978-3-030-58663-8_1

1. Life Course: Developmental and


Transitional Events
Rajesh Tampi1 and Deena Tampi2
(1) Department of Medicine, Cleveland Clinic Lerner College of
Medicine of Case Western Reserve University, Cleveland, OH, USA
(2) Behavioral Health Advisory Group, Princeton, NJ, USA

Keywords Developmental and transitional events – Bereavement –


Retirement – Labor force

Questions
1.
Which of the following is true of bereavement among older adults?
A.
Bereavement, especially loss of a spouse, is experienced
disproportionately by older adults
B.
Most bereaved individuals experience grief that decreases in
intensity over time
C.
Bereavement can be a trigger for the onset of both physical
and/or mental health problems
D.
Complicated grief is not included as a mental health disorder in
the DSM-5
E.
All of the above
2. Which of the following is a true statement regarding the effect of
retirement on cognition among older adults?
A.
There is no evidence that retirement accelerates the rate of
decline in crystallized cognitive abilities
B.
There is strong evidence that retirement accelerates the rate of
decline in crystallized cognitive abilities
C.
There is con licting evidence that retirement accelerates the
rate of decline in luid cognitive abilities
D.
There is strong evidence that retirement slows the rate of
decline in crystallized cognitive abilities
E.
There is strong evidence that retirement slows the rate of
decline in luid cognitive abilities
3.
Which of the following is a false statement regarding bereavement?
A.
The prevalence of spousal loss is higher among men than
women
B.
Bereavement includes psychological, social, and physiological
processes
C.
Widowhood has been correlated with increased rates of
morbidity and mortality
D.
Bereavement-related depressive episodes when compared to
general depressive episodes are associated with lower rates of
suicidal ideation
E.
Bereavement-related depressive episode do not have an
increased risk of subsequent depressive episodes
4. Which of the following is true regarding older adults and the labor
force in the United States?
A Trends show a decrease in the labor force participation for
A. Trends show a decrease in the labor force participation for
both older men and women

B.
Older women who are married, widowed, or never married
had higher labor force participation rates than older women
who were divorced or separated
C.
Labor force participation rates for older men who identi ied as
White or as Asian were higher than those for older men
identifying as Black
D.
A higher percentage of older men tend to work part-time when
compared to older women
E.
All of the above

Answers
Answer 1E Bereavement, especially the loss of a spouse, is
experienced disproportionately by older adults. Most bereaved
individuals experience grief that increases in intensity over time.
Bereavement can also be a trigger for the onset of both physical and/or
mental health problems. Complicated grief is not included as a mental
health disorder in the DSM-5.

Answer 2C A systematic review that included data from seven


studies found weak evidence that retirement accelerates the rate of
cognitive decline in crystallized abilities, but it was only true among
individuals retiring from jobs high in complexity and dealing with
people. The evidence of the impact of retirement on the rate of decline
in luid cognitive abilities is con licting. Retirement does not improve
crystallized or luid cognitive abilities.

Answer 3A The prevalence of spousal loss is higher among women


than men, with 39.9% of women over 65 years and 72.9% of women
over 85 years being widows. Bereavement not only includes
psychological but also social and physiological processes. Widowhood
has been correlated with increased rates of morbidity and mortality
due to various reasons including accidents, strokes, heart disease, and
cancer. Bereavement-related depressive episodes when compared to
general depressive episodes are associated with lower rates of suicidal
ideation. Individuals with bereavement-related depressive episode do
not have an increased risk for subsequent depressive episodes.

Answer 4C Trends show an increase in the labor force participation


for both older men and women. Older women who are divorced or
separated had higher labor force participation rates than older women
who are married, widowed, or never married. Labor force participation
rates for older men who identi ied as White or as Asian were higher
than those for older men identifying as Black. A higher percentage of
older women work part-time when compared to older men.

Further Reading
Federal Interagency Forum on Aging-Related Statistics. Older Americans. Key
indicators of well-being. Federal Interagency forum on aging-related statistics.
Washington, DC: U.S. Government Printing Of ice; 2012. p. 2012.

Ghesquiere A, Shear MK, Duan N. Outcomes of bereavement care among widowed


older adults with complicated grief and depression. J Prim Care Community Health.
2013;4(4):256–64.
[Crossref]

Hart CL, Hole DJ, Lawlor DA, et al. Effect of conjugal bereavement on mortality of the
bereaved spouse in participants of the Renfrew/Paisley Study. J Epidemiol
Community Health. 2007;61(5):455–60.
[Crossref]

Meng A, Nexø MA, Borg V. The impact of retirement on age related cognitive decline -
a systematic review. BMC Geriatr. 2017;17(1):160.
[Crossref]

Mojitbai R. Bereavement-related depressive episodes: characteristics, 3-year course,


and implications for the DSM-5. Arch Gen Psychiatry. 2011;68(9):920–8.
[Crossref]

Pies RW. The bereavement exclusion and DSM-5: an update and commentary. Innov
Clin Neurosci. 2014;11(7–8):19–22.
[PubMed][PubMedCentral]

West LA, Cole S, Goodkind D, He W. 65+ in the United States: 2010. Special studies
current population reports. https://www.census.gov/content/dam/Census/library/
publications/2014/demo/p23-212.pdf. Accessed 16 June 2019.
© Springer Nature Switzerland AG 2021
R. Tampi et al. (eds.), Absolute Geriatric Psychiatry Review
https://doi.org/10.1007/978-3-030-58663-8_2

2. Demography and Epidemiology


Rajesh Tampi1 and Deena Tampi2
(1) Department of Medicine, Cleveland Clinic Lerner College of
Medicine of Case Western Reserve University, Cleveland, OH, USA
(2) Behavioral Health Advisory Group, Princeton, NJ, USA

Keywords Demography – Epidemiology – Psychiatric disorders –


Aging population – Substance use disorders

Questions
1.
Which of the following is true of the demographics of aging in the
United States?
A.
The population of older Americans (ages ≥65 years) is
projected to increase by 30% by 2060
B.
The population of older non-Hispanic White Americans is
projected to increase by 24% by 2060
C.
The percentage of older Americans with completed bachelor’s
degree or more increased by 20% over the past ive decades
D.
The number of older Americans who are working has reduced
signi icantly over the past ive decades
E.
All of the above
2. According to the World Health Organization (WHO), which of the
following is the most common psychiatric disorder among older
g p y g
adults?
A.
Anxiety disorders
B.
Schizophrenia
C.
Alcohol use disorder
D.
Depression
E.
Posttraumatic stress disorder
3.
Which of the following statements is true of the older adult
population in the United States?
A.
More than twice as many Hispanic Americans live in poverty
when compared to African Americans
B.
More than twice as many Hispanic Americans live in poverty
when compared to non-Hispanic White Americans
C.
More than twice as many African Americans live in poverty
when compared to Hispanic Americans
D.
More than four times as many African Americans live in
poverty when compared to Hispanic Americans
E.
More than four times as many Hispanic Americans live in
poverty when compared to African Americans
4. Which of the following is the most commonly used psychoactive
substance among older adults in the United States?
A.
Alcohol
B.
Benzodiazepines
C. Cannabis

D.
Cocaine
E.
Methamphetamine
5.
Which of the following is true of the older adult population in the
United States?
A.
Obesity rates among older adults have been increasing
B.
Less older adults are divorced when compared with previous
generations
C.
Poverty rate for adults has increased sharply over the past 5
decades
D.
The gender gap in life expectancy between men and women is
widening
E.
All of the above

Answers
Answer 1C The population of older Americans (ages ≥65 years) is
projected to increase by 9% by 2060 (from 15% to 24%). The
population of older non-Hispanic White Americans is projected to
decrease by 24% by 2060 (from 78.3% to 54.6%). The percentage of
older Americans with completed bachelor’s degree or more increased
by 20% over the past ive decades (from 5% to 25%). The number of
older Americans who are working has increased over the past ive
decades. By 2014, 23% of older men and about 15% of older women
were in the labor force.
Answer 2D According to the WHO, the most common psychiatric
disorders among older adults are depression (7%), dementia (5%),
anxiety disorders (3.8%), and substance use disorders (1%).

Answer 3B In the United States, there are wide economic disparities


across the population of older adults. It is estimated that 18% of
Hispanic Americans and 19% of African Americans live in poverty,
which is more than twice the rate among older non-Hispanic Whites
Americans at 8%.

Answer 4A Alcohol is the most commonly used psychoactive


substance among older adults in the United States followed by cannabis
and cocaine.

Answer 5A The obesity rates among older adults in the United


States have been increasing with 40% of 65-to-74-year-olds now being
considered obese. There are more older Americans who are divorced
when compared with previous generations. The number of divorced
older women increased from 3% in 1980 to 13% in 2015, and for men
it rose from 4% to 11% during the same period. The poverty rates for
older Americans have dropped sharply during the past 50 years, from
nearly 30% in 1966 to 10% today. The gender gap in life expectancy
between men and women in the United States is narrowing. In 1990,
there was a 7-year gap in life expectancy between men and women. By
2013, this gap had narrowed to less than 5 years (76.4 years vs.
81.2 years).

Further Reading
Fact sheet: aging in the United States – population reference bureau. https://www.
prb.org/aging-unitedstates-fact-sheet/. Accessed 17 June 2019.

Fact sheet: aging in the United States – population reference bureau. https://www.
prb.org/aging-unitedstates-fact-sheet/. Accessed 22 June 2019.

Mattson M, Lipari RN, Hays C, Van Horn SL. A day in the life of older adults:
substance use facts. https://www.samhsa.gov/data/sites/default/ iles/report_2792/
ShortReport-2792.html. Accessed 22 June 2019.
Mental health of older adults. https://www.who.int/news-room/fact-sheets/detail/
mental-health-of-older-adults. Accessed 21 June 2019.
© Springer Nature Switzerland AG 2021
R. Tampi et al. (eds.), Absolute Geriatric Psychiatry Review
https://doi.org/10.1007/978-3-030-58663-8_3

3. Culture and Gender


Rajesh Tampi1 and Deena Tampi2
(1) Department of Medicine, Cleveland Clinic Lerner College of
Medicine of Case Western Reserve University, Cleveland, OH, USA
(2) Behavioral Health Advisory Group, Princeton, NJ, USA

Keywords Culture – Gender – Race – Disparity – Culture bound


syndrome (CBS) – Lesbian, gay, and bisexual (LGB)

Questions
1. Which of the following statements is false regarding racial
disparities among older adults with mental health disorders ?
A.
African Americans suffer more psychological distress when
compared to non-Hispanic White Americans
B.
The rates of depression are similar among African Americans
and non-Hispanic White Americans
C.
African Americans are as likely to seek mental health
treatment as non-Hispanic White Americans
D.
African Americans are more likely to prematurely terminate
mental health treatment when compared to non-Hispanic
White Americans
E.
African Americans are more likely to seek mental health
treatment in primary care settings rather than in specialty
mental health settings
2.
Which of the following culture-bound syndromes are you most
likely to ind in a 75-year-old man from Nigeria?
A.
Brain Fag
B.
Koro
C.
Amok
D.
Latah
E.
Hwa-Byung
3.
Which of the following is false regarding health disparities among
lesbian, gay, and bisexual (LGB) older adults?
A.
Lesbians and bisexual women have higher odds than
heterosexual women for disability
B.
Lesbians and bisexual women have higher odds than
heterosexual women for having poor mental health
C.
Lesbians and bisexual women have higher odds than
heterosexual women for having poor physical health
D.
Lesbians and bisexual women have similar rates of poor
physical health, disability, and poor mental health
E.
Gay and bisexual men were more likely than heterosexual men
to have poor physical health, disability, and poor mental health
4. Which of the following is true of mental illness among older adults?
A.
Male sex is the most consistent risk factor for depression
B.
The prevalence of most anxiety disorders is higher in men than
i
in women

C. Late-onset schizophrenia is more common among men than


women

D.
Suicide attempts among the elderly are often characterized by
a greater degree of lethal intent
E.
Female gender is a risk factor for alcohol use disorder

Answers
Answer 1C Older African Americans suffer more psychological
distress when compared to older non-Hispanic White Americans due to
their life-long exposure to and experiences with racism, discrimination,
prejudice, poverty, and violence. In addition, they also tend to have
fewer psychological, social, and inancial resources for coping with
stress than older non-Hispanic White Americans. The rates of
depression are similar among older African Americans and older non-
Hispanic White Americans. Older African Americans are half as likely to
seek mental health treatment when compared to older non-Hispanic
White Americans. Older African Americans attend fewer sessions when
they do seek specialty mental health treatment and are more likely than
non-Hispanic White Americans to terminate treatment prematurely.
Older African Americans are more likely to seek mental health
treatment in primary care than in specialty mental health settings.

Answer 2A Brain Fag is the culture-bound syndrome (CBS) that is


more likely to be seen in men from Nigeria and Saharan Africa. It
primarily occurs among students but can also be seen in other age
groups. Individuals often complain of unpleasant feelings in the head,
visual problems, poor memory, fatigue, and sleepiness. Occasionally
they may also complain of feeling worms crawling in the head. Koro is a
CBS seen mainly among men from South East Asia but can sometimes
affect women. Amok is a CBS seen among individuals from the
Philippines and Malaysia. Latah is another CBS seen mainly among
women from Malaysia. Hwa-Byung mainly affects Korean women.

Answer 3C A pivotal study that investigated health disparities


among lesbian, gay, and bisexual (LGB) older adults found that lesbians
and bisexual women had higher odds than heterosexual women for
disability and poor mental health but not for poor physical health after
adjustment for age, income, and education. It was also found that
lesbians and bisexual women had similar rates of poor physical health,
disability, and poor mental health. Gay and bisexual men were more
likely than heterosexual men to have poor physical health, disability,
and poor mental health. There were no observed differences in health
outcomes between gay and bisexual men.

Answer 4D Female sex is the most consistent risk factor for


depression and for some anxiety disorders. The prevalence of most
anxiety disorders is higher in women than in men. Late-onset
schizophrenia is more common among women, with female-to-male
ratios ranging from 1.9:1.0 to 22.5:1.0. Suicide attempts in the elderly
are often characterized by a greater degree of lethal intent. Male gender
is a risk factor for alcohol use disorder among older adults, whereas
female sex is a risk factor for prescription drug misuse.

Further Reading
Conner KO, Copeland VC, Grote NK, et al. Mental health treatment seeking among
older adults with depression: the impact of stigma and race. Am J Geriatr Psychiatry.
2010;18(6):531–43.
[Crossref]

Faison WE, Armstrong D. Cultural aspects of psychosis in the elderly. J Geriatr


Psychiatry Neurol. 2003;16(4):225–31.
[Crossref]

Fredriksen-Goldsen KI, Kim HJ, Barkan SE, et al. Health disparities among lesbian,
gay, and bisexual older adults: results from a population-based study. Am J Public
Health. 2013;103(10):1802–9.
[Crossref]

Kohrt BA, Rasmussen A, Kaiser BN, et al. Cultural concepts of distress and
psychiatric disorders: literature review and research recommendations for global
mental health epidemiology. Int J Epidemiol. 2014;43(2):365–406.
[Crossref]

Kuerbis A, Sacco P, Blazer DG, et al. Substance abuse among older adults. Clin Geriatr
Med. 2014;30(3):629–54.
[Crossref]

Skoog I. Psychiatric disorders in the elderly. Can J Psychiatr. 2011;56(7):387–97.


[Crossref]
© Springer Nature Switzerland AG 2021
R. Tampi et al. (eds.), Absolute Geriatric Psychiatry Review
https://doi.org/10.1007/978-3-030-58663-8_4

4. Family and Community


Rajesh Tampi1 and Deena Tampi2
(1) Department of Medicine, Cleveland Clinic Lerner College of
Medicine of Case Western Reserve University, Cleveland, OH, USA
(2) Behavioral Health Advisory Group, Princeton, NJ, USA

Keywords Family – Community – Veterans affairs (VA) – Caregiver


burden – Community mental health

Questions
1.
Which of the following groups is the single largest provider of
community care for older adults in the United States?
A.
The Veterans Affairs (VA) system
B.
Medicare
C.
Medicaid
D.
Area Agencies on Aging
E.
Family caregivers
2. Which of the following is a risk factor for caregiver burden among
caregivers of older adults in the United States?
A.
Lack of choice in being a caregiver
B E h d i lif
B. Enhanced purpose in life

C.
Positive life experience
D.
Higher life satisfaction
E.
Higher quality of life
3.
Which of the following is true of current family caregivers of older
adults in the United States?
A.
The primary caregivers are mainly siblings
B.
The primary caregivers on average provided less than 20 hours
of care per week
C.
Dementia caregivers are less likely to report substantial
physical and inancial dif iculties
D.
Dementia caregivers are less likely to use respite care
E.
All of the above
4. Which of the following statements is true regarding the
effectiveness of community-based mental health outreach services
in identifying isolated older adults with mental illness?
A.
There is no evidence to support the effectiveness of these
services
B.
There is limited evidence to support the effectiveness of these
services
C.
There is strong evidence to support the effectiveness of these
services
D There is limited evidence to indicate the ineffectiveness of
D. There is limited evidence to indicate the ineffectiveness of
these services
E.
There is strong evidence to indicate the ineffectiveness of these
services

Answers
Answer 1E For the 6.6 million older adults in the United States who
receive care in the community, two-thirds (66%) tend to depend
exclusively family, friends, or neighbors (family caregivers). The VA
system and Area Agencies on Aging provide care to older veterans but
not to the extent done by the family caregivers. Medicare and Medicaid
pay for services but do not provide direct care to individuals.

Answer 2A Risk factors for caregiver burden among caregivers of


older adults in the United States include lower income, lower
education, living with the care recipient, higher number of hours spent
caregiving, self-reported depression, social isolation, inancial stress,
and the lack of choice in being a care-giver. Caregivers report little or no
burden with positive life experience, enhanced purpose in life, a higher
life satisfaction, and a higher quality of life.

Answer 3C A recent study found that primary caregivers of older


adults in the United States were mainly spouses and adult children. On
average, primary caregivers provided about or in excess of 30 hours per
week of care. When compared to 1999 in 2015, spouses provided fewer
hours of care, were twice as likely to work and half as likely to report
substantial emotional, physical, and inancial dif iculty due to
caregiving. Adult children provided comparable hours of care to a more
impaired population and a similar proportion reported substantial
caregiving-related dif iculty at each time. The use of respite care nearly
doubled from 8.5% in 1999 to 15.7% in 2015. Dementia caregivers
were less likely to report substantial physical and inancial dif iculty
and more likely to use respite care in 2015 than 1999.

Answer 4B An excellent systematic review that included data from


14 studies found that there is limited evidence to support the
effectiveness of outreach services in identifying isolated older adults
with mental illness.

Further Reading
Freedman VA, Spillman BC. Disability and care needs among older Americans.
Milbank Q. 2014;92:509–41.
[Crossref]

Musich S, Wang SS, Kraemer S, et al. Caregivers for older adults: prevalence,
characteristics and health care utilization and expenditures. Geriatr Nurs.
2017;38(1):9–16.
[Crossref]

Van Citters AD, Bartels SJ. A systematic review of the effectiveness of community-
based mental health outreach services for older adults. Psychiatr Serv.
2004;55(11):1237–49.
[Crossref]

Wolff JL, Mulcahy J, Huang J, et al. Family caregivers of older adults, 1999–2015:
trends in characteristics, circumstances, and role-related appraisal. Gerontologist.
2018;58(6):1021–32.
[Crossref]
© Springer Nature Switzerland AG 2021
R. Tampi et al. (eds.), Absolute Geriatric Psychiatry Review
https://doi.org/10.1007/978-3-030-58663-8_5

5. Ethics and Forensics


Rajesh Tampi1 and Deena Tampi2
(1) Department of Medicine, Cleveland Clinic Lerner College of
Medicine of Case Western Reserve University, Cleveland, OH, USA
(2) Behavioral Health Advisory Group, Princeton, NJ, USA

Keywords Ethics – Autonomy – Bene icence – Non-male icence –


Justice – Competence to stand trial

Questions
1. A 70-year-old married Caucasian man with a diagnosis of major
depressive disorder, recurrent, severe with psychotic features
refuses to take prescribed sertraline and olanzapine despite having
worsening mood, poor appetite, weight loss of 5 lbs. in 10 days,
poor sleep, poor energy, and worsening nihilistic delusions. The
psychiatric team decides to apply to the local probate court for a
court-ordered involuntary electroconvulsive therapy to improve
the patient’s symptoms of depression. Which of the two ethical
principles are at play against each other in this situation?
A.
Justice versus bene icence
B.
Autonomy versus justice
C.
Autonomy versus non-male icence
D.
Autonomy versus bene icence
E. Bene icence versus non-male icence
2.
Which of the following items is not needed for medical decision-
making capacity in an older adult?
A.
Knowledge of the state laws regarding a particular medical
situation
B.
Understanding of the facts of a particular medical situation
C.
Appreciating the consequences of a particular medical
situation
D.
Making a reasonable choice regarding a particular medical
situation
E.
Stating a reasonable choice regarding a particular medical
situation
3. Which of the following is a true statement regarding older
prisoners in the legal system in the United States?
A.
The annual cost of keeping an older inmate in prison is
approximately half the cost of keeping a younger inmate in
prison
B.
The healthcare costs for older prisoners is approximately half
the cost for younger prisoners
C.
Older inmates tend to have a substantially higher number of
chronic illnesses than younger inmates
D.
The number of older inmates with dementia is expected to
increase by 10% in the next three decades
E.
All of the above
4.

Which of the following is a false statement regarding competence


to stand trial among older adults?
A.
The defendant must be able to understand the objectives of the
legal proceedings and assist their counsel in their defense in
order to be deemed competent to stand trial
B.
The rates of incompetence to stand trial among older criminal
offenders is between 30% and 50%
C.
Among older adults, psychotic disorders are the most common
reason for incompetence to stand trial
D.
The lack of abstraction may affect the competence of
individuals to stand trial in early stages of dementia
E.
The restoration of competence is possible even among
individuals with dementia

Answers
Answer 1D In this scenario, the two opposing ethical principles are
autonomy versus bene icence. The patient is exercising his civil right of
autonomy by refusing the prescribed medications. The treatment team
is exercising the principle bene icence as without appropriate
treatment, the patients’ symptoms of depression will continue to
deteriorate. In healthcare, autonomy is the individual’s right to make an
informed medical decision. Bene icence is the ethical principle to help
or to do good for the patient and non-male icence is the principle to do
no harm to the patient. Justice in healthcare is the appropriate use of
medical resources.

Answer 2A The knowledge of the state laws regarding a particular


medical situation is not required to have medical decision-making
capacity. However, understanding of the facts of a particular medical
situation, appreciating the consequences of a particular medical
situation, making a reasonable choice regarding a particular medical
situation, and stating a reasonable choice regarding a particular
medical situation are essential elements of medical decision-making
capacity.

Answer 3C Older inmates tend to have a substantially higher


number of chronic illnesses than younger inmates. The annual cost of
keeping an older inmate in prison is approximately three times the cost
of keeping a younger inmate in prison. The healthcare costs for older
prisoners are approximately three and a half times the cost for younger
prisoners. The number of older inmates with dementia is expected to
increase three times in the next three decades.

Answer 4C Competence to stand trial is the legally determined


capacity of a defendant to proceed with criminal adjudication. The
Dusky Standard mandates that a defendant must be able to understand
the objectives of the legal proceedings and be able to assist their
counsel with their defense in order to be deemed competent to stand
trial. The rates of incompetence to stand trial among older criminal
offenders are between 30% and 50%. Among older adults, cognitive
disorders rather than psychotic disorders are the most common reason
for incompetence to stand trial. The lack of abstraction may affect the
competence of individuals in the early stages of dementia. The
restoration of competence is possible in individuals with dementia
despite the association of dementia with incompetence to stand trial.

Further Reading
Marson DC, Earnst KS, Jamil F, Bartolucci A, Harrell LE. Consistency of physicians’
legal standard and personal judgments of competency in patients with Alzheimer’s
disease. J Am Geriatr Soc. 2000;48(8):911–8.
[Crossref]

Tampi RR, Young J, Balachandran S, Dasarathy D, Tampi D. Ethical, legal and forensic
issues in geriatric psychiatry. Curr Psychiatry Rep. 2018a;20(1):1.
[Crossref]
Walaszek A. Clinical ethics issues in geriatric psychiatry. Psychiatr Clin North Am.
2009;32(2):343–59.
[Crossref]
© Springer Nature Switzerland AG 2021
R. Tampi et al. (eds.), Absolute Geriatric Psychiatry Review
https://doi.org/10.1007/978-3-030-58663-8_6

6. Economics and Health Policy


Rajesh Tampi1 and Deena Tampi2
(1) Department of Medicine, Cleveland Clinic Lerner College of
Medicine of Case Western Reserve University, Cleveland, OH, USA
(2) Behavioral Health Advisory Group, Princeton, NJ, USA

Keywords Economics – Health policy – Medicare – Medicaid – Patient


Protection and Affordable Care Act (ACA)

Questions
1.
Which of the following is a false statement regarding Medicare Part
A?
A.
It is a hospital insurance program that covers inpatient care in
hospitals
B.
It covers care in critical access hospitals and skilled nursing
facilities
C.
It covers hospice care
D.
It covers custodial or long-term care
E.
It is paid for by individuals from their payroll taxes while they
are employed
2. Which of the following is false regarding Medicare Part B?
A. It covers physician services and outpatient care
B.
It covers payment for prescription medications
C.
It covers physical and occupational therapy services
D.
It covers medical supplies when necessary
E.
It is paid for by monthly premiums obtained from the
individuals
3.
Which of the following is not part of the Patient Protection and
Affordable Care Act (ACA) as it relates to the care of older adults?
A.
The restructuring of Medicare reimbursements from a fee-for-
service model to a bundled payments model
B.
The provision for single payments to hospitals or physician
groups for de ined episodes of care
C.
The creation of opportunities for the recruitment of additional
geriatric healthcare professionals
D.
The development of process for measuring healthcare quality
E.
The provision of support for the development of health
information technology
4. Which of the following is true of the outcomes regarding the
Centers for Medicare & Medicaid Services National Partnership to
Improve Dementia Care in Nursing Homes to improve the quality of
care for individuals with dementia?
A.
Since initiation of the partnership, the prescription of
antipsychotics has increased in long-term care
B. Since initiation of the partnership, the prescription of mood
B. Since initiation of the partnership, the prescription of mood
stabilizers has declined in long-term care

C.

Since initiation of the partnership, the prescription of


antidepressants has increased in long-term care
D.
Antidepressants are still the most commonly prescribed
psychotropic medication overall in long-term care
E.
All of the above

Answers
Answer 1D Medicare Part A is a hospital insurance program that
covers inpatient hospital care. It pays for critical access hospital and
skilled nursing facility care. Part A also pays for hospice care and some
home healthcare. However, it does not cover custodial or long-term
care. Part A is paid for by individuals from their payroll taxes while they
are employed, and so there are no premiums levied.

Answer 2B Medicare Part B covers physician services, outpatient


care, and physical and occupational therapy services. It pays for
medical supplies when necessary. Part B is paid for by monthly
premiums obtained from individuals. It does not pay for prescription
drugs, which is covered by the Parts C and D.

Answer 3C The important ACA reforms that are related to care of


older adults include the following: the restructuring of Medicare
reimbursements from a fee-for-service model to a bundled payments
model, the provision for single payments to hospitals or physician
groups for de ined episodes of care, the development of process for
measuring healthcare quality, and the provision of support for the
development of health information technology. The creation of
opportunities for the recruitment of additional geriatric healthcare
professionals is not one of the ACA’s mandates. Additionally, the ACA
provides opportunities for the development of accountable care
organizations (ACOs), patient-centered medical homes (PCMHs),
programs to reduce hospital readmissions and greater emphasis on the
development of preventive healthcare services.

Answer 4D Maust et al. found that the prescription of psychotropic


medications to individuals in long-term care has declined, although the
partnership did not accelerate this decrease. The use of mood
stabilizers however, possibly as a substitute for antipsychotics, has
increased and accelerated after initiation of the partnership in both
long-term care residents overall and in those with dementia.
Antidepressants are still the most commonly prescribed medication
overall but as with antipsychotics, the use of antidepressants declined
both before and after initiation of the partnership.

Further Reading
Bartels SJ, Gill L, Naslund JA. The affordable care act, accountable care organizations,
and mental health care for older adults: implications and opportunities. Harv Rev
Psychiatry. 2015;23(5):304–19.
[Crossref]

https://www.cms.gov/Medicare/Medicare.html. Accessed 11 Aug 2019.

Maust DT, Kim HM, Chiang C, et al. Association of the Centers for Medicare &
Medicaid Services’ National Partnership to improve dementia care with the use of
antipsychotics and other Psychotropics in long-term Care in the United States from
2009 to 2014. JAMA Intern Med. 2018;178(5):640–7.
[Crossref]
Part II
Biological Aspects of Aging
In this part, we review the important topic of biological aspects of
aging. The topics that we will cover include neuroanatomy,
neuropathology, biochemistry, neuropharmacology, anatomy and
physiology.
Neuroanatomy · Neuropathology · Biochemistry ·
Neuropharmacology · Anatomy and physiology
Rajesh R. Tampi
Deena J. Tampi
© Springer Nature Switzerland AG 2021
R. Tampi et al. (eds.), Absolute Geriatric Psychiatry Review
https://doi.org/10.1007/978-3-030-58663-8_7

7. Neuroanatomy and Neuropathology


Rajesh Tampi1 and Deena Tampi2
(1) Department of Medicine, Cleveland Clinic Lerner College of
Medicine of Case Western Reserve University, Cleveland, OH, USA
(2) Behavioral Health Advisory Group, Princeton, NJ, USA

Keywords Neuroanatomy – Neuropathology – Aging brain – Thalamus


– Circadian rhythms – Atrophy – Limbic system – Basal ganglia –
Ventricular system

Questions
1.
Which one of the following is false regarding thalamus in an aging
brain ?
A.
It is located between the cerebral cortex and the midbrain
B.
It is composed of white matter
C.
It relays sensation, spatial sense, and motor signals to the
cerebral cortex
D.
It is preferentially affected in aging
E.
The decreased volume of the thalamus with aging results in
reduced performance on tests of cognitive speed
2. Atrophy of which of the following nuclei is associated with the age-
related loss of circadian rhythms?
A. Supraoptic nucleus

B.
Paraventricular nucleus
C.
Ventromedial nucleus
D.
Suprachiasmatic nucleus
E.
Lateral hypothalamic nuclei
3.
In the aging brain, which area tends to show preferential atrophy?
A.
Brain stem
B.
Basal ganglia
C.
Prefrontal cortex
D.
Pons
E.
Cerebellum
4. Which of the following is a true statement regarding an aging
brain?
A.
Pontine volumes decline across the age span in both men and
women
B.
Cortical white matter volumes decline across the age span in
both men and women
C.
Thalamic volume remains stable across the age span in both
men and women
D.
Cortical gray matter volume declines at a more rapid rate with
g y p
age in men than in women
E. All the above

5.
Which of the following structures in not part of the limbic system?
A.
Caudate nucleus
B.
Amygdala
C.
Hippocampus
D.
Hypothalamus
E.
Cingulate gyrus
6.
Caudate nucleus and putamen are part of which of the following
structures?
A.
Globus pallidus
B.
Striatum
C.
Subthalamic nucleus
D.
Substantia nigra
E.
Hippocampus
7. Which of the following is true of the ventricular system in the
human brain?
A.
There are two paralleling lateral ventricles
B.
There are two adjacent third ventricles
C.
There are two superimposed fourth ventricles
D. Cerebral aqueduct connects the third ventricle system to the
lateral ventricles

E.
Cerebrospinal luid is produced by the arachnoid villi
8.
Which of the following is a correct statement regarding an aging
brain?
A.
The frontal lobe volume decreases by about 2%
B.
The temporal lobe volume decreases by about 1%
C.
The parietal and occipital lobe volumes decrease by about 4%
D.
Despite brain volume loss, there is usually minimal neuronal
loss
E.
All of the above
9.
Which of the cranial nerves are paired incorrectly?
A.
Cranial nerve I → Optic nerve
B.
Cranial nerve II → Olfactory nerve
C.
Cranial nerve III → Oculomotor nerve
D.
Cranial nerve V → Trigeminal nerve
E.
Choices A and B
Answers
Answer 1B The thalamus is a symmetrical paired structure that is
situated between the cerebral cortex and the midbrain. It lies above the
hypothalamus and surrounds the third ventricle. The thalamus is
composed of gray matter. It relays sensation, spatial sense, and motor
signals to the cerebral cortex. The thalamus also regulates sleep and
consciousness. It is preferentially affected in aging. The decreased
volume of the thalamus with aging results in reduced performance on
tests of cognitive speed.

Answer 2D Atrophy of the suprachiasmatic nucleus is associated


with age-related loss of circadian rhythms. The supraoptic and
paraventricular nuclei are associated with osmoregulation. These
nuclei release vasopressin, and lesions to these nuclei produce diabetes
insipidus. Ventromedial and lateral hypothalamic nuclei are associated
with satiety. Lesion to the ventromedial nucleus results in overeating,
while a lesion to the lateral hypothalamus produces anorexia.

Answer 3C
In an aging brain, the prefrontal cortex along with the lateral, parietal,
and sensorimotor regions tend to show preferential atrophy when
compared to other areas of the brain.

Answer 4D Both pontine and cortical white matter volumes remain


stable across the age span in both men and women. Thalamic volume
declines linearly with age at a similar rate in both men and women.
Cortical gray matter volume declines at a more rapid rate with age in
men than in women. Thalamic, pontine, and cortical white matter
volumes do not differ between men and women, but men had more
cortical gray matter than women.

Answer 5A The limbic system is a functional concept that consists


of the limbic lobe and other subcortical structures. These structures
and their connections form a complex network for controlling
emotions. The structures that constitute the limbic system include the
limbic cortex (cingulate gyrus and parahippocampal gyrus),
hippocampal formation (the dentate gyrus, hippocampus, and subicular
complex), the amygdala, the septal area, and the hypothalamus.
Caudate nucleus is part of the basal ganglia.

Answer 6B Caudate nucleus and putamen together make up the


striatum. The basal ganglia is a constellation of a set of deep forebrain
nuclei including the striatum (caudate and putamen), globus pallidus
(internal and external segments), subthalamic nucleus, and substantia
nigra (pars reticulata and pars compacta).

Answer 7A In the human brain, there are two lateral ventricles, one
third ventricle, and one fourth ventricle. The cerebral aqueduct
connects the third ventricle to the fourth ventricle. The cerebrospinal
luid is produced by the choroid plexus and not by the arachnoid villi.

Answer 8D As individuals age (>50 years), the frontal lobe volumes


decrease by about 12% and the temporal lobe volume decreases by
about 9%. However, the parietal and occipital lobes show little to no
atrophy. The annual shrinkage rates for brain volumes is 0.32% for the
total brain volume with the hippocampal and temporal atrophy at
about 0.68% and 0.82%, respectively. Despite the brain volume loss,
there is usually minimal neuronal loss if any.

Answer 9E Human beings have 12 pairs of cranial nerves (I–XII).


They are as follows: the olfactory nerve (I), the optic nerve (II),
oculomotor nerve (III), trochlear nerve (IV), trigeminal nerve (V),
abducens nerve (VI), facial nerve (VII), vestibulocochlear nerve (VIII),
glossopharyngeal nerve (IX), vagus nerve (X), accessory nerve (XI), and
the hypoglossal nerve (XII).

Further Reading
Bakkour A, Morris JC, Wolk DA, Dickerson BC. The effects of aging and Alzheimer’s
disease on cerebral cortical anatomy: speci icity and differential relationships with
cognition. NeuroImage. 2013;76:332–44.
[Crossref]

Dickstein DL, Morrison JH, Hof PR. Neuropathology of aging. In: Jagust W, D'Esposito
M, editors. Imaging the aging brain. New York: Oxford; 2010. p. 1–35.
Jacobson S, Marcus EM. hypothalamus, neuroendocrine system, and autonomic
nervous system. Neuroanatomy for the neuroscientist. Boston: Springer; 2008a. p.
165–87.

Jacobson S, Marcus EM. Hypothalamus, neuroendocrine system, and autonomic


nervous system. Neuroanatomy for the neuroscientist. Boston: Springer; 2008b. p.
121–45.

Nelson AB, Kreitzer AC. Reassessing models of basal ganglia function and
dysfunction. Annu Rev. Neurosci. 2014;37:117–35.
[Crossref]

Rajmohan V, Mohandas E. The limbic system. Indian J Psychiatry. 2007;49(2):132–9.


[Crossref]

Sollars PJ, Pickard GE. The neurobiology of circadian rhythms. Psychiatr Clin North
Am. 2015;38(4):645–65.
[Crossref]

Stratchko L, Filatova I, Agarwal A, Kanekar S. The ventricular system of the brain:


anatomy and normal variations. Semin Ultrasound CT MR. 2016;37(2):72–83.
[Crossref]

Sullivan EV, Rosenbloom M, Serventi KL, Pfefferbaum A. Effects of age and sex on
volumes of the thalamus, pons, and cortex. Neurobiol Aging. 2004;25(2):185–92.
[Crossref]

Van Der Werf YD, Tisserand DJ, Visser PJ, Hofman PA, Vuurman E, Uylings HB, Jolles J.
Thalamic volume predicts performance on tests of cognitive speed and decreases in
healthy aging. A magnetic resonance imaging-based volumetric analysis. Brain Res
Cogn Brain Res. 2001;11(3):377–85.
[Crossref]
© Springer Nature Switzerland AG 2021
R. Tampi et al. (eds.), Absolute Geriatric Psychiatry Review
https://doi.org/10.1007/978-3-030-58663-8_8

8. Biochemistry and
Neuropharmacology
Rajesh Tampi1 and Deena Tampi2
(1) Department of Medicine, Cleveland Clinic Lerner College of
Medicine of Case Western Reserve University, Cleveland, OH, USA
(2) Behavioral Health Advisory Group, Princeton, NJ, USA

Keywords Biochemistry – Neuropharmacology – Neurons – Receptors


– Neurotransmitters – Amino acids – Excitatory – Inhibitory –
Catecholamines

Questions
1.
Which one of the following is an amino acid neurotransmitter?
A.
Vasopressin
B.
Serotonin
C.
Dopamine
D.
Glutamate
E.
Acetylcholine
2. Which of the following choices correctly pair the type of neurons
with the primary areas from which they project?
A Noradrenergic neurons → Raphe nuclei
A. Noradrenergic neurons → Raphe nuclei

B.
Serotonergic neurons → Locus coeruleus
C.
Dopaminergic neuron → Pontomesencephalotegmental
complex
D.
Cholinergic neurons → Basal forebrain complex
E.
All of the above
3.
Which of the following statements is correct?
A.
Α-ketoglutarate is converted to γ-aminobutyric acid (GABA)
using the enzyme γ-aminobutyric acid transferase (GABA-T)
B.
Glutamine is converted to glutamate using the enzyme
glutamic acid decarboxylase (GAD)
C.
5-Hydroxytryptophan is converted to serotonin using the
enzyme 5-hydroxytryptophan decarboxylase
D.
Dopamine is converted to epinephrine using the enzyme
tyrosine hydroxylase
E.
All of the above
4. Which of the following is the main excitatory neurotransmitter in
the human brain and the spinal cord?
A.
GABA
B.
Serotonin
C.
Dopamine
D Gl t t
D. Glutamate

E.
Acetylcholine
5.
Which of the following is the main inhibitory neurotransmitter in
the human brain and the spinal cord?
A.
GABA
B.
Serotonin
C.
Dopamine
D.
Glutamate
E.
Acetylcholine
6.
Which one of the following is not a metabotropic receptor?
A.
Nicotinic receptors
B.
Dopamine receptors
C.
Noradrenaline receptors
D.
Muscarinic receptors
E.
5HT2A (Serotonin) receptors
7. Riluzole, memantine and amantadine are thought to exert their
clinical effects via which of the following mechanisms?
A.
Serotonin receptor agonism
Serotonin receptor agonism
B.
Dopamine receptor antagonism

C. NMDA receptor antagonism

D.
Norepinephrine receptor antagonism
E.
Acetylcholine receptor agonism
8.
Which of the following is the rate-limiting step in the synthesis of
catecholamines?
A.
The conversion of tyrosine to dihydroxyphenylalanine (DOPA)
by the enzyme tyrosine hydroxylase
B.
The conversion of dihydroxyphenylalanine (DOPA) to
dopamine by the enzyme amino acid decarboxylase
C.
The conversion of dopamine to norepinephrine by the enzyme
dopamine-β-hydroxylase
D.
The conversion of norepinephrine to epinephrine by the
enzyme phenylethanolamine-N-methyltransferase
E.
The conversion of norepinephrine to 3,4-dihydroxymandelic
acid by the enzyme monoamine oxidase
9. Which of the following is associated the irst pain sensation?
A.
Dopamine
B.
Serotonin
C.
Glutamate
D.
Histamine
E.
Substance P

Answers
Answer 1D There are two main classes of neurotransmitters: small-
molecule transmitters and large-molecule transmitters. The small-
molecule transmitters include amino acid neurotransmitters
(glutamate, gamma aminobutyric acid (GABA), and glycine),
acetylcholine, monoamines (norepinephrine, epinephrine, dopamine,
and serotonin), and adenosine triphosphate (ATP). Large-molecule
transmitters include neuropeptides such as substance P, enkephalin,
and vasopressin.

Answer 2D Cholinergic neurons originate primarily from the basal


forebrain complex and innervate the hippocampus and the neocortex.
Cholinergic neurons do also originate in the
pontomesencephalotegmental cholinergic complex and innervate the
dorsal thalamus and the forebrain. Noradrenergic neurons primarily
originate from the locus coeruleus and innervate the spinal cord,
cerebellum, hypothalamus, thalamus, basal telencephalon, and
neocortex. Serotonergic neurons originate in the raphe nuclei and
innervate the hypothalamus, limbic system, neocortex, cerebellum, and
spinal cord. The dopaminergic neurons that originate in the substantia
nigra innervate the striatum, whereas dopaminergic neurons
originating from the ventral tegmental area of the midbrain innervate
the prefrontal cortex of the limbic system.

Answer 3C Glutamate is synthesized by converting α-ketoglutarate


using the enzyme γ-aminobutyric acid transferase (GABA-T) or in nerve
terminals by the hydrolysis of glutamine using the enzyme glutaminase.
γ-aminobutyric acid (GABA) is synthesized by converting glutamate
using the enzyme glutamic acid decarboxylase (GAD). Serotonin (5-
hydroxytryptamine) is synthesized from tryptophan by a two-step
process – irst through enzymatic hydroxylation of tryptophan to 5-
hydroxytryptophan and then the enzymatic decarboxylation of 5-
hydroxytryptophan to 5-hydroxytryptamine (serotonin).
Catecholamines are synthesized from tyrosine by a multi-step process:
First tyrosine is oxidized to dihydroxyphenylalanine (DOPA) by the
enzyme tyrosine hydroxylase. Then the DOPA is decarboxylated to
dopamine using the enzyme DOPA decarboxylase. In noradrenergic
neurons, dopamine is converted to norepinephrine using dopamine-β-
hydroxylase. Norepinephrine can then be converted to epinephrine
using the enzyme phenylethanolamine-N-methyltransferase.

Answer 4D The amino acid neurotransmitter glutamate is the main


excitatory neurotransmitter in the human brain and the spinal cord. It
is responsible for 3/4th of all the excitatory transmission within the
central nervous system (CNS).

Answer 5A GABA is the main inhibitory neurotransmitter in the


human brain and spinal cord. It mediates both presynaptic inhibition
and postsynaptic inhibition.

Answer 6A The dopamine, noradrenaline, and muscarinic receptors


are all metabotropic receptors. The nicotinic receptors are ionotropic
receptors. All serotonin receptors except the 5HT3 receptors are
metabotropic receptors. The 5HT3 receptors are directly coupled to the
ion channels.

Answer 7C Riluzole, memantine, and amantadine are thought to


exert their clinical effects via NMDA receptor antagonism.

Answer 8A The rate-limiting step in the synthesis of catecholamines


is the conversion of tyrosine to DOPA by the enzyme tyrosine
hydroxylase. Tyrosine hydroxylase is controlled by feedback inhibition
by dopamine and norepinephrine thus providing the internal control of
the synthesis of catecholamines.

Answer 9C Impulses are transmitted from the nociceptors via thinly


myelinated Aδ ibers and unmyelinated C ibers. The activation of Aδ
ibers release glutamate, which is responsible for irst pain (fast pain).
Activation of C ibers releases a combination of glutamate and
substance P, which is responsible for the delayed second pain (slow
pain).
Further Reading
Neurotransmitters & neuromodulators. In: Barrett KE, Barman SM, Brooks HL, Yuan
JJ, editors. Ganong’s review of medical physiology. 26th ed. New York: McGraw-Hill;
http://accessmedicine.mhmedical.com/content.aspx?bookid=2525& sectionid=
204291077. Accessed 15 Sept 2019.

Neurotransmitters & neuromodulators. In: Barrett KE, Barman SM, Brooks HL, Yuan
JJ, editors. Ganong’s review of medical physiology. 26th ed. New York: McGraw-Hill;
http://accessmedicine.mhmedical.com/content.aspx?bookid=2525& sectionid=
204291077. Accessed 9 Oct 2019.

Neurotransmitters & neuromodulators. In: Barrett KE, Barman SM, Brooks HL, Yuan
JJ, editors. Ganong’s review of medical physiology. 26th ed. New York: McGraw-Hill;
http://accessmedicine.mhmedical.com/content.aspx?bookid=2525& sectionid=
204291077. Accessed 10 Oct 2019.

Neurotransmitters & neuromodulators. In: Barrett KE, Barman SM, Brooks HL, Yuan
JJ, editors. Ganong’s review of medical physiology. 26th ed. New York: McGraw-Hill;
http://accessmedicine.mhmedical.com/content.aspx?bookid=2525& sectionid=
204291077. Accessed 11 Oct 2019.

Neurotransmitters & neuromodulators. In: Barrett KE, Barman SM, Brooks HL, Yuan
JJ, editors. Ganong’s review of medical physiology. 26th ed. New York: McGraw-Hill;
https://accessmedicine.mhmedical.com/content.aspx?bookid=2525& sectionid=
204291198#1159051761. Accessed 14 June 2020.
© Springer Nature Switzerland AG 2021
R. Tampi et al. (eds.), Absolute Geriatric Psychiatry Review
https://doi.org/10.1007/978-3-030-58663-8_9

9. Anatomy and Physiology


Rajesh Tampi1 and Deena Tampi2
(1) Department of Medicine, Cleveland Clinic Lerner College of
Medicine of Case Western Reserve University, Cleveland, OH, USA
(2) Behavioral Health Advisory Group, Princeton, NJ, USA

Keywords Anatomy – Physiology – Horner’s syndrome – Hemianopsia


– Aphasia – Hemi-neglect – Ataxia – Apraxia – Lower motor neuron
(LMN) lesion

Questions
1. A 70-year-old man develops sudden onset of ipsilateral Horner’s
syndrome; ipsilateral loss of pain and temperature sensation of the
face; cerebellar ataxia; weakness of the palate, pharynx, and vocal
cords; and the contralateral loss of pain and temperature sensation
of the body. These symptoms occur due to the occlusion of the
which of the following blood vessels?
A.
Vertebral artery
B.
Anterior inferior cerebellar artery
C.
Anterior spinal artery syndrome
D.
Posterior cerebral artery
E.
Initial segment of the middle cerebral artery
2.
A 75-year-old woman presents with sudden onset of bitemporal
hemianopsia. Her symptoms have occurred due to a lesion in which
part of the visual pathway?
A.
The right temporal lobe
B.
The right occipital lobe
C.
The right lateral geniculate nucleus
D.
The left optic nerve
E.
The optic chiasm

3. A 79-year-old woman presents to a geriatric psychiatry clinic with


a 1-month history of inability to repeat instructions that her family
is giving her. She is able to comprehend information and converse
normally. The woman is also able to read and write information but
cannot repeat information verbally. She is independent in all her
activities of daily living. The family thinks that the woman is being
oppositional, as she could not go on vacation to Florida as they had
planned. However, they do not describe any other behavioral
symptoms. The examination is normal except for the woman’s
inability to repeat verbal information. She also scored of 30 out of
30 on the Montreal Cognitive Assessment (MoCA). What is the most
likely diagnosis for this patient given the available information?
A.
The woman has Broca’s aphasia
B.
The woman has conduction aphasia
C.
The woman has Wernicke’s aphasia
D.
The woman is faking her symptoms
E.
The woman has frontotemporal dementia
4.
An 80-year-old right-handed woman does not acknowledge that
the left side of her body is paralyzed following a cerebrovascular
accident (CVA). Where is the lesion in her brain most likely located?
A.
Right parietal lobe
B.
Left parietal lobe
C.
Left temporal lobe
D.
Right temporal lobe
E.
Left occipital lobe
5.
A 78-year-old woman is seen at your clinic with a 2-month history
of repeated falls. On examination, the woman is able to stand
straight with her feet together and eyes open, but becomes
unsteady when her eyes are closed. Which part of the woman’s
nervous system is most affected to produce this inding?
A.
Cerebellum
B.
Dorsal columns of spinal cord
C.
Temporal lobe
D.
Pons
E.
Optic chiasma
6. Which of the following is incorrectly paired?
A.
Gait apraxia → Pontine damage
B.
Gait ataxia → Cerebellar damage
C. Festinating gait → Parkinson’s disease

D.
Hemiparetic gait → Cerebrovascular accident
E.
High stepping gait → Peripheral neuropathy
7.
Which of the following is not a characteristic sign of lower motor
neuron lesion?
A.
Atrophy
B.
Hypoactive deep tendon re lexes
C.
Babinski sign
D.
Weakness
E.
Muscle laccidity

Answers
Answer 1A This patient is presenting with symptoms of
Wallenberg’s syndrome or lateral medullary syndrome, which is caused
by the occlusion of the vertebral artery. Individuals with this syndrome
present with a loss of pain and temperature sensation on the
contralateral side of the body below the level of the lesion along with
ipsilateral Horner’s syndrome, motor and sensory loss in the face, and
cerebellar ataxia. Occlusion of the anterior inferior cerebellar artery
will result in vertigo and nystagmus, ipsilateral deafness, ipsilateral
Horner’s syndrome, and contralateral loss of pain and temperature
sensation in the body. Occlusion of the anterior spinal artery will result
in a laccid paralysis below the level of the lesion and loss of pain and
temperature sensation with preserved light touch, position sense, and
vibration sense. Occlusion of the posterior cerebral artery causes
Weber’s syndrome, which is characterized by ipsilateral pupillary
dilation, ipsilateral oculomotor paresis, and contralateral paralysis of
the face, arm, and leg. Occlusion of the initial segment of the middle
cerebral artery will cause an acute onset of a dense contralateral
paralysis and hemi-anesthesia involving the face, arm, and leg along
with contralateral homonymous hemianopia and deviation of the eyes
to the ipsilateral side of the infarct. When the dominant cerebral
hemisphere is involved, the individual will also be mute with no
understanding of speech.

Answer 2E Bitemporal hemianopsia occurs due to a lesion in the


optic chiasm, for example, a pituitary tumor or a craniopharyngioma. A
lesion in the right temporal lobe part of the visual pathway will result in
a left homonymous upper quadrant defect. A lesion in the right
occipital lobe will produce a left homonymous hemianopia. A lesion in
the right lateral geniculate nucleus will cause a left homonymous
hemianopia. A lesion in the left optic nerve will result in no perception
of light from the left eye.

Answer 3B Unless proven otherwise, this woman has conduction


aphasia. Individuals with this type of aphasia have relatively intact
auditory comprehension and spontaneous speech, but they have
trouble repeating words due to damage to the arcuate fasciculus, which
connects the Wernicke’s and Broca’s areas in the brain. Broca’s aphasia
(non- luent or expressive aphasia) is characterized by impaired verbal
luency and repetition, but intact auditory comprehension. However,
these individuals are unable to read or write appropriately. In
Wernicke’s aphasia (sensory aphasia or receptive aphasia), reading
comprehension is usually abnormal and verbal output is either normal
or increased. Additionally, sentences have no meaning and paraphasic
errors are common. There is no evidence to suggest that the patient is
faking her symptoms. Frontotemporal dementia is unlikely at this time
as there is no cognitive and functional decline and there are no
behavioral symptoms.
Answer 4A This woman has a lesion of the right parietal lobe that is
causing left hemineglect. Additional symptoms from a right parietal
lobe CVA would include a left-sided paralysis and anosognosia.

Answer 5B This woman is presenting with a positive Romberg sign.


Individuals who have lesions of the dorsal columns of the spinal cord
present with sensory ataxia. They lose balance when they close their
eyes, as they are unaware of the position of their legs. Individuals with
cerebellar ataxia have gait and balance problems irrespective of their
eyes being open or closed.

Answer 6A Gait apraxia (magnetic gait) is seen among individuals


with normal pressure hydrocephalus and not pontine damage.

Answer 7C Atrophy, hypoactive deep tendon re lexes, the absence of


a Babinski sign, weakness, and laccidity are all signs of lower motor
neuron lesion. Extensor plantar response (Babinski sign present),
weakness, spasticity, rigidity, and hyperactive deep tendon re lexes are
all signs of an upper motor neuron (UMN) lesion.

Further Reading
Jacobson S, Marcus EM. Cerebral vascular disease. Neuroanatomy for the
neuroscientist. Boston: Springer; 2008a. p. 409–33.
[Crossref]

Jacobson S, Marcus EM. Visual system and occipital lobe. Neuroanatomy for the
neuroscientist. Boston: Springer; 2008b. p. 311–36.
[Crossref]

Jacobson S, Marcus EM. Cerebral cortex functional localization. Neuroanatomy for


the neuroscientist. Boston: Springer; 2008c. p. 169–86.
[Crossref]

Jacobson S, Marcus EM. Spinal cord. Neuroanatomy for the neuroscientist. Boston:
Spring; 2008d. p. 169–86.
[Crossref]

Khasnis A, Gokula RM. Romberg’s test. J Postgrad Med. 2003;49(2):169–72.


[PubMed]
Mazzoni P, Pearson TS, Rowland LP. Aphasia, apraxia, agnosia Merritt’s neurology
handbook. Philadelphia: Lippincott Williams & Wilkins; 2006. p. 6–9.

Pearce JM. Romberg and his sign. Eur Neurol. 2005;53(4):210–3.


[Crossref]

Salzman B. Gait and balance disorders in older adults. Am Fam Physician.


2010;82(1):61–8.
[PubMed]
Part III
Psychiatric Diagnosis
In this part, we review common psychiatric disorders seen among older
adults including neurocognitive disorders, depressive and bipolar
disorders, sleep-wake disorder, schizophrenia spectrum and other
psychotic disorders, substance-related and addictive disorders, anxiety
disorders and obsessive-compulsive and related disorders and sexual
dysfunctions, gender dysphoria and paraphilic disorders.
Neurocognitive disorders · Depressive and bipolar disorders · Sleep-
wake disorder · Schizophrenia spectrum and other psychotic disorders
· Substance-related and addictive disorders · Anxiety disorders and
obsessive-compulsive and related disorders and sexual dysfunctions ·
Gender dysphoria · Paraphilic disorders
Rajesh R. Tampi
Deena J. Tampi
Juan Joseph Young
Insiya Nasrulla
Nery A. Diaz
Pallavi Joshi
© Springer Nature Switzerland AG 2021
R. Tampi et al. (eds.), Absolute Geriatric Psychiatry Review
https://doi.org/10.1007/978-3-030-58663-8_10

10. Neurocognitive Disorders


Rajesh Tampi1 and Deena Tampi2
(1) Department of Medicine, Cleveland Clinic Lerner College of
Medicine of Case Western Reserve University, Cleveland, OH, USA
(2) Behavioral Health Advisory Group, Princeton, NJ, USA

Keywords Mild neurocognitive disorder – Major neurocognitive


disorder – Dementia – Traumatic brain injury – Delirium – Alzheimer’s
disease (AD) – Vascular disease (VD) – Frontotemporal degeneration –
Lewy body disease

Questions
1.
Which of the following factors differentiates mild neurocognitive
disorder from major neurocognitive disorder?
A.
Age
B.
Family history
C.
Level of education
D.
Subjective and objective concerns
E.
Absence of decline in activities of daily living
2. Which of the following is not a known risk factor for mild
neurocognitive disorder?
A Old
A. Older age

B.
Female sex
C.
Lower educational level
D.
Vascular risk factors
E.
Apolipoprotein E (APOE) e4 genotype
3.
What is the rate of progression of mild neurocognitive disorder to
major neurocognitive disorder?
A.
≤1% per year
B.
2–4% per year
C.
5–7% per year
D.
8–15% per year
E.
20–25% per year
4. Which of the following is a risk factor for progression of mild
neurocognitive disorder to major neurocognitive disorder?
A.
Younger age
B.
More years of education
C.
Higher baseline cognitive function
D.
APOE e4 allele
E.
Non-amnestic single mild neurocognitive disorder type
5.
Among the choices listed, which is the best screening tool for mild
neurocognitive disorder?
A.
Mini Mental State Examination
B.
Montreal Cognitive Assessment
C.
Mini Cog
D.
Clock Drawing Test
E.
Memory Impairment Screen

6.
What is the most common etiology for a major neurocognitive
disorder among older adults?
A.
Alzheimer’s disease
B.
Vascular disease
C.
Lewy body disease
D.
Parkinson’s disease
E.
Frontotemporal lobar degeneration
7. Which of the following is associated with decreased risk for the
development of Alzheimer’s disease (AD)?
A.
Head injury in males
B.
Older age
C.
Diabetes mellitus
D.
APOE ε2
E. Current smoking

8.
Which of the following is a true statement regarding the
association between late life depression (LLD) and major
neurocognitive disorder?
A.
LLD is consistently and similarly associated with a tenfold
increased risk for development of a major neurocognitive
disorder
B.
LLD is consistently and similarly associated with a twofold
increased risk for development of a major neurocognitive
disorder
C.
LLD is not associated with any increased risk for development
of a major neurocognitive disorder
D.
LLD is consistently and similarly associated with a ivefold
reduced risk for development of a major neurocognitive
disorder
E.
LLD is consistently and similarly associated with a threefold
reduced risk for development of a major neurocognitive
disorder
9. Which of the following is the most common mode of inheritance
for familial early onset Alzheimer’s disease (EOAD)?
A.
Autosomal dominant
B.
Autosomal recessive
C.
X-linked recessive
D.
Y-linked
E.
Polygenic
10.
Which of the following is the most prevalent genetic risk factor for
Alzheimer’s disease?
A.
Amyloid precursor protein
B.
Presenilin 1
C.
Presenilin 2
D.
Apolipoprotein E4 (APOE4)
E.
Apolipoprotein E2 (APOE2)
11.
Which of the following is a core feature for the diagnosis of major
neurocognitive disorder with Lewy bodies?
A.
Rapid eye movement behavior disorder
B.
Severe neuroleptic sensitivity
C.
Fluctuating cognition with pronounced variations in attention
and alertness
D.
Apathy or inertia
E.
Loss of sympathy or empathy
12. Which of the following is not true of major frontotemporal
neurocognitive disorder?
A.
It is the second most common type of major neurocognitive
yp j g
among individuals under 65 years in age
B.
The average age of onset is between 45 years and 65 years
C. The peak prevalence is between the ages of 65 years and
69 years

D.

Men are more commonly affected than women


E.
Behavioral variant is four times more common than the
language/aphasia variant
13.
Which of the following is true regarding major neurocognitive
disorder due to vascular disease?
A.
It is more common among men than women
B.
It is more common among individuals with higher
educational status
C.
The average rate of cognitive decline is faster than that of
Alzheimer’s disease
D.
The mean survival is between 3 years and 5 years
E.
All of the above
14. Which one of the following features is most helpful in
differentiating dementia with Lewy bodies (DLB) from
Parkinson’s disease dementia (PDD)?
A.
Age of onset of symptoms
B.
Female preponderance
C.
Onset of cognitive symptoms in relation to motor symptoms
g y p y p
D.
Family history
E.
Type of motor symptoms
15.

Which of the following is not a potentially modi iable risk factor


for major neurocognitive disorder due to Alzheimer’s disease?
A.
Marital status
B.
Diabetes
C.
Obesity
D.
Depression
E.
Low educational attainment

16.
If it were possible to eliminate the seven most important
modi iable risk factors for major neurocognitive disorder due to
Alzheimer’s disease, by what percentage would the incidence of
the illness decline worldwide?
A.
2%
B.
5%
C.
10%
D.
13%
E.
30%
17. Which of the following disorders is part of the major
neurodegenerative clinicopathological entity called the Parkinson
plus syndromes?
A.
Progressive supranuclear palsy (PSP)
B.
Corticobasal degeneration (CBD)
C.
Multiple system atrophy (MSA)
D. Dementia with Lewy bodies (DLB)

E.
All of the above
18.
Which of the following features is more common in Parkinson’s
disease when compared to Parkinson plus syndromes?
A.
Rapid progression rate
B.
Asymmetric distribution
C.
Poor response to levodopa or other dopaminergic
medications
D.
Presentation of atypical clinical features
E.
All of the above
19. Which of the following is true regarding the genetics of major
frontotemporal neurocognitive disorder?
A.
10–20% of cases have a genetic component
B.
X linked recessive is the common form of inheritance
C.
Most familial cases of have mutations in the gene encoding
Most familial cases of have mutations in the gene encoding
valosin-containing protein (VCP)
D.
Minority of the cases are due to mutations in the microtubule
associated protein tau (MAPT), the progranulin (GRN) genes,
or the hexanucleotide repeat expansion in the C9ORF72 gene
E.
All of the above
20.
Which one of the following is not commonly associated with
sporadic Creutzfeldt-Jakob disease (CJD)?
A.
Rapidly progressive dementia
B.
REM behavior disorder
C.
EEG readings of 1 Hz to 2 Hz periodic sharp-wave complexes
D.
Elevated CSF protein 14-3-3
E.
85% to 90% of individuals die within 1 year

21. A 60-year-old woman presents with right-sided limb rigidity,


akinesia, limb dystonia and myoclonus, orobuccal apraxia, cortical
sensory de icit, alien hand phenomena, and progressive cognitive
decline. The brain positron emission tomography (PET) scan in
this woman shows decreased glucose uptake suggestive of
hypometabolism in multiple cortical and subcortical regions.
What is the most probable diagnosis for this patient?
A.
Alzheimer’s disease
B.
Vascular dementia
C.
Multisystem atrophy
D.
Corticobasal degeneration
Corticobasal degeneration
E.
Parkinson’s disease
22.
Which of the following is the least likely clinical feature to be seen
in a 70-year-old man recently diagnosed with multiple-system
atrophy?
A.
Severe autonomic failure
B.
Levodopa-refractory parkinsonism
C.
Dementia
D.
Cerebellar ataxia
E.
Respiratory disturbances

23.
Which of the following features would exclude an individual from
a diagnosis of progressive supranuclear palsy (PSP)?
A.
Gradually progressive bradykinesia
B.
Cerebellar signs
C.
Vertical gaze palsy
D.
Disproportionate postural instability
E.
Onset at age ≥40 years
24. Which of the following is true regarding motor symptoms and
frontotemporal dementia (FTD)?
A.
Mild features of motor neuron disease (MND) can occur in up
to 5% of individuals with FTD
B.
MND is more common among individuals with semantic-
variant primary progressive aphasia than with behavioral-
variant FTD
C.
Early parkinsonism is present in up to 80% of individuals
with FTD
D.
Individuals with FTD may present with features of
corticobasal syndrome or progressive supranuclear palsy
syndrome
E. All of the above

25.
Which of the following is true regarding cognitive decline among
individuals with multiple sclerosis (MS)?
A.
About 10% of individuals with MS develop some type of
cognitive decline
B.
Cognitive decline in MS tends to occur late in the illness
C.
Cognitive decline in MS is seen more commonly among
women
D.
Brain atrophy is the strongest correlate for cognitive decline
in MS
E.
All of the above
26. Which of the following is not true of HIV-associated
neurocognitive disorder (HAND)?
A.
HAND is characterized by a rapid onset and progression of
symptoms
B.
HAND does not reduce the level of consciousness
C.
When HAND progresses, signs of psychomotor slowing occur
D.
HAND is caused by a multi-dimensional and complex
immunopathological processes
E.
Combined antiretroviral therapy (CART) is the treatment of
choice for HAND
27.
Which of the following is the most common psychiatric
complication of traumatic brain injury (TBI)?
A.
Major depression
B.
Bipolar disorder
C.
Generalized anxiety disorder
D.
Obsessive compulsive disorder (OCD)
E.
Posttraumatic stress disorder (PTSD)

28. Which of the following is true of the association with TBI in older
age compared to TBI in younger age?
A.
Shorter periods of agitation
B.
Lower risk for developing cognitive impairment
C.
Lower risk for developing permanent disability
D.
Higher risk for developing mass lesions
E.
All of the above
29.
Which of the following is true of the progression of cognitive
impairment and the timing in its relationship to the TBI event?
A.
There are ive stages of evolution of cognitive impairment
after TBI
B.
The irst stage involves posttraumatic amnesia
C.
The second stage is related to loss of consciousness
D.
The third stage involves a relatively rapid recovery over the
next several months
E.
All of the above
30.
Which of the following is the most common behavioral and
psychological symptom of dementia (BPSD)?
A.
Delusions
B.
Hallucinations
C.
Anxiety
D.
Apathy
E.
Inappropriate sexual behaviors

31. Which of the following is not consistently associated with a higher


level of apathy among individuals with Alzheimer’s disease (AD)?
A.
Neuronal loss
B.
Increased neuro ibrillary tangles (NFT)
C
C.
Increased levels of tau proteins in the cerebrospinal luid
(CSF)
D.
Beta-amyloid plaque burden
E.
All of the above
32.
Which of the following is not true regarding delirium among older
adults?
A.
Delirium has a high prevalence among older adults
B.
Delirium is often unrecognized among older adults
C.
Delirium is associated with long-term functional decline
among older adults
D.
Delirium is associated with increased mortality among older
adults
E.
Clouding of consciousness is a hallmark feature of delirium
among older adults

33. Which one of the following is the most commonly used delirium
instrument in the world?
A.
Confusion Assessment Method (CAM)
B.
4A’s Test (4AT)
C.
Neelon and Champagne (NEECHAM) Confusion Scale
D.
Delirium Rating Scale-Revised-98 (DRS-R-98)
E.
Memorial Delirium Assessment Scale (MDAS)
34.
Which of the following is not a known leading risk factor for
delirium among older adults?
A.
Dementia or cognitive impairment
B.
Functional impairment
C.
Vision impairment
D.
Presence of PTSD
E.
Age >70 years
35.
Which of the following is a precipitating factor for delirium among
older adults?
A.
Dementia or cognitive impairment
B.
Functional impairment
C.
Vision impairment
D.
Use of physical restraints
E.
Age >70 years

36. Which of the following is the most prevalent subtype of delirium


seen among older adults?
A.
Hyperactive
B.
Hypoactive
C.
Mixed
Mixed
D.
Subsyndromal
E.
Nocturnal delirium
37.
What percentage of cases of delirium among older adults is
thought to be preventable?
A.
2–3%
B.
10–15%
C.
20–25%
D.
30–40%
E.
60–70%
38.
Which of the following is true regarding sundowning in
individuals with dementia?
A.
A clear and univocal de inition of the sundowning has been
achieved
B.
It is considered to be a syndrome and not a symptom
C.
It is seen among all individuals who develop dementia
D.
Cholinergic de icit has been identi ied as the de inite
causative factor for sundowning
E.
All of the above
39. Which of the following statements regarding the association
between alcohol use and the development of dementia is
supported by the best current evidence?
pp y
A.
Light-to-moderate alcohol use in young adulthood is
associated with an increased risk for dementia
B.
Light-to-moderate alcohol use in young adulthood is
associated with a decreased risk for dementia
C.
Light-to-moderate alcohol use in middle to late adulthood is
associated with an increased risk for dementia
D.
Light-to-moderate alcohol use in middle to late adulthood is
associated with a decreased risk for dementia
E.
Alcohol use has no association with the development of
dementia at any age range
40.
Which of the following is true regarding the relationship between
TBI and dementia ?
A.
TBI appears to increase the age of onset of dementia
B.
Increasing severity of a single moderate-to-severe TBI
increases the risk of developing AD
C.
APOE ɛ4 alleles have no effect on the risk of individuals with
TBI developing AD
D.
Acute TBI results in the development of vascular pathology
E.
All of the above

Answers
Answer 1E Of the answer choices listed, it is the absence of decline
in activities of daily living that differentiates mild neurocognitive
disorder from major neurocognitive disorder.
Answer 2B Among the answer choices listed, only female sex is not
a known risk factor for mild neurocognitive disorder. Older age is the
strongest risk factor for developing mild neurocognitive disorder. Other
known risk factors include hypertension, hyperlipidemia, coronary
artery disease, stroke, family history of cognitive impairment,
apolipoprotein E (APOE) e4 genotype, vitamin D de iciency, sleep-
disordered breathing, and prior critical illness (sepsis).

Answer 3D The annual rate of progression of mild neurocognitive


disorder to major neurocognitive disorder varies between 8% and 15%
per year.

Answer 4D Risk factors for the progression of mild neurocognitive


disorder to major neurocognitive disorder include older age, fewer
years of education, stroke, diabetes, APOE e4 allele, and amnestic
subtype of mild neurocognitive disorder. Factors associated with
increased likelihood of reverting from mild neurocognitive disorder to
normal cognition include younger age, more years of education, higher
baseline cognitive function, and non-amnestic single mild
neurocognitive disorder type.

Answer 5B Among the listed choices, the Montreal Cognitive


Assessment had the best performance in detecting mild neurocognitive
disorder.

Answer 6A Alzheimer’s disease (AD) accounts for an estimated 60%


to 80% of older adults presenting with a major neurocognitive disorder.

Answer 7D Head injury among males, older age, diabetes mellitus,


the use of conjugated-equine estrogen with medroxyprogesterone
acetate, current smoking, and lower social engagement are all
associated with an increased risk for AD. Among genetic factors, APOE
ε4 remains the strongest genetic risk factor for development of AD, but
the presence of APOE ε2 allele is actually associated with decreased
risk of development of AD. Other factors associated with decreased risk
of development of AD include the use of statins, light-to-moderate
alcohol consumption, compliance with a Mediterranean diet, higher
educational attainment, and involvement in physically and cognitively
stimulating activities.

Answer 8B A recent meta-analysis indicated that LLD is consistently


and similarly associated with a twofold increase in risk for
development of a major neurocognitive disorder, and did not
signi icantly differ by type of major neurocognitive disorder.

Answer 9A The EOAD is an almost entirely genetically determined


disorder with a heritability rate of 92–100%. Approximately 35% to
60% of individuals that have EOAD have at least one affected irst-
degree relative (familial EOAD). The mode of inheritance for familial
EOAD varies, but the most common is autosomal dominant
transmission, which accounts for 10–15% of familial cases.

Answer 10D Apolipoprotein E4 (APOE4) is the most prevalent


genetic risk factor of AD and is expressed in more than half of
individuals with AD.

Answer 11C Fluctuating cognition with pronounced variations in


attention and alertness, recurrent visual hallucinations that are well
formed and detailed, and spontaneous features of parkinsonism with
onset subsequent to the development of cognitive decline are the core
features of major neurocognitive disorder with Lewy bodies. Suggestive
features include rapid eye movement behavior disorder and severe
neuroleptic sensitivity, but these are not core features for diagnosis of
the illness.

Answer 12D Men and women are equally affected by major


frontotemporal neurocognitive disorder. All of the other facts are true
of major frontotemporal neurocognitive disorder.

Answer 13D Risk factors for major neurocognitive disorder due to


vascular disease include increasing age, lower educational status,
female sex, vascular risk factors, and global and medial temporal
atrophy. The average rates of cognitive decline are similar for major
neurocognitive disorder due to vascular disease and Alzheimer’s
disease, but mortality is higher in major neurocognitive disorder due to
vascular disease largely because of cardiovascular and cerebrovascular
causes with mean survival rate of 3 years to 5 years.

Answer 14C The clinical features of DLB and PDD are similar and
include hallucinations, cognitive luctuations, and dementia in the
setting of the extrapyramidal motor impairments (Parkinsonism). The
cognitive domains that are involved in both DLB and PDD include
prominent executive dysfunction, visual-spatial abnormalities, and
variable impairments in memory. But in DLB, the cognitive impairment
often heralds the onset of illness in advance of the development of
Parkinsonian motor symptoms, while a diagnosis of PDD is made when
cognitive impairments develop in the setting of already well-
established motor symptoms of Parkinson’s disease (PD).

Answer 15A Available evidence indicates that diabetes, obesity,


physical and mental inactivity, depression, smoking, low educational
attainment, and diet are potentially modi iable risk factors for major
neurocognitive disorder due to Alzheimer’s disease.

Answer 16E It has been modelled that elimination of the seven


most important modi iable risk factors for Alzheimer’s disease would
lead to a 30% reduction in incidence of major neurocognitive disorder
due to Alzheimer’s disease worldwide.

Answer 17E The term “Parkinson plus syndromes” or “atypical


parkinsonian disorders” (APDs) is used to describe four distinct major
neurodegenerative clinicopathological entities: progressive
supranuclear palsy (PSP), corticobasal degeneration (CBD), multiple
system atrophy (MSA), and dementia with Lewy bodies (DLB).

Answer 18B Parkinson plus syndromes often present with a


parkinsonian syndrome that is atypical in terms of symptoms, including
a rapid rate of progression, symmetric distribution, and poor response
to levodopa or other dopaminergic medications. In addition to having
parkinsonism, the clinical presentation of Parkinson plus syndromes
are also characterized by supranuclear gaze palsy, early postural
instability, early dementia, asymmetrical apraxia, and autonomic
dysfunction.
Answer 19A Approximately, 10–20% of cases of major
frontotemporal neurocognitive disorder have genetic component with
an autosomal dominant pattern of inheritance or identi iable disease-
causing mutations. Most familial cases of major frontotemporal
neurocognitive disorder have mutations in the microtubule associated
protein tau (MAPT), progranulin (GRN) genes, or a hexanucleotide
repeat expansion in the C9ORF72 gene. Less commonly, mutations in
the gene encoding for valosin-containing protein (VCP) may cause
major frontotemporal neurocognitive disorder in association with
inclusion body myositis and Paget’s disease of bone.

Answer 20B The classic symptoms of sporadic CJD are a rapidly


progressive dementia with behavioral abnormalities, ataxia (usually
gait), extrapyramidal features, and eventually myoclonus. EEG shows 1
Hz to 2 Hz periodic sharp-wave (often biphasic or triphasic) complexes.
The American Academy of Neurology suggests checking for elevation of
14-3-3 protein in the CSF when a strong suspicion of CJD exists, but the
diagnosis is still uncertain. Approximately 85% to 90% of individuals
with CJD die within 1 year.

Answer 21D The prompt describes the classic clinical presentation


for corticobasal degeneration (CBD), which includes asymmetric
parkinsonism with a variable combination of ideomotor apraxia,
rigidity, myoclonus and dystonia, and the presence of an alien limb
phenomenon. However, in reality CBD is a dif icult disorder to diagnose
antemortem as it can present with multiple phenotypes, none of which
are speci ic enough to lead to a de initive diagnosis. Additionally,
multiple other neurodegenerative disorders including Alzheimer’s
disease (AD) can mimic its clinical course. The etiology for CBD remains
unclear, but the current neuropathological and genetic evidence
indicates a role for microtubule-associated protein tau. Positron
emission tomography (PET) scan shows hypometabolism in multiple
cortical and subcortical regions including frontal, temporal,
sensorimotor, parietal association cortices, the caudate nucleus,
lenticular nucleus, and thalamus. Current pharmacological therapies for
CBD have limited clinical bene it, and there are no disease-modifying
agents available.
Answer 22C Multiple system atrophy is a fatal neurodegenerative
disorder that presents with progressive autonomic failure, and various
combinations of parkinsonian features and cerebellar. It can be
classi ied into parkinsonian subtype and cerebellar subtype. Poor
response to levodopa is a mandatory diagnostic criterion for probable
multiple-system atrophy of the parkinsonian subtype. Early and severe
autonomic failure is a key feature of multiple-system atrophy, and the
most frequently affected domains are the urogenital and cardiovascular
systems. Respiratory disturbances are characteristic with diurnal or
nocturnal inspiratory stridor developing in as many as 50% of
individuals. The presence of dementia or visual hallucinations is not
consistent with a diagnosis of multiple-system atrophy. If these
symptoms occur in the presence of parkinsonism and autonomic
failure, a diagnosis of dementia with Lewy bodies should be considered.

Answer 23B Progressive supranuclear palsy (PSP) is a disorder


characterized by the development of bradykinesia, disproportionate
amount of postural instability, erect posture with nuchal rigidity, frontal
behavioral and cognitive changes, and vertical gaze palsy. It is caused by
the aggregation of tau proteins. PSP is almost entirely sporadic and has
a prevalence of 5–6 persons per 100,000. The mean onset age of
63 years with a median survival of 7 years. The diagnosis of PSP is
clinical, but magnetic resonance imaging and cerebrospinal luid
measures appear to hold promise as early-stage screening tools. The
treatment of PSP is supportive, but coenzyme Q-10 has shown some
preliminary symptomatic ef icacy. Levodopa may also provide modest
but transient bene it. Features that would exclude a diagnosis of PSP
include a recent episode of encephalitis, alien limb syndrome, cortical
sensory defects or temporoparietal atrophy, psychosis that is unrelated
to dopaminergic treatment, cerebellar signs, severe unexplained
dysautonomia, severe and asymmetric parkinsonian signs, and
structural abnormality in the basal ganglia on neuroimaging.

Answer 24D Mild features of motor neuron disease (MND) can


occur in up to 40% of individuals with FTD. These features include
upper motor neuron signs (hyper-re lexia, extensor plantar response,
spasticity), lower motor neuron signs (weakness, muscle atrophy,
fasciculations), dysarthria, dysphagia, and pseudobulbar affect. Among
the FTD variants, MND occurs more frequently among individuals with
behavioral-variant FTD and less often among individuals with
semantic-variant primary progressive aphasia or non- luent variant
primary progressive aphasia. Early parkinsonism is seen in
approximately 20% of individuals with FTD. Parkinsonism is most often
seen among individuals with behavioral-variant FTD followed by those
with non- luent variant primary progressive aphasia. Individuals with
FTD may also present with features of corticobasal syndrome or
progressive supranuclear palsy syndrome.

Answer 25D It is estimated that more than 50% of individuals with


MS present with some type of cognitive decline during the course of the
illness. It has been noted that cognitive changes can occur even in the
earliest stages of the disease, and the most vulnerable period is the irst
5 years of illness. Men appear to be more vulnerable to the cognitive
decline in MS. Brain atrophy has been found to be the strongest
correlate for cognitive decline even at early stages of the illness.
Treatments with anti-in lammatory and immunosuppressive
immunomodulators may bene it the cognitive de icit by reducing the
brain’s lesion burden.

Answer 26A HAND is characterized by an insidious onset with slow


progression of symptoms. HAND is associated with shortened survival
among individuals with HIV, but it is not associated with reduced the
level of consciousness and does not cause focal neurological signs or
neck stiffness. In the earlier stages of HAND, individuals not only
present with dif iculties in concentration and memory, but also present
with impaired executive functioning. With the disease progression,
signs of psychomotor slowing with depressive and other affective
symptoms can develop. HAND is caused by a multi-dimensional and
complex immunopathological processes. For the prevention of HAND,
an early start of CART might be bene icial. CART is also the treatment of
choice for individuals who have developed HAND.

Answer 27A The most common psychiatric complication of TBI is


mood disorder, particularly major depression. The second most
common psychiatric complication is anxiety disorders.

Answer 28D Older adults who suffer TBI are at greater risk for
developing longer periods of agitation, cognitive impairment,
permanent disability, and mass lesions when compared to younger
adults who suffer TBI.

Answer 29D Cognitive impairments after TBI evolve in four stages


based on the timing in its relationship to the TBI event. The irst stage
corresponds to the period of loss of consciousness, which occurs soon
after the injury. This ranges from a brief loss of consciousness to
prolonged coma. The second stage is a form of posttraumatic delirium
(labeled posttraumatic confusional state) and is associated with
agitation, confusion, disorientation, and a variable amount of
psychomotor activity. The third stage involves relatively rapid recovery
over the next several months and the return of the brain’s ability to
attend to, process, and remember information. The fourth stage, which
is associated with moderate to severe TBI, is characterized by
permanent cognitive de icits in which the individual must learn to
accept, adapt, and adjust to their cognitive disability.

Answer 30D Apathy is noted to be the most common BPSD. It is


highly prevalent across the different types and stages of neurocognitive
disorders including in mild cognitive impairment (MCI), Alzheimer’s
disease (AD), frontotemporal dementia (FTD), and vascular dementia.
It is also seen in individuals having Parkinson’s disease (PD),
schizophrenia, strokes, multiple sclerosis, traumatic brain injury, and
major depression.

Answer 31D Among individuals with AD, a higher level of apathy


has been consistently associated with neuronal loss, increased NFT
burden, and increased levels of total and phospho-tau proteins in the
cerebrospinal luid (CSF). However, the association between beta-
amyloid plaque burden or the CSF level of beta amyloid and level of
apathy in AD has been inconsistent.

Answer 32E Inattention is a hallmark feature of delirium among


older adults. Delirium has a high prevalence, but it is often
unrecognized among older adults. Delirium is associated with long-
term functional decline and increased mortality among older adults.

Answer 33A The Confusion Assessment Method (CAM) is the most


widely used delirium assessment instrument in the world. All other
tests listed are also validated instruments for use among individuals
with delirium but are just not used as commonly.

Answer 34D Presence of PTSD is not a known leading risk factor for
delirium among older adults. The known leading risk factors for
delirium among older adults include pre-existing dementia or cognitive
impairment, functional impairment, vision impairment, history of
alcohol abuse, and age >70 years. Presence of speci ic comorbidities
such as stroke or depression is also associated with an increased risk
for delirium among all patient populations.

Answer 35D Polypharmacy, psychoactive medication use, sedative-


hypnotic use, the use of physical restraints, the use of bladder catheter,
elevated BUN/creatinine ratio, abnormal sodium, glucose or potassium
levels, metabolic acidosis, infections, any iatrogenic event, major
surgery, and trauma are important precipitating factors for delirium
among older adults. The presence of dementia or cognitive impairment,
functional impairment, vision and hearing impairment, history of
alcohol abuse, age >70 years, presence of speci ic comorbidities such as
stroke or depression are all predisposing factors for delirium among
older adults but not necessarily precipitating factors.

Answer 36B Hypoactive delirium is the most prevalent subtype of


delirium among older adults and accounts for almost 3/4th of the cases
of delirium.

Answer 37D Available evidence indicates that approximately 30–


40% of cases of delirium among older adults are preventable.

Answer 38B A clear and univocal de inition of the sundowning has


not been yet achieved. Sundowning is considered to be a syndrome and
not a symptom that is seen between 2.5% and 66% of individuals with
dementia, depending on the study setting. The pathophysiology of
sundowning remains poorly understood, and no de initive causative
factors have been identi ied. The current understanding is that
sundowning occurs due to multiple interacting factors including
neurobiological, physiological, medical, and environmental causes.

Answer 39D The current best evidence indicates that light-to-


moderate alcohol use in middle-to-late adulthood is associated with a
decreased risk of cognitive impairment and dementia. Heavy alcohol
use is associated with changes in brain structures, cognitive
impairments, and an increased risk of all types of dementia.

Answer 40B Recent research data indicate that TBI appears to


lower the age of onset of TBI-related neurocognitive syndromes.
Evidence also indicates that increasing severity of a single moderate-to-
severe TBI increases the risk of subsequent AD. Any blast or blunt
physical force to the head as long as there is violent head displacement
increases the risk for dementia. The presence of apolipoprotein E ɛ4
alleles increases the risk for AD among individuals with TBI. Acute TBI
results in amyloid pathology and other neurodegenerative
proteinopathies.

Further Reading
Ahmed S, Venigalla H, Mekala HM, et al. Traumatic brain injury and neuropsychiatric
complications. Indian J Psychol Med. 2017;39(2):114–21.
[Crossref]

American Psychiatric Association. Diagnostic and statistical manual of mental


disorders. 5th ed. Arlington: American Psychiatric Association; 2013. Neurocognitive
Disorders; 308–309.
[Crossref]

Bang J, Spina S, Miller BL. Frontotemporal dementia. Lancet. 2015;386(10004):1672–


82.
[Crossref]

Cacace R, Sleegers K, Van Broeckhoven C. Molecular genetics of early-onset


Alzheimer’s disease revisited. Alzheimers Dement. 2016;12(6):733–48.
[Crossref]
Canevelli M, Valletta M, Trebbastoni A, et al. Sundowning in dementia: clinical
relevance, pathophysiological determinants, and therapeutic approaches. Front Med
(Lausanne). 2016;3:73.

Cherbuin N, Kim S, Anstey KJ. Dementia risk estimates associated with measures of
depression: a systematic review and meta-analysis. BMJ Open. 2015;5(12):e008853.
[Crossref]

Eggers C, Arendt G, Hahn K, et al. HIV-1-associated neurocognitive disorder:


epidemiology, pathogenesis, diagnosis, and treatment. J Neurol. 2017;264(8):1715–
27.
[Crossref]

Fanciulli A, Wenning GK. Multiple-system atrophy. N Engl J Med. 2015;372(3):249–


63.
[Crossref]

Fong TG, Davis D, Growdon ME, Albuquerque A, Inouye SK. The interface between
delirium and dementia in elderly adults. Lancet Neurol. 2015;14(8):823–32.
[Crossref]

Geschwind MD. Prion diseases. Continuum (Minneap Minn). 2015;21(6


Neuroinfectious Disease):1612–38.

Golbe LI. Progressive supranuclear palsy. Semin Neurol. 2014;34(2):151–9.


[Crossref]

Gomperts SN. Lewy body dementias: dementia with lewy bodies and Parkinson
disease dementia. Continuum (Minneap Minn). 2016;22(2 Dementia):435–63.

Grijalvo-Perez AM, Litvan I. Corticobasal degeneration. Semin Neurol.


2014;34(2):160–73.
[Crossref]

Harrison F, Aerts L, Brodaty H. Apathy in dementia: systematic review of recent


evidence on pharmacological treatments. Curr Psychiatry Rep. 2016;18(11):103.
[Crossref]

Hersi M, Irvine B, Gupta P, et al. Risk factors associated with the onset and
progression of Alzheimer’s disease: a systematic review of the evidence.
Neurotoxicology. 2017;61:143–87.
[Crossref]

Hshieh TT, Inouye SK, Oh ES. Delirium in the elderly. Psychiatr Clin North Am.
2018;41(1):1–17.
[Crossref]

Langa KM, Levine DA. The diagnosis and management of mild cognitive impairment:
a clinical review. JAMA. 2014;312(23):2551–61.
[Crossref]

Ljubenkov PA, Geschwind MD. Dementia. Semin Neurol. 2016;36(4):397–404.


[Crossref]

Marcantonio ER. Delirium in Hospitalized Older Adults. N Engl J Med.


2017;377(15):1456–66.
[Crossref]

Mendez MF. What is the relationship of traumatic brain injury to dementia? J


Alzheimers Dis. 2017;57(3):667–81.
[Crossref]

Mittal V, Muralee S, Williamson D, et al. Review: delirium in the elderly: a


comprehensive review. Am J Alzheimers Dis Other Dement. 2011;26(2):97–109.
[Crossref]

O’Brien JT, Thomas A. Vascular dementia. Lancet. 2015;386(10004):1698–706.


[Crossref]

Oh ES, Fong TG, Hshieh TT, Inouye SK. Delirium in older persons: advances in
diagnosis and treatment. JAMA. 2017;318(12):1161–74.
[Crossref]

Olfati N, Shoeibi A, Litvan I. Progress in the treatment of Parkinson-plus syndromes.


Parkinsonism Relat Disord. 2019;59:101–10.
[Crossref]

Olney NT, Spina S, Miller BL. Frontotemporal Dementia. Neurol Clin. 2017;35(2):339–
74.
[Crossref]

Petersen RC. Mild cognitive impairment. Continuum (Minneap Minn). 2016;22(2


Dementia):404–18.

Rao V, Koliatsos V, Ahmed F, et al. Neuropsychiatric disturbances associated with


traumatic brain injury: a practical approach to evaluation and management. Semin
Neurol. 2015;35(1):64–82.
[Crossref]

Rehm J, Hasan OSM, Black SE, Shield KD, Schwarzinger M. Alcohol use and dementia:
a systematic scoping review. Alzheimers Res Ther. 2019;11(1):1–11.
[Crossref]

Sachdev PS, Blacker D, Blazer DG, et al. Classifying neurocognitive disorders: the
DSM-5 approach. Nat Rev Neurol. 2014;10(11):634–42.
[Crossref]

Sa ieh M, Korczyn AD, Michaelson DM. ApoE4: an emerging therapeutic target for
Alzheimer’s disease. BMC Med. 2019;17(1):64.
[Crossref]

Sanford AM. Mild cognitive impairment. Clin Geriatr Med. 2017;33(3):325–37.


[Crossref]

Scheltens P, Blennow K, Breteler MM, de Strooper B, Frisoni GB, Salloway S, Van der
Flier WM. Alzheimer’s disease. Lancet. 2016;388(10043):505–17.
[Crossref]

Silveira C, Guedes R, Maia D, Curral R, Coelho R. Neuropsychiatric symptoms of


multiple sclerosis: state of the art. Psychiatry Investig. 2019;16(12):877–88.
[Crossref]

Wang Z, Dong B. Screening for cognitive impairment in geriatrics. Clin Geriatr Med.
2018;34(4):515–36.
[Crossref]

Warren JD, Rohrer JD, Rossor MN. Clinical review. Frontotemporal dementia. BMJ.
2013;347:f4827.
[Crossref]
© Springer Nature Switzerland AG 2021
R. Tampi et al. (eds.), Absolute Geriatric Psychiatry Review
https://doi.org/10.1007/978-3-030-58663-8_11

11. Depressive Disorders and Bipolar


and Related Disorders
Juan Young1
(1) Department of Psychiatry, Yale School of Medicine, New Haven, CT,
USA

Keywords Depressive disorders – Bipolar disorders – Late-life –


Diagnosis – Screening tools – Risk factors – DSM-5 criteria

Questions
1. A 72-year-old Caucasian male presents with his daughter to an
outpatient clinic for worsening memory, diminished functioning,
decreased activities of daily living, and lack of motivation to care
for himself for the past couple of months. Per the daughter’s
report, he has no known history of depression, psychosis, mania,
or other psychiatric disorders. Additionally, the patient does not
appear unhappy, is eating well, and appears to enjoy interactions
with his family when they initiate interactions with him. The
daughter reports that his medical history is signi icant for a recent
stroke and Alzheimer’s disease. Which of the following is most
consistent with the patient’s current presentation?
A.
Pseudodementia
B.
Hypnotic/sedative abuse
C.
Apathy
D. Delirium

E.
Acute myocardial infarction
2.
A 68-year-old African American woman is brought to the
Emergency Department by her family for hoarding behavior and
inability to care for herself. Her presenting symptoms have been
gradually worsening for the past year. Additionally, her family
reports she has been suffering from weight loss, insomnia,
psychomotor retardation, decreased energy, and feelings of
worthlessness for the past month. The woman is tearful
throughout the entirety of her emergency department (ED) visit,
and repeatedly laments on being a burden to her family. Which of
the following signs or symptoms of depression is the most
noticeable in older adults?
A.
Weight loss
B.
Decreased energy
C.
Feelings of worthlessness
D.
Insomnia
E.
Psychomotor retardation
3. Which of the following is a core symptom of a major depressive
episode according to the DSM-5?
A.
Lack of interest or pleasure
B.
Psychomotor disturbance
C.
Decreased concentration
D. Decrease appetite
E.
Suicidal ideation
4.
Which of the following types of delusions is more common in late-
life depression with psychotic features?
A.
Delusions of guilt
B.
Grandiose delusions
C.
Erotomaniac delusions
D.
Nihilistic delusions
E.
Delusions of jealousy
5.
Which of the following is a clinician rated scale that is used in the
evaluation of late-life depression?
A.
Geriatric Depression Scale
B.
Zung Depression Scale
C.
Beck Depression Inventory-II
D.
Hamilton Rating Scale for Depression
E.
Center for Epidemiologic Studies Depression Scale – 20 item
6. Which of the following is most commonly associated with poorer
outcomes for major depression and higher rates of suicidal
ideation among older adults?
A
A.
Limited social support

B. Suf icient inancial support

C.
Employment
D.
Lack of access to antidepressants
E.
History of being prescribed mood stabilizers
7.
Which of the following factors has been associated with improved
outcomes in late-life depression?
A.
Introverted personality
B.
Stimulant use history
C.
Male gender
D.
A history of severe depressive symptomatology
E.
A family history of depression
8. Which of the following is true of the difference between a
depressive-episode among older adults when compared to
younger adults?
A.
Recurrent episodes of depression in older adults tend to be
associated with similar symptoms and last as long as prior
episodes
B.
Depressive episodes in older adults last approximately 9
months if untreated
C.
Episodes of depression in older adults occur more frequently
h
as they age
D. Major depressive episodes in older adults do not typically
merge into a chronic condition

E.
Depressive episodes in older adults are more likely due to
alcohol consumption
9.
What is the minimum duration of symptoms necessary for a
patient to meet criteria for a manic episode according to the DSM-
5?
A.
1 week
B.
2 weeks
C.
4 weeks
D.
6 months
E.
12 months
10. Which of the following is true about hypomanic and manic
episodes?
A.
Hypomanic episodes are more likely to present with
irritability than expansive mood
B.
Hypomanic and manic episodes are often of comparable
intensity in symptoms
C.
A patient cannot be diagnosed with a hypomanic episode if he
or she has experienced a major depressive episode in the past
D.
A hypomanic episode has a minimum duration of 4 days when
compared to a manic episode’s minimum duration of 1 week
E.
Hypomanic episodes can only occur in bipolar II disorder
11.
A 67-year-old Caucasian woman presents to your clinic with her
husband reporting a history of recurrent depressive episodes that
last a month or more with predominant symptoms including
anhedonia, decreased concentration, and feelings of
worthlessness. Her husband reports that during these episodes,
she does not get out of bed and does not appear to have any
motivation to take care of herself. At times, the patient becomes
very irritable, has a decreased need for sleep, feels restless due to
her “boundless energy,” and cannot help but think of all the things
that she “needs to do” despite her husband telling her that he will
take care of those issues until she gets better. These reassurances
fall on deaf ears, however, as her husband reports inding her
cleaning their home multiple times during the night. These
episodes of high energy last for not more than 3 days. The woman
has never been hospitalized for her psychiatric illness. What is the
most likely diagnosis for this woman given her history?
A.
Major depressive disorder, moderate, without psychotic
symptoms
B.
Major depressive disorder, severe, with psychotic symptoms
C.
Bipolar I disorder
D.
Bipolar II disorder
E.
Cyclothymic disorder

12. What is a change in the bipolar disorder diagnostic criteria in the


DSM-5 when compared to the DSM-IV criteria?
A.
Irritable mood symptoms are now a part of criterion A
B.
Hypomania cannot be preceded by a major depressive
episode
C.
The presence of expansive mood requires three additional
associated symptoms in order to meet criteria for a manic or
hypomanic symptom
D. Depressive episodes are no longer dominant in bipolar II
disorder
E.
The presence of a mood symptom must now be accompanied
by persistently increased activity or energy levels

13.
Which of the following is a change in the DSM-5 diagnostic criteria
for bipolar II disorder when compared to the DSM-IV criteria?
A.
Persistence of hypomanic episodes is suggestive of
transformation into bipolar I disorder
B.
Transformation from a major depressive episode into a
hypomanic episode under antidepressant treatment is no
longer an explicit exclusion criterion, as long as there is
persistence into a fully syndromal level beyond the
physiological effect of antidepressants
C.
Irritable mood is no longer one of the mood states in criterion
A
D.
A major depressive episode is necessary before a hypomanic
episode
E.
There has been no change in bipolar II disorder diagnosis in
the DSM-5
14. Which of the following is an example of an initial presentation for
an older adult with bipolar disorder?
A.
A 67-year-old African American man presenting to an
emergency department (ED) who was brought in by family
for worsening impulsive behaviors
B
B.
A 73-year-old Caucasian man presenting to an outpatient
physician with deteriorating short-term memory
C. A 72-year-old Caucasian woman presenting at an outpatient
clinic requesting treatment for recurrent depressive episodes
with irritable mood

D.
An 80-year-old Asian man who is following up with his
psychiatrist reporting changes in mood after a stroke
E.
A 77-year-old African American woman who presents at an
outpatient clinic with worsening upper extremity rigidity
causing severe anxiety and sleepless nights
15.
Which of the following is true regarding the clinical course of late-
life bipolar disorder in older adults whose irst manic episode
occurred around age 60 years?
A.
If the irst manic episode occurred after 60 years of age, it is
unlikely to be related to a cerebrovascular accident
B.
Older adults with late-life bipolar disorder are more likely to
experience psychotic symptoms 2 weeks after the initial
manic episode
C.
New-onset manic episodes are more common in late-life than
early-onset depressive episodes
D.
Late-life bipolar disorder patients are more likely to
experience an initial depressive episode in mid-life followed
by a long latency period before the onset of manic symptoms
E.
Older adults with late-life bipolar disorder experience more
euphoric episodes than those with earlier onset of manic
symptoms
16. Which of the following is true regarding the prevalence of
psychotic symptoms in older adults with bipolar disorder?
A.
Most studies indicate that psychotic symptoms are less
frequent during manic episodes when compared to
depressive episodes
B.
Manic-psychotic symptoms are more frequently seen among
individuals >60 years of age
C. Psychotic symptoms occur with similar frequency in both
manic and depressive episodes

D.
Psychotic symptoms during manic episodes in late-life are
more likely to be due to Alzheimer’s disease-like pathology
than other causes
E.
Older patients with bipolar disorder never get psychotic
symptoms
17.
How long should someone experience depressive symptoms in
order to meet criteria for persistent depressive disorder?
A.
1 week
B.
2 weeks
C.
6 months
D.
1 year
E.
2 years
18. What is the most common personality disorder found among
older adults diagnosed with persistent depressive disorder
(dysthymia)?
A.
Histrionic personality disorder
B
B.
Schizotypal personality disorder
C.
Obsessive-compulsive personality disorder
D.
Dependent personality disorder
E. Antisocial personality disorder

19.
Which of the following is true regarding subsyndromal depression
among older adults?
A.
Anxiety disorders are not a common comorbidity
B.
Only 1% of elderly adults exhibits signs of subsyndromal
depression
C.
The irst signs of subsyndromal depression are always a
mixture of apathy, poor appetite, and cognitive impairment
D.
Adults with subsyndromal depression have signi icantly
increased odds of developing new-onset major depressive
disorder and anxiety disorders
E.
More men than women experience subsyndromal depression
in old age
20. Development of subsyndromal depression among older adults is
associated with which of the following?
A.
Impaired ability to do basic self-care tasks
B.
Male gender
C.
Being antidepressant naı̈ve
D.
Married status
E.
Higher self-rated health
21.
Which of the following is a warning sign for the development of
subsyndromal depression among older adults?
A.
Inability to handle one’s inances
B.
Somatic complaints such as severe neck or low back pain
C.
Multiple marriages in a short span of time
D.
Multiple hospitalizations with delirium episodes
E.
Having a college-level education

22. Which of the following is true about diagnosing subsyndromal


depression?
A.
An individual could be diagnosed with subsyndromal
depression if he or she demonstrates a depressed affect and
experiences at least one other symptom of a major depressive
episode that persists for at least 2 weeks
B.
An individual could have met criteria for a mood disorder in
the past, but must not have met criteria for a mood disorder
within a year of the current depressive symptoms
C.
In the DSM-5, it is coded under the category “subthreshold
depression”
D.
There were no changes in diagnostic criteria from DSM-IV to
DSM-5
E.
An individual could have experienced a previous hypomanic
episode and still be diagnosed with subsyndromal depression
23.
What is the requirement regarding the temporal relationship
between a stressful event and the onset of the mood symptoms
when diagnosing adjustment disorder with depressed mood,
when the stressful event is readily identi iable?
A.
The onset of mood symptoms must occur within 3 months of
the stressor
B.
The onset of mood symptoms must occur within 6 months of
the stressor
C.
The onset of mood symptoms must occur within 1 year of the
stressor
D.
The onset of mood symptoms must occur within 2 years of
the stressor
E.
The onset of mood symptoms must occur within 3 years of
the stressor
24. A 65-year-old Caucasian woman presents to her primary care
physician reporting depressed mood, low energy, laky skin, and
unexplained weight loss despite reporting no change in her
caloric intake. She reports that these symptoms started about 6
weeks ago. About 4 months ago, she started rationing her
medications in order to save money when she found out that her
disability payments were halved. She denies any prior psychiatric
history. What is the most likely cause of her presentation?
A.
Discontinuation of her citalopram
B.
Halving her levothyroxine dose
C.
She is experiencing an adjustment disorder with depressed
mood
mood
D.
Late-onset persistent depressive disorder
E.
She started drinking alcohol to self-medicate
25.
Which of the following personality traits is strongly related with
depression in late-life?
A.
Openness
B.
Conscientiousness
C.
Extraversion
D.
Neuroticism
E.
Agreeableness

Answers
Answer 1C Apathy has been de ined as a lack of motivation or
emotion not attributed to impaired consciousness, cognitive
impairment, or a mood disorder. It has been proposed as a syndrome
ev`idenced by diminished goal-directed overt behavior, diminished
goal-directed cognition, and diminished emotional concomitants of
goal-directed behavior. Studies have indicated that it may be a result of
disruption of the frontal-subcortical circuit. It should be distinguished
from major depression, which entails emotional distress, vegetative
symptoms, suicidal ideation, and hopelessness as serotonergic
reuptake inhibitors typically prescribed to treat depression may induce
or worsen apathy.

Answer 2E Although the patient’s presentation is clearly consistent


with a major depressive episode, other older adults may not present
with such clear-cut symptomatology. Psychomotor disturbances and
other symptoms of melancholia are more likely to emerge in later age
compared to non-melancholic symptoms. Thus, elderly patients that
exhibit psychomotor retardation should be screened for depressive
symptoms and managed accordingly.

Answer 3A According to the DSM-5, a diagnosis of a major


depressive episode is made when an individual exhibits one or both of
the core symptoms (depressed mood and anhedonia/lack of interest or
pleasure) and four or more other depressive symptoms such as weight
loss, psychomotor disturbance, insomnia/hypersomnia, feelings of
worthlessness, excessive or inappropriate guilt, decreased
concentration or decision-making ability, and recurrent thoughts of
death and suicide. The episode must last at least 2 weeks.

Answer 4D Nihilistic delusions, persecutory delusions, and


delusions of illness are more commonly encountered in late-life when
compared to other forms of delusions. Nihilistic delusions speci ically
are more commonly seen in late-life depression with psychotic features.

Answer 5D Among the following, only Hamilton Rating Scale for


Depression is the clinician-administered scale for depression that is
used in the evaluation of individuals with late-life depression.

Answer 6A Several studies have suggested that older adults with


fewer social resources and a general feeling of disconnect from their
respective communities had higher levels of depression, suicidal
ideation, and were less likely to endorse positive affect items (e.g.,
“feeling happy”) on screening instruments for depression.

Answer 7E Several factors have been associated with improved


outcomes in late-life depression. Some of these factors include a history
of recovery from previous episodes, a family history of depression,
female gender, extroverted personality, current or recent employment,
absence of substance abuse, no history of major psychiatric disorders,
less severe depressive symptomatology, and absence of major life
events and serious medical illness.
Answer 8C Studies investigating the clinical course of major
depressive episodes have historically focused on younger populations
with limited focus on older adults. Major depressive episodes typically
include a remission and relapse cycle in which new episodes tend to
have similar presenting symptoms and durations to previous episodes.
Classically, the duration of an untreated major depressive episode is
approximately 9 months. In older adults, these episodes may occur
more frequently and could eventually merge into a chronic condition.

Answer 9A According to the DSM-5, a manic episode is an alteration


of mood that is euphoric, expansive, or irritable and is associated with
increased energy and is accompanied by at least three of the seven
associated symptoms of mania including decreased need for sleep,
racing thoughts, pressured speech, and increased high-risk behaviors,
and it should last for at least 1 week.

Answer 10D Hypomanic episodes consist of a distinct period of


abnormally elevated, expansive, or irritable mood with persistently
increased activity or energy. Like manic episodes, they must be
accompanied by three or more associated symptoms such as
grandiosity, decreased need for sleep, pressured speech, light of ideas,
distractibility, increase in goal-directed activity, or excessive
involvement in high-risk behaviors. According to the DSM-5, a
hypomanic episode has a minimum duration of 4 days of symptoms
while manic episodes must last for at least 1 week. Although bipolar II
disorder patients experience hypomanic episodes, it should be noted
that bipolar II disorder is not a milder form of bipolar I disorder as
patients diagnosed with bipolar II disorder have a greater chronicity of
illness and spend more time in the depressive phase of their illness
potentially causing more disability.

Answer 11D The patient reports a history of recurrent major


depressive episodes with periods of what appears to be hypomania
interspersed in her clinical course. Thus, her clinical presentation is
most consistent with a bipolar II disorder diagnosis. As she is exhibiting
hypomanic episodes, it is unlikely that she only has a unipolar
depressive disorder. As the patient meets criteria for major depressive
and hypomanic episodes, it is unlikely that the patient has a
cyclothymic disorder, which entails less intense symptomatology.

Answer 12E The DSM-IV required only the presence of one of two
mood symptoms (elevated/euphoric mood or irritable mood) to meet
criterion A for bipolar disorder while in DSM-5, this mood change now
needs to be accompanied by persistently increased activity or energy
levels.

Answer 13B Unlike in the DSM-IV where transformation from a


major depressive episode into a hypomanic episode under
antidepressant treatment is considered an exclusion criterion, in the
DSM-5, as long as there is a persistence into a fully syndromal level
beyond the physiological effect of antidepressants, bipolar II disorder
could still be diagnosed.

Answer 14C Both type I and type II bipolar disorders have been
dif icult to diagnosis accurately in clinical practice due to the dif iculty
differentiating both types from unipolar disorder. This is more
pronounced in individuals who may initially present with a history of
recurrent depressive episodes with no clear history of mania or
hypomania.

Answer 15D Most individuals with late-life bipolar disorder have


depression as their irst mood episode and this is associated with
female gender, high number of recurrences and longer illness.

Answer 16A Most studies of individuals with older age bipolar


disorder (i.e., bipolar disorder in adults ≥60 years of age) indicate that
psychotic symptoms are more common in depressive rather than manic
episodes.

Answer 17E To make a diagnosis of persistent depressive disorder,


the individual should exhibit depressed mood for most of the day and
for more days than not, as indicated by the individual or by the
observation of others for at least 2 years.
Answer 18C According to one study, the most common personality
disorders in older patients diagnosed with dysthymic disorder were
obsessive-compulsive and avoidant personality disorders. These
disorders are also the most commonly found in patients with a history
of major depression.

Answer 19D According to data obtained from US adults aged 55


years and older who participated in the National Epidemiologic Survey
on Alcohol and Related Conditions, older adults with subsyndromal
depression had signi icantly greater odds of developing new-onset
major depressive disorder and eventually developing an anxiety
disorder.

Answer 20A Subsyndromal depression has been associated with


poorer outcomes in basic self-care tasks such as feeding or grooming.

Answer 21B At times, it may be dif icult for a clinician to detect


subsyndromal depressive symptoms among older adults, which could
still impact the individual’s functioning and quality of life. The presence
of somatic complaints such as severe neck or low back pain should alert
healthcare providers that an individual may have an underlying mood
disorder that should be evaluated further and treated appropriately.

Answer 22A A depressive episode with insuf icient symptoms to


meet the criteria for a major depressive episode is called a minor or
subsyndromal depression. It is coded in the DSM-5 under the category
“other speci ied depressive disorder.” An individual can be diagnosed
with this condition if they never met criteria for a mood disorder and
experienced a depressed affect with at least one symptom from a major
depressive episode. The presenting symptoms must last at least 2
weeks.

Answer 23A In diagnosing adjustment disorder with depressed


mood, if a stressful event is readily identi iable and the relationship
between the stressful event and onset of mood symptoms is clear, the
syndrome must develop within 3 months of the identi ied stressful
event. Additionally, these mood symptoms must resolve within 6
months after the resolution of the stressor or the consequences of said
stressors.

Answer 24B The patient’s presentation of new-onset depression


with dry skin, low energy, and unexplained weight loss in the context of
a decrease in her medication regimen is suggestive of an undertreated
medical disorder rather than a primary psychiatric disorder. The above
symptoms are most consistent with subclinical hypothyroidism, which
has been associated with increasing the risk of depression in the elderly
by four times compared to those without subclinical hypothyroidism.

Answer 25D High neuroticism scores on personality scales have


been strongly associated with depression in late-life. It has been
proposed that ruminating and avoidance behaviors exhibited by
individuals with neurotic personality traits could be psychological risk
factors associated with depression across the lifespan. The relationship
between neuroticism and depression in middle-aged and older adults
have also been reported to be a function of a shared genetic
vulnerability.

Further Reading
American Psychiatric Association. Major Depressive Disorder. In: Diagnostic and
statistical manual of mental disorders. 5th ed. Arlington, VA: American Psychiatric
Publishing, a division of American Psychiatric Association; 2013a. p. 160–1.
[Crossref]

American Psychiatric Association. Bipolar and related disorders. In: Diagnostic and
statistical manual of mental disorders. 5th ed. Arlington, VA: American Psychiatric
Publishing, a division of American Psychiatric Association; 2013b. p. 124.
[Crossref]

American Psychiatric Association. Bipolar and related disorders. In: Diagnostic and
statistical manual of mental disorders. 5th ed. Arlington, VA: American Psychiatric
Publishing, a division of American Psychiatric Association; 2013c. p. 132–6.
[Crossref]

American Psychiatric Association. Depressive Disorders. In: Diagnostic and


statistical manual of mental disorders. 5th ed. Arlington, VA: American Psychiatric
Publishing, a division of American Psychiatric Association; 2013d. p. 168–71.
[Crossref]

American Psychiatric Association. Depressive disorders. In: Diagnostic and


statistical manual of mental disorders. 5th ed. Arlington, VA: American Psychiatric
Publishing, a division of American Psychiatric Association; 2013e. p. 183–4.
[Crossref]

American Psychiatric Association. Trauma- and stressor-related disorders. In:


Diagnostic and statistical manual of mental disorders. 5th ed. Arlington, VA:
American Psychiatric Publishing, a division of American Psychiatric Association;
2013f. p. 286–9.
[Crossref]

Angst J. Bipolar disorders in DSM-5: strengths, problems and perspectives. Int J


Bipolar Disord. 2013;1:12.
[Crossref]

Balsamo M, Cataldi F, Carlucci L, et al. Assessment of late-life depression via self-


report measures: a review. Clin Interv Aging. 2018;13:2021–44.
[Crossref]

Blazer DG. Depression and social support in late life: a clear but not obvious
relationship. Aging Ment Health. 2005;9(6):497–9.
[Crossref]

Chueire VB, Romaldini JH, Ward LS. Subclinical hypothyroidism increases the risk for
depression in the elderly. Arch Gerontol Geriatr. 2007;44(1):21–8.
[Crossref]

Devanand DP, Turret N, Moody BJ, et al. Personality disorders in elderly patients with
dysthymic disorder. Am J Geriatr Psychiatry. 2000;8(3):188–95.
[Crossref]

Dols A, Beekman A. Older Age Bipolar Disorder. Psychiatr Clin North Am.
2018;41(1):95–110.
[Crossref]

Dols A, Korten N, Comijs H, et al. The clinical course of late-life bipolar disorder,
looking back and forward. Bipolar Disord. 2018;20(5):459–69.
[Crossref]

Fiske A, Wetherell JL, Gatz M. Depression in older adults. Annu Rev. Clin Psychol.
2009;5:363–89.
[Crossref]
Grabovich A, Lu N, Tang W, et al. Outcomes of subsyndromal depression in older
primary care patients. Am J Geriatr Psychiatry. 2010;18(3):227–35.
[Crossref]

Ishii S, Weintraub N, Mervis JR. Apathy: a common psychiatric syndrome in the


elderly. J Am Med Dir Assoc. 2009;10(6):381–93.
[Crossref]

Laborde-Lahoz P, El-Gabalawy R, Kinley J, et al. Subsyndromal depression among


older adults in the USA: prevalence, comorbidity, and risk for new-onset psychiatric
disorders in late life. Int J Geriatr Psychiatry. 2015;30(7):677–85.
[Crossref]

Phillips ML, Kupfer DJ. Bipolar disorder diagnosis: challenges and future directions.
Lancet. 2013;381(9878):1663–71.
[Crossref]

Sö zeri-Varma G. Depression in the elderly: clinical features and risk factors. Aging
Dis. 2012;3(6):465–71.
[PubMed][PubMedCentral]

Steffens DC, Blazer DG. Depressive disorders. In: The American Psychiatric
Publishing textbook of geriatric psychiatry. 5th ed. Washington, DC: American
Psychiatric Publishing, a division of American Psychiatric Association; 2015. p. 243–
72.

VanItallie TB. Subsyndromal depression in the elderly: underdiagnosed and


undertreated. Metabolism. 2005;54(5 Suppl 1):39–44.
[Crossref]
© Springer Nature Switzerland AG 2021
R. Tampi et al. (eds.), Absolute Geriatric Psychiatry Review
https://doi.org/10.1007/978-3-030-58663-8_12

12. Sleep-Wake Disorders


Insiya Nasrulla1
(1) Department of Psychiatry, Medstar Georgetown University
Hospital/Medstar Washington Hospital Center, Washington, DC,
USA

Insiya Nasrulla
Email: Insiya.H.Nasrulla@medstar.net

Keywords Aging – Insomnia – Sleep disorders – Rapid eye movement


(REM) sleep – Non-rapid eye movement (NREM) sleep – Geriatric
syndromes

Questions
1. Which of the following is the accurate de inition of sleep
ef iciency?
A.
The amount of rapid eye movement (REM) sleep to non-rapid
eye movement (NREM) sleep
B.
The amount of wakefulness during the day to rest at night
C.
The ratio of time asleep to time in bed
D.
Average sleep duration for that particular age cohort
E.
Average sleep duration compared to previous duration during
individual’s lifetime
2.
Changes in circadian rhythm with age occurs due to the
degeneration of which of the following brain area?
A.
Paraventricular nucleus
B.
Ventromedial nucleus
C.
Suprachiasmatic nucleus
D.
Supraoptic nucleus
E.
Preoptic nucleus

3.
Which one of the following sleep changes is not usually associated
with aging?
A.
Increase in sleep latency
B.
Reduction in sleep ef iciency
C.
Reduction in stage 3 and 4 sleep
D.
Increase in nighttime arousals
E.
Increase in REM sleep
4. Which of the following is the most common sleep disorder among
older adults?
A.
Insomnia
B.
Hypersomnia
C.
Restless leg syndrome
D.
Periodic limb movement disorder (PLMD)
E. REM sleep behavior disorder (RBD)

5.
Which of the following disorders may coexist with or precede the
diagnosis of RBD by many years?
A.
Multiple sclerosis
B.
Parkinson’s disease
C.
Frontotemporal dementia
D.
Prion disease
E.
HIV-associated dementia
6.
Which of the following nutritional de iciencies in associated with
restless leg syndrome (RLS), a condition that generally tends to
worsen with age?
A.
Calcium
B.
Iron
C.
Thyroid
D.
Melatonin
E.
Vitamin B12
7. Which of the following is not a part of the instruction for stimulus
control that is used for the management of insomnia?
A.
Using the bed for sleep and sex only
B.
Going to bed only when tired

C. Getting out of bed when unable to sleep after 20 minutes

D.
Sleeping at the same time at night
E.
Awakening at the same time each day
8.
Which of the following is the only antidepressant approved by the
FDA for the treatment of insomnia?
A.
Doxepin
B.
Trazodone
C.
Mirtazapine
D.
Amitriptyline
E.
Sertraline
9. Which of the following is true of advanced circadian rhythms
among older adults ?
A.
It is de ined as a circadian rhythm that is longer than
24 hours
B.
Older adults tend to go to bed earlier and wake up earlier
C.
Older adults tend to go to bed later and wake up later
D.
It can be treated using bright light therapy in the morning
E.
N f h b
None of the above
10.
Which of the following would be considered a best irst-line
treatment for sundowning among older adults?
A.
Melatonin
B.
Light therapy
C.
N-methyl- -aspartate (NMDA) receptor antagonists
D.
Antipsychotics
E.
Benzodiazepines

Answers
Answer 1C The ratio of time asleep to time in bed is called sleep
ef iciency. Sleep ef iciency tends to decline with age.

Answer 2C Suprachiasmatic nucleus sets the clock for the body’s


circadian rhythms and adaptations to the reproducible 24-hour day-
night cycle of life on earth.

Answer 3E Changes in sleep associated with age include all of the


following: decrease in sleep ef iciency, phase advance of circadian
rhythms, increase in sleep latency, increase in nighttime arousals,
reductions in rapid eye movement (REM) sleep, and decrease in stages
3 and 4 sleep.

Answer 4A Insomnia is the most common sleep disorder among


older adults.

Answer 5B RBD is very common among individuals with


synucleinopathies with an estimated prevalence of approximately 50%
in Parkinson’s disease, 70–90% in multiple system atrophy, and 80% in
dementia with Lewy bodies (DLB). RBD also may occur less commonly
in a variety of other neurologic disorders, including Alzheimer disease,
amyotrophic lateral sclerosis (ALS), progressive supranuclear palsy
(PSP), and Huntington disease.

Answer 6B Iron is a necessary cofactor in the brain for the synthesis


of dopamine and the regulation of dopamine receptors and for
dopamine available in the synapse. Iron and ferritin have been found to
be abnormally low in the cerebrospinal luid of patients with RLS. Iron
supplementation is recommended in the presence of low serum
ferritin, but even with normal serum ferritin, an iron de iciency may
exist in the brain that is responsible for RLS.

Answer 7D Stimulus control is designed to help individuals with


insomnia establish a consistent sleep-wake rhythm, strengthen the bed
and bedroom as cues for sleep, and weaken them as cues for activities
that might interfere with sleep. Instructions for stimulus control
include the following: using the bed only for sleep and sex, going to bed
only when tired, getting out of bed if unable to sleep within 20 minutes,
and awakening at the same time each day.

Answer 8A Doxepin is a sedating tricyclic antidepressant (TCA)


with a high af inity for histamine (H1) receptors that is FDA approved
for the treatment of sleep maintenance insomnia. The initial dose in
older patients is 3 mg, which may be titrated to 6 mg if clinically
indicated. The dose should not exceed 3 mg in patients with hepatic
impairment. Other TCAs (amitriptyline, nortriptyline, and imipramine)
as well as trazodone and mirtazapine are often used off label for the
treatment of insomnia among older adults.

Answer 9B There is an advance (early timing) in the phase of the


major sleep episode in relation to the desired or required sleep time
and wake-up time. It typically presents as a chronic or recurrent
complaint of dif iculty staying awake until one’s desired conventional
bedtime, together with an inability to remain asleep until their desired
time for awakening. Options A, C, and D are associated with delayed
sleep wake phase disorder.

Answer 10B Individually tailored non-pharmacological approaches


should be considered as the irst-line therapy for sundowning. In
particular, environmental modi ications have been reported to be
potentially bene icial to reduce sundown-related behavioral disorders.
Among these, light therapy (i.e., the exposition to bright light during the
afternoon/evening hours) has been observed to produce a signi icant
reduction of sundowning episodes and motor restless behaviors in
open-label studies conducted on patients with dementia, as well as to
improve agitated behaviors in institutionalized older individuals.

Further Reading
Boeve BF. REM sleep behavior disorder: updated review of the core features, the
REM sleep behavior-disorder-neurodegenerative disease association, evolving
concepts, controversies, and future directions. Ann N Y Acad Sci. 2010;1184:15–54.
[Crossref]

Canevelli M, Valletta M, Trebbastoni A, et al. Sundowning in dementia: clinical


relevance, pathophysiological determinants, and therapeutic approaches. Front Med
(Lausanne). 2016;3:73.

Cornelissen G, Otsuka K. Chronobiology of aging: a mini-review. Chronobiology of


aging: a mini-review. Gerontology. 2017;63(2):118–28.
[Crossref]

Feinsilver SH, Hernandez AB. Sleep in the elderly: unanswered questions. Clin Geriatr
Med. 2017;33(4):579–96.
[Crossref]

Lie JD, Tu KN, Shen DD, Wong BM. Pharmacological treatment of insomnia. P T.
2015;40(11):759–71.
[PubMed][PubMedCentral]

Patel D, Steinberg J, Patel P. Insomnia in the elderly: a review. J Clin Sleep Med.
2018;14(6):1017–24.
[Crossref]

Pavlova M. Circadian rhythm sleep-wake disorders. Continuum (Minneap Minn).


2017;23(4, Sleep Neurology):1051–63.
Sharma MP, Andrade C. Behavioral interventions for insomnia: theory and practice.
Indian J Psychiatry. 2012;54(4):359–66.
[Crossref]

Trenkwalder C, Allen R, Hö gl B, et al. Comorbidities, treatment, and pathophysiology


in restless legs syndrome. Lancet Neurol. 2018;17(11):994–1005.
[Crossref]

Yaremchuk K. Sleep disorders in the elderly. Clin Geriatr Med. 2018;34(2):205–16.


[Crossref]

Zdanys KF, Steffens DC. Sleep disturbances in the elderly. Sleep disturbances in the
elderly. Psychiatr Clin North Am. 2015;38(4):723–41.
[Crossref]
© Springer Nature Switzerland AG 2021
R. Tampi et al. (eds.), Absolute Geriatric Psychiatry Review
https://doi.org/10.1007/978-3-030-58663-8_13

13. Schizophrenia and Other Psychotic


Disorders
Nery A. Diaz1
(1) Columbia University Irving Medical Center, New York, NY, USA

Nery A. Diaz
Email: NAD2149@cumc.columbia.edu

Keywords Late-onset schizophrenia – Very-late-onset schizophrenia-


like psychosis – Primary psychotic disorders – Secondary psychotic
disorders – Schizoaffective disorder schizophreniform disorder –
Delusional disorder – Delirium – Dementia praecox – Catatonia –
Negative symptoms

Questions
1. According to the consensus statement by the International Late-
Onset Schizophrenia Group, what are the suggested ages of onset
for a late-onset schizophrenia versus a very-late-onset
schizophrenia-like psychosis?
A.
Late onset after the age of 30 years and very-late-onset after
the age of 45 years
B.
Late onset after the age of 35 years and very-late-onset after
the age of 55 years
C.
Late onset after the age of 40 years and very-late-onset after
the age of 60 years
D. Late onset after the age of 50 years and very-late-onset after
the age of 60 years
E.
Late onset after the age of 60 years and very-late-onset after
the age of 75 years

2.
What proportion of individuals with schizophrenia develop the
illness after the age of 40 years?
A.
5%
B.
10%
C.
15.5%
D.
23.5%
E.
45%
3.
What is the 1-year incidence rate for the development of
psychosis of Alzheimer’s disease (AD)?
A.
10.5–15.3%
B.
20–25%
C.
32.5–36.1%
D.
43.1–44.5%
E.
47.5–49.4%
4. What is the estimated lifetime prevalence of schizophrenia among
older adults?
A.
0 1 0 5%
0.1–0.5%

B. 1%

C.
1–2%
D.
2%
E.
2–3%
5.
Which of the following is not considered a secondary form of
psychotic disorders?
A.
Psychotic disorder due to dementia
B.
Psychotic disorder due to delirium
C.
Psychotic disorder due to substance use
D.
Psychotic disorder due to mood disorders
E.
Psychotic disorder due to medical disorders
6. Which of the following disorders is the most common reason for
the occurrence of psychotic symptoms among older adults?
A.
Schizophrenia
B.
Delirium
C.
Dementia
D.
Major depressive disorder
E.
Bipolar affective disorder
7.
Which of the following differentiates late-onset schizophrenia
(LOS) from early onset schizophrenia (EOS)?
A.
Female preponderance
B.
More positive symptoms
C.
Worse premorbid functioning
D.
More severe neurocognitive impairments
E.
All of the above
8.
Which of the following would be seen more commonly in late-
onset schizophrenia (LOS) when compared to early onset
schizophrenia (EOS)?
A.
Visual hallucinations
B.
Formal thought disorder
C.
Affective lattening
D.
Negative symptoms
E.
All of the above
9. Which of the following is more common symptoms in psychosis in
Alzheimer’s disease when compared to primary psychotic
disorders among older adults?
A.
Auditory hallucination
B.
Visual hallucination
C
C.
Tactile hallucination
D.
Gustatory hallucination

E. Olfactory hallucination

10.
According to DSM-5 diagnostic criteria, what is the duration
criteria required to make a diagnosis of delusional disorder?
A.
≥2 weeks
B.
≥1 month
C.
≥3 months
D.
≥6 months
E.
≥9 months
11.
Which of the following is true of delusional disorder among older
adults?
A.
The prevalence is approximately 1%
B.
It causes social dysfunction
C.
It causes cognitive dysfunction
D.
It causes occupational impairment
E.
All of the above
12. Which of the following is false regarding schizoaffective disorder
among older adults?
A.
Greater severity of illness
B.
Greater treatment resistance
C. Higher risk for suicide

D.
More likely to be married
E.
Less likely to live independently
13.
Which of the following is a not a risk factor for developing very
late-onset schizophrenia (VLOS)?
A.
Male gender
B.
Immigrant status
C.
Paranoid personality disorder
D.
Hearing loss
E.
Lower socioeconomic status
14. Which of the following is the most common psychotic symptom
seen among individuals with Parkinson’s disease (PD)?
A.
Persecutory delusions
B.
Auditory hallucinations
C.
Visual hallucinations
D.
Olfactory hallucinations
E.
Tactile hallucinations
15.
Which of the following would not be considered a risk factor for
the development of psychotic symptoms among individuals with
Parkinson’s disease (PD)?
A.
Older age
B.
Greater severity of illness
C.
Longer duration of illness
D.
Axial rigidity subtype of PD
E.
Reduced hearing

16.
Among the following disorders, which is the most common
etiology for the development of catatonia among older adults?
A.
Major depressive disorder
B.
Generalized anxiety disorder
C.
Schizophrenia
D.
Schizoaffective disorder
E.
Major neurocognitive disorder
17. Which of the following is false regarding psychotic major
depression when compared to non-psychotic major depression
among older adults?
A.
Their prevalence decreases with age
p g
B.
They exhibit greater cognitive dif iculties
C.
They have more pronounced brain atrophy
D.
They have higher relapse rates
E. They have greater mortality rates

18.
You have been asked to evaluate a 70-year-old woman who was
hospitalized for increasing agitation and persistent thoughts that
people are stealing things from her home. The patient cannot be
reassured that no one is stealing from her home. These symptoms
occurred suddenly and have worsened over the past several
weeks. Which one of the following would not be an essential irst
step in the assessment of this patient’s presentation?
A.
Gathering collateral information from family
B.
Reviewing the patient’s past medical and psychiatric history
C.
Reviewing the patient’s current medication list
D.
Completing a focused physical examination
E.
Ordering an MRI scan of the brain
19. Which of the following is true of suicide among individuals with
schizophrenia?
A.
Compared with the general population, individuals with
schizophrenia have a two-fold greater risk of suicide
B.
Among individuals with schizophrenia, most completed
suicides appear to occur early in the disease process
suicides appear to occur early in the disease process
C.
10% of individuals with schizophrenia have a history of
suicide attempts
D.
Overall prevalence estimate for completed suicide among
individuals with schizophrenia is 2%
E.
All of the above
20.
Which of the following symptoms would argue against a diagnosis
of Charles Bonnet syndrome?
A.
The presence of formed and complex hallucinations
B.
Persistent or repetitive visual hallucinations
C.
Full or partial retention of insight
D.
The presence of complex delusions
E.
The absence of hallucinations in other sensory modalities

Answers
Answer 1C The consensus statement by the International Late-
Onset Schizophrenia Group suggested that schizophrenia with an onset
after age 40 years should be called “late-onset schizophrenia” and
schizophrenia after that age of 60 years should be called “very-late-
onset schizophrenia-like psychosis.”

Answer 2D The proportion of individuals with schizophrenia who


develop the illness after the age of 40 years has been estimated at
23.5%.
Answer 3B The 1-year incidence rates for the development of
psychosis among individuals with AD ranges from 20% to 25%,
whereas the 2-year incidence rates range from 32.5% to 36.1%.

Answer 4A While the lifetime prevalence of schizophrenia has been


estimated at 1%, the lifetime prevalence of schizophrenia among older
adults is estimated between 0.1% and 0.5%.

Answer 5D Among older adults, approximately 60% of the cases of


newly diagnosed psychotic disorder have a secondary psychotic
disorder. A newly diagnosed psychotic disorder in late-life requires a
thorough evaluation for secondary causes such as dementia, delirium,
drugs, and medical disorders. Mood disorders are a primary form of
psychotic disorder among older adults. Clinically, it is important to
distinguish between primary and secondary forms of newly diagnosed
psychotic disorders to formulate a treatment plan.

Answer 6C Dementia is the most common contributing factor for


the development of psychotic symptoms among older adults.

Answer 7A Late-onset schizophrenia is more commonly seen in


women than men. In addition, individuals with late-onset
schizophrenia tend to have better premorbid functioning, fewer
negative symptoms, and less severe neurocognitive impairments.

Answer 8A Individuals with LOS complain of visual, tactile, and


olfactory hallucinations; persecutory delusions; partition delusions;
and third-person, running commentary, and accusatory or abusive
auditory hallucinations. They are less likely to display formal thought
disorder, affective lattening, or blunting and negative symptoms.

Answer 9B Visual hallucinations are the most common type of


hallucination in Alzheimer’s disease followed by auditory and less
commonly, olfactory, tactile, gustatory hallucinations. In primary
psychoses, auditory hallucinations are most common type of
hallucinations.
Answer 10B According to the DSM-5, the essential feature of
delusional disorder is the presence of one or more delusions that
persist for at least 1 month.

Answer 11B The estimated prevalence of delusional disorder


among older adults is approximately 0.03%. Delusional disorder among
older adults typically causes social dysfunction, but otherwise these
individuals appear to have normal cognitive, personal, and occupational
functioning.

Answer 12D Older adults with schizoaffective disorder tend to have


greater severity of illness with worse outcomes including greater
treatment resistance and risk for suicide. In addition, older adults with
schizoaffective disorder are less likely to drive, are less likely to be
married, and are less likely to live independently.

Answer 13A Female gender, being from an immigrant population,


greater abnormalities in brain structures, family history of
schizophrenia or avoidant personality, paranoid or schizoid personality
disorder, hearing loss, and being from a lower socioeconomic status are
all risk factors for developing VLOS.

Answer 14C Among individuals with PD, visual hallucinations (VHs)


are the most prevalent psychotic symptom.

Answer 15E Risk factors for the development of psychotic


symptoms among individuals with PD are older age, greater severity of
illness, longer duration of illness, the axial rigidity subtype of PD,
idiopathic PD, underlying dementia, delirium or depression, the
presence of sleep disturbance including REM behavior disorder,
reduced vision, and the use of dopaminergic agents.

Answer 16A Affective disorders such as major depressive disorder


and bipolar disorder are the most common psychiatric causes for
catatonia among older adults.

Answer 17A The prevalence of psychotic major depression


increases with age. Additionally, individuals with psychotic major
depression exhibit greater cognitive dif iculties, have more pronounced
brain atrophy, have higher relapse rates, and have greater mortality
rates when compared to age-matched non-psychotic major depression.

Answer 18E Essential irst step in assessing older individuals


presenting with new onset psychotic symptoms include gathering
collateral information from family, reviewing the patient’s past medical
and psychiatric history, reviewing the patient’s current medication list,
and completing a focused physical examination. Ordering CT or MRI
scan of the brain along with laboratory work-up would be the next step
in the assessment of older adults with new onset psychotic symptoms.

Answer 19B When compared with the general population,


individuals with schizophrenia have an 8.5-fold greater risk for suicide.
Most completed suicides appear to occur early in the disease process.
Between 20% and 50% of individuals with schizophrenia report a
history of suicide attempts, and 4–13% of individuals eventually die by
suicide.

Answer 20D Charles Bonnet syndrome (CBS) is an uncommon


cause for visual hallucinations in the presence of visual impairment.
The risk factors for CBS include older age (>64 years), social isolation,
low cognitive function, a past history of stroke, and poor bilateral visual
acuity. The criteria for CBS include the following: the presence of
formed and complex hallucinations, persistent or repetitive visual
hallucinations, full or partial retention of insight, the absence of
delusions, and the absence of hallucinations in other sensory
modalities. The individuals with CBS are aware that their symptoms are
unreal, but the hallucinations often seem to it in logically with their
current surroundings.

Further Reading
Chang A, Fox SH. Psychosis in Parkinson’s disease: epidemiology, pathophysiology,
and management. Drugs. 2016;76(11):1093–118.
[Crossref]

Folsom DP, Lebowitz BD, Lindamer LA, et al. Schizophrenia in late life: emerging
issues. Dialogues Clin Neurosci. 2006;8(1):45–52.
[Crossref]

Gournellis R, Oulis P, Michalopoulou P, et al. Dimensional approach to delusions in


psychotic depression in the elderly: factor structure and clinical correlates. Int J
Geriatr Psychiatry. 2009;24(4):363–8.
[Crossref]

Howard R, Rabins PV, Seeman MV, et al. Late-onset schizophrenia and very-late-onset
schizophrenia-like psychosis: an international consensus. The International Late-
Onset Schizophrenia Group. Am J Psychiatry. 2000;157(2):172–8.
[Crossref]

Schizophrenia Spectrum and Other Psychotic Disorders. Diagnostic and Statistical


Manual of Mental Disorders Fifth Edition, Washington, DC: American Psychiatric
Association Publishing; https://doi.org/10.1176/appi.books.9780890425596.dsm02.
Accessed 12 Jan 2020.

Iglewicz A, Meeks TW, Jeste DV. New wine in old bottle: late-life psychosis. Psychiatr
Clin North Am. 2011;34(2):295–318, vii.
[Crossref]

Kasckow J, Felmet K, Zisook S. Managing suicide risk in patients with schizophrenia.


CNS Drugs. 2011;25(2):129–43.
[Crossref]

Nair AG, Nair AG, Shah BR, Gandhi RA. Seeing the unseen: Charles bonnet syndrome
revisited. Psychogeriatrics. 2015;15(3):204–8.
[Crossref]

Reinhardt MM, Cohen CI. Late-life psychosis: diagnosis and treatment. Curr
Psychiatry Rep. 2015;17(2):1.
[Crossref]

Serra-Mestres J, Jaimes-Albornoz W. Recognizing catatonia in medically hospitalized


older adults: why it matters. Geriatrics (Basel). 2018;3(3):E37.
[Crossref]

Tampi RR, Young J, Hoq R, et al. Psychotic disorders in late life: a narrative review.
Ther Adv Psychopharmacol. 2019;9:2045125319882798.
[Crossref]
© Springer Nature Switzerland AG 2021
R. Tampi et al. (eds.), Absolute Geriatric Psychiatry Review
https://doi.org/10.1007/978-3-030-58663-8_14

14. Substance-Related and Addictive


Disorders
Juan Young1
(1) Department of Psychiatry, Yale School of Medicine, New Haven, CT,
USA

Keywords Substance use disorders – Drug use disorder – Late-life –


Diagnosis – Screening tools – Risk factors – DSM-5 criteria –
Withdrawal

Questions
1.
Which of the following is the most commonly used substance over
a lifetime among older adults in addition to caffeine and tobacco?
A.
Marijuana
B.
Opioids
C.
Alcohol
D.
Methamphetamine
E.
Sedatives
2. Which of the following would not be considered a risk factor for
substance use disorders in late-life?
A Being male
A. Being male

B.
Being African American
C.
Having less than a high school education
D.
Being a caregiver
E.
Having a disability
3. A 68-year-old Caucasian man with history of PTSD and major
depressive disorder presents to an outpatient clinic for worsening
anxiety after being non-compliant with treatment and follow-up
appointments for the past year. He reports to you that he has been
drinking a glass of wine a couple of times a week to “take the edge
off,” but denies a history of delirium tremens, withdrawals, or
seizures related to alcohol use. He also reports using crack
cocaine 1 week prior but reported that it was a “one-time thing”
and denies any other history of cocaine use or dependence.
Towards the end of the interview, he asks you for a prescription
for medical marijuana as he heard that it helps with PTSD and
anxiety symptoms. After further discussion, you ind out that in
the past year, he has been buying more and more marijuana, and
has been smoking at least 2–3 “joints” several times throughout
the day. He reports cravings when he cannot obtain marijuana, is
late to work at a local grocery when he smokes too much the night
before, and spends a signi icant amount of time trying to ind
stronger strains of cannabis at local dispensaries or on the
internet. When asked what his family thinks about his cannabis
use, he replies that his second wife is in the process of divorcing
him because he secretly used hundreds of dollars from their joint
bank account in order to buy increasingly expensive cannabis
paraphernalia. What is the most likely diagnosis for this man?
A.
Cannabis use disorder, mild
B C bi di d d t
B. Cannabis use disorder, moderate

C.
Cannabis use disorder, severe
D.
Alcohol use disorder, mild
E.
Stimulant use disorder, in sustained remission
4.
Which of the following is a predictor of increased alcohol
consumption drinking in older age?
A.
Being male
B.
Having more inancial resources
C.
Being Caucasian
D.
The young-old
E.
All of the above
5. What changes were introduced into the updated DSM-5 criteria
for alcohol use disorder when compared to the DSM-IV?
A.
Important occupational activities are reduced due to
continued alcohol use
B.
Recurrent alcohol use in situations in which it is physically
hazardous
C.
Withdrawal symptoms or withdrawal syndrome of alcohol
D.
Cravings or a strong desire or urge to use alcohol
E. Alcohol use is continued despite knowledge of having a
persistent or recurrent physical or psychological problem
persistent or recurrent physical or psychological problem
that is likely to have been caused or exacerbated by alcohol

6.
A 72-year-old African American man with history of major
depressive disorder and anxiety presents to an outpatient clinic
reporting worsening anhedonia and feelings of worthlessness for
the past 3 weeks. He also relapsed on alcohol use after being
sober for 3 years because he wanted to “not feel anything
anymore.” He reports that he has been drinking approximately
four to ive 6-packs every day for the past 2 weeks. After an
extensive discussion with the patient, he is agreeable to admission
to a dual-diagnosis inpatient unit for further treatment of his
depression and alcohol use disorder with an eventual plan of
being placed on naltrexone. However, routine laboratory workup
is remarkable for signi icantly elevated transaminitis suggestive of
hepatotoxicity. Medication reconciliation reveals that the patient
is taking aspirin, atorvastatin, metoprolol, buspirone, and
sertraline (although he reports being non-compliant with
sertraline for the past 3 months). Which of the following
medications is the mostly likely cause of the abnormal liver
function tests?
A.
Aspirin
B.
Atorvastatin
C.
Metoprolol
D.
Buspirone
E.
Sertraline
7. Which of the following medications have been found to have the
most evidence in preventing relapse in older patients diagnosed
with alcohol use disorder ?
A.
Disul iram
Disul iram
B. Buprenorphine

C.
Buspirone
D.
Bupropion
E.
Naltrexone
8.
A 78-year-old Caucasian man with a history of alcohol use
disorder and cirrhosis presents to the clinic requesting
medications to help maintain abstinence from alcohol use after
recently undergoing detoxi ication. He exhibits normal sinus
rhythm on EKG and his renal function and creatinine are within
normal limits. Which of the following medications is most
appropriate for this speci ic patient?
A.
Buprenorphine
B.
Naloxone
C.
Acamprosate
D.
Naltrexone
E.
Disul iram
9. Which of the following choices is considered to be “one standard
drink”?
A.
12 oz. of wine
B.
5 oz. of hard liquor (80-proof distilled spirits)
C
C.
4 oz. of malt liquor
D. 12 oz. of beer

E.
10 oz. of liquer
10.
Which of the following instruments is not validated for use among
older adults with alcohol use disorder?
A.
CAGE
B.
Michigan Alcohol Screening Test-Geriatric Version (MAST-G)
C.
Alcohol Use Disorders Identi ication Test (AUDIT)
D.
Alcohol, Smoking, and Substance Involvement Screening Test
(ASSIST)
E.
Comorbidity-Alcohol Risk Evaluation Tool (CARET)
11. Which of the following processes are most consistent with a
typical alcohol withdrawal syndrome presentation?
A.
Initial appearance of autonomic instability → 72 hours after
cessation of drinking
B.
Tactile, auditory, and visual hallucinations → 14 days after
cessation of drinking
C.
Acute onset of seizures → 7 days after cessation of drinking
D.
Onset of delirium tremens → 48-72 hours after cessation of
drinking
E.
Hypersomnia → 4 hours after cessation of drinking
yp g
12.
Which of the following substances are more commonly associated
with a decreased risk of any type of dementia with light-to-
moderate consumption?
A.
Alcohol
B.
Cocaine
C.
Marijuana
D.
PCP
E.
Opioids
13.
Which of the following psychiatric diagnoses have been associated
with an increased risk of suicide in veterans?
A.
Major depressive disorder in men and substance use disorder
in women
B.
Bipolar disorder in men and substance use disorder in
women
C.
Schizoaffective disorder in men and substance use disorder in
women
D.
Substance use disorder in men and bipolar disorder in
women
E.
Substance use disorder in men and schizoaffective disorder in
women
14. What is the estimated percentage of older women who misuse
prescription drugs?
A.
1%
1%
B.
3%
C.
11%

D. 23%

E.
35%
15.
Which of the following cognitive and/or behavioral symptoms are
most consistent with Korsakoff’s syndrome?
A.
Fluctuating orientation
B.
Irritability
C.
Anomic aphasia
D.
Flight of ideas
E.
Confabulation
16. A 70-year-old Caucasian male veteran presents to your clinic
intoxicated and smelling of alcohol. He reports signi icant
depression and feels like a failure for relapsing on alcohol use
after being sober for 5 years. He indicates that he recently started
drinking again 5 days ago and reports signi icant amount of vodka
consumption every day. Which of the following characteristics
could this patient have that are strong predictors of relapse in
those that have been diagnosed with alcohol use disorder in the
past?
A.
Low-risk drinking and 2–3 alcohol use disorder lifetime
symptoms
B.
B.
Medium-risk drinking and 6+ alcohol use disorder lifetime
symptoms
C.
High-risk drinking and 6+ alcohol use disorder lifetime
symptoms
D. Both low- and medium-risk drinking and 6+ alcohol use

E. disorder lifetime symptoms

Both medium- and high-risk drinking and 6+ alcohol use


disorder lifetime symptoms

17.
Approximately what percentage of drug users aged ≥50 years
meet the criteria for a drug use disorder?
A.
1%
B.
3%
C.
5%
D.
10%
E.
15%
18. Which of the following is less common among older adults in
relation to prescription drug misuse/abuse of benzodiazepines,
opiate analgesics, and certain skeletal muscle relaxants?
A.
Tolerance
B.
Physical dependence
C.
Chronic pain
D.
Insomnia
E.
Anxiety
19.
Which of the following personality disorders have been found to
be the most comorbid with alcohol use disorder?
A.
Schizotypal personality disorder
B.
Schizoid personality disorder
C.
Histrionic personality disorder
D.
Dependent personality disorders
E.
Antisocial personality disorder

20. Which of the following interventions are most likely to result in


greater engagement in mental health and substance abuse
services by older primary care patients?
A.
Enhanced referral to a specialty mental health/substance
abuse clinic
B.
Integrated care with mental health and substance abuse
providers co-located in primary care
C.
Screening in the primary care setting with instruments such
as CAGE and further inquiring greater details about a
patient’s substance abuse history
D.
Prescribing buprenorphine to any patient that has a high
likelihood of meeting criteria for an opioid use disorder
E.
Referral to a social worker who can provide lyers with
information regarding various chemical dependency
g g p y
resources in the community
21.
Which of the following biological markers is considered the most
sensitive for detecting long-term heavy alcohol use?
A.
Blood alcohol level
B.
Acetate
C.
γ-glutamyltransferase (GGT)
D.
Aspartate aminotransferase (AST)
E.
Alanine aminotransferase (ALT)

22.
Which of the following characteristics is more likely to be found in
adults ≥50 years in age who continue to smoke cigarettes?
A.
Female gender
B.
Native American ethnicity
C.
Lower income
D.
Higher level of education
E.
Being married
23. Which of the following is a side effect that could occur shortly
after marijuana use among older adults?
A.
Diarrhea
B.
Pinpoint pupils
C.
Sialorrhea
D.
Heart attack
E.
Vertigo
24.
Which of following is not a sign or symptom of potential alcohol
use disorder among older adults?
A.
Anxiety
B.
Excessive mood swings
C.
Unexplained falls, bruises, or burns
D.
New-onset dif iculties in decision making
E.
Decrease tolerance to alcohol use

25.
Which of the following characteristics is more likely to be found
among adults aged ≥55 years with prescription opioids as their
primary drug of choice?
A.
Female gender
B.
African American ethnicity
C.
Hispanic ethnicity
D.
Intranasal route is preferred
E.
Use of other illicit substances is common
Answers
Answer 1C Almost 80% of older adults had used any substance over
their lifetimes with more than 50% reporting such use over the
previous 12 months. Alcohol was the most commonly used substance
over the lifetime (74%) and in the previous 12 months (45%). This was
followed by tobacco (52% lifetime and 14% previous 12 months).

Answer 2C Being male, White, or African American race; having a


high school education or beyond; being unmarried or divorced; having
depression or bereavement; having chronic physical illness or pain,
disability, loneliness or social isolation; being a caregiver; having a
change in living situation or career; and not being religiously active are
all known risk factors for substance use disorders in late-life.

Answer 3B The patient exhibits several symptoms and behaviors


indicative of a substance use disorder relating to cannabis use. He
speci ically reported that he uses signi icant amounts of cannabis each
day or whenever he can get access to cannabis, has cravings for
cannabis when he is unable to obtain any, reports problems at work
and at home as a result of his cannabis use, and he continues to ind
reasons and excuses to continue using cannabis despite the obvious
negative consequences of his substance abuse. He presently meets
criteria for cannabis use disorder with moderate severity as he
presents with at least 4 of the 11 criteria needed to diagnose a
substance use disorder.

Answer 4E Although being male, more af luent, being Caucasian,


and being young-old are all consistently associated with unhealthy
drinking in late-life, the only one choice that is actually a predictor of
increased drinking in older age is having more inancial resources.

Answer 5D Notable changes in the DSM-5 criteria for alcohol use


disorder include the emphasis of the cravings criteria in alcohol use
disorder diagnosis, and the removal of recurring legal problems criteria
to improve screening accuracy and to avoid excluding certain portions
of the population who would meet criteria for alcohol use disorder
otherwise.
Answer 6B A sudden elevation in liver function tests could be a sign
of acute hepatotoxicity (or acute hepatitis) that could be a result of an
interaction between severe alcohol use and the intake of statins. In this
case, the patient is taking atorvastatin, which with elevated alcohol use,
is the most likely cause of the abnormal indings. Of the other
medications he is currently taking, aspirin is more likely to increase
risk of bleeding and metoprolol would be more likely to cause
autonomic instability in alcoholic patients. Neurological side effects
such as dizziness or drowsiness can occur or worsen in alcoholics using
antidepressants or anxiolytics.

Answer 7E Although there is limited evidence in the utilization of


pharmacological therapy in preventing relapse of alcohol use disorder
among older adults, there is some evidence to suggest that the opioid-
receptor antagonist naltrexone has some utility in preventing relapse of
alcohol use among the older adult population.

Answer 8C Most older adults present with comorbid medical


disorders that could limit the therapeutic options available to help
them maintain their abstinence from substance use. Acamprosate is a
safe and well-tolerated pharmacotherapeutic agent that has been
approved by the Food and Drug Administration (FDA) in maintaining
abstinence from alcohol in detoxi ied alcohol-dependent individuals,
although it is relatively contraindicated in patients that have comorbid
renal failure or dysfunction. The patient discussed in the vignette
reports a history of cirrhosis, which makes naltrexone and disul iram
poor choices for him. However, laboratory workup indicates that he still
has normal renal function indicating acamprosate is the most
appropriate pharmacotherapy for him to maintain abstinence from
alcohol use. The rest of the choices are not indicated or approved in
maintaining abstinence from alcohol use.

Answer 9D One “standard drink” is equivalent to 12 oz. of beer, 5 oz.


of wine, 1.5 oz. of hard liquor, and 4 oz. of liquer or aperitif per the
National Institute on Alcohol Abuse and Alcoholism.
As per the Center for Substance Abuse Treatment protocol, it is
recommended that adults age 65 years of age and older should
consume no more than one standard drink per day.

Answer 10D The CAGE, SMAST, AUDIT, and Comorbidity-Alcohol


Risk Evaluation Tool (CARET) are validated for use among older adults.
The ASSIST is yet to be validated for use among older adults.

Answer 11D Alcohol withdrawal symptoms typically emerge 6


hours after cessation or a signi icant decrease in intake of alcohol. The
initial presentation may include autonomic instability, tremors,
hyperactivity, insomnia, and headaches. If the withdrawal symptoms
are minor, the patient is typically conscious and oriented.
Hallucinations and illusions may occur and can last up to 6 days. Acute
symptomatic seizures may emerge 6-48 hours after the last drink.
Delirium tremens (DTs) are indicative of severe withdrawal symptoms
and have a time of onset typically between 48 and 72 hours after
cessation of drinking. More severe withdrawal presentations may last
for up to 2 weeks.

Answer 12A Light-to-moderate alcohol intake is protective against


the development of dementia, while excessive drinking increases the
risk of dementia. A recent meta-analysis noted that the alcohol dose
associated with lower risk of dementia was con ined to at most
12.5 gm/day with the risk hitting bottom (RR ≈ 0.9) at roughly 6 g/day.
The risk for dementia seemed to be elevated (≈10%) when the dose
surpasses 23 drinks/week or 38 gm/day. For the alcohol type,
recommendation for wine is prioritized. The subgroup analysis further
indicated that the effect of alcohol may be greater in younger adults
(<60 years old) with regard to protection against dementia.

Answer 13B The examination of the National Death Index data and
Veterans Health Administration patient treatment records from the
iscal year 1999 indicated that out of the 7684 veterans who died by
suicide 7 years onwards, being diagnosed with bipolar disorder was
found to have the greatest estimated risk of suicide among men, and a
substance use disorder diagnosis was found to have the greatest
estimated risk of suicide among women.
Answer 14C It is estimated that up to 11% of older adult women are
misusing prescription drugs and that number of older adults aged
greater than or equal to 50 years using prescription drugs for
nonmedical uses will rise to 2.7 million by the year 2020. This suggests
that soon, specialized interventions may be required to treat substance
use disorders in older adults who will present with multiple
comorbidities and varying levels of functional impairment.

Answer 15E Confabulations are “false or erroneous memories


arising involuntarily in the context of neurological amnesia” that are
typical in Korsakoff syndrome presentations. Other symptoms
commonly found in Korsakoff syndrome include anterograde and
retrograde amnesia, executive dysfunction, apathy, and other social and
affective issues related to alcohol-related cognitive impairment.
Fluctuating orientation is typically found in delirious patients.
Irritability and light of ideas could occur in mania/hypomania. Anomic
aphasia may occur in different types of dementias including
Alzheimer’s disease.

Answer 16E According to data obtained from the Netherlands


Mental Health Survey and Incidence Study-2 (NEMESIS-2), the greatest
predictors of risk of relapse were medium-risk (15–28 drinks for
women, and 42 drinks weekly for men) and high-risk (≥29 drinks
weekly for women, and 43 drinks weekly for men) drinking behavior in
those diagnosed with severe alcohol use disorder (AUD) wherein they
experienced at least 6 or more AUD symptoms in their lifetime.

Answer 17D Among community non-institutionalized adults aged


≥50 years, 60% have used alcohol, 3% used illicit drugs, and 1–2% used
nonmedical prescription drugs in the past year. Among all drug users
aged ≥50 years, 10–12% meet the criteria for a drug use disorder.

Answer 18A Tolerance, withdrawal syndrome, and dose escalation


may be less common among older adults in relation to misuse/abuse of
benzodiazepines, opiates, and skeletal muscle relaxants. Chronic pain,
insomnia, and anxiety are common complaints that lead to prescription
of controlled substances and the potential for their abuse/misuse.
Lower doses may decrease the risk of abuse and dependence, although
fear of abuse often results in failure to adequately treat these
symptoms.

Answer 19E According to the Epidemiological Catchment Area


(ECA) survey, antisocial personality was found to be present in 14.3%
of those considered to be “alcoholics” with an odds ratio of 21.0. It was
also reported that 73.6% of ECA respondents diagnosed with antisocial
personality disorder also met criteria for “alcoholism.”

Answer 20B One randomized trial comparing treatment


engagement with integrated care versus enhanced referral care for
depression, anxiety, and at-risk alcohol use in geriatric primary care
patients found that older patients were more likely to accept
collaborative mental health treatment with primary care than in
specialized mental health and substance abuse clinic (i.e., integrated
service arrangements appear to improve engagement with mental
health and substance abuse services in older adults who may otherwise
not utilize these services).

Answer 21C γ-Glutamyltransferase (GGT) is a glycoprotein


biomarker implicated in intracellular glutathione homeostasis. High
levels of GGT are an early indicator of liver disease and have been
associated with signi icant long-term heavy drinking. However, it is not
a very sensitive marker as it shows up in only 30-50 percent of drinkers
in the general population. Despite this, GGT is still more sensitive than
the other biomarkers listed, which are more indicative of short-term
alcohol use (blood alcohol level) or less sensitive indicators of liver
disease associated with alcohol use (AST and ALT).

Answer 22C The data from the 2008 to 2009 US National Surveys
on Drug Use and Health showed that the past-year tobacco use was
twice as frequent among adults 50 to 64 years in age (30.2%) when
compared to adults ≥65 years in age (14.1%). The investigators found
that the odds of increased cigarette smoking were noted among men,
whites, African Americans, and those who had less education, had
lower income, were not currently married, or were binge drinkers or
illicit/non-medical drug users.
Answer 23D The rise in marijuana use in both medicinal and
recreational capacity has the potential to predispose older adult users
to unique risks, as it has been known to cause short-term memory
impairment, increased pulse rate, increased respiratory rate, and
elevated blood pressure. Additionally, it has also been reported to
increase the risk of a heart attack by four times within the irst hour of
smoking marijuana. Thus, older adults with signi icant cardiovascular
risk factors should be warned of the potential adverse effects that
marijuana use could cause.

Answer 24E Despite a relatively high prevalence of alcohol abuse in


the elderly, problems with alcohol often go undetected by health care
providers, especially as older adults may be less likely to seek help or
see their alcohol use as problematic. Due to the higher prevalence of
medical comorbidities in the geriatric population that can worsen with
alcohol abuse, rapid detection and prompt treatment should be
provided when appropriate. Some signs and symptoms of potential
alcohol use disorder among older adults include anxiety, depression,
disorientation, excessive mood swings, unexplained falls, bruises, or
burns, socioeconomic problems, increased tolerance to alcohol,
memory loss, new-onset dif iculties in decision-making, poor hygiene
and nutrition, seizures, sleep problems, and social isolation.

Answer 25B Older adults with prescription opioids as their primary


drug of choice were more likely to be male, African American, less likely
to be Hispanic, preferred an oral route for consumption (and were less
likely to use the intranasal route), were more likely to use alcohol (but
less likely to use other illicit substances), and were more likely to live in
urban areas. In contrast, older adults with heroin as their primary drug
of choice preferred using intravenous and intranasal routes for
consumption and were more likely to be either African American or
Hispanic. It should be noted that there is still limited data regarding the
prevalence of opioid use disorders (OUD) and medication-assisted
treatment in the geriatric population despite an increasing number of
older adults presenting with a diagnosis of OUD and requesting opioid
maintenance therapy.
Further Reading
American Psychiatric Association. Cannabis-related disorders. In: Diagnostic and
statistical manual of mental disorders. 5th ed. Arlington: American Psychiatric
Publishing, a division of American Psychiatric Association; 2013. p. 509–16.
[Crossref]

Arts NJ, Walvoort SJ, Kessels RP. Korsakoff’s syndrome: a critical review.
Neuropsychiatr Dis Treat. 2017;13:2875–90.
[Crossref]

Barry KL, Blow FC, Oslin DW. Substance abuse in older adults: review and
recommendations for education and practice in medical settings. Subst Abus.
2002;23(3 Suppl):105–31.
[Crossref]

Bartels SJ, Coakley EH, Zubritsky C, et al. Improving access to geriatric mental health
services: a randomized trial comparing treatment engagement with integrated
versus enhanced referral care for depression, anxiety, and at-risk alcohol use. Am J
Psychiatry. 2004;161(8):1455–62.
[Crossref]

Blazer DG, Wu LT. Patterns of tobacco use and tobacco-related psychiatric morbidity
and substance use among middle-aged and older adults in the United States. Aging
Ment Health. 2012;16(3):296–304.
[Crossref]

Culberson JW, Ziska M. Prescription drug misuse/abuse in the elderly. Geriatrics.


2008;63(9):22–31.
[PubMed]

Huhn AS, Strain EC, Tompkins DA, et al. A hidden aspect of the U.S. opioid crisis: rise
in irst-time treatment admissions for older adults with opioid use disorder. Drug
Alcohol Depend. 2018;193:142–7.
[Crossref]

Ilgen MA, Bohnert AS, Ignacio RV, et al. Psychiatric diagnoses and risk of suicide in
veterans. Arch Gen Psychiatry. 2010;67(11):1152–8.
[Crossref]

Kuerbis A, Sacco P, Blazer DG, Moore AA. Substance abuse among older adults. Clin
Geriatr Med. 2014;30(3):629–54.
[Crossref]
Mason BJ, Heyser CJ. Acamprosate: a prototypic neuromodulator in the treatment of
alcohol dependence. CNS Neurol Disord Drug Targets. 2010;9(1):23–32.
[Crossref]

Mirijello A, D'Angelo C, Ferrulli A, et al. Identi ication and management of alcohol


withdrawal syndrome. Drugs. 2015;75(4):353–65.
[Crossref]

Moore AA, Whiteman EJ, Ward KT. Risks of combined alcohol/medication use in
older adults. Am J Geriatr Pharmacother. 2007;5(1):64–74.
[Crossref]

Moore AA, Karno MP, Grella CE, et al. Alcohol, tobacco, and nonmedical drug use in
older U.S. Adults: data from the 2001/02 national epidemiologic survey of alcohol
and related conditions. J Am Geriatr Soc. 2009;57(12):2275–81.
[Crossref]

Oslin D, Liberto JG, O’Brien J, et al. Naltrexone as an adjunctive treatment for older
patients with alcohol dependence. Am J Geriatr Psychiatry. 1997;5(4):324–32.
[Crossref]

Peterson K. Biomarkers for alcohol use and abuse–a summary. Alcohol Res Health.
2004-2005;28(1):30–7.
[PubMed][PubMedCentral]

Regier DA, Farmer ME, Rae DS, et al. Comorbidity of mental disorders with alcohol
and other drug abuse. Results from the Epidemiologic Catchment Area (ECA) Study.
JAMA. 1990;264(19):2511–8.
[Crossref]

Simoni-Wastila L, Yang HK. Psychoactive drug abuse in older adults. Am J Geriatr


Pharmacother. 2006;4(4):380–94.
[Crossref]

Substance Abuse Among Older Adults. Center for substance abuse treatment. source.
Rockville: Substance Abuse and Mental Health Services Administration; 1998.
Report No.: (SMA) 98-3179.

Tuithof M, ten Have M, van den Brink W, et al. Alcohol consumption and symptoms
as predictors for relapse of DSM-5 alcohol use disorder. Drug Alcohol Depend.
2014;140:85–91.
[Crossref]

Tyburski EM, Sokolowski A, Samochowiec J, et al. New diagnostic criteria for alcohol
use disorders and novel treatment approaches – 2014 update. Arch Med Sci.
2014;10(6):1191–7.
[Crossref]

van Beek JHDA, de Moor MHM, Geels LM, et al. The association of alcohol intake with
γ-glutamyl transferase (GGT) levels: evidence for correlated genetic effects. Drug
Alcohol Depend. 2014;134:99–105.
[Crossref]

Wu L-T, Blazer DG. Substance use disorders and psychiatric comorbidity in mid and
later life: a review. Int J Epidemiol. 2014;43(2):304–17.
[Crossref]

Xu W, Wang H, Wan Y, et al. Alcohol consumption and dementia risk: a dose-response


meta-analysis of prospective studies. Eur J Epidemiol. 2017;32(1):31–42.
[Crossref]

Yarnell S, Li L, MacGrory B, Trevisan L, et al. Substance use disorders in later life: a


review and synthesis of the literature of an emerging public health concern. Am J
Geriatr Psychiatry. 2020;28(2):226–36.
[Crossref]
© Springer Nature Switzerland AG 2021
R. Tampi et al. (eds.), Absolute Geriatric Psychiatry Review
https://doi.org/10.1007/978-3-030-58663-8_15

15. Anxiety Disorders and Obsessive


Compulsive and Related Disorders
Pallavi Joshi1
(1) Department of Psychiatry, Yale School of Medicine, New Haven, CT,
USA

Keywords Anxiety – Generalized anxiety disorder – Phobia –


Obsessive-compulsive disorder – Diagnosis

Questions
1.
What is the estimated lifetime prevalence of generalized anxiety
disorder (GAD) among older adults?
A.
1–5%
B.
5–10%
C.
10–15%
D.
15–20%
E.
20–25%
2. Anxiety disorders and subsyndromal anxiety among older adults
are associated with which of the following?
A.
Increased physical disability
p y y
B. Increased risk for cognitive impairment and dementia

C.
Poorer quality of life
D.
Increased health service use
E.
All of the above
3.
Which of the following makes the recognition of GAD among older
adults particularly dif icult?
A.
Associated physical symptoms overlap signi icantly with
symptoms of normal aging and medical conditions common
in later life
B.
Associated physical symptoms overlap signi icantly with
medication side effects
C.
Older adults infrequently use psychological terms to describe
anxiety
D.
Worry content among older adults re lects problems that
arise in later stages of life
E.
All of the above
4. Which of the following is not a major risk factor for late-onset
GAD?
A.
Male gender
B.
Adverse life events
C.
Chronic physical or mental health disorders
D.
P t
Poverty

E. Parental loss

5.
Which of the following ethnic groups has the lowest lifetime
prevalence of any anxiety disorder among individuals over 50
years in age?
A.
Latinos
B.
Non-Latino whites
C.
Asians
D.
African Americans
E.
Afro-Caribbeans
6.
Which of the following is implicated in the neurobiology of worry
among older adults with GAD?
A.
Decreased activation in left orbitofrontal cortex and
ventromedial prefrontal cortex
B.
Decreased activation in the bilateral amygdala and left insula
C.
A circuit extending from ventromedial through dorsolateral
prefrontal cortices, converging on the amygdala
D.
A circuit extending only from the dorsolateral prefrontal
regions to the amygdala
E.
A second circuit from ventromedial prefrontal cortex to the
insula
7. Which of the following is true about the relationship between
anxiety and stroke?
A. Higher anxiety symptoms are associated with increased risk
of incident stroke among men, but not women

B.
Higher anxiety symptoms are associated with increased risk
of incident stroke in individuals with comorbid depressive
symptoms
C.
Higher anxiety symptoms are associated with increased risk
of incident stroke in individuals with cardiovascular
conditions
D.
Higher anxiety symptoms are associated with increased risk
of incident stroke among women, but not men
E.
Higher anxiety symptoms are associated with increased risk
of incident stroke independent of cardiovascular risk factors
and gender
8.
Which of the following is associated with increased anxiety in
cognitively normal older adults?
A.
Aβ+ women who were APOE ε4 allele carriers
B.
Aβ+ women who were not APOE ε4 allele carriers
C.
Aβ+ men who were APOE ε4 allele carriers
D.
Aβ+ men who were not APOE ε4 allele carriers
E.
Aβ+ women who were APOE ε4 allele carriers with Val66Met
polymorphism of BDNF gene
9. Which of the following is the most prevalent anxiety disorder
among older adults?
A.
Generalized anxiety disorder
B.
Speci ic phobia
C. Social anxiety disorder

D.
Post-traumatic stress disorder
E.
Panic disorder
10.
What is the most common speci ic phobia among older adults?
A.
Fear of heights
B.
Blood-injection-injury phobia
C.
Fear of enclosed spaces
D.
Fear of falling
E.
Fear of driving
11. What is the postulated neuropathologic process by which anxiety
may lead to cognitive decline?
A.
Cortisol-induced overstimulation of glucocorticoid receptors
in the medial temporal lobe leading to hippocampal atrophy
B.
Anxiety is associated with cardiovascular disease, which
increases the risk of vascular dementia
C.
Increased levels of interleukin-6 and tumor necrosis factor
have been found in anxiety disorders
D.
Anxiety disorders are associated with decreased levels of
BDNF
E.
All of the above
12.
Which of the following is true about older adults with panic
disorder?
A.
Onset of symptoms is usually in later life
B.
Compared to younger adults, older adults have more panic
symptoms
C.
Compared to younger adults, older adults have worse global
functioning
D.
Compared to younger adults, older adults have less severe
comorbid depressive symptoms
E.
Compared to younger adults, older adults experience more
distress during panic attacks
13. Which of the following statements about anxiety in individuals
with neurocognitive disorders is false?
A.
Up to 15% of individuals with neurocognitive disorders have
signi icant anxiety symptoms
B.
The overlap between symptoms of anxiety and agitation in
neurocognitive disorders complicates the identi ication of
anxiety in people with neurocognitive disorders
C.
Comorbid anxiety and neurocognitive disorders are
associated with increased functional limitations
D.
Comorbid anxiety and neurocognitive disorders are
associated with poorer physical health
E.
There may be an increased rate of conversion from mild
neurocognitive disorder to major neurocognitive disorder
h i t t t
when anxiety symptoms are present
14.
How does OCD present differently among older adults when
compared to younger adults?
A.
The severity of symptoms on the Yale-Brown Obsessive
Compulsive Scale is higher among younger adults
B.
Older adults have more frequent counting rituals
C.
Older adults have more frequent concerns about symmetry
D.
Older adults have more frequent concerns about toileting
E.
Older adults have more frequent concerns about needing to
know
15.
Which of the following is false about OCD in older adults?
A.
Older adults with OCD have a later age of onset when
compared to younger adults
B.
Older adults with OCD have a higher rate of adult onset when
compared to younger adults
C.
Younger adults with OCD are more likely to be male
D.
Older adults have greater OCD severity and higher prevalence
of symptoms
E.
Younger adults with OCD have a higher incidence of comorbid
tic disorder
16. Which of the following cognitive functions in older adults is not
negatively affected by worry?
A.
Social cognition
B.
Complex cognition
C.
Executive function
D.
Short-delay recall
E. Long-delay recall

17.
According to the DSM-5, what is the minimum duration of
symptoms required for a diagnosis of speci ic phobia?
A.
2 months
B.
3 months
C.
6 months
D.
12 months
E.
No minimum duration of symptoms is required for diagnosis
if the symptoms cause signi icant impairment
18. According to the DSM-5, which of the following physical
symptoms is not a characteristic of worry in GAD?
A.
Restlessness
B.
Fatigue
C.
Increased muscle aches or soreness
D.
Palpitations
E.
Insomnia
19.
What is the most common category of mental illness among older
adults?
A.
Mood disorders
B.
Anxiety disorders
C.
Psychotic disorders
D.
Somatoform disorders
E.
Eating disorders

20.
Which of the following is true regarding the diagnosis of panic
disorder?
A.
At least two panic attacks are followed by 1 month of worry
about additional panic attacks
B.
Severe anxiety in response to social situations is a diagnostic
criterion
C.
Panic attacks are always preceded by worry about a
circumscribed event
D.
A panic attack can occur from a calm state or an anxious state
E.
A signi icant maladaptive change in behavior related to
attacks is not required for diagnosis

Answers
Answer 1C In a study of 1974 community-dwelling individuals aged
65 years or over, the lifetime prevalence of GAD was found to be 11%
(95% CI = 9.6–12.4%), with 24.6% reporting onset of symptoms after
50 years of age.

Answer 2E Anxiety disorders and subsyndromal anxiety among


older adults are associated with increased physical disability, increased
risk for cognitive impairment and dementia, poorer quality of life, and
increased health service use.

Answer 3E Recognizing GAD among older adults is particularly


dif icult because associated physical symptoms, such as sleep
disturbance, fatigue, restlessness, dif iculty concentrating, overlap
signi icantly with symptoms of normal aging and medical conditions
common in later life. Older adults are less accurate than younger adults
in describing symptoms of anxiety and in using psychological terms
(worry, concern, etc.) to describe anxiety. Worry content among older
adults typically re lects problems that are common in later stages of
life, such as health concerns, retirement, loss of loved ones, and end of
life planning.

Answer 4A A prospective cohort study of 1711 individuals aged


65 years and over without a symptoms of GAD at baseline found that
the principal predictors of late-onset incident GAD over 12 years were
as follows: being female, recent adverse life events, and having chronic
physical and mental health disorders. Female gender was associated
with an increased risk of GAD with a hazard ratio of 3.43 (CI = 1.83–
6.41).

Answer 5C Prevalence studies show that the gender-adjusted


lifetime prevalence for any anxiety disorder among adults over 50 years
is as follows: Latinos (18.2%, 1.9); non-Latino whites (18.7%, 1.2);
Asians (9.45%, 1.5); African Americans (15.5%, 1.2); Afro-Caribbeans
(11.5%, 2.7).

Answer 6C A recent study compared 20 older adults with GAD and


16 age-matched controls (mean age = 67.88 years) on clinical measures
and neural activity during worry using functional magnetic resonance
imaging. Compared to older adults without GAD, older adults with GAD
demonstrated increased activation in the left frontal orbitofrontal
cortex and ventromedial prefrontal cortex, increased activation in the
bilateral amygdala and left insula, and a circuit extending from
ventromedial through dorsolateral prefrontal cortices, converging on
the amygdala. Older adults without GAD demonstrated a separate
circuit from the ventromedial prefrontal cortex to the amygdala.

Answer 7E A recent prospective study of anxiety and incident


stroke found that higher anxiety symptoms levels are associated with
higher risk of stroke with a dose-response relationship, with every 1
standard deviation (SD) increase in anxiety associated with a 17%
increase in stroke risk when adjusting for demographic factors. The
association of anxiety with risk of incident stroke remained signi icant
after controlling for gender, cardiovascular, and behavioral risk factors.

Answer 8A APOE ε4 allele carriage has been associated with greater


severity of both depressive and anxiety symptoms in cognitively normal
older adults in which 4.6% and 13.7% screened positive for depression
and anxiety, respectively. Aβ+ in conjunction with ε4 allele carriage was
associated with increased anxiety symptoms in women (d = 0.83), but
not in men. The Val66Met polymorphism of the BDNF gene is
associated with a greater severity of depressive symptoms, but not
anxiety symptoms in women.

Answer 9B Speci ic phobia is the most common anxiety disorder


among older adults with a prevalence of 5–6%.

Answer 10D The most common speci ic phobia among older adults
is the fear of falling. The prevalence ranges from 12% to 65% and is
higher among people with a history of falls.

Answer 11E Anxiety is associated with an increased risk for


cognitive decline and dementia in the community. Postulated biological
pathways include the following: hypercortisolism – anxiety leads to
stress, which leads to higher levels of cortisol and cortisol-induced
overstimulation of glucocorticoid receptors in the medial temporal lobe
leads to hippocampus atrophy; mental stress as a result of anxiety
triggers physiological reactions such as increased heart rate, blood
pressure and platelet activity, which increases the risk for vascular
dementia; increased cytokine levels are found in stress-related
disorders and negatively affect cognitive functioning; anxiety disorders
are associated with decreased levels of BDNF, which is essential for
synaptic plasticity, neuronal repair, and memory.

Answer 12D Onset of panic disorder is typically present in younger


adulthood. Older adults with panic disorder have fewer panic
symptoms, better global functioning, and less severe comorbid
depressive symptoms than younger adults with the disorder.

Answer 13A A critical review of 74 studies of assessment and


clinical characteristics of anxiety in dementia noted that the prevalence
of anxiety disorders among people with neurocognitive disorders range
from 5% to 21%, and up to 71% have signi icant anxiety symptoms.
Diagnosis of anxiety in neurocognitive disorders is challenging because
of the overlap of symptoms of anxiety and agitation associated with
neurocognitive disorders. Neurocognitive disorders are associated with
increased functional limitations and poorer physical health, and there
may be an increased rate of conversion from mild neurocognitive
disorder to major neurocognitive disorder when anxiety symptoms are
present.

Answer 14D The severity of symptoms on the Yale-Brown


Obsessive Compulsive Scale does not substantially differ among older
and younger adults. Older adults have less frequent concerns about
symmetry, need to know, and counting rituals and more frequent
concerns about toileting, fear of having sinned, and medication
schedules.

Answer 15D OCD in younger adults is associated with greater global


severity and higher prevalence of obsessive-compulsive symptoms
when compared to older adults.

Answer 16E A recent study examined the association between the


cognitive symptoms of anxiety (i.e., worry) on cognitive performance in
156 older adults. Cognitive performance was measured by the Penn
Computerized Neurocognitive Battery (Penn CNB), the Delis-Kaplan
Executive Function System (D-KEFS), and the Rey Auditory Verbal
Learning Test (RAVLT). Worry symptoms were a signi icant predictor of
Penn CNB social cognition, complex cognition, executive function, and
episodic memory performance as well as RAVLT immediate and short-
delay recall, but not of D-KEFS performance or RAVLT long-delay recall.
In contrast, anxiety and depressive symptoms had few unique
associations with cognitive performance.

Answer 17C According to the DSM-5, a minimum duration of


symptoms of 6 months is required for a diagnosis of speci ic phobia.

Answer 18D According to the DSM-5, anxiety or worry in GAD is


associated with three or more of the following symptoms: restlessness,
fatigue, dif iculty concentrating, irritability, muscle tension, or sleep
disturbance.

Answer 19B Anxiety disorders are the most common category of


mental illness among older adults. The MentDis_ICF65+ study is a
cross-sectional, multicenter study in Europe based on an age-strati ied,
random sample of 3142 older men and women (aged 65–84 years). The
study estimated the lifetime prevalence of any anxiety disorder among
older adults at 25.6% (95% CI = 21.4–29.7%) and 12-month prevalence
at 17.2% (95% CI = 14.0–20.4%).

Answer 20D A panic attack can occur from a calm state or an


anxious state. A panic attack is not always preceded by worry about a
circumscribed event. For a diagnosis of panic disorder, a panic attack is
followed by at least 1 month of worry about additional panic attacks
and involves a signi icant maladaptive change in behavior related to
attacks. Anxiety in response to social situations is a diagnostic criterion
of social anxiety disorder.

Further Reading
American Psychiatric Association. Anxiety disorders in diagnostic and statistical
manual of mental disorders. 5th ed. Arlington: American Psychiatric Association;
2013. https://doi.org/10.1176/appi.books. 9780890425596, Accessed 4 Jan 2020.
[Crossref]

Andreas S, Schulz H, Volkert J, et al. Prevalence of mental disorders in elderly people:


the European MentDis_ICF65+ study. Br J Psychiatry. 2017 Feb;210(2):125–31.
[Crossref]

Bower ES, Wetherell JL, Mon T, Lenze EJ. Treating anxiety disorders in older adults:
current treatments and future directions. Harv Rev Psychiatry. 2015;23(5):329–42.
[Crossref]

de Vito A, Calamia M, Greening S, Roye S. The association of anxiety, depression, and


worry symptoms on cognitive performance in older adults. Neuropsychol Dev Cogn
B Aging Neuropsychol Cogn. 2019;26(2):161–73.
[Crossref]

Dell’Osso B, Benatti B, Rodriguez CI, et al. Obsessive-compulsive disorder in the


elderly: a report from the International College of Obsessive-Compulsive Spectrum
Disorders (ICOCS). Eur Psychiatry. 2017;45:36–40.
[Crossref]

Gulpers B, Ramakers I, Hamel R, et al. Anxiety as a predictor for cognitive decline


and dementia: a systematic review and meta-analysis. Am J Geriatr Psychiatry.
2016;24(10):823–42.
[Crossref]

Holmes SE, Esterlis I, Mazure CM, et al. β-Amyloid, APOE and BDNF genotype, and
depressive and anxiety symptoms in cognitively normal older women and men. Am J
Geriatr Psychiatry. 2016;24(12):1191–5.
[Crossref]

Jimenez DE, Alegrı́a M, Chen CN, et al. Prevalence of psychiatric illnesses in older
ethnic minority adults. J Am Geriatr Soc. 2010;58(2):256–64.
[Crossref]

Lambiase MJ, Kubzansky LD, Thurston RC. Prospective study of anxiety and incident
stroke. Stroke. 2014;45(2):438–43.
[Crossref]

Mohlman J, Eldreth DA, Price RB, Staples AM, Hanson C. Prefrontal-limbic


connectivity during worry in older adults with generalized anxiety disorder. Aging
Ment Health. 2017;21(4):426–38.
[Crossref]

Ramos K, Stanley MA. Anxiety disorders in late life. Psychiatr Clin North Am.
2018;41(1):55–64.
[Crossref]

Reynolds K, Pietrzak RH, El-Gabalawy R, et al. Prevalence of psychiatric disorders in


U.S. older adults: indings from a nationally representative survey. World Psychiatry.
2015;14(1):74–81.
[Crossref]

Seignourel PJ, Kunik ME, Snow L, et al. Anxiety in dementia: a critical review. Clin
Psychol Rev. 2008;28(7):1071–82.
[Crossref]

Sheikh JI, Swales PJ, Carlson EB, et al. Aging and panic disorder: phenomenology,
comorbidity, and risk factors. Am J Geriatr Psychiatry. 2004;12(1):102–9.
[Crossref]

Taylor S. Early versus late onset obsessive-compulsive disorder: evidence for distinct
subtypes. Clin Psychol Rev. 2011;31(7):1083–100.
[Crossref]

Wetherell JL, Petkus AJ, McChesney K, et al. Older adults are less accurate than
younger adults at identifying symptoms of anxiety and depression. J Nerv Ment Dis.
2009;197(8):623–6.
[Crossref]

Zhang X, Norton J, Carriè re I, Ritchie K, Chaudieu I, Ancelin ML. Generalized anxiety


in community-dwelling elderly: prevalence and clinical characteristics. J Affect
Disord. 2015;172:24–9.
[Crossref]

Zhang X, Norton J, Carriè re I, et al. Risk factors for late-onset generalized anxiety
disorder: results from a 12-year prospective cohort (the ESPRIT study). Transl
Psychiatry. 2015;5:e536.
[Crossref]
© Springer Nature Switzerland AG 2021
R. Tampi et al. (eds.), Absolute Geriatric Psychiatry Review
https://doi.org/10.1007/978-3-030-58663-8_16

16. Sexuality, Gender Dysphoria, and


Paraphilic Disorders
Insiya Nasrulla1
(1) Department of Psychiatry, Medstar Georgetown University
Hospital/Medstar Washington Hospital Center, Washington, DC,
USA

Insiya Nasrulla
Email: Insiya.H.Nasrulla@medstar.net

Keywords Sexuality – Older adults – Menopause – Sexual dysfunction


– Erectile dysfunction – Libido – LGBTQ

Questions
1.
Which of the following is the most commonly reported sexual
dysfunction among older men?
A.
Trouble achieving or maintaining an erection
B.
Lack of interest in sex
C.
Achieving orgasm too quickly
D.
Performance anxiety
E.
Inability to orgasm
Whi h f h f ll i i h l d l
2. Which of the following is the most commonly reported sexual
dysfunction among older women?
A.
Trouble with vaginal lubrication
B.
Finding sex not pleasurable
C.
Pain with sex
D.
Lack of interest in sex
E.
Inability to achieve orgasm
3.
Which of the following is the correct order of sexual response
cycle?
A.
Excitement, plateau, resolution, orgasm
B.
Excitement, resolution, plateau, orgasm
C.
Excitement, plateau, orgasm, resolution
D.
Excitement, resolution, orgasm, plateau
E.
Excitement, orgasm, resolution, plateau
4. A 70-year-old recently widowed (for approximately 6 months)
white man with a history of anxiety and depression that is well
controlled with medications comes into your of ice hesitatingly
complaining of trouble with sustained erections during an
attempted intercourse with his new girlfriend. Medical history
also includes type 2 diabetes, hypertension, and remote history of
melanoma. Which of the following is a de inite indicator that the
patient has a psychogenic cause for his erectile dysfunction (ED)?
A.
Patient is recently widowed
B. Has a history of anxiety and depression

C.
New relationship with girlfriend
D.
Nocturnal erections are present
E.
Has well-controlled diabetes
5.
What does the clinician do if a patient expresses interest in sexual
activity but lacks capacity to engage in sexual activity?
A.
If the individual expresses an interest in sexual activity, then
they can consent for sexual activity and should be able to
engage in the activity
B.
It depends on the jurisdiction where the individual lives and
the laws regarding capacity
C.
The spouse or potential sexual partner can make this decision
for the individual
D.
A surrogate decision-maker other than the spouse/potential
sexual partner can consent
E.
The physician cannot comment on this issue
6. A 67-year-old woman comes to your of ice with complaints of
vaginal dryness, decreased lubrication, pain with intercourse, and
symptoms of urgency. She has had recurrent UTIs this year and is
on suppression antibiotics. Which of the following is the most
appropriate diagnosis for this woman?
A.
Female orgasmic disorder
B.
Menopause
Menopause
C. Major depressive disorder

D.
Medication-induced sexual dysfunction
E.
Genito-pelvic pain/penetration disorder
7.
A 62-year-old post-menopausal woman comes into your of ice
asking for help with decreased libido. You diagnose her with
hypoactive sexual desire disorder and decide to treat her with
libanserin (Addyi). However, which of the following elements in
the patient’s history is a contraindication for use due to black box
warning?
A.
She is an older adult in her 60s and post-menopausal
B.
She has diabetes mellitus and uses daily insulin
C.
She is a 1 pack per day (ppd) smoker for the last 30 years
D.
She drinks half a bottle of wine daily
E.
She has essential hypertension and uses an ACE inhibitor
8. Which antihypertensive has been shown to reduce inappropriate
sexual behaviors in dementia?
A.
Hydrochlorothiazide
B.
Amlodipine
C.
Lisinopril
D.
Prazosin
Prazosin
E.
Pindolol
9.
What is the meaning of the term “trans-woman”?
A.
Transgender person assigned male sex at birth but whose
gender identity is that of a woman
B.
Transgender person assigned female sex at birth but whose
gender identity is that of a man
C.
Can be A or B depending on the institution and state
terminology or the individual’s own preference
D.
Non-binary gender
E.
None of the above

10.
Which of the following is least likely to occur among lesbian, gay,
bisexual, transgender, or queer (LGBTQ) older adults?
A.
Substance use disorders
B.
Major depressive disorder
C.
Prostitution
D.
Suicide
E.
Anxiety disorders

Answers
Answer 1A Among older men, the most prevalent sexual problems
reported are trouble achieving or maintaining an erection, lack of
interest in sex, achieving orgasm too quickly, performance anxiety, and
inability to orgasm in this order.

Answer 2D Among older women, the most prevalent sexual


problems reported are lack of interest in sex, trouble with vaginal
lubrication, inability to achieve orgasm, inding sex not pleasurable, and
pain with sex in this order.

Answer 3C The four stages of sexual response are as follows:


Excitement: interest or urge for sexual activity; plateau: vascular
system and body’s response to stimulation; orgasm: climax, height of
physiologic response; resolution: recovery period after orgasm in
which body returns to resting state.

Answer 4D There are often historical clues that suggest certain


diagnoses. Rapid onset of ED usually indicates psychogenic causes or
genitourinary trauma or surgery. A history of no sustained erection is
suggestive of anxiety or vascular steal. If there is a history of depression
or substance use, then the sexual dysfunction is likely related to these
issues. With underlying psychogenic causes, nocturnal erections still
occur. If there is complete loss of nocturnal erections, this suggests
underlying vascular disease or neurologic disease.

Answer 5B If it is determined that the individual lacks sexual


consent capacity but displays continued interest in sexual activity, a
surrogate decision-maker may be necessary. In the event the surrogate
decision-maker is the spouse or potential sexual partner, con lict of
interest may necessitate an alternative surrogate. Of note, some
jurisdictions prohibit substituted judgment for sexual activity in
incapacitated individuals.

Answer 6B Loss of estrogen with menopause is a signi icant


physiologic event that can affect various aspects of women’s sexual
function. Low levels of estrogen lead to changes within the
genitourinary tract, including shortening of the vagina and vaginal
dryness, Changes in the bacterial lora and pH balance, thinning of the
labia, and a decrease in fat pad under mons pubis may also occur. This
urogenital atrophy makes the vaginal mucosa more susceptible to
trauma from sexual activity, potentially leading to dyspareunia and
vaginal bleeding. There is also increased risk of urinary tract infections
and urinary incontinence secondary to changes in urethral epithelium.

Answer 7D Flibanserin carries a boxed warning for hypotension


and syncope in certain settings, including with the concurrent use of
alcohol. A Risk Evaluation and Mitigation Strategy (REMS) program has
been established for libanserin speci ically due to the risk of
hypotension and syncope associated with concomitant alcohol use.
Prescribers must complete certi ication training before being able to
prescribe libanserin and must document that they counseled the
patient on avoiding alcohol use with libanserin treatment. Although it
is not FDA approved for treatment of sexual dysfunction in post-
menopausal women, the safety and ef icacy of libanserin was assessed
in this population in the SNOWDROP trial as well as the PLUMERIA
trial.

Answer 8E Pindolol, a beta-blocker has been shown to decrease the


adrenergic drive and thus decreasing agitation, aggression, and
inappropriate sexual behaviors among individuals with dementia.
Common side effects of pindolol include fatigue and hypotension.

Answer 9A Transgender persons assigned male sex at birth but


whose gender identity is that of a woman. Cisgender or cis indicates
that sex and gender identity are both the same, whereas transgender or
trans indicates that sex is incongruent with gender identity. Non-binary
gender describes individuals who do not identify with either being a
man or a woman.

Answer 10C It is estimated that 29% of the transgender population


lives in poverty when compared with 12% of the US population. Some
studies have shown that marijuana, crack cocaine, and alcohol are the
most commonly used drugs by transgender people. Studies have also
found alarming rates of methamphetamine use (4–46%) and injection
drug use (2–40%). Some studies suggest that tobacco use rates can
range from 45% to 74%. Approximately 39% had serious psychological
distress in the previous month when compared with only 5% of the US
population; 40% had attempted suicide in their lifetime when
compared with 4.6% of the general population.

Further Reading
English C, Muhleisen A, Rey JA. Flibanserin (Addyi): the irst FDA-approved
treatment for female sexual interest/arousal disorder in premenopausal women. P T.
2017;42(4):237–41.
[PubMed][PubMedCentral]

Hillman J. Sexual consent capacity: ethical issues and challenges in long-term care.
Clin Gerontol. 2017;40(1):43–50.
[Crossref]

Johnson K, Yarns BC, Abrams JM, et al. Gay and gray session: an interdisciplinary
approach to transgender aging. Am J Geriatr Psychiatry. 2018;26(7):719–38.
[Crossref]

Lapinski J, Covas T, Perkins JM, et al. Best practices in transgender health: a


Clinician’s guide. Prim Care. 2018;45(4):687–703.
[Crossref]

Morton L. Sexuality in the older adult. Prim Care. 2017;44(3):429–38.


[Crossref]

Ozkan B, Wilkins K, Muralee S, Tampi RR. Pharmacotherapy for inappropriate sexual


behaviors in dementia: a systematic review of literature. Am J Alzheimers Dis Other
Dement. 2008;23(4):344–54.
[Crossref]

Portman DJ, Brown L, Yuan J, et al. Flibanserin in postmenopausal women with


hypoactive sexual desire disorder: results of the PLUMERIA study. J Sex Med.
2017;14(6):834–42.
[Crossref]

Srinivasan S, Glover J, Tampi RR, Tampi DJ, Sewell DD. Sexuality and the older adult.
Curr Psychiatry Rep. 2019;21(10):97.
[Crossref]

Streed CG Jr, McCarthy EP, Haas JS. Self-reported physical and mental health of
gender nonconforming transgender adults in the United States. GBT Health.
2018;5(7):443–8.
Wilkins JM. More than capacity: alternatives for sexual decision making for
individuals with dementia. Gerontologist. 2015;55(5):716–23.
[Crossref]

Yarns BC, Abrams JM, Meeks TW, et al. The mental health of older LGBT adults. Curr
Psychiatry Rep. 2016;18(6):60.
[Crossref]
Part IV
Diagnostic Methods
In this part, we review the topics of patient interview, psychiatric
history, attitudinal bias, collateral, suicide risk assessment, family
assessment, mental status examination, cognitive screening, functional
assessment, psychological and neuropsychological testing, neurologic
examination, clinical laboratory testing, structural and functional
imaging in relation to the care of older adults with psychiatric
disorders.
Patient interview · Psychiatric history · Attitudinal bias · Collateral ·
Suicide risk assessment · Mental status examination · Cognitive
screening · Functional assessment · Psychological and
neuropsychological testing · Neurologic examination · Clinical
laboratory testing · Structural and functional imaging
Kristina Zdanys
Rabin Dahal
Rosemary Szparagowski
© Springer Nature Switzerland AG 2021
R. Tampi et al. (eds.), Absolute Geriatric Psychiatry Review
https://doi.org/10.1007/978-3-030-58663-8_17

17. Interviewing and History Taking


Kristina Zdanys1
(1) University of Connecticut School of Medicine, Farmington, CT, USA

Kristina Zdanys
Email: zdanys@uchc.edu

Keywords Patient interview – Psychiatric history – Attitudinal bias –


Collateral – Suicide risk assessment – Family assessment

Questions
1. A 79-year-old man schedules a irst psychiatric appointment for
assessment of subjective cognitive changes. Which of the following
is the best initial approach to the interview?
A.
Interview the patient irst
B.
Interview a collateral informant before the initial appointment
with the patient’s consent
C.
Interview a collateral informant before the initial appointment
without the patient’s knowledge
D.
Interview a collateral informant at the initial appointment
before meeting with the patient
E.
Interview a collateral informant irst at the initial appointment,
but with the patient in the room
2.
Which of the following is true regarding attitudinal biases in the
psychiatric interview of an older adult?
A.
Biases of the interviewer are unlikely to impact data collection
about symptoms
B.
Biases of both the patient and the interviewer may result in the
over-attribution of psychiatric symptoms to the aging process
C.
The patient’s biases are usually unconscious, while the
interviewer’s biases are usually conscious
D.
The patient’s biases are usually conscious, while the
interviewer’s biases are usually unconscious
E.
An older patient cannot have attitudinal biases about aging

3. A patient with moderate stage vascular dementia presents for


psychiatric interview. However, based on her answers she does not
appear to be an accurate historian. What is the next best step in the
assessment of this individual?
A.
Terminate the patient interview and consult caregivers for
collateral information
B.
Provide reassurance and elicit the most concerning subjective
symptoms from the patient; then, ill in gaps with collateral
from caregivers
C.
Provide reassurance and repeat questions until understood
D.
Avoid open-ended questions and offer the patient multiple-
choice answers
E.
Continue the full interview before consulting caregivers for
collateral information
4.

Which of the following is true regarding assessment of suicide risk


during the psychiatric interview of older adults?
A.
Suicide risk should be assessed with caregivers present who
can con irm or dispute the patient’s answer
B.
Suicidal thoughts should be prioritized as the irst symptom to
be assessed in a psychiatric review of systems, so that
hospitalization may be considered
C.
Access to means of suicide should not be discussed, as the
patient may previously not have considered those means
D.
Suicide assessment involves step-wise questioning, exploring
hopelessness, passive death wish, and active suicidality
E.
Suicide risk assessment may be omitted from psychiatric
interview for patients living in skilled nursing facilities and
other institutions with 24-hour staff
5. Family assessment is a key component of the psychiatric interview
of older adults, in part to assess supports available to the patient
through the family network and thereby identify additional
services that may be needed. Which of the following behavioral
disturbances has been suggested to be the least tolerated by family
caregivers of patients with functional impairment?
A.
Physical aggression
B.
Personality con lict
C.
Sleep disturbance
D.
Daytime wandering
E.
Anxiety

Answers
Answer 1A Whenever possible, the patient should be interviewed
irst.

Answer 2B Both patients and interviewers may attribute


psychiatric symptoms such as loss of energy, loss of appetite, weight
loss, concentration dif iculty, cognitive decline, and psychomotor
changes to the aging process rather than to an underlying psychiatric
illness or neurocognitive disorder.

Answer 3B Patients with moderate- and later-stage dementias often


are unable to provide speci ic details when answering questions posed
in a psychiatric interview, but may be able to report more general
subjective concerns. The interviewer should elicit subjective concerns
from the patient and ill in gaps from collateral informants, rather than
terminate the interview entirely. Repeating questions would not
improve the patient’s ability to provide accurate answers, and patients
may be overwhelmed by multiple-choice options. If the patient is
unable to provide an accurate history, it would not be helpful to
continue the full interview before consulting caregivers for collateral.

Answer 4D Suicide assessment involves step-wise questioning that


explores feelings of hopelessness (e.g., “Do you see any future for
yourself?”), passive death wish (e.g., “Do you ever feel life is not worth
living?”), and suicidality (e.g., “Do you ever think about taking your own
life?”). Suicide risk should be assessed one-on-one with the patient as
he or she may be hiding suicidal thoughts from family and other
caregivers. Suicidal thoughts should always be addressed, but should be
part of a more comprehensive assessment of depressive symptoms
rather than the irst item discussed in the interview in order to
establish rapport and trust. Patients may die by suicide in long-term
care facilities, and thus suicide risk assessment should not be omitted
for this population.
Answer 5C Sleep disturbance is generally least tolerated by family
caregivers of older adults. It is important to screen for behavioral
disturbances and assess the impact of these disturbances on family and
other caregivers as part of the psychiatric interview of older adults.

Further Reading
Blazer DG. The psychiatric interview of older adults. In: Steffens DC, Blazer DG,
Thakur ME, editors. The American Psychiatric Publishing textbook of geriatric
psychiatry. 5th ed. Washington, DC: American Psychiatric Publishing; 2015. p. 89–
106.

Grossberg GT, Beck D, Zaidi SNY. Rapid depression assessment in geriatric patients.
Clin Geriatr Med. 2017;33(3):383–91.
[Crossref]

Mezuk B, Rock A, Lohman MC, et al. Suicide risk in long-term care facilities: a
systematic review. Int J Geriatr Psychiatry. 2014;29(12):1198–211.
[Crossref]

Sanford JR. Tolerance of debility in elderly dependants by supporters at home: its


signi icance for hospital practice. Br Med J. 1975;3(5981):471–3.
[Crossref]

Varghese M, Dahale AB. The geropsychiatric interview – assessment and diagnosis.


Indian J Psychiatry. 2018;60(Suppl 3):S301–11.
[PubMed][PubMedCentral]
© Springer Nature Switzerland AG 2021
R. Tampi et al. (eds.), Absolute Geriatric Psychiatry Review
https://doi.org/10.1007/978-3-030-58663-8_18

18. Mental Status Examination


Kristina Zdanys1
(1) University of Connecticut School of Medicine, Farmington, CT, USA

Kristina Zdanys
Email: zdanys@uchc.edu

Keywords Behavior – Speech – Psychomotor function – Psychosis –


Suicide assessment

Questions
1.
Which of the following descriptors of speech would be consistent
with a patient who suffers from Broca’s aphasia?
A.
Echolalia
B.
Echopraxia
C.
Word salad
D.
Non- luent
E.
Pressured
2. Which of the following indings on the mental status examination is
observed in excited, or “malignant” catatonia?
A.
Severe psychomotor agitation
Severe psychomotor agitation
B. Waxy lexibility

C.
Grimacing
D.
Posturing
E.
Echolalia
3.
A 74-year-old man is evaluated in an outpatient psychiatry clinic.
He endorses 6 months of sad mood, low energy, impaired
concentration, decreased appetite with 10lb weight loss, restless
sleep, and anhedonia. He denied suicidal ideation. The mental
status examination reveals additional new-onset auditory
hallucinations telling him he is worthless, and he thinks that he has
caused his children great harm. His wife denies any new stressors
and states her husband’s thoughts about their children are not real.
He scored 27 out of 30 points on a standardized cognitive screen
and was fully oriented to time and place. He feels medically well
and his vital signs are stable. This presentation is most consistent
with which of the following disorders?
A.
Late-onset schizophrenia
B.
Very-late-onset schizophrenia-like psychosis
C.
Major depressive disorder, severe with psychotic features
D.
Delirium
E.
Dementia with Lewy bodies (DLB)
4. What distinguishes the Mini-Mental State Examination (MMSE)
from the mental status examination?
A.
The MMSE is the same as a mental status examination
The MMSE is the same as a mental status examination

B. The MMSE is a structured alternative to the mental status


examination
C.
The MMSE identi ies delirium and the mental status
examination does not
D.
The MMSE speci ically screens for cognitive impairment, while
the mental status examination includes assessment of
cognition among other assessments of the psychiatric
interview
E.
The MMSE is an independent diagnostic tool
5.
The question, “do you ever feel like someone is out to get you?”
gives information regarding which part of the mental status
examination?
A.
Thought process
B.
Thought content
C.
Perceptual disturbances
D.
Cognition
E.
Judgment

Answers
Answer 1D Broca’s aphasia is characterized by a lack of luency in
speech.

Answer 2A Severe psychomotor agitation is seen in excited


catatonia, which can lead to complications such as hyperthermia and
autonomic dysfunction. The other choices are consistent with catatonia
of the retarded type.

Answer 3C The patient meets criteria for an active major depressive


episode, and the symptoms of hallucinations and delusions on mental
status examination are consistent with the modi ier “severe with
psychotic features.” Psychotic symptoms may emerge in delirium and
DLB, although this patient does not demonstrate any alteration in
consciousness and/or medical symptoms consistent with delirium, and
visual hallucinations would be more consistent with DLB than auditory
hallucinations.

Answer 4D The MMSE is a screen for cognitive impairment. The


mental status examination is part of the psychiatric interview and
includes a cognitive screen as well as other assessments such as
appearance, affect, mood, psychomotor-activity, perception, thought
process, thought content, insight, and judgment. The two terms are
frequently mistaken for one another. Cognitive impairment identi ied
on the MMSE may be in the context of delirium, dementia, or other
underlying neuropsychiatric or cognitive process. The mental status
examination takes into account other presentations consistent with
delirium, such as psychomotor changes and changes in thought
process/content. The MMSE is not a diagnostic tool on its own.

Answer 5B This question examines thought content, speci ically


probing for paranoid ideation.

Further Reading
Acharya AB, Wroten M. Broca Aphasia. StatPearls [Internet]. Treasure Island:
StatPearls Publishing; 2020.

American Psychiatric Association. Depressive disorders. Desk reference to the


Diagnositc criteria from DSM-5. Arlington: American Psychiatric Publishing, a
division of American Psychiatric Association; 2013. p. 93–114.

Blazer DG. The psychiatric interview of older adults. In: Steffens DC, Blazer DG,
Thakur ME, editors. The American Psychiatric Publishing textbook of geriatric
psychiatry. 5th ed. Washington, DC: American Psychiatric Publishing; 2015. p. 89–
106.

Folstein MF, Folstein SE, McHugh PR. “Mini-mental state”. A practical method for
grading the cognitive state of patients for the clinician. J Psychiatr Res.
1975;12(3):189–98.
[Crossref]

McIntyre KM, Norton JR, McIntyre JS. Psychiatric interview, history, and mental
status examination. In: Sadock BJ, Sadock VA, Ruiz P, editors. Kaplan & Sadock’s
comprehensive textbook of psychiatry. 9th ed. Philadelphia: Wolters Kluwer
Health/Lippincott Williams & Wilkins; 2009. p. 886–906.

Rasmussen SA, Mazurek MF, Rosebush PI. Catatonia: our current understanding of its
diagnosis, treatment and pathophysiology. World J Psychiatry. 2016;6(4):391–8.
[Crossref]

Tombaugh TN, McIntyre NJ. The mini-mental state examination: a comprehensive


review. Am Geriatr Soc. 1992;40(9):922–35.
[Crossref]
© Springer Nature Switzerland AG 2021
R. Tampi et al. (eds.), Absolute Geriatric Psychiatry Review
https://doi.org/10.1007/978-3-030-58663-8_19

19. Functional Assessment


Kristina Zdanys1
(1) University of Connecticut School of Medicine, Farmington, CT, USA

Kristina Zdanys
Email: zdanys@uchc.edu

Keywords Instrumental Activities of Daily Living (IADLs) – Basic


Activities of Daily Living (BADLs) – Neurocognitive disorder –
Functional Assessment Staging (FAST) scale

Questions
1.
Impairment in Instrumental Activities of Daily Living (IADLs)
among older adults with geriatric depression has been correlated
with which of the following conditions?
A.
Anxiety
B.
Greater depression severity
C.
Weight gain
D.
A&B
E.
A, B, and C
2. Which of the following is considered an Instrumental Activity of
Daily Living (IADL)?
y g( )

A. Bathing

B.
Dressing
C.
Eating
D.
Toileting
E.
Shopping
3.
When an older individual develops impairment in Basic Activities
of Daily Living (BADLs), which one of the following abilities do they
tend to lose irst?
A.
Bathing
B.
Dressing
C.
Toileting
D.
Transferring
E.
Maintaining continence
4. In addition to Basic Activities of Daily Living (BADLs) and
Instrumental Activities of Daily Living (IADLs), what additional
parameter is critical to a functional assessment among older
adults?
A.
Social functioning
B.
Technological functioning
C.
Driving safety
D. Financial resources

E.
Marital status
5.
An individual with mild Alzheimer’s disease is unable to manage
his own inances, but does not require assistance choosing clothing.
This correlates with what stage on the Functional Assessment
Staging (FAST) scale?
A.
3
B.
4
C.
5
D.
6A
E.
7A

Answers
Answer 1D IADL impairment has been correlated with anxiety,
greater depression severity, weight loss, and psychomotor retardation.

Answer 2E IADLs include shopping, cooking, using the telephone,


managing medications, driving, performing housework, and managing
inances.

Answer 3B BADLs include self-care activities that an individual


must accomplish to be self-suf icient. These activities include bathing,
dressing, toileting, transferring, maintaining continence, and feeding.
Individuals tend to lose these abilities in the order noted before and
tend to regain their function in the opposite order during rehabilitation.
Answer 4A Social functioning assessment evaluates the social
interactions of the older adult and includes frequency of getting out of
the house, in-person social contacts, and other means of contact with
supports (e.g., telephone calls). A change in social functioning may be
indicative of a change in present illness. Technological functioning and
driving safety would be considered through IADL assessment. Financial
resources are important to understand but are not a direct functional
assessment. Marital status is part of social history and may contribute
to history of present illness, particularly if there has been a change such
as divorce or death of spouse.

Answer 5B The FAST scale assigns a numeric value (from 1 to 7F)


that correlates with speci ic levels of functional impairment for patients
with a neurocognitive disorder. A patient demonstrating dif iculty
performing complex tasks such as managing inances, but who does not
require assistance with choosing clothing or have other impaired
BADLs would be assigned a score of 4.

Further Reading
Alexpooulos GS, Vrontou C, Kakuma T, et al. Disability in geriatric depression. Am J
Psychiatry. 1996;153(7):877–85.
[Crossref]

Blazer DG. The psychiatric interview of older adults. In: Steffens DC, Blazer DG,
Thakur ME, editors. The American Psychiatric Publishing textbook of geriatric
psychiatry. 5th ed. Washington, DC: American Psychiatric Publishing; 2015. p. 89–
106.

Katz S. Assessing self-maintenance: activities of daily living, mobility, and


instrumental activities of daily living. J Am Geriatr Soc. 1983;31(12):721–7.
[Crossref]

Lawton MP, Brody EM. Assessment of older people: self-maintaining and


instrumental activities of daily living. Gerontologist. 1969;9(3):179–86.
[Crossref]

Reisberg B. Functional assessment staging (FAST). Psychopharmacol Bull.


1988;24(4):653–9.
[PubMed]
Tran HT, Leonard SD. Geriatric assessment for primary care providers. Prim Care.
2017;44(3):399–411.
[Crossref]
© Springer Nature Switzerland AG 2021
R. Tampi et al. (eds.), Absolute Geriatric Psychiatry Review
https://doi.org/10.1007/978-3-030-58663-8_20

20. Psychological and


Neuropsychological Testing
Rabin Dahal1
(1) Yale New Haven Hospital, New Haven, CT, USA

Rabin Dahal
Email: rabin.dahal@yale.edu

Keywords Neuropsychological evaluation – Cognitive domains –


Dementia – Neuropsychological pro ile – Cognitive test interpretation

Questions
1. Which of the following statements is true of neuropsychological
evaluation?
A.
Neuropsychological evaluation can identify the onset and type
of mild and major neurocognitive disorder so that early
intervention can occur
B.
Neuropsychological evaluation can be useful in predicting the
degree of driving risk in persons with major neurocognitive
disorder
C.
Neuropsychological evaluation can be useful in determining
decision-making capacity in persons with cognitive
impairment
D. Neuropsychological evaluation can identify cognitive de icits,
predict functional outcomes, and monitor patient recovery
after traumatic brain injury
after traumatic brain injury
E.

All of the above


2.
Which of the following evaluates executive functioning?
A.
Wisconsin Card-Sorting Test
B.
Token Test
C.
Wechsler Adult Intelligence Scale Revised (WAIS-R)
D.
Rey-Osterrieth Complex Figure
E.
All of the above
3.
Which of the following evaluates language functions?
A.
Wisconsin Card-Sorting Test
B.
Token Test
C.
Wechsler Adult Intelligence Scale Revised (WAIS-R)
D.
Rey-Osterrieth Complex Figure
E.
Symbol Digit Modalities Test
4.
Which of the following neurocognitive pro iles would be consistent
with a diagnosis of major neurocognitive disorder, probably due to
Alzheimer’s disease, mild stage?
Cognitive Perception Memory Attention Executive Language Praxis
domains function
Choices
A. Impaired Impaired
B. Impaired Impaired Impaired
C. Impaired Impaired
D. Impaired
E. Impaired Impaired Impaired

5.
Which of the following neurocognitive pro iles would be consistent
with a diagnosis of probable major frontotemporal neurocognitive
disorder, mild stage?

Cognitive Perception Memory Attention Executive Language Praxis


domains function
Choice
A. Impaired Impaired
B. Impaired Impaired Impaired
C. Impaired Impaired
D. Impaired
E. Impaired Impaired Impaired

6.
Which of the following pairs correctly differentiates cortical from
subcortical patterns of cognitive impairment?

Impairment Features Cortical pattern Subcortical


pattern
Choice
A. Alertness Normal Reduced
B. Cognitive processing Slowed Normal
speed
Impairment Features Cortical pattern Subcortical
pattern
C. Executive functioning Impaired from the Normal in early
onset stages
D. Language Dysarthria Impaired
E. Praxis Normal Impaired

7.
Which of the following is not correctly paired for a progressive
neurocognitive disorder and neurocognitive symptoms associated
with a major depressive disorder?

Disorder Features Progressive Neurocognitive symptoms


neurocognitive associated with a major
disorder depressive disorder
Choice
A. Onset Insidious Acute
B. Progress Slow Rapid
C. Awareness Minimal or Present
absent
D. Mood Normal Depressed
E. Neuropsychological Improves or Worsens on repeated testing
testing results stable on
repeated testing

8. When a healthy individual is given a neuropsychological battery of


tests, the chances of this individual obtaining a low score
increases with which of the following?
A.
Fewer number of tests being administered and interpreted
B.
Higher years of education
C.
Higher levels of intelligence at baseline
D. Belonging to an ethnic minority

E.
Frequent testing
9.
Which of the following is not a logical principle in interpreting a
neuropsychological test?
A.
Interpretation begins with the analysis of the information
from interview, medical records, behavioral observations, and
tests
B.
Test score pro iles are optimally meaningful when interpreted
within the context of background information, observed
behaviors, and an approach to each problem-solving task
C.
Hypotheses generated from the test pro ile can be used as a
stand-alone factor in inferring someone’s cognition
D.
Scores in neuropsychological test are compared to norms for
that particular age and sex, among others
E.
None of the above
10. Symbol Digit Modalities Test evaluates which of the following
cognitive domains?
A.
Speech
B.
Perception
C.
Attention
D.
Motor control
E.
Executive function
11.

Which of the following is a key component of a


neuropsychological evaluation?
A.
Interview with the patient, family member, and caregiver
B.
Qualitative assessment of mood, cognition, and behavior
C.
Administration of neuropsychological battery
D.
Final report and review
E.
All of the above

12.
In which of the following situations would you not consider a
referral for a neuropsychiatric evaluation?
A.
Patient is very severely affected
B.
There is already a clear diagnosis
C.
Patient has severe distress or anxiety before a test
D.
Patient had neuropsychological testing performed 2 months
ago
E.
All of the above
13.
Which of the following neurocognitive pro iles would be
consistent with a diagnosis of corticobasal degeneration?

Cognitive Perception Memory Attention Executive Language Praxis


domains function
Cognitive Perception Memory Attention Executive Language Praxis
domains function
Choice
A. Impaired Impaired
B. Impaired Impaired Impaired
C. Impaired Impaired
D. Impaired
E. Impaired Impaired Impaired

14.
Which of the following is not true for the difference between
Montreal Cognitive Assessment (MoCA) and the Mini Mental State
Examination (MMSE)?
A.
Floor and ceiling effects are less common with MoCA
B.
MoCA is more likely to detect subtle cognitive de icits
C.
MoCA can yield lower scores in diverse healthy population-
based samples
D.
MMSE not only yields a total score but also index scores based
on individual items tapping domains of attention, retentive
memory, orientation, language, and executive function
E.
The most common version of MMSE tests the ability to make
intersecting pentagons, whereas the most common version of
MoCA tests the ability to copy a cube
15. A neurocognitive pro ile that includes poor executive function that
includes poor planning, dif iculties with working memory,
attention, problem solving, verbal reasoning, inhibition, mental
lexibility, multitasking, monitoring of actions, task changing, and
decreased speed of processing would be consistent with which of
the following disorders?
A. Binswanger’s disease
g

B.
Alzheimer’s disease
C.
Lewy body dementia
D.
Corticobasal degeneration
E.
Frontotemporal dementia

Answers
Answer 1E Neuropsychological evaluation can identify the onset
and type of mild and major neurocognitive disorders so that early
intervention can occur. Neuropsychological evaluation can also be
useful in predicting the degree of driving risk in persons with major
neurocognitive disorder. Additionally, neuropsychological evaluation is
useful in determining decision-making capacity in persons with
cognitive impairment. Furthermore, neuropsychological evaluation can
identify cognitive de icits, predict functional outcomes, and monitor
patient recovery after traumatic brain injury.

Answer 2A Of the following, the Wisconsin Card-Sorting Test


evaluates executive functioning. It evaluates reasoning, cognitive
lexibility, and abstraction. The test takes about 20–30 minutes to
complete.

Answer 3B Of the following, the Token Test evaluates language


functioning especially verbal comprehension. The test takes about 10–
15 minutes to complete.

Answer 4B Individuals with major neurocognitive disorder,


probably due to Alzheimer’s disease, often will present with
impairments in memory, language, and praxis in the mild stages of the
illness. All other cognitive domains tend to get affected as the illness
progresses to the more severe stages of the illness.

Answer 5A Individuals with probable major frontotemporal


neurocognitive disorder will often present with impairments in
attention and executive functioning in the mild stages of the illness. All
other cognitive domains tend to get affected as the illness progresses to
the more severe stages of the illness.

Answer 6A Individuals with cortical pattern of cognitive


impairment present with normal alertness, normal cognitive
processing speed, preservation of attention and executive functioning
in the early stages of the illness, and impairments in memory, language,
praxis, and perception. Individuals with subcortical pattern of cognitive
impairment often present with reduced alertness, reduced cognitive
processing speed, impaired attention and executive functioning from
the onset of illness, impairments in memory with recognition better
memory, normal language except for dysarthria, normal praxis except
for ideomotor slowing, and normal perception.

Answer 7E Individuals with progressive neurocognitive disorder


present with insidious onset and slow progression of symptoms. They
may or may not have a psychiatric history. They often have limited or
no awareness of their symptoms and/or de icits. Their mood is often
good and their neuropsychological test results worsen with repeated
trials. Among individuals who have neurocognitive symptoms
associated with a major depressive disorder, the onset and progression
of symptoms are sudden and rapid. Psychiatric history is often present,
and the individual is aware of their symptoms and/or de icits. Their
neuropsychological test results remain stable or improve with repeated
trials.

Answer 8D A certain level of variability is expected with healthy


individuals and low scores are not that uncommon. This is related to
individual variation in test performance. For example, a higher number
of tests administered are likely to produce a lower score, even in
healthy individuals. Lower education levels and lower intelligence at
baseline will also affect people’s familiarity with the questions and
their ability to answer the questions correctly. Ethnic minorities are
likely to score lower on the tests, especially if the questionnaire is not
culturally modi ied, e.g., in a study in Northern India, the participants
were unable to answer the months correctly because they use a
different calendar. Caution should be applied while interpreting data in
these scenarios.

Answer 9C Neuropsychological test results need to be carefully


interpreted. They are clinically meaningful only when compared to the
clinical pro ile of the individual. Neuropsychological test results should
not be used as a stand-alone factor in inferring someone’s cognition, as
there are many ways by which a healthy individual can score low on
these tests.

Answer 10C Symbol Digit Modalities Test evaluates attention.

Answer 11E A neuropsychological evaluation includes an interview


with the patient, family member, and caregiver. In addition, there is a
qualitative assessment of mood, cognition, and behaviors. This followed
by the administration of the neuropsychological battery of tests
followed by a written inal report and review of the evaluation with
patient, family member, and/or caregiver.

Answer 12E Neuropsychological testing remains a vital component


of geriatric assessment, but there are certain scenarios where one
should reevaluate the risks and bene its. If a person is affected severely,
multiple domains will be affected, and additional information from
testing may not outweigh the distress and cost to the patient. In cases
where a clear clinical diagnosis is made (e.g., amnestic mild
neurocognitive disorder in a patient with strong family history, with
amyloid uptake positive scans, cognitive loss matching that of
Alzheimer’s disease (AD)), neuropsychological testing can be
reconsidered. However, it can still be performed to establish a baseline
and monitor future decline. Generally, if patients have had testing less
than 6 months ago, the test does not need to be repeated, unless there
is a major neurological event or if the patient has rapidly progressive
dementia. Repeated evaluation may give distorted result due to patient
learning effect. When it is indicated, a parallel version of the test can be
utilized. For patients who are severely confused (e.g., delirious
patients) or those who are responding to internal stimuli, it is hard to
reliably gauge the level of cognition.

Answer 13E Important cognitive features of corticobasal


degeneration include executive dysfunction, aphasia, apraxia,
behavioral changes, and visuospatial dysfunction with relatively
preserved episodic memory.

Answer 14D The MoCA and not the MMSE yields a sub-score based
on domains. MoCA is more sensitive than MMSE in identifying mild
cognitive impairment (MCI). The common version of MMSE has
intersecting pentagons versus copying a cube in the MoCA.

Answer 15A The neurocognitive pro ile described here is that of a


subcortical ischemic vascular dementia. Binswanger’s disease is an
example of arteriolosclerotic cerebral small vessel disease that
prominently affects subcortical brain regions including the basal
ganglia and corona radiata. Patients with arteriolosclerotic cerebral
small vessel disease typically have multiple lacunes or extensive,
con luent white matter lesions as the pathogenic basis for their
symptoms. Cognitive symptoms include poor executive function that
includes poor planning, dif iculties with working memory, attention,
problem solving, verbal reasoning, inhibition, mental lexibility,
multitasking, monitoring of actions, task changing, and decreased
speed of processing.

Further Reading
Alamri Y. Scoring neuropsychological tests: what corrections need to be considered?
Eur Neurol. 2017;78(1–2):84–5.
[Crossref]

American Academy of Clinical Neuropsychology. (AACN) practice guidelines for


neuropsychological assessment and consultation. Clin Neuropsychol.
2007;21(2):209–31.
[Crossref]

Chui HC. Subcortical ischemic vascular dementia. Neurol Clin. 2007;25(3):717–40, vi.
[Crossref]

Larner AJ. Screening utility of the Montreal Cognitive Assessment (MoCA): in place
of–or as well as–the MMSE? Int Psychogeriatr. 2012;24(3):391–6.
[Crossref]

Schroeder RW, Martin PK, Walling A. Neuropsychological evaluations in adults. Am


Fam Physician. 2019;99(2):101–8.
[PubMed]

Teng EL, Manly JJ. Neuropsychological testing: helpful or harmful? Alzheimer Dis
Assoc Disord. 2005;19(4):267–71.
[Crossref]

Zucchella C, Federico A, Martini A. Neuropsychological testing. Pract Neurol.


2018;18(3):227–37.
[Crossref]
© Springer Nature Switzerland AG 2021
R. Tampi et al. (eds.), Absolute Geriatric Psychiatry Review
https://doi.org/10.1007/978-3-030-58663-8_21

21. The Neurological Examination


Rosemary Szparagowski1
(1) Department of Psychiatry, University of Pittsburgh Medical Center,
Pittsburgh, PA, USA

Rosemary Szparagowski
Email: szparagowskirl@upmc.edu

Keywords Medication side effects – Common exam indings – Normal


aging – Medication management – Potentially inappropriate
prescribing – The mental status examination

Questions
1.
Which one of these physical examination indings is not
associated with normal aging?
A.
Absent Achilles tendon re lex
B.
Decreased vibratory sensation in lower extremity
C.
Presence of extensor plantar response
D.
Mild swaying on the Romberg Test
E.
Decreased gait velocity
2. Patient is a 78-year-old woman with a history of ibromyalgia,
panic disorder, diabetes mellitus, and hyperlipidemia is
p yp p

presenting to your of ice for quarterly psychiatric examination


along with her daughter. Patient’s daughter says that in the past 3
months the patient has fallen on ive separate occasions. The
patient reports that she feels most unsteady when standing up
from the table after dinner and when getting out of bed to use the
bathroom at night. On physical examination, the patient’s vital
signs are as follows: 124/86 mm Hg when sitting and 98/64 mm
Hg on standing. She states that her primary physician recently
changed one of her medications but cannot remember which one.
You obtain a list of medications from her pharmacy. Which of the
following medications is most likely to be contributing to her
physical examination indings?
A.
Metformin 500 mg daily
B.
Atorvastatin 40 mg daily
C.
Amitriptyline 50 mg at bedtime
D.
Buspirone 10 mg twice daily
E.
Trazodone 25 mg at bedtime
3. Which of the following would be considered an abnormal inding
in an 83-year-old man with a history of alcohol use disorder in
sustained remission and generalized anxiety disorder?
A.
Decreased gait velocity
B.
Reduced stride length
C.
Gait apraxia
D.
Increased heel contact skid velocity
E.
None of the above
No e o t e above
4.
A 67-year-old man with a history of schizophrenia presents to
your of ice with new onset movement disorder after recently
switching from olanzapine to lurasidone. His neurological
examination is signi icant for stereotyped movements of his
tongue, lips, and jaw. Which of the following is true for the
management of this condition?
A.
Stopping antipsychotic therapy is highly recommended
B.
Movements may worsen as medication doses are lowered
C.
It is recommended to increase dose of antipsychotic to mask
movements
D.
Switching to an antipsychotic with a lower D2 af inity may
reduce movements
E.
Choices B and D
5. A 66-year-old man with a history of hypertension, alcohol use
disorder and generalized anxiety disorder presents to an
appointment with you accompanied by his wife. According to the
patient’s wife, the patient has been having increased frequency of
falls for the past 6 months. Patient’s wife has noticed that he
seems to have dif iculty walking down stairs. His wife has also
noticed that the patient seems more irritable lately. She describes
that he is typically very even tempered, however lately has been
quick to become frustrated or angry. Patient states that he has just
been stressed about inances since his retirement 2 years ago. His
neurological examination is signi icant for diminished facial
movements and increased rigidity of limbs although no tremor is
noted. On examination of his eyes, he is not able to look up or
down on command; however, his eyes are able to move up and
down with lexion and extension of head. Based on his
presentation, what is the most likely diagnosis for this gentleman?
A
A.
Parkinson’s disease
B. Conversion disorder

C.
Lewy body dementia
D.
Progressive supranuclear palsy
E.
Alzheimer’s disease
6. A right-handed 66-year-old man with hypertension,
hyperlipidemia and vascular dementia who presents to your of ice
after being recently hospitalized with a stroke. You ask him to
draw a clock as part of your evaluation. The patient draws a clock
pictured below. Based on the patient’s inability to draw a clock,
where would you most likely see a lesion on the MRI of this man’s
brain?

A.
Left parietal lobe
B.
Right parietal lobe
C.
Ri ht f t ll b
Right frontal lobe

D. Left frontal lobe

E.
Right occipital lobe
7.
A 78-year-old woman with a history of depression and anxiety
presents to your of ice with a 6-week history of numbness and
tingling in her feet. She has fallen twice in the past 6 weeks and
describes feeling off balance. Patient states that she had
previously felt stable on her feet. Her history is also signi icant for
a 10-pound weight loss in the past 6 months. Patient associates
this weight loss with her ill- itting dentures. She states that she
must use her denture cream 6–10 times daily to avoid pain. On
neurological examination, the patient demonstrates hyperre lexia
and spasticity bilaterally. She also exhibits decreased vibratory
sense on plantar surface of feet. Her vitamin B12 and folate levels
are normal. De iciency of which of following would you expect to
ind in this patient?
A.
Thiamine
B.
Zinc
C.
Copper
D.
Thyroid hormones (T4 and T3)
E.
Vitamin D
8. Which of the following tremors is most likely to occur in an
individual with Parkinson’s disease?
A.
A resting tremor that oscillates in a single plane with a
regular rate and variable amplitude
B.
A resting tremor that oscillates in multiple planes with
variable rate and regular amplitude
C. A ine rapid oscillating tremor in a single plane that is
accentuated by action or holding affected limb in a certain
posture
D.
A coarse irregular tremor elicited by movement
E.
A characteristic “wing beating” tremor
9.
A 73-year-old woman with Alzheimer’s disease presents to the
psychiatric emergency department with altered mental status. On
examination, you note that the patient has constricted pupils with
excessive salivation and lacrimation. She is noted to have
decreased re lexes in upper and lower extremities and slurred
speech. Which medication did she did she likely overdose on?
A.
Quetiapine
B.
Donepezil
C.
Diphenhydramine
D.
Memantine hydrochloride
E.
Sertraline
10. Which test on a neurological examination would help isolate a
cerebellar lesion?
A.
Finger-to-nose testing
B.
Alternating movement test
C.
Observing gait
D.
A and B
E. All of the above

Answers
Answer 1C The presence of an extensor plantar response is a sign of
upper motor neuro damage and should never be considered a normal
physical examination inding.

Answer 2C Tricyclic antidepressants (TCAs) are commonly known


to cause orthostasis among other undesirable symptoms. Amitriptyline
is on 2019 BEERS list of medication to avoid in older adults due to its
anti-cholinergic side effects and associated increased risk of falls. If
TCAs must be used, they should be used with caution with careful
monitoring for side effects.

Answer 3C Gait apraxia, also described as a “magnetic” or


“shuf ling” gait , is considered abnormal at any age and is typically
indicative of underlying pathology in the frontal lobe. Decreased gait
velocity, reduced stride length, and increased heel contact skid velocity
are common indings in normal aging. Increased heel contact skid
velocity is the result of decreased and delay hamstring muscle
activation. It can be particularly problematic in an aging population as
it increases risk for falls.

Answer 4E This patient has tardive dyskinesia, a common anti-


psychotic-induced movement disorder. The patient described in the
question is exhibiting the oral-buccal-lingual variety of this condition.
Current evidence suggests that choices B and D are both correct.
According to the dopamine receptor sensitivity hypothesis, it is thought
that creating a D2 blockade using antipsychotic medication makes
dopamine receptors in the striatum “hypersensitive” to the effects of L-
dopa. Thus, reduction in antipsychotic agents may transiently worsen
movement disorder. To the same effect, switching to an antipsychotic
with a lower D2 af inity may mitigate the “hypersensitivity” mediated
by a robust D2 blockade associated with a high D2 af inity
antipsychotic. Stopping the antipsychotic therapy in a patient with
schizophrenia is associated with high rates of relapse and increased
mental health hospitalizations. Increasing the dose of antipsychotic
may transiently reduce symptoms; however, long-term studies have
indicated that it may exacerbate the movement disorder and is not
recommended.

Answer 5D Progressive supranuclear palsy (PSP) is an atypical


parkinsonian disorder associated with vertical supranuclear gaze palsy,
postural instability, and progressive cognitive decline. One key feature
of PSP is preserved vertical movement of eye on lexion and extension
of neck, but inability to move eyes vertically in response to request
made by others. Patients with this condition often struggle with
descending steps due to inability to look downwards. In addition to
motor symptoms, patients also experience cognitive and behavioral
changes due to involvement of the frontal lobe. The most common
behavioral symptoms include apathy, disinhibition, and anxiety.

Answer 6B The patient is demonstrating a hemispatial neglect.


Typically clock drawings by individuals with hemispatial neglect are
characterized by lack of numbers on the left side of the drawing.
Neglect is caused by damage to the non-dominant parietal lobe of the
brain. For a right-handed individual, the non-dominant hemisphere will
most likely be on the right side.

Answer 7C This patient is exhibiting signs of copper de iciency on


neurologic examination. This de iciency was most likely caused by her
excessive use of a zinc containing denture cream. Excessive ingestion of
zinc over time can deplete copper levels in the body. Copper de iciency
can lead to sensory ataxia due to dorsal column involvement. In severe
cases, it can cause degeneration of the spinal cord and spasticity.

Answer 8A A resting tremor that oscillates in a single plane with a


regular rate and variable amplitude describes a parkinsonian tremor. A
ine rapid oscillating tremor in a single plane that is accentuated by
action or holding affected limb in a certain posture describes an
essential tremor. A coarse irregular tremor elicited by movement
describes a cerebellar tremor. A characteristic “wing beating” tremor
describes a tremor brought on by Wilson’s disease.

Answer 9B The patient in the question is experiencing cholinergic


toxicity from the ingestion of donepezil. Cholinergic agents in excess
are commonly known to cause miosis, excessive secretions,
hypotension, and diarrhea. If cholinergic burden is severe, it can even
cause laccid paralysis due to its effects on the peripheral nervous
systems, fatal arrhythmias, and death. Sertraline overdose would cause
serotonin syndrome, which is characterized by brisk re lexes,
mydriasis, and myoclonus. A patient overdosing on diphenhydramine
would present with anti-cholinergic syndrome: dry mucosal surfaces,
urinary retention, elevated heart rate, and mydriasis.

Answer 10E Patients with cerebellar lesions will have abnormal


testing when performing inger-to-nose testing, alternating movements,
and they will have ataxic gait. On inger-to-nose testing and rapid
alternating movements, patient with cerebellar lesions will exhibit
intention tremor and incoordination.

Further Reading
Ahmad J, Hasan MJ, Anam AM, Barua DK. Donepezil: an unusual therapy for acute
diphenhydramine overdose. BMJ Case Rep. 2019;12(3):e226836.
[Crossref]

Arya D, Khan T, Margolius AJ, Fernandez HH. Tardive dyskinesia: treatment update.
Curr Neurol Neurosci Rep. 2019;19(9):69.
[Crossref]

By the 2019 American Geriatrics Society Beers Criteria® Update Expert Panel.
American Geriatrics Society 2019 updated AGS Beers criteria® for potentially
inappropriate medication use in older adults. J Am Geriatr Soc. 2019;67(4):674–94.
[Crossref]

Chen P, Goedert KM. Clock drawing in spatial neglect: a comprehensive analysis of


clock perimeter, placement, and accuracy. J Neuropsychol. 2012;6(2):270–89.
[Crossref]
Crawford P, Zimmerman EE. Tremor: sorting through the differential diagnosis. Am
Fam Physician. 2018;97(3):180–6.
[PubMed]

Galvin JE. Chapter 12. Mental status and neurological examination in older adults. In:
Halter JB, Ouslander JG, Tinetti ME, Studenski S, High KP, Asthana S, Halter JB,
Ouslander JG, Tinetti ME, Studenski S, High KP, Asthana S, et al., editors. Hazzard’s
geriatric medicine and gerontology. 6e ed. New York: McGraw-Hill; 2009. http://
accessmedicine.mhmedical.com/content.aspx?bookid=371& sectionid=41587617.
Accessed 30 Jan 2020.

Greene P. Progressive Supranuclear palsy, Corticobasal degeneration, and multiple


system atrophy. Continuum (Minneap Minn). 2019;25(4):919–35.

Kumar N, Gross JB Jr, Ahlskog JE. Copper de iciency myelopathy produces a clinical
picture like subacute combined degeneration. Neurology. 2004;63(1):33–9.
[Crossref]

Louis ED. Diagnosis and Management of Tremor. Continuum (Minneap Minn).


2016;22(4 Movement Disorders):1143–58.

Marshall FJ. Approach to the elderly patient with gait disturbance. Neurol Clin Pract.
2012;2(2):103–11.
[Crossref]

Pourmand A, Shay C, Redha W, et al. Cholinergic symptoms and QTc prolongation


following donepezil overdose. Am J Emerg Med. 2017;35(9):1386.e1–3.
[Crossref]

Ricciardi L, Pringsheim T, Barnes TRE, et al. Treatment recommendations for tardive


dyskinesia. Can J Psychiatr. 2019;64(6):388–99.
[Crossref]

Schmahmann JD. Disorders of the cerebellum: ataxia, dysmetria of thought, and the
cerebellar cognitive affective syndrome. J Neuropsychiatry Clin Neurosci.
2004;16(3):367–78.
[Crossref]

Sultana J, Spina E, Tri irò G. Antidepressant use in the elderly: the role of
pharmacodynamics and pharmacokinetics in drug safety. Expert Opin Drug Metab
Toxicol. 2015;11(6):883–92.
[Crossref]

Varghese M, Dahale AB. The geropsychiatric interview-assessment and diagnosis.


Indian J Psychiatry. 2018;60(Suppl 3):S301–11.
[PubMed][PubMedCentral]

Viswanathan A, Sudarsky L. Balance and gait problems in the elderly. Handb Clin
Neurol. 2012;103:623–34.
[Crossref]

Wazir SM, Ghobrial I. Copper de iciency, a new triad: anemia, leucopenia, and
myeloneuropathy. J Community Hosp Intern Med Perspect. 2017;7(4):265–8.
[Crossref]
© Springer Nature Switzerland AG 2021
R. Tampi et al. (eds.), Absolute Geriatric Psychiatry Review
https://doi.org/10.1007/978-3-030-58663-8_22

22. Clinical Laboratory Testing


Rosemary Szparagowski1
(1) Department of Psychiatry, University of Pittsburgh Medical Center,
Pittsburgh, PA, USA

Rosemary Szparagowski
Email: szparagowskirl@upmc.edu

Keywords Laboratory testing – Guidelines routine testing –


Medication side effects – Drug monitoring – Pharmacokinetics

Questions
1. A 75-year-old woman with a history of major depressive disorder,
coronary artery disease, and essential hypertension is presenting
to your of ice for evaluation of worsening depression. She has been
off of all psychotropic medication for several years and is interested
in restarting sertraline. You decide to restart her on sertraline. Two
months later after starting sertraline and titrating to a therapeutic
dose, the patient’s husband calls your of ice and reports that the
patient is experiencing nausea, malaise, and has developed an
unsteady gait. There have been no other changes in her medical
history since she last saw you. You immediately send the patient for
laboratory testing. What laboratory inding would you expect to
ind that is related to her use of sertraline?
A.
Hyperkalemia
B.
Hyponatremia
C. Hypernatremia

D.
Hypocalcemia
E.
Hypercalcemia
2.
A 67-year-old man with a history of bipolar I disorder on lithium
presents to your of ice for ongoing medication management. He
complains of breakthrough hypomanic symptoms in recent weeks
and is interested in medication adjustments to treat these
symptoms. He has a recent lithium level from a week ago that is
0.8 meq/l. You decide on starting low-dose olanzapine for
augmentation. What initial and follow-up testing is recommended
for blood glucose monitoring for this gentleman?
A.
Fasting glucose and HbA1c at baseline and annually thereafter
B.
Fasting glucose and HbA1c at baseline and every 6 months
thereafter
C.
Fasting glucose and HbA1c at baseline, at 12 weeks, and
annually thereafter
D.
Fasting glucose and HbA1c at baseline, at 12 weeks, and every
6 months thereafter
E.
None of the above
3. You are evaluating a 65-year-old woman with bipolar II disorder
who has been on maintenance lithium therapy for over 1 year.
According to American Psychiatric Association (APA) practice
guideline, what routine lab testing should be ordered every
6 months?
A.
Lithium level
B. Basic metabolic panel
p

C.
TSH
D.
Lipid panel
E.
A, B, and C
4.
In your clinic you evaluate a 78-year-old African American male
veteran with a history of chronic back pain, hypothyroidism,
obstructive sleep apnea, and post-traumatic stress disorder who
has been stable after completing cognitive processing therapy 10
years ago. He comes to your of ice reporting worsening depression
over the course of the past 6 months. He reports additional
symptoms of fatigue, lightheadedness, and tingling in his hands and
feet. You order a complete blood count (CBC), basic metabolic panel
(BMP), and thyroid-stimulating hormone (TSH). The TSH and BMP
are found to be unremarkable. The CBC is shown below. An
abnormality in which additional lab result(s) would most likely
explain his symptoms?
A.
Decreased serum iron
B.
Elevated serum ferritin
C.
Elevated serum homocysteine
D.
Decreased serum vitamin B12
E.
Choices C and D

Analyte Patient’s value Reference range


Hemoglobin 6.8 12.0–16.0 g/dL
Analyte Patient’s value Reference range
Hematocrit 16 35–47%
RBC 1.3 3.8–5.3 × 106 μL
MCV 134 80–100 fL
MCH 46 26–32 pg
MCHC 33 32–36

5.
An 82-year-old woman who resides at a nursing home presents to
your of ice complaining of weakness and depression. She has a
history of vitamin D de iciency in the past and currently is not on
vitamin D supplementation. So as part of the initial work-up, you
check her vitamin D level. What is the minimum vitamin D serum
level that is recommended among older adults?
A.
10 ng/mL
B.
30 ng/mL
C.
50 ng/mL
D.
75 ng/mL
E.
100 ng/mL
6. Which of the following would be considered a normal age-related
change in thyroid function in an older adult?
A.
Decreased T4
B.
Decreased T3
C.
Decreased TSH
D.
Decreased thyroxine-binding globulin
y gg

E. A and B

7.
An 82-year-old man is brought to your of ice by his family. They
state that for the last 6 months the patient has been losing weight,
has gotten progressively weaker and more forgetful. In the last
year, he has lost 17 pounds and now weighs only 130 pounds. He
becomes quickly fatigued and cannot “get up and go” like he used
to. He is needing signi icantly more assistance with his activities
of daily living including bathing and dressing. Which of the
following should be included in the initial laboratory work-up for
his symptoms?
A.
CBC with differential, BMP, liver function studies (LFTs)
B.
Urinalysis, calcium, phosphate, TSH
C.
CT head without contrast, lumbar puncture, heavy metal
panel
D.
Vitamin B12 and folate levels, albumin, and lipid panel
E.
Choices A, B, and D
8. You evaluate a 66-year-old man with a history of bipolar II
disorder. He has been on a regimen of valproate 1000 mg at
bedtime and mirtazapine 15 mg at bedtime for 4 months. He is
also on metformin and atorvastatin for his type II diabetes and
hyperlipidemia. On routine laboratory work, the patient is found
to have thrombocytopenia. Which medication(s) could be
implicated in the patient’s condition?
A.
Valproate
B.
Metformin
C.
C.
Mirtazapine
D. Atorvastatin

E.
A and C
9.
An 84-year-old woman with a history of depression in sustained
remission presents to your of ice for a routine follow-up
appointment. She notes that she was recently diagnosed with
osteomalacia by her primary care doctor. Which of the following
laboratory abnormalities is consistent with this diagnosis?
A.
Decreased parathyroid hormone (PTH)
B.
Elevated urine calcium
C.
Serum phosphorus high
D.
Elevated alkaline phosphatase
E.
All of the above
10. A 70-year-old man with a history of bipolar I disorder presents to
an emergency department with altered mental status. His family
reports that he has become progressively more confused and
somnolent over the course of the last 3 days. It is noted that he
was recently started on valproate during an inpatient psychiatric
admission for treatment of a manic episode. Which of the
following laboratory tests would reveal the possible reason for his
presentation?
A.
Valproate level
B.
Hepatic function panel
C.
CBC with differential
CBC with differential
D. Serum ammonia

E.
Choices A&D

Answers
Answer 1B Hyponatremia is a commonly known complication of
selective serotonin-reuptake inhibitors in older adults, with the
incidence varying from 0.5% to 32%. This is a potentially life-
threatening complication that should be monitored for when initiating
treatment with SSRIs. Symptoms of hyponatremia are non-speci ic, but
can include fatigue, nausea, lethargy, dizziness, gait disturbance, and
muscle cramps. If sodium concentration falls below 115–120 mEq/L,
patient is at risk of respiratory depression, coma, and death.
Hyperkalemia, hypocalcemia, and hypernatremia are not commonly
associated with the initiation of SSRI. However, these electrolytes
should also be observed in older adults as part of routine monitoring.

Answer 2C According to the American Diabetes Association and the


American Psychiatric Association routine monitoring for patients on
antipsychotic medication should include fasting glucose and HbAa1c at
baseline, again at 12 weeks, and then annually thereafter. It is also
recommended that fasting lipid levels be drawn at baseline, at 12
weeks, and then every 5 years after starting an antipsychotic
medication.

Answer 3E As part of routine monitoring for patients taking lithium


TSH, lithium level, and basic metabolic panel should be drawn every 6–
12 months. Hypothyroidism occurs in 5–35% of patients treated with
lithium. Older adults are at higher risk for lithium toxicity due to
physiologic changes and presence of medical comorbidities; therefore,
it is essential to monitor patients closely for complications. As per APA
guidelines: “In general, renal function should be tested every 2–3
months during the irst 6 months of treatment and thyroid function
should be evaluated once or twice during the irst 6 months of lithium
treatment. Subsequently, renal and thyroid function may be checked
every 6 months to 1 year in stable patients or whenever clinically
indicated (e.g., in the presence of breakthrough affective symptoms,
changes in side effects, or new medical or psychiatric signs or
symptoms).” Lipid panel and HgA1c are not part of routine lithium
monitoring and both would require less frequent routine screening in
non-diabetic patients.

Answer 4E The patient in this question has vitamin B12 de iciency


resulting in pernicious anemia as evidenced by their decreased
hemoglobin and red blood cell count with increased mean corpuscular
volume. Common symptoms of pernicious anemia include lethargy,
apathy, and depression. The patient is also complaining of peripheral
neuropathy, a symptom of vitamin B12 de iciency. Vitamin B12 is an
essential coenzyme in both energy metabolism (the conversion of
methylmalonyl-CoenzymeA to succinyl-coenzymeA) and amino acid
synthesis (the conversion of homocysteine to methionine). When levels
of vitamin B12 are low, persons can be expected to have elevations in
homocysteine and methylmalonic acid in the serum as a result.

Answer 5B Higher rates of depression have been shown to be


associated with lower vitamin D levels. According to the American
Geriatric Society Workgroup on vitamin D supplementation for older
adults, a serum vitamin D level of 30 ng/mL should be a minimum goal
to maintain in older adults, particularly for those who are frail, at risk
for falls, and for injuries or fractures.

Answer 6B Decreased T3 is a common inding in older adults due to


the decreased peripheral deiodination of T4. On the other hand, T4 and
thyroxine-binding globulin remain normal with aging. Of note, the
synthesis of T4 does decline with aging, however, so does its
metabolism; as such, the T4 level remains the same. There is evidence
that TSH increases with age rather than decreasing.

Answer 7E This patient is demonstrating failure to thrive in an


older adult. When evaluating a patient for failure to thrive, it is
important to do a thorough initial work-up to exclude infection, organ
failure, and malignancy. If you have speci ic concerns for an
in lammatory process such as polymyalgia rheumatica or chronic
infection, you should consider ordering erythrocyte sedimentation rate
or C-reactive protein as well. A patient with risk factors for STDs should
also be provided with HIV and syphilis testing. A lipid panel can also
help from a diagnostic standpoint, as low total cholesterol is commonly
found in failure to thrive and can be indicative of nutritional status. A
lumbar puncture, CT of the brain, and heavy metal panel would not be
indicted unless patient has speci ic risk factors.

Answer 8E According to the literature, both valproic acid and


mirtazapine can cause thrombocytopenia, albeit, via different
mechanisms. Valproic acid is known to cause thrombocytopenia via
dose-dependent bone marrow suppression. In patients treated with
valproic acid, the probability of thrombocytopenia increases with dose
and is often mild to moderate in severity. On the other hand,
mirtazapine can cause low platelet counts via drug-induced immune
thrombocytopenia (DITP). DITP is mediated by drug-dependent
autoantibodies. In DITP platelet counts are ussually extremely low and
are decreased within 2 weeks of drug exposure; most patients recover
quickly with cessation of the offending agent.

Answer 9D Elevated alkaline phosphatase (AKP) is a common


inding from increased bone remodeling. Individuals with osteomalacia
have reduced mineral deposits within their bone matrix resulting in
weaker bones and increased risk of fractures. Among individuals with
osteomalacia, the blood calcium levels are often low with low
phosphorus. PTH concentrations are often elevated among these
individuals.

Answer 10E The individual in this vignette has most likely


developed hyperammonemia encephalopathy from valproate. Valproate
disrupts the urea cycle resulting in elevated ammonia levels. Older
adults have been shown to develop hyperammonemia from valproate at
higher rates when compared to younger adults. Because
hyperammonemia is a toxic side effect, it is recommended to check
both an ammonia level and to assess a valproate level in this individual.

Further Reading
American Diabetes Association, American Psychiatric Association, American
Association of Clinical Endocrinologists, North American Association for the Study
of Obesity. Consensus development conference on antipsychotic drugs and obesity
and diabetes. Diabetes Care. 2004;27:596–601.
[Crossref]

Buoli M, Serati M, Botturi A, et al. The risk of thrombocytopenia during valproic acid
therapy: a critical summary of available clinical data. Drugs R D. 2018;18(1):1–5.
[Crossref]

Camilleri L. Lesson of the month 1: sodium valproate-induced encephalopathy. Clin


Med (Lond). 2018;18(5):430–2.
[Crossref]

Fotso Soh J, Klil-Drori S, Rej S. Using lithium in older age bipolar disorder: special
considerations. Drugs Aging. 2019;36(2):147–54.
[Crossref]

Francis RM, Selby PL. Osteomalacia. Bailliere Clin Endocrinol Metab.


1997;11(1):145–63.
[Crossref]

Geriatrics Society Workgroup on Vitamin D Supplementation for Older Adults.


Recommendations abstracted from the American Geriatrics Society consensus
statement on vitamin D for prevention of falls and their consequences. J Am Geriatr
Soc. 2014;62(1):147–52.
[Crossref]

Hirschfeld RMA, Bowden CL, Gitlin MJ, et al. Practice guideline for the treatment of
patients with bipolar disorder. 2nd ed. https://psychiatryonline.org/pb/assets/raw/
sitewide/practice_guidelines/guidelines/bipolar.pdf. Accessed retrieved 30 Jan 2020.

Jacob S, Spinler SA. Hyponatremia associated with selective serotonin-reuptake


inhibitors in older adults. Ann Pharmacother. 2006;40(9):1618–22.
[Crossref]

Papaleontiou M, Haymart MR. Approach to and treatment of thyroid disorders in the


elderly. Med Clin North Am. 2012;96(2):297–310.
[Crossref]

Sarkisian CA, Lachs MS. “Failure to thrive” in older adults. Ann Intern Med.
1996;124(12):1072–8.
[Crossref]

Stuhec M, Alisky J, Malesic I. Mirtazapine associated with drug-related


thrombocytopenia: a case report. J Clin Psychopharmacol. 2014;34(5):662–4.
[Crossref]

Sztajnkrycer MD. Valproic acid toxicity: overview and management. J Toxicol Clin
Toxicol. 2002;40(6):789–801.
[Crossref]

Verdery RB. Clinical evaluation of failure to thrive in older people. Clin Geriatr Med.
1997;13(4):769–78.
[Crossref]

Wolffenbuttel BHR, Wouters HJCM, Heiner-Fokkema MR, et al. The many faces of
cobalamin (vitamin B12) de iciency. Mayo Clin Proc Innov Qual Outcomes.
2019;3(2):200–14.
[Crossref]
© Springer Nature Switzerland AG 2021
R. Tampi et al. (eds.), Absolute Geriatric Psychiatry Review
https://doi.org/10.1007/978-3-030-58663-8_23

23. Structural and Functional Imaging


Kristina Zdanys1
(1) University of Connecticut School of Medicine, Farmington, CT, USA

Kristina Zdanys
Email: zdanys@uchc.edu

Keywords Magnetic resonance imaging (MRI) – Computerized


tomography (CT) – Positron emission tomography (PET) – Single-
photon emission computerized tomography (SPECT) – Gadolinium –
Atrophy

Questions
1. An 82-year-old man presented to the emergency department with
acute onset of right-sided weakness and aphasia. He was last
observed in his usual state of health 1 hour prior. A middle cerebral
artery infarction is suspected. Which imaging study of the head is
the most appropriate next step in work-up?
A.
Magnetic resonance imaging (MRI) without intravenous
contrast
B.
MRI with and without intravenous contrast
C.
Non-contrast computerized tomography (CT)
D.
CT with intravenous contrast
E. Cervicocerebral arteriography
2.
Which of the following brain changes observed on a magnetic
resonance imaging (MRI) scan would be characteristic of
individuals with Alzheimer’s disease?
A.
Asymmetrical brain atrophy
B.
Bilateral, symmetrical atrophy of cerebellum
C.
Occipital lobe atrophy greater than temporal lobe atrophy
D.
Moderate to severe ventricular dilation
E.
Atrophy localized to the bilateral hippocampi
3.
A 78-year-old woman presents to her primary care physician
reporting 6 months of increasing forgetfulness and word- inding
dif iculty. What is the gold-standard brain imaging study the
primary care physician should order irst?
A.
Magnetic resonance imaging (MRI) without intravenous
contrast
B.
MRI with and without intravenous contrast
C.
Non-contrast computerized tomography (CT)
D.
CT with intravenous contrast
E.
Functional MRI without contrast
4. Which of the following is an absolute contraindication for magnetic
resonance imaging (MRI)?
A.
Known gadolinium allergy
B. Cardiac implantable electronic devices, such as a pacemaker

C.
Coronary artery stents
D.
Tattoos
E.
Joint replacement
5.
Which of the following imaging techniques may be useful for early
diagnosis of AD?
A.
Magnetic resonance imaging (MRI) with contrast
B.
Single-photon emission computerized tomography (SPECT)
C.
Positron emission tomography (PET)
D.
All of the above
E.
B and C only

Answers
Answer 1C Non-contrast head CT is the gold-standard imaging to be
ordered for new-onset focal neurological changes with suspected
stroke. It is used to assess for hemorrhage and large infarct. MRI is
more sensitive than CT for acute infarct, but for quick assessment in
order to assess for safety of thrombolytic agents a CT would be
performed irst.

Answer 2D Moderate to severe ventricular dilation is common


among patients with Alzheimer’s disease. Brain atrophy is typically
symmetrical. The cerebellum is usually spared. The occipital lobe
demonstrates atrophy late in the disease. Although there is shrinkage in
the hippocampal region/medial temporal lobe, there is associated
global atrophy as well.

Answer 3A An MRI scan without contrast would be ordered irst for


non-acute work-up of possible neurocognitive disorder.

Answer 4B There is an absolute contraindication for patients with


pacemakers or other cardiac implantable devices to undergo MRI.
Patients with a known gadolinium allergy may obtain MRI without
contrast. Coronary artery stents, tattoos, or joint replacements are
relative contraindications, and patients with these factors should
undergo a cautious evaluation before undergoing MRI.

Answer 5E SPECT and PET are promising potential biomarkers for


AD and may be useful for both early diagnosis and as predictors for
conversion from mild cognitive impairment to AD. Two forms of PET of
interest include beta-amyloid PET imaging and 18F- luorodeoxyglucose
(FDG) PET, which identi ies areas of hypoperfusion and
hypometabolism that have characteristic distribution in AD.

Further Reading
Expert Panel on Neurologic Imaging, Salmela MB, Mortazavi S, Jagadeesan BD, et al.
ACR appropriateness criteria® cerebrovascular disease. J Am Coll Radiol.
2017;14(5S):S34–61.

Ghadimi M, Sapra A. Magnetic resonance imaging (MRI), contraindications.


StatPearls [Internet]. Treasure Island: StatPearls Publishing; 2019–2020.

Kaufman DM, Geyer HL, Milstein MJ. TIAs and strokes. In: Kaufman’s clinical
neurology for psychiatrists. Philadelphia: Elsevier; 2017. p. 235–56.
[Crossref]

Taylor WD, Moore SD, Chin SS. Neuroanatomy, neurophysiology, and neuropathology
of aging. In: Blazer DG, Steffens DC, editors. The American Psychiatric Publishing
textbook of geriatric psychiatry. 4th ed. Washington, DC: American Psychiatric
Publishing; 2009. p. 63–95.
Part V
Treatments
In this part, we review various aspects of the treatment of psychiatric
disorders seen among older adults including pharmacokinetics and
pharmacodynamics, cytochrome (CYP)-450 system, antidepressants,
mood stabilizers, antipsychotics, anxiolytics, hypnotics, cognitive
enhancers, stimulants, drugs for tardive dyskinesia, adverse effects of
drugs, interventional psychiatry, psychotherapy, interdisciplinary
approaches, setting-speci ic treatment and special management
problems.
Pharmacokinetics and pharmacodynamics · Antidepressants · Mood
stabilizers · Antipsychotics · Anxiolytics · Hypnotics · Cognitive
enhancers · Stimulants · Interventional psychiatry · Psychotherapy
Arushi Kapoor
Mara Storto
Meera Balasubramaniam
Emily E. Bay
Kyle Resnick
Rakin Hoq
Rabin Dahal
Rajesh R. Tampi
Kristen Cannon
Edward Singh
Laurel J. Bessey
Deena J. Tampi
© Springer Nature Switzerland AG 2021
R. Tampi et al. (eds.), Absolute Geriatric Psychiatry Review
https://doi.org/10.1007/978-3-030-58663-8_24

24. Pharmacokinetics and


Pharmacodynamics
Arushi Kapoor1
(1) Department of Psychiatry, Perelman School of Medicine at the
University of Pennsylvania, Philadelphia, PA, USA

Keywords Drug metabolism – Absorption – Clearance – Excretion –


Drug-drug interaction – Normal physiological changes of aging

Questions
1. While rounding on the inpatient geriatric psychiatry unit, a
medical student on the team asks, “What are age-associated
changes seen in older adults that impact prescribing psychotropic
medications?” Which of the following is an accurate response to
this question?
A.
Decreased glomerular iltration rate and decreased renal
clearance of active drug metabolites
B.
Increased lean body mass and decreased adipose tissue
C.
Increased glomerular iltration rate and increased renal
clearance of active drug metabolites
D.
Increased hepatic blood low resulting in increased activity of
cytochrome p450 enzymes
E.
Increased lean body mass and increased adipose tissue
2.
Which of the following statements is true regarding the
pharmacokinetics of lipid-soluble psychotropics in older adults?
A.
Lower volume of distribution results in shorter half-lives and
decreased drug accumulation
B.
Higher volume of distribution results in longer half-lives and
increased drug accumulation in the body
C.
Lower volume of distribution results in longer half-lives and
increased drug accumulation in the body
D.
Higher volume of distribution results in shorter half-lives and
decreased drug accumulation
E.
The volume of distribution is typically unaffected in older
adults
3. Which of the following statements is false regarding the
proportionality of dosage to plasma concentration for
antidepressants when used in older adults?
A.
Proportionality of dosage to plasma concentration for
sertraline and citalopram is linear across therapeutic range,
but nonlinear for escitalopram
B.
Proportionality of dosage to plasma concentration for
luvoxamine and luoxetine is nonlinear across therapeutic
range
C.
Proportionality of dosage to plasma concentration for
sertraline, citalopram, and escitalopram is linear across
therapeutic range
D.
Proportionality of dosage to plasma concentration for
luoxetine and paroxetine is nonlinear across the therapeutic
range
E. Proportionality of dosage to plasma concentration for
p y g p
luoxetine is nonlinear across the therapeutic range while that
of escitalopram is linear
4.
Which of the following medications interacts with escitalopram to
increase risk of upper gastrointestinal bleeding?
A.
Haloperidol
B.
Losartan
C.
Ibuprofen
D.
Omeprazole
E.
Aripiprazole
5. A 91-year-old woman with a history of multiple comorbid medical
conditions is referred to you by her primary care provider for
assessment and management of depression. She is currently
taking a calcium-channel blocker, several antidiabetic
medications, and a non-steroidal anti-in lammatory drug for her
in lammatory arthritis. Which one of the following antidepressant
medications has the least likelihood of interacting with any other
of her current medications by changing their blood levels or
ef icacy?
A.
Amitriptyline
B.
Sertraline
C.
Doxepin
D.
Trazodone
E.
Paroxetine
6.
Which of the following statements is false regarding the
pharmacokinetics of antipsychotics in older adults?
A.
Rate of absorption of antipsychotics can be slowed and lag
time to effect can be lengthened by interactions antacids and
anticholinergics
B.
Antipsychotics are highly protein-bound
C.
Due to slower elimination of antipsychotic drugs in older
adults, antipsychotic blood levels for geriatric patients are
regularly recommended for appropriate clinical management
D.
Antipsychotics are highly lipid soluble and thus have a large
volume of distribution and slow elimination
E.
Antipsychotics are largely metabolized by the hepatic system,
which tends to have reduced functioning in older adults
7. Which of the following recommendations regarding the use of
benzodiazepines in older adults is inappropriate?
A.
Benzodiazepines should be avoided for the treatment of
insomnia, agitation, or delirium in older adults
B.
Benzodiazepines may be appropriate for the treatment of
certain disorders including seizure disorders, rapid eye
movement sleep disorders, and end-of-life care
C.
Diazepam is considered to be relatively safe in older adults
when compared to other benzodiazepines and is therefore
the recommended benzodiazepine of choice
D.
Older adults have increased sensitivity to benzodiazepines
and decreased metabolism of longer acting agents
E.
Combination of benzodiazepines and alcohol use may be fatal
in older adults
8.
Which of the following statements is false regarding the
absorption stage of drug pharmacokinetics?
A.
The small intestine is the primary absorption site
B.
Absorption is a passive process, mostly determined by the
size of the dose
C.
In older adults, the rate of absorption can be slowed by
reduced gastric emptying and motility
D.
Antacids can delay absorption of drugs
E.
Due to the effect that proton pump inhibitors have on gastric
pH, they can affect the absorption of certain drugs
9.
Which of the following statements is false regarding the
metabolism stage of drug pharmacokinetics?
A.
Drug metabolism occurs by Phase 1 oxidation and/or Phase II
conjugation processes
B.
Oxidation reactions are unaffected by aging, whereas
conjugation is affected by aging
C.
Genetic polymorphisms and CYP inhibitors and inducers can
alter an individual’s rate for metabolism of drugs
D.
Medications metabolized primarily through conjugation are
less affected by normal aging
E.
With increasing age, there is a decrease in liver size and
hepatic enzyme activity
10. Which of the following is false regarding the pharmacokinetics
and pharmacodynamics of psychotropic medications among older
adults?
adults?
A.
Older adults are more sensitive to adverse effects of
psychotropics even at lower concentrations
B. Homeostatic mechanisms such as postural control, water
balance, orthostatic circulatory responses, and
thermoregulation are frequently unchanged with aging

C.
Reductions in dopamine or acetylcholine function with age
may increase sensitivity to antipsychotics and SSRIs
D.
Age-associated decline in renal clearance may affect excretion
of psychotropic drug metabolites and lithium
E.
Age-associated hepatic changes affect drug metabolism in the
elderly

Answers
Answer 1A Increased susceptibility to adverse effects in older
adults may be a result of the pharmacokinetic changes associated with
aging, such as diminished glomerular iltration leading to decreased
renal clearance of active metabolites. Other signi icant changes include
decreased liver size and decreased hepatic blood low with variable
effects on cytochrome P450 enzyme leading to decreased hepatic
clearance. With physiological changes in aging, lean body mass
decreases while there is an increase in adipose tissue altering the
volume of distribution of lipid-soluble drugs and subsequently leading
to an increased elimination half-life.

Answer 2B For most psychotropics that are lipid soluble, the loss of
lean body mass with aging will lead to increases in their volume of
distribution, resulting in longer half-lives and drug accumulation. This
is because a drug’s half-life is directly proportional to its apparent
volume of distribution. Conversely, for water-soluble drugs such as
lithium and digoxin, volumes of distribution will be diminished in older
patients, reducing the margin of safety after acute increases in plasma
drug concentration.

Answer 3A Sertraline, citalopram, and escitalopram have a linear


dosage to plasma concentration across therapeutic range and a low risk
of discontinuation symptoms. Fluoxetine, luvoxamine, and paroxetine
do not have a linear dose to plasma concentration. Available data from
head-to-head randomized comparisons indicate that all SSRIs currently
available have similar ef icacy and tolerability in the treatment of
depression.

Answer 4C SSRIs may directly affect platelet activation, and they act
synergistically with other medications that increase the risk of
gastrointestinal or post-surgical bleeding. Both selective serotonin
reuptake inhibitors (SSRIs) and non-steroidal anti-in lammatory drugs,
including Ibuprofen, ketorolac, indomethacin, individually increase the
risk of upper GI bleeding. Concomitant use increases the cumulative
risk and should be monitored closely. SSRIs and beta-blockers can
interact together to result in bradycardia. Simvastatin and losartan do
not have known clinically signi icant interactions. Omeprazole is a
potent CYP450 2C19 inhibitor and can theoretically increase
escitalopram levels.

Answer 5B Sertraline, along with citalopram and escitalopram, has


the lowest pro ile of cytochrome P450 induction, inhibition, or
competition among the SSRI antidepressants. Fluoxetine, paroxetine,
and luvoxamine are much more likely to interact in major ways with
other commonly prescribed medications due to their interaction with
cytochrome P450 enzymes. In several randomized controlled trials
assessing the ef icacy of SSRIs for acute treatment of geriatric
depression, sertraline was found to be more ef icacious than placebo
and as ef icacious as amitriptyline, luoxetine, luvoxamine, and other
SSRIs.

Answer 6C Antipsychotics are generally well absorbed from the


gastrointestinal tract, but their rate of absorption may be affected by
antacids and anticholinergic medications. With a few exceptions,
therapeutic antipsychotic blood levels for older adults are generally not
well established and therefore are not a regularly used tool in clinical
management. Antipsychotics are highly protein-bound, though the age-
related changes in protein binding do not have as much clinical
importance as the decreases in hepatic function and renal clearance
associated with aging. Bioavailability of most low-potency
antipsychotics (chlorpromazine, thioridazine, clozapine, and
quetiapine) is variable, so dosing ranges are wide and no single
“correct” dose is associated with an optimal reference. Antipsychotic
drugs are also highly lipid soluble leading to a large volume of
distribution, slow elimination, and high concentrations in the brain
relative to blood in older adults. Measurable antipsychotic drug
metabolites can persist in urine for as long as 3 months after
discontinuation of the drug.

Answer 7C According to The Beers Criteria for Potentially


Inappropriate Medication Use, published by the American Geriatrics
Society, benzodiazepines should be avoided for the treatment of
insomnia, agitation , or delirium among older adults. All
benzodiazepines increase the risk of cognitive impairment, falls,
fractures, and motor vehicle accident (MVA) in older adults. Diazepam,
which is a long-acting benzodiazepine, is highly lipophilic and moves
rapidly into fat storage sites, which can result in signi icant
accumulation (due to higher ratios of adipose tissue in older adults)
therefore incurring higher risk of adverse effect.

Answer 8B A drug taken orally passes from the stomach to the


proximal small intestine, where most of the absorption takes place.
Absorption is an active process that is regulated by the cells lining the
gut (enterocytes). These enterocytes express the drug metabolizing
enzyme cytochrome P450 (CYP450) 3A4 and P-glycoprotein – both
assist with regulating drug absorption. With aging, the rate of
absorption can be slowed by age-related slowing in gastric motility or
delay in gastric emptying. In the absence of overt diseases, the extent of
absorption is relatively unaffected with aging. Consumption of antacids
or cathartics containing aluminum, magnesium, or calcium can delay
absorption. Proton pump inhibitors have the potential to interfere with
absorption of certain drugs because of effects on gastric pH.
Answer 9B The principal drug-metabolizing enzymes include the
Phase 1 cytochrome p450 (CYP) enzymes and Phase II uridine 5′-
diphosphate glucuronosyltransferase (UGT) for glucuronidation. While
oxidation reactions of several CYP enzymes are signi icantly affected by
aging, conjugation reactions (acetylation and methylation and
glucuronidation) are unaffected by normal aging. Psychotropics
metabolized primarily by glucuronidation include lorazepam,
temazepam, oxazepam, valproate, lamotrigine, morphine, and codeine.
Aging effects on the liver include a gradual reduction in liver mass,
reduced blood low, and reduced hepatic metabolic rate. Currently used
“liver function tests” poorly correlate with drug metabolizing capacity.

Answer 10B Interindividual differences in pharmacodynamics


become evident when those with similar plasma drug concentrations
experience different effects. In general, older individuals are more
sensitive to adverse effects of psychotropics at lower concentrations.
Homeostatic mechanisms such as postural control, water balance,
orthostatic circulatory responses, and thermoregulation are frequently
less robust in the aged and may interfere with the ability to
physiologically adapt to a medication. Reductions in dopamine or
acetylcholine function with age may increase sensitivity to
antipsychotics and SSRIs (which indirectly reduce dopamine out low)
as well as medications with antimuscarinic effects. Hepatic and renal
functions tend to have age-related reductions resulting in changes in
hepatic metabolism and renal clearance.

Further Reading
Alexopoulos GS, Streim J, Carpenter D, et al. Using antipsychotic drugs in older
patients. J Clin Psychiatry. 2004;65(Suppl 2):5–99.
[PubMed]

Andres TM, McGrane T, et al. Geriatric pharmacology: an update. Anesthesiol Clin.


2019;37(3):475–92.
[Crossref]

By the 2019 American Geriatrics Society Beers Criteria® Update Expert Panel.
American Geriatrics Society 2019 updated AGS beers criteria® for potentially
inappropriate medication use in older adults. J Am Geriatr Soc. 2019;67(4):674–94.
[Crossref]

Dalton SO, Johansen C, Mellemkjaer L, et al. Use of selective serotonin reuptake


inhibitors and risk of upper gastrointestinal tract bleeding: a population-based
cohort study. Arch Intern Med. 2003;163(1):59–64.
[Crossref]

English BA, Dortch M, Ereshefsky L, Jhee S. Clinically signi icant psychotropic drug-
drug interactions in the primary care setting. Curr Psychiatry Rep. 2012;14(4):376–
90.
[Crossref]

Ereshefsky L. Pharmacokinetics and drug interactions: update for new


antipsychotics. J Clin Psychiatry. 1996;57(Suppl 11):12–25.
[PubMed]

Graff-Guerrero A, Rajji TK, Mulsant BH, et al. Evaluation of antipsychotic dose


reduction in late-life schizophrenia: a prospective dopamine D2/3 receptor
occupancy study. JAMA Psychiat. 2015;72(9):927–34.
[Crossref]

Klotz U. Pharmacokinetics and drug metabolism in the elderly. Drug Metab Rev.
2009;41(2):67–76.
[Crossref]

Lotrich FE, Pollock BG. Aging and clinical pharmacology: implications for
antidepressants. J Clin Pharmacol. 2005;45(10):1106–22.
[Crossref]

Mangoni AA, Jackson SH. Age-related changes in pharmacokinetics and


pharmacodynamics: basic principles and practical applications. Br J Clin Pharmacol.
2004;57(1):6–14.
[Crossref]

Mulsant BH, Blumberger DM, Ismail Z, et al. A systematic approach to the


pharmacotherapy of geriatric major depression. Clin Geriatr Med. 2014;30(3):517–
34.
[Crossref]

Probst-Schendzielorz K, Viviani R, Stingl JC. Effect of cytochrome P450


polymorphism on the action and metabolism of selective serotonin reuptake
inhibitors. Expert Opin Drug Metab Toxicol. 2015;11(8):1219–32.
[Crossref]

Steffens D, Blazer D, Thakur M. The American Psychiatric Publishing textbook of


geriatric psychiatry. 5th ed. Arlington: American Psychiatric Publishing, Inc.; 2015. p.
529–30.
© Springer Nature Switzerland AG 2021
R. Tampi et al. (eds.), Absolute Geriatric Psychiatry Review
https://doi.org/10.1007/978-3-030-58663-8_25

25. Cytochrome P450


Mara Storto1 and Meera Balasubramaniam1
(1) Department of Psychiatry, NYU Grossman School of Medicine, New
York, NY, USA

Mara Storto
Email: mara.storto@nyulangone.org

Meera Balasubramaniam (Corresponding author)


Email: meera.balasubramaniam@nyulangone.org

Keywords Cytochrome (CYP)P450 – Enzyme inhibition – Enzyme


induction – Drug-drug interactions – Drug metabolism – Adverse
effects – Drug toxicity

Questions
1. Which of the following antidepressant medications has the greatest
risk of causing signi icant drug-drug interactions due to overall
CYP450 enzyme inhibition?
A.
Bupropion
B.
Sertraline
C.
Duloxetine
D.
Mirtazapine
E. Fluoxetine
2.
Ms. Jones is an 82-year-old woman who presents with persistent
depressive symptoms following a trial with an SSRI. She requests to
try an SNRI, speci ically venlafaxine, which was recommended to
her by a peer. You order a genetic test that reveals that she has a
genetic de iciency causing decreased activity of CYP2D6, which is
the enzyme primarily responsible for metabolism of both
venlafaxine and duloxetine. What would be most important to
consider if you do proceed with prescribing venlafaxine, given her
genetic de iciency?
A.
Need for higher dosing
B.
Increased risk of side effects
C.
Decreased potential for therapeutic effect
D.
Higher likelihood for drug-to-drug interaction with this
medication
E.
Potential worsening of her depression
3. When a 75-year-old man with a history of chronic pain syndrome
who is being treated with codeine by a pain specialist is started on
luoxetine for depressive symptoms, what is most likely to happen
in this individual?
A.
The severity of his depression will get worse
B.
The severity of his pain will get better
C.
There will be no change in his severity of pain
D.
The severity of his pain will worsen
E Ch i A dB
E. Choices A and B

4.
Mr. Smith is a 68-year-old man with a diagnosis of schizophrenia
has been stable on clozapine 300 mg orally twice a day for the past
15 years. He has a remote history of tobacco use but has not
smoked in the past 10 years. He comes to your of ice and states that
since his last appointment 3 months ago, he has started smoking
again. He currently smokes about 1.5 packs of cigarettes per day. He
is not interested in quitting smoking at this time. What would be
the safest recommendation to make regarding the patient’s
clozapine regimen to reduce risk of decompensation and/or
toxicity?
A.
Decrease current dose and monitor clozapine level
B.
Increase current dose and monitor clozapine level
C.
Discontinue medication entirely, as clozapine is
contraindicated among individuals who are actively smoking
cigarettes
D.
Discontinue medication entirely, as clozapine will be ineffective
among individuals who are actively smoking cigarettes
E.
There is no indication for medication change
5. Carbamazepine is predominantly metabolized by the CYP3A4
isoenzymes. Additionally, the rate of action of CYP3A4 isoenzymes
decreases with age. Based on this information, which of the
following side effects would you be least worried about in an older
adult taking carbamazepine for mood stabilization?
A.
Increased sedation
B.
Ataxia
C. Hyponatremia

D.
Thrombocytopenia
E.
Rash

Answers
Answer 1E Fluoxetine strongly inhibits four major cytochrome P450
isozymes: CYP1A2, CYP2C9/2C19, CYP2D6, and CYP3A4. Fluoxetine
interacts with a higher number of CYP450 enzyme systems compared
to other antidepressant drugs listed. Drug metabolizing enzymes can
also be affected by aging either by post-translational modi ications or
by transcriptional modi ications. As a result, it is important to be aware
of the ways in which other prescribed medications are metabolized in
older individuals on luoxetine, as this population already has overall
decreased CYP450 enzymatic activity. Of the medications listed,
mirtazapine has been seen to have the least signi icant inhibition of
CYP450 enzymes.

Answer 2B Venlafaxine and duloxetine (SNRI drugs) are


metabolized by CYP2D6. Their levels will increase in patients who have
poor enzymatic activity of CYP2D6 or are taking another medication
that inhibits this enzyme. Therefore, in the case of this patient, you
would expect higher plasma levels of the drug that could increase risk
of venlafaxine toxicity. Since decreased CYP2D6 activity would likely
lead to accumulation of the drug, it would require lower dosing if
prescribed and not higher dosing. This also would not be likely to
decrease the potential for therapeutic effect, but moreover would
increase the risk of adverse effect. Decreased enzymatic activity would
also not affect the drug’s potential for drug-to-drug interaction or
worsen her depression.
Answer 3D Codeine is metabolized into morphine via the
cytochrome P450 2D6 isoenzymes (CYP2D6). The clinical effect of
codeine depends largely on CYP2D6 activity. This activity is impaired by
SSRI and SNRI antidepressant that are moderate or strong CYP2D6
inhibitors. Fluoxetine and paroxetine are considered strong CYP2D6
inhibitors with sertraline being considered a moderate inhibitor.
Among SNRIs, duloxetine is considered as a moderate CYP2D6
inhibitor. When codeine is co-prescribed with antidepressants that are
moderate to strong CYP2D6 inhibitors, it will lead to impaired pain
management. The dose of codeine may need to up titrated to maintain
good pain control.

Answer 4B Cigarette use has been shown to induce hepatic


cytochrome P450 enzymes, due to the action of polycyclic aromatic
hydrocarbons (PAH) in tobacco smoke. In particular, there is signi icant
evidence that PAH induces CYP1A1, CYP1A2, and CYP2E1. Clozapine is
primarily metabolized by CYP1A2 and is therefore metabolized at a
higher rate in patients who are active smokers. Therefore, in this
individual who has recently started smoking again, his clozapine levels
are expected to go down from the level that previously kept his
psychotic symptoms in remission. In this case, the recommendation
would be for clozapine to be titrated with close monitoring of the
clozapine level in order to avoid acute decompensation. Similarly,
olanzapine is also metabolized by CYP1A2 and dose adjustments may
need to be made for patients who have signi icant tobacco use.

Answer 5D Carbamazepine is primarily metabolized by CYP3A4.


Therefore, there is higher potential for carbamazepine toxicity if an
individual is also taking a medication that exhibits CYP3A4 inhibition
such as macrolide antibiotics, antifungals, and protease inhibitors.
Additionally, CYP3A4 activity is decreased with aging. As a result, it is
important to monitor for toxicity of medications metabolized by this
enzyme in the older adult population. Initial signs of increased plasma
levels of carbamazepine are increased sedation, ataxia, and rash. Older
adults should also be monitored for less common, but life-threatening
side effects of carbamazepine toxicity such as hyponatremia hepatitis
and cardiac conduction abnormalities. Thrombocytopenia has not been
shown to be a potential side effect of carbamazepine toxicity but is
noted to be a serious adverse effect of valproic acid.

Further Reading
Cazet L, Bulteau S, Evin A, et al. Interaction between CYP2D6 inhibitor
antidepressants and codeine: is this relevant? Expert Opin Drug Metab Toxicol.
2018;14(8):879–86.
[Crossref]

Crews KR, Gaedigk A, Dunnenberger HM, et al. Clinical Pharmacogenetics


Implementation Consortium guidelines for cytochrome P450 2D6 genotype and
codeine therapy: 2014 update. Clin Pharmacol Ther. 2014;95(4):376–82.
[Crossref]

Fricke-Galindo I, LLerena A, Jung-Cook H, Ló pez-Ló pez M. Carbamazepine adverse


drug reactions. Expert Rev Clin Pharmacol. 2018;11(7):705–18.
[Crossref]

Gross T, Daniel J. Overview of pharmacogenomic testing in clinical practice. Ment


Health Clin. 2018;8(5):235–41.
[Crossref]

Horstmann S, Binder EB. Pharmacogenomics of antidepressant drugs. Pharmacol


Ther. 2009;124(1):57–73.
[Crossref]

Kennedy WK, Jann MW, Kutscher EC. Clinically signi icant drug interactions with
atypical antipsychotics. CNS Drugs. 2013;27(12):1021–48.
[Crossref]

Lowe EJ, Ackman ML. Impact of tobacco smoking cessation on stable clozapine or
olanzapine treatment. Ann Pharmacother. 2010;44(4):727–32.
[Crossref]

Lozupone M, Panza F, Stella E, et al. Pharmacogenetics of neurological and


psychiatric diseases at older age: has the time come? Expert Opin Drug Metab
Toxicol. 2017;13(3):259–77.
[Crossref]
© Springer Nature Switzerland AG 2021
R. Tampi et al. (eds.), Absolute Geriatric Psychiatry Review
https://doi.org/10.1007/978-3-030-58663-8_26

26. Antidepressants
Emily E. Bay1
(1) Medical University of South Carolina, Charleston, SC, USA

Emily E. Bay
Email: baye@musc.edu

Keywords Selective serotonin reuptake inhibitor (SSRI) – Serotonin-


norepinephrine reuptake inhibitor (SNRI) – Tricyclic antidepressant
(TCA) – Monoamine oxidase inhibitor (MAOI) – Atypical
antidepressants – Electroconvulsive therapy (ECT) – Depression –
Anxiety – Behavioral and psychological symptoms of dementia

Questions
1. Which of the following statements correctly re lects the current
data regarding the relationship between antidepressants and
management of strokes?
A.
The use of antidepressants after stroke for both depressed
and non-depressed people has been associated with
worsening cognitive functioning
B.
The use of antidepressants has been shown to increase the
risk of recurrent stroke in all people who have had a stroke
C.
The use of antidepressants in non-depressed people who
have had stroke has been associated with lengthened
recovery time and increased disability
D. The use of antidepressants has only been shown to be helpful
in stroke recovery for those who develop post-stroke
depression
E.
The use of antidepressants in non-depressed people who
have suffered stroke has been associated with reduced
recovery time and reduced disability

2.
Which of the following antidepressants has the shortest half-life?
A.
Paroxetine
B.
Fluoxetine
C.
Venlafaxine
D.
Sertraline
E.
Bupropion
3. An 82-year-old woman with Alzheimer’s dementia presents with
her son for worsening agitation at home over the last 2 months.
She is currently only prescribed memantine. Underlying medical
issues that could cause his agitation have been ruled out. Which of
the following medication options would be the safest and still
effective medication to treat his behavioral symptoms?
A.
Citalopram
B.
Lorazepam
C.
Haloperidol
D.
Diphenhydramine
E.
Amitriptyline
4.

Which of the following accurately describes the effect that SSRIs


have on sleep architecture?
A.
Decreased sleep latency and increased slow-wave sleep
B.
Rapid-eye movement (REM) suppression and increased REM
latency
C.
Increased sleep latency and increased REM latency
D.
Increased sleep latency and REM suppression
E.
Decreased sleep latency, REM suppression, and increased
REM latency
5. A 70-year-old man with a history of coronary artery disease and
hypertension presents to the Emergency Department with
excessive fatigue in the last 2 days. An EKG is obtained and he is
found to be in sinus bradycardia with a heart rate in the mid
30/minute. The patient’s caregiver notes that the patient was
recently started a new medication for depression a few days ago
but he does not recall its name. The caregiver does remember that
the patient is taking metoprolol to help manage his blood
pressure. Which medication is likely contributing to this patient’s
presentation?
A.
Aripiprazole
B.
Mirtazapine
C.
Fluoxetine
D.
Duloxetine
E.
Venlafaxine
Venlafaxine
6.
Which antidepressant should be avoided in patients with a history
of stroke?
A.
Bupropion
B.
Sertraline
C.
Mirtazapine
D.
Duloxetine
E.
Trazodone

7.
A 72-year-old man with a history of severe depression who had
achieved remission with amitriptyline presents for follow-up. In
the last several months his family reports he has been increasingly
depressed, not sleeping, and has lost over 30 pounds due to not
eating. He appears malnourished, withdrawn, and his responses
are greatly delayed. A medical work-up by his primary care
physician is unremarkable. What is the next best course of action?
A.
Increase amitriptyline
B.
Start risperidone
C.
Augment with bupropion
D.
Start electroconvulsive therapy (ECT)
E.
Augment with mirtazapine
8. Which of the following classes of medication has been shown to
have association with accelerated bone loss in post-menopausal
women?
A.
Norepinephrine-dopamine reuptake inhibitors
B.
Acetylcholinesterase inhibitors

C. NMDA-receptor antagonists

D.
Benzodiazepines
E.
Selective-serotonin reuptake inhibitors
9.
A 70-year-old woman with a history of a right middle cerebral
artery stroke 2 years ago (thought to be related to poorly
controlled hypertension and atherosclerosis) is brought in by her
daughter for worsening depressive symptoms in the last
6 months. She has signi icant disability from her stroke and now
lives in a nursing facility. She has gained signi icant weight due to
her physical imitations from her stroke. She is morbidly obese and
spends most of her day in bed. She is also noted to be
hypertensive at 150/70 but her daughter reports her amlodipine
was just increased to 10 mg at her primary care appointment
earlier today. What antidepressant would be most appropriate for
this patient?
A.
Bupropion
B.
Mirtazapine
C.
Sertraline
D.
Venlafaxine
E.
Trazodone
10. Which of the following statements is true regarding the
t h lth ff t f tid t i ld d lt ?
concurrent health effects of antidepressant use in older adults?
A.
The current use of antidepressants has been associated with a
decreased risk of falls
B. The use of antidepressants has been associated with
decreased risk of intracranial hemorrhage

C.
The use of antidepressants does not increase risk of cardiac
arrhythmias

D.
The use of antidepressants does not have a known association
with causing confusion in older adults
E.
The use of antidepressants has been associated with
decreased risk of myocardial infarction
11.
A 67-year-old woman with a history of pancreatic cancer treated
with resection and chemotherapy presents to you with worsening
depressive symptoms. She states she constantly feels nauseated
and has also lost a signi icant amount of weight since her surgery.
Which of the following medications could potentially alleviate
both her depressive symptoms and nausea?
A.
Sertraline
B.
Mirtazapine
C.
Bupropion
D.
Venlafaxine
E.
Fluoxetine
12. A 74-year-old man with a history of major depression successfully
treated in his 40s with tricyclic antidepressants presents for
recurrent depressive symptoms. He has recently been treated
ecu e t dep ess ve sy pto s e as ece t y bee t eated
with an SSRI followed by SNRI without symptomatic
improvement on either trial. He requests a trial of another
tricyclic antidepressant as this worked well in the past but he
does not recall which medication he was prescribed. You discuss
risks and bene its associated with this class of medication, and he
resolves to still go through with a medication trial but requests

the one with the least associated side effects. Which of the
following would be the best recommendation for him?
A.
Amitriptyline
B.
Nortriptyline
C.
Doxepin
D.
Clomipramine
E.
Imipramine
13.
By which mechanism does selegiline transdermal patch reduce
the risk for hypertensive crisis associated with monoamine
oxidase inhibitor (MAOI) drugs?
A.
Increased MAO inhibition in the gastrointestinal tract
B.
Decreased MAO inhibition in the central nervous system
C.
Decreased MAO inhibition in the gastrointestinal tract
D.
Increased MAO inhibition in the central nervous system
E.
Bypass of irst-pass metabolism results in overall less potent
MAO inhibition
14. A 70-year-old man with history of atrial ibrillation presents to
f t t t fd i Hi EKG f li thi k
you for treatment of depression. His EKG from earlier this week
shows a QTc of 480 m/s. Which of the following antidepressants
should be avoided for this patient?
A.
Citalopram
B.
Duloxetine

C. Sertraline

D.
Paroxetine
E.
Fluvoxamine
15. A 70-year-old woman with a history of hypertension, stroke, and
mild cognitive impairment presents to you for persistent
depressive symptoms including mild anhedonia and frequent
insomnia. She is unable to tolerate the gastrointestinal side effects
of SSRIs and was signi icantly hypertensive when taking
venlafaxine. She is currently prescribed mirtazapine 45 mg, which
has been helpful for her depression but she continues to endorse
insomnia. She reports taking trazodone 200 mg in the past, which
helped her sleep but only modestly improved her mood. Which is
the best course of action for the treatment of the patient’s
symptoms?
A.
Discontinue mirtazapine and start duloxetine 20 mg daily
B.
Continue mirtazapine and titrate to 60 mg
C.
Retrial with adjunctive trazodone and consider dose titration
to or above her previous dosing
D.
Start adjunctive bupropion 150 mg. daily
E.
Start adjunctive quetiapine
16.
A 75-year-old female with a past medical history of hypertension
presents to the emergency department with her husband due to
concern for increasing confusion and somnolence in past 2 days.
Head imaging obtained was unremarkable; however, her sodium
is noted to be 121. Her husband recalls that she recently started a
new medication for depression. Which of the following
medications is most likely contributing to this patient’s
presentation?
A.
Nortriptyline
B.
Mirtazapine
C.
Paroxetine
D.
Clomipramine
E.
Lurasidone

17.
Which of the following medications has been shown in a
randomized controlled trial to have bene it in treating behavioral
symptoms associated with frontotemporal dementia?
A.
Bupropion
B.
Trazodone
C.
Duloxetine
D.
Escitalopram
E.
Mirtazapine
Antidepressants have been associated with which of the following
18. Antidepressants have been associated with which of the following
ocular complications?
A.
Acute closed angle glaucoma
B.
Abnormal color perception
C.
Retinal detachment
D. Cataracts

E.
Impairment of eye movements
19.
The risk of hyponatremia from SSRIs increases with concomitant
use of which of the following medications?
A.
Antacids
B.
Levothyroxine
C.
Oxycodone
D.
Amlodipine
E.
Insulin
20. Which of the following factors is associated with increased risk of
falls in older adults taking SSRIs?
A.
High dosages
B.
Low dosages
C.
Use >2 years
D
D.
Use <2 years
E.
There is no association between SSRIs and increased fall risk

Answers
Answer 1E Depression following a stroke is a well-recognized
phenomenon that has led to the extensive study of the role of
antidepressants in stroke management. There is now emerging
literature supporting that antidepressants can be bene icial in
shortening stroke recovery and decreasing disability for both
depressed and non-depressed subjects, particularly.

Answer 2C Of the antidepressants listed, venlafaxine has the


shortest half-life of about 5 hours, making withdrawal symptoms very
common and often severe when doses are missed or the medicine is
discontinued abruptly. Due to the short half-life, venlafaxine is
frequently prescribed in an extended-release formulation to both
simplify dosing and to reduce incidence of withdrawal symptoms, but
still risk of withdrawal remains high if doses are missed.

Antidepressant Mechanism Half-life


Paroxetine 5-HT reuptake inhibitor 18–24 hours
Fluoxetine 5-HT reuptake inhibitor 24–96 hours
Venlafaxine 5-HT, NE reuptake inhibitor ~5 hours
Sertraline 5-HT reuptake inhibitor 24–36 hours
Bupropion NE, DA reuptake inhibitor ~12 hours

5-HT serotonin, NE norepinephrine, DA dopamine

Answer 3A The CitAD trial in 2014 was a randomized, placebo-


controlled, double-blind, parallel group trial that enrolled 186 patients
with probable Alzheimer disease and clinically signi icant agitation in
Canada and the United States. Participants who received citalopram
showed signi icant decrease in agitation and caregiver distress when
compared to placebo. Of the treatment options listed, an SSRI such as
citalopram would be the safest treatment option for an older adult.
Although antipsychotics and benzodiazepines have been studied for
treating behavioral symptoms of dementia, they carry signi icant side-
effect burden and would not be preferable treatment option. Tricyclic
antidepressants also carry signi icantly more side effect burden when
compared to SSRIs and therefore would not be a preferable irst-choice
treatment. Antihistamines should be avoided when possible in older
adults with cognitive impairment as they can increase risk of confusion
and falls.

Answer 4B SSRIs affect sleep architecture by suppressing REM


stages and increasing REM latency.

Medication Mechanism affecting Effect on sleep


class sleep
SSRI/SNRI 5-HT reuptake inhibition REM suppression, increased REM
latency
Trazodone, 5-HT2 antagonism Decreased sleep latency, increased
mirtazapine SWS
Tricyclic 5-HT, NE reuptake Decreased sleep latency, REM
antidepressants inhibition, H 1 antagonism suppression, increased REM latency

5-HT serotonin, NE norepinephrine, H histamine, SWS slow-wave sleep,


REM rapid-eye movement

Answer 5C SSRIs, SNRIs, and other second-generation


antidepressants such as trazodone are metabolized by the cytochrome
P450 system. Of the SSRIs, the three most commonly implicated in
causing adverse medication interactions are luoxetine and paroxetine
(due to both being strong CYP2D6 inhibitors) in addition to
luvoxamine (a strong CYP1A2, CYP2C9, and CYP2C19 inhibitor). This
patient is taking metoprolol, which is metabolized by the CYP2D6
enzyme. So due to CYP2D6 inhibition by their antidepressant (among
the choices luoxetine is the most likely culprit), she is having
accumulation of metoprolol causing a severe bradycardia.
Answer 6A Strokes increase seizure risk because ischemic or
necrotic cerebral tissue can become epileptogenic. While there is not an
absolute contraindication against using bupropion in the post-stroke
population, the FDA still warns against using bupropion in post-stroke
patients due to potential for further decreasing seizure threshold and
possibly inducing seizures; therefore, it is recommended it be avoided
when possible. The other medications listed do not have known
adverse effects speci ic to stroke that would contraindicate them for
post-stroke patients.

Answer 7D This patient is presenting with a severe decompensation


in depressive symptoms, which is strongly deduced by his severe
weight loss and malnourished presentation. Given his degree of decline,
this patient should be considered for electroconvulsive therapy (ECT)
due to the high ef icacy and rapid onset of therapeutic effect. Increasing
amitriptyline is unlikely to be bene icial as he had previously achieved
remission at his current dosage. This patient is not experiencing
psychotic symptoms making risperidone a less viable option. The
severity and acuity of his current depressive state makes addition of a
second antidepressant a less preferable option due to delay in
therapeutic effect.

Answer 8E The use of serotonergic antidepressants, particularly


SSRIs, have been associated with accelerated bone loss in post-
menopausal women when compared to controls who were not taking
antidepressants. The mechanism by which this accelerated bone loss
occurs is not yet clear, but it is established that serotonin has a
regulatory effect on the osteoblast cell line. The other medications in
the answer choices do not have known associations with accelerated
bone loss.

Answer 9C SSRIs have proven ef icacy in geriatric patients with a


history of stroke or vascular disease in general. This patient’s obesity
makes mirtazapine a less suitable option due to high association with
weight gain. She is also already bedridden most of the day, and
additional sedation from mirtazapine could potentially limit her
mobility further. Because she has hypertension, venlafaxine is a less
suitable option due to the possibility of dose-dependent increases in
systolic blood pressure. Bupropion is not recommended in post-stroke
patients as they have higher seizure risk, and bupropion can further
lower seizure threshold. Trazodone is not considered irst line for
depression.

Answer 10E Antidepressants have been associated with a


cardioprotective effect, particularly in association with a decreased risk
of myocardial infarction in older adults. This effect also appears to be
more speci ic to serotonergic antidepressants (SSRIs). One theory as to
why there is this association is because of how SSRIs are associated
with decreased platelet activation and aggregation. Unfortunately, it is
also likely due to that same mechanism that there have been
associations between antidepressant use and the increased risk of
intracranial hemorrhage.

Answer 11B Mirtazapine is a unique antidepressant that works


through alpha-2 receptor antagonism, selective stimulation of 5-HT-1
receptors, and selective inhibition of 5-HT2 and 5-HT3 receptors. The
5-HT3 inhibition has an antiemetic effect and is also the mechanism by
which the anti-emetic medication ondansetron works.

Answer 12B Secondary amines like nortriptyline are typically


preferred in the geriatric population as they are less anti-alpha-1
antagonism side effects (likely to cause orthostatic hypotension and
falls), and have less prominent anticholinergic and antihistaminergic
effects when compared with tertiary amine tricyclic antidepressants
(the remaining answer choices are all tertiary amines).

Answer 13C Monoamine oxidase inhibitors (MAOI) increase levels


of all monoamines including serotonin, dopamine, and norepinephrine
by inhibiting monoamine oxidase (MAO) enzymes, which catabolize
these neuropeptides. Normally these medications are consumed orally,
which results in inhibition of MAO enzymes in the gastrointestinal tract
lining, so when foods rich in dietary amines (e.g., tyramine) are
consumed, it can result in toxic accumulation of pressor monoamines
like dopamine causing severe hypertension. The selegiline transdermal
patch bypasses the gastrointestinal tract and therefore reduces the
amount of MAO inhibition in the gastrointestinal tract. Because of this
mechanism, it has been shown not to cause hypertensive crisis even
when taken with a tyramine-inclusive diet.

Answer 14A The patient in this question has a prolonged QTc


interval. Certain psychotropic medications have the tendency to
signi icantly lengthen QTc which increases the risk of potentially lethal
arrhythmias such as “torsades de pointes.” Among the antidepressants,
citalopram has been shown most consistently to cause signi icant QTc
prolongation.

Answer 15C The patient in question is presenting with a partial


response to her current treatment (mirtazapine) but is still having
residual depressive symptoms. Her mirtazapine is already at the
maximum recommended dosage, and so titrating the dosage further
would not be a suitable option. In these cases, the current medication
should be continued and adjunctive therapy should be considered.
Given that the patient had a partial response to trazodone in the past
and it helped with her insomnia, the best initial plan would be to retrial
her with this medication. Since she only had partial response at a
dosage of 200 mg in the past, she may need a higher dosage to have a
full response. This patient would be particularly vulnerable to adverse
effects of antipsychotics with her history of cerebrovascular disease
and mild cognitive impairment. Bupropion would not be likely to help
with her complaint of insomnia. An SNRI such as duloxetine would be
likely to have similar gastrointestinal side effects to SSRIs which she did
not tolerate in the past making it not the best choice. Duloxetine would
also not likely help her complaint of insomnia.

Answer 16C Serotonergic antidepressants have a well-known


association with causing hypernatremia particularly in older adults. In
a comparison of antidepressant medications, SSRIs and SNRIs have
been shown to have a relatively higher association with new onset
hyponatremia than other classes of antidepressants, including tricyclic
antidepressants and atypical antidepressants such as mirtazapine.
Antipsychotics do not have a known association with inducing
hyponatremia.
Answer 17B There have been limited randomized controlled trials
assessing the ef icacy of antidepressants for treating behavioral
symptoms associated with frontotemporal dementia. The drugs that
have been studied in randomized controlled trials so far include
citalopram, paroxetine, and trazodone. Trazodone was shown in a small
trial to have signi icant improvements in treating irritability, agitation,
depressive symptoms, and eating disorder symptoms for patients with
frontotemporal dementia.

Answer 18A SSRIs have been associated with an increased risk of


acute closed angle glaucoma with associated vision loss; therefore, they
should be prescribed with informed caution or otherwise avoided in
patients with a premorbid history of glaucoma. The exact mechanism is
unknown, but it has been hypothesized that SSRIs increase intraocular
pressure due to the serotonergic effects on the ciliary bodies.

Answer 19C SSRIs, when given concomitantly with opioids, have an


increased risk of hyponatremia.

Answer 20D It is well established that SSRIs have an association


with increased risk of falls in older adults. The mechanism by which
SSRIs lead to falls though is unknown. Increased fall risk for older
adults taking SSRIs was demonstrated in those taking moderate
dosages and with a short duration of use.

Further Reading
Baek JH, Nierenberg AA, Fava M. Pharmacological approaches to treatment-resistant
depression. In: Massachusetts General Hospital comprehensive clinical psychiatry.
2nd ed. London: Elsevier; 2016. p. 506–9.

Buoli M, Serati M, Caldiroli A, Galimberti D, Scarpini E, Altamura AC.


Pharmacological management of psychiatric symptoms in frontotemporal dementia:
a systematic review. J Geriatr Psychiatry Neurol. 2017;30(3):162–9. https://doi.org/
10.1177/0891988717700506.
[Crossref][PubMed]

Carvalho AF, Sharma MS, Brunoni AR, et al. The safety, tolerability and risks
associated with the use of newer generation antidepressant drugs: a critical review
of the literature. Psychother Psychosom. 2016;85(5):270–88.
[Crossref]

Chen VC, Ng MH, Chiu WC, et al. Effects of selective serotonin reuptake inhibitors on
glaucoma: a nationwide population-based study. PLoS One. 2017;12(3):e0173005.
[Crossref]

Cremens MC. Geriatric psychiatry. In: Massachusetts General Hospital


comprehensive clinical psychiatry. 2nd ed. London: Elsevier; 2016. p. 763–9.

DeMartinis NA, Winokur A. Pharmacology of psychotropic drugs. In: Therapy in


sleep medicine. 1st ed. Philadelphia: Saunders; 2011. p. 109–25.

Fabian TJ, Amico JA, Kroboth PD, et al. Paroxetine-induced hyponatremia in older
adults: a 12-week prospective study. Arch Intern Med. 2004;164(3):327–32.
[Crossref]

Fava M, Papakostas GI. Antidepressants. In: Massachusetts General Hospital


comprehensive clinical psychiatry. 2nd ed. London: Elsevier; 2016. p. 489–505.

Fernandes BS, Hodge JM, Pasco JA, et al. Effects of depression and serotonergic
antidepressants on bone: mechanisms and implications for the treatment of
depression. Drugs Aging. 2016;33(1):21–5.
[Crossref]

Hilas O, Avena-Woods C. Potential role of mirtazapine in underweight older adults.


Consult Pharm. 2014;29(2):124–30.
[Crossref]

Hirsch CH, Maharaj S, Bourgeois JA. Pharmacotherapy: safe prescribing and adverse
drug events. In: Hategan A, Bourgeois J, Hirsch C, Giroux C, editors. Geriatric
psychiatry. Cham: Springer; 2018.

Krishnan KR, Doraiswamy PM, Clary CM. Clinical and treatment response
characteristics of late-life depression associated with vascular disease: a pooled
analysis of two multicenter trials with sertraline. Prog Neuro-Psychopharmacol Biol
Psychiatry. 2001;25(2):347–61.
[Crossref]

Lavoie KL, Paine NJ, Pelletier R, et al. Relationship between antidepressant therapy
and risk for cardiovascular events in patients with and without cardiovascular
disease. Health Psychol. 2018;37(11):989–99.
[Crossref]

Lien Y-HH. Antidepressants and hyponatremia. Am J Med. 2018;131(1):7–8. https://


doi.org/10.1016/j.amjmed.2017.09.002.
[Crossref][PubMed]

Low Y, Setia S, Lima G. Drug-drug interactions involving antidepressants: focus on


desvenlafaxine. Neuropsychiatr Dis Treat. 2018;14:567–80.
[Crossref]

Marcum ZA, Perera S, Thorpe JM, et al. Antidepressant use and recurrent falls in
community-dwelling older adults: indings from the health ABC study. Ann
Pharmacother. 2016;50(7):525–33.
[Crossref]

Mead GE, Hsieh CF, Hackett M. Selective serotonin reuptake inhibitors for stroke
recovery. JAMA. 2013;310(10):1066–7.
[Crossref]

Mulsant BH, Blumberger DM, Ismail Z, et al. A systematic approach to


pharmacotherapy for geriatric major depression. Clin Geriatr Med. 2014;30(3):517–
34.
[Crossref]

Myint PK, Staufenberg EF, Sabanathan K. Post-stroke seizure and post-stroke


epilepsy. Postgrad Med J. 2006;82(971):568–72.
[Crossref]

Nandagopal JJ, Delbello MP. Selegiline transdermal system: a novel treatment option
for major depressive disorder. Expert Opin Pharmacother. 2009;10(10):1665–73.
[Crossref]

Nardell M, Tampi RR. Pharmacological treatments for frontotemporal dementias. Am


J Alzheimers Dis Other Dement. 2013;29(2):123–32. https://doi.org/10.1177/
1533317513507375.
[Crossref]

Noordam R, Aarts N, Leening MJ, et al. Use of antidepressants and the risk of
myocardial infarction in middle-aged and older adults: a matched case-control study.
Eur J Clin Pharmacol. 2016;72(2):211–8.
[Crossref]

Porsteinsson AP, Drye LT, Pollock BG, et al. Effect of citalopram on agitation in
Alzheimer disease: the CitAD randomized clinical trial. JAMA. 2014;311(7):682–91.
[Crossref]

Raji MA, Barnum PD, Freeman J, et al. Mirtazapine for depression and comorbidities
in older patients with cancer. Ann Pharmacother. 2007;41(9):1548–9.
[Crossref]

Rauma PH, Honkanen RJ, Williams LJ, et al. Effects of antidepressants on


postmenopausal bone loss – a 5-year longitudinal study from the OSTPRE cohort.
Bone. 2016;89:25–31.
[Crossref]

Richa S, Yazbek J. Ocular adverse effects of common psychotropic agents. CNS Drugs.
2010;24(6):501–26. https://doi.org/10.2165/11533180-000000000-00000.
[Crossref][PubMed]

Ritter JM, Flower R, Henderson G, Loke YK, MacEwan D, Rang HP. Antidepressant
drugs. In: Rang & Dale’s pharmacology. 9th ed. Edinburgh: Elsevier; 2020. p. 603–22.

Robinson RG, Jorge RE. Post-stroke depression: a review. Am J Psychiatry.


2016;173(3):221–31.
[Crossref]

Rochester MP, Kane AM, Linnebur SA, Fixen DR. Evaluating the risk of QTc
prolongation associated with antidepressant use in older adults: a review of the
evidence. Ther Adv Drug Saf. 2018;9(6):297–308.
[Crossref]

Sultana J, Spina E, Tri irò G. Antidepressant use in the elderly: the role of
pharmacodynamics and pharmacokinetics in drug safety. Expert Opin Drug Metab
Toxicol. 2015;11(6):883–92.
[Crossref]

Welch CA. Electroconvulsive therapy. In: Massachusetts General Hospital


comprehensive clinical psychiatry. 2nd ed. London: Elsevier; 2016. p. 510–7.

Wellbutrin XL. 2017, May. Retrieved from https://www.accessdata.fda.gov/


drugsatfda_docs/label/2017/021515s036lbl.pdf. Accessed 19 Apr 2020.
© Springer Nature Switzerland AG 2021
R. Tampi et al. (eds.), Absolute Geriatric Psychiatry Review
https://doi.org/10.1007/978-3-030-58663-8_27

27. Mood Stabilizers


Kyle Resnick1
(1) Department of Psychiatry & Behavioral Sciences, Cleveland Clinic
Akron General, Akron, OH, USA

Kyle Resnick
Email: kresnick@neomed.edu

Keywords Mood stabilizers – Anticonvulsants – Bipolar disorder –


Bipolar depression – Bipolar mania – Manic depressive disorder –
Older age bipolar disorder (OABD)

Questions
1. An 81-year-old man taking multiple medications for different
chronic health conditions presents to the emergency department
with diarrhea, tremors, weakness, and confusion. The emergency
department physician notices that the patient’s complicated
medication list includes lithium. The physician suspects lithium
toxicity as a possible cause of the patient’s presentation. Which of
the following medications is not known to affect lithium levels
when co-administered?
A.
Ibuprofen
B.
Enalapril
C.
Hydrochlorothiazide
D. Acetaminophen
E.
Acetazolamide
2.
A 70-year old woman with a past medical history signi icant for
bipolar disorder presents to your clinic as a new patient. Her
symptoms have been well controlled with lithium since adulthood
after numerous failed medication trials. Patient has been on a
stable dose of lithium for many years. She reports worsening
tremors and feelings of fogginess over the past several months. A
family member who accompanies also shares the patient’s
concerns. What might be a reasonable next step in management of
the patient’s lithium dosing?
A.
Increase the lithium dose to account for increased renal
clearance
B.
Increase the lithium dose to account for decreased renal
clearance
C.
Decrease the lithium dose to account for increased renal
clearance
D.
Decrease the lithium dose to account for decreased renal
clearance
E.
Add propranolol to control the tremors
3. Which of the following mood stabilizers can cause drug-drug
interactions via the induction of cytochrome P450 enzyme system
and is particularly problematic for use among older adults?
A.
Carbamazepine
B.
Lithium
C. Valproate
D.
Lamotrigine
E.
Topiramate
4.
A 74-year-old man presents to your clinic after being admitted
following a manic episode. He was started on lithium in the
hospital. The man asks you about the use of lithium for someone
of his age. Which of the following statements would be true of the
use of lithium among older adults with bipolar disorder?
A.
Lithium can be effective in lower plasma concentrations
among older adults when compared to younger adults
B.
Lithium is much less effective in treating bipolar disorder
among older adults when compared to younger adults
C.
Lithium causes similar rates of side effects among older
adults when compared to younger adults
D.
Lithium is not as effective at treating bipolar disorder among
older adults when compared to other mood stabilizers
E.
All of the above
5. A 75-year-old man is started on valproate for symptoms of bipolar
disorder. Due to the patient’s age and drug-drug interactions
potentially altering serum levels, the prescribing physician plans
to regularly check the valproate level. Which of the following
medications when co-administered in this man would not result
in any potential drug-drug interaction?
A.
Aspirin
B.
Phenytoin
Phenytoin

C. Warfarin

D.
Lamotrigine
E.
Hydrochlorothiazide
6.
A 75-year-old woman is found to be confused and poorly
responsive to verbal and physical stimuli. Following an initial
work-up in the emergency department, the patient is found to
have an elevated serum ammonia level. Which of the following
mood stabilizers is most likely to be responsible for the elevated
ammonia level?
A.
Lithium
B.
Valproate
C.
Carbamazepine
D.
Oxcarbazepine
E.
Topiramate
7. Which of the following mood stabilizers has been shown to have
ef icacy in preventing relapse of depressive symptoms among
older adults with bipolar disorder?
A.
Oxcarbazepine
B.
Lamotrigine
C.
Gabapentin
D.
Carbamazepine
Carbamazepine
E. Topiramate

8.
A 71-year-old patient on lithium monotherapy arrives for his well
visit. On review, his serum lithium level is 0.4 meq/L. What may
be the most reasonable next step to ensure the patient’s
symptoms remain in good control?
A.
Increase the patient’s lithium dose
B.
Decrease the patient’s lithium dose
C.
Do not change the patient’s lithium dose
D.
Change to a new mood stabilizer
E.
Add hydrochlorothiazide to his medication regimen
9. Valproate can be effective in treatment of bipolar disorder among
older adults at doses lower than expected for younger adults.
Which of the following pharmacokinetic mechanisms is not
presumed to be a contributing factor to this inding?
A.
Increased free valproate due to decreased protein binding
ef iciency
B.
Elimination of valproate is reduced due to increased volume
of distribution
C.
Increased free valproate due to decreased serum albumin
levels
D.
Increased free valproate due to improved absorption of drug
E.
All of the above
All of the above
10.
A psychiatrist treating an 80-year-old patient for bipolar
depression wants to prescribe valproate but notes that the patient
is taking warfarin. What is the mechanism of action that results in
a serious drug-drug interaction between these two drugs?
A.
Valproate induces oxidative metabolism of warfarin, resulting
in decreased levels of warfarin
B.
Valproate displaces warfarin from albumin-binding sites,
resulting in increased levels of warfarin
C.
Warfarin induces oxidative metabolism of valproate, resulting
in decreased levels of valproate
D.
Warfarin displaces valproate from albumin-binding sites,
resulting in increased levels of valproate
E.
Valproate decreases the renal clearance of warfarin

11.
Which of the following drug classes used for the treatment of
bipolar disorder carries an FDA boxed warning about increased
mortality when used among older adults?
A.
Atypical antipsychotics
B.
Anticonvulsants
C.
Selective serotonin reuptake inhibitors (SSRIs)
D.
Salts (lithium)
E.
Benzodiazepines
12. A 69-year-old woman who is taking carbamazepine has for
bipolar disorder presents to the clinic complaining of recent skin
rashes that have occurred after a dose increase of carbamazepine.
Which laboratory study should be monitored closely in this
individual given her recent condition?
A. Complete blood count (CBC)

B.
Basic metabolic panel (BMP)
C.
Beta natriuretic peptide (BNP)
D.
Liver function test (LFT)
E.
Urine analysis (UA)
13.
Which of the following medications may have mood-stabilizing
functions in the treatment of bipolar disorder among older
adults?
A.
Oxcarbazepine
B.
Gabapentin
C.
Zonisamide
D.
Topiramate
E.
All of the above
14. Which of the following is not a side effect of lithium when used
among older adults with bipolar disorder?
A.
Weight loss
B.
Memory loss
C.
Fine hand tremors
D.
Ataxia
E. Polyuria

15.
In the irst randomized controlled trial of late-life mania (GERI-
BD), the investigators compared the tolerability and ef icacy of
lithium carbonate and divalproex among 224 inpatients and
outpatients aged ≥60 years with bipolar I disorder who presented
with a manic, hypomanic, or mixed episode. Which of the
following was inding from this pivotal study?
A.
The response rates did not differ signi icantly between the
lithium and divalproex groups
B.
The need for adjunctive risperidone was more in the lithium
group
C.
Sedation was signi icantly greater in the divalproex group
D.
Participants in the divalproex group experienced more
tremors
E.
All of the above
16. Which of the following does not represent a concern when
carbamazepine is used among older adults with bipolar disorder?
A.
Carbamazepine has signi icant drug-drug interactions with
many common medications
B.
Carbamazepine is ineffective in the treatment of bipolar
disorder with non-classical features
C.
C
Carbamazepine can cause cardiac conduction delays among
older adults
D.
Carbamazepine may cause sedation among older adults
E. Carbamazepine may cause cognitive impairment among older
adults

17.
Which of the following mood stabilizers has been shown to reduce
the rates of dementia among individuals with bipolar disorder?
A.
Lithium
B.
Divalproex
C.
Carbamazepine
D.
Gabapentin
E.
Quetiapine
18.
Which of the following has shown the least convincing evidence as
an adjunctive therapy when co-administered with lithium or
anticonvulsant mood stabilizers?
A.
Antidepressants
B.
Benzodiazepines
C.
Antipsychotics
D.
Psychotherapy
E.
Electroconvulsive therapy
Which of the following mood stabilizers has demonstrated anti-
g
19. suicidal effects and may be very useful among older adults with
bipolar disorder?
A.
Lithium
B.
Divalproex
C. Carbamazepine

D.
Gabapentin
E.
Quetiapine
20.
Which mood stabilizer has the most favorable cognitive pro ile
when used to treat older adults with bipolar disorder?
A.
Topiramate
B.
Lamotrigine
C.
Carbamazepine
D.
Valproate
E.
Lithium

Answers
Answer 1D Many medications are known to increase the risk of
lithium toxicity when co-administered with lithium. Special
consideration should be made when treating older adults who are more
likely to have issues of polypharmacy and renal dysfunction.
Acetaminophen, a hepatically metabolized drug, is not known to
increase the risk of lithium toxicity. Nonsteroidal anti-in lammatory
drugs (NSAIDs) such as ibuprofen and angiotensin-converting enzyme
(ACE) inhibitors such as enalapril can increase the risk of lithium
toxicity by their effects on renal blood low. Thiazide diuretics such as
hydrochlorothiazide have been shown to increase lithium levels by
anywhere between 25% and 400%. Carbonic anhydrase inhibitors such
as acetazolamide decrease serum lithium levels.

Answer 2D The glomerular iltration rate decreases over time as


individuals age. Because lithium is renally cleared, accurate and
effective dosing of lithium among older adults may be lower than
anticipated and when compared to younger adults with better kidney
function. Symptoms of chronic lithium toxicity, such as seen in this
patient, may appear even when the serum lithium levels are not
elevated. In this patient, the dose of lithium should be decreased to
account for decreased kidney function. Adding propranolol to control
her tremors will not be helpful, and in some cases make the patient
dizzy by lowering her blood pressure.

Answer 3A Carbamazepine is a potent inducer of the cytochrome


P450 enzyme pathway. Induction of this pathway hastens degradation
and bioavailability of many medications. Carbamazepine is usually not
utilized as irst-line pharmacotherapy for bipolar disorder among older
adults.

Answer 4A Lithium has been found to be effective in treating


bipolar disorder at relatively lower plasma concentrations among older
adults when compared to younger adults. Lithium is about equally
effective in treating bipolar disorder in both age groups (72% vs. 78%,
older adults vs. younger adults). Lithium causes side effects more
frequently among older adults. Lithium has been demonstrated to be
equal to or more effective at treating bipolar disorder among older
adults when compared to other mood stabilizers.

Answer 5E Due to drug-drug interactions, valproate levels should be


carefully monitored in among older adults. This is especially true
among individuals who are also taking aspirin, phenytoin, lamotrigine,
and warfarin.
Answer 6B Hyperammonemia is a serious potential adverse effect
of valproate administration. Ammonia levels can be elevated even when
valproate levels are normal.

Answer 7C Among the mood stabilizers that are listed, lamotrigine


has been shown to be effective at preventing relapse of depressive
symptoms.

Answer 8C Dosing of lithium in elderly adults are typically lower


relative to dosing among younger adults. This is due to the greater risk
of toxicity, drug-drug interactions, and evidence that lower levels of
lithium may provide similar symptomatic control in the older
population. The target levels have been suggested to range from 0.4 to
0.7 among older adults with bipolar disorder.

Answer 9D Valproate binds to albumin in the serum, so any


decrease in serum albumin in the older adult population will result in a
higher fraction of the active unbound drug. Protein binding ef iciency
also decreases with age. In addition, the volume of distribution
increases with age resulting in a longer half-life for valproate. The
absorption of valproate does not change with age.

Answer 10B Valproate and its metabolites have higher af inity for
albumin than warfarin, thus displacing this drug from protein binding
and resulting in increased levels of warfarin in the blood. Valproate is
an inducer of oxidative metabolism resulting in decreased levels of
drugs that are oxidatively metabolized such as phenytoin or tricyclic
antidepressants (TCAs).

Answer 11A Atypical antipsychotics that are often used for the
treatment of bipolar disorder have been shown in trials to have up to
1.6–1.7 times the risk of increased mortality when used among older
adults with dementia-related psychosis. While the boxed warning
speci ically refers to use of these drugs for the treatment of dementia-
related psychosis, the potential risk can be reasonably extrapolated to
their use among older adults with bipolar disorder.
Answer 12A A serious potential effect of carbamazepine is blood
dyscrasias such as aplastic anemia or agranulocytosis. While these
complications are very rare, the risk of occurrence increases with age,
and there may be an associative relationship between carbamazepine-
induced skin rashes and blood dyscrasias.

Answer 13E Any of the listed medications may prove useful as a


mood stabilizer and are employed as such among younger adults with
bipolar disorder. Evidence for their use among older adults is mostly
limited to case studies, but they may be considered as viable options in
circumstances where more commonly used mood stabilizers are
unhelpful or limited by their potential harm.

Answer 14A Weight gain, and not weight loss, is the common side
effect of lithium. Other side effects are memory loss, ataxia, ine hand
tremors, polyuria, hypothyroidism, seizures, diabetes insipidus, cardiac
arrhythmia, raised intracranial pressure, and in rare cases coma.

Answer 15A In the GERI-BD study, the response rates did not differ
signi icantly between the lithium and divalproex groups (79% vs. 73%).
Attrition rates were similar for lithium and divalproex (51% vs. 44%).
The groups did not differ signi icantly in sedation. Participants in the
lithium group tended to experience more tremors. Similar proportions
of participants in the lithium and divalproex groups achieved target
concentrations (57% vs. 56%). The need for adjunctive risperidone was
low and similar between groups (17% vs. 14%).

Answer 16B Carbamazepine has been suggested to be effective in


preventing relapse of bipolar symptoms among individuals presenting
with non-classical features. However, studies show that it remains
inferior to lithium in overall long-term treatment of bipolar disorder.
Carbamazepine is a cytochrome P450 enzyme inducer, which can result
in signi icant changes to blood levels of drugs metabolized by this
enzyme. Carbamazepine has been shown to cause dangerous
bradyarrhythmia and atrioventricular (AV) conduction delays
especially in elderly women. Carbamazepine may cause neurological
side effects such as dizziness, sedation, ataxia, and cognitive
impairment.
Answer 17A One study from Denmark has shown that continued
treatment with lithium was associated with a reduced rate of dementia
in individuals with bipolar disorder when compared to continued
treatment with anticonvulsants, antidepressants, or antipsychotics.

Answer 18D Studies examining the use of benzodiazepines as an


adjunctive therapy have shown limited utility in the control of both
manic and depressive symptoms in older age bipolar disorder.
Antipsychotics have stronger evidence for their use as adjuncts to mood
stabilizers. Antidepressants may be useful as short-term adjuncts
during depressive episodes. Psychotherapy may be helpful in improving
social functioning and decreasing risk of relapse with the strongest
evidence cognitive behavioral therapy. ECT is a highly effective
intervention for acute mania and depression, both as monotherapy and
as an adjunct to primary mood stabilizer treatment among older adults
with bipolar disorder.

Answer 19A Evidence from recent studies indicates that lithium has
an anti-suicidal effect that is independent of its mood-stabilizing effect.
This effect has been attributed to a neurotrophic effect of lithium where
the use of lithium is associated with enlargement in the anterior
cingulated gyrus, hippocampus, and prefrontal cortex, which is thought
to promote a “top-down braking” of aggression toward self. This anti-
suicidal effect is highly relevant among older adults with bipolar
disorder, as old age itself is associated with an increased risk for
suicide. This bene it when using lithium versus other anticonvulsants
among older adults with bipolar disorder must be considered as most
anticonvulsants have been associated with an increased risk of suicidal
acts and violent deaths.

Answer 20B Lamotrigine appears to have a safer neurocognitive


pro ile among individuals with bipolar disorder when compared with
other mood stabilizers.

Further Reading
Aziz R, Lorberg B, Tampi RR. Treatments for late-life bipolar disorder. Am J Geriatr
Pharmacother. 2006;4:347–64.
[Crossref]

D’Souza R, Rajji TK, Mulsant BH, Pollock BG. Use of lithium in the treatment of
bipolar disorder in late-life. Curr Psychiatry Rep. 2011;13(6):488–92.
[Crossref]

De Fazio P, Gaetano R, Caroleo M, et al. Lithium in late-life mania: a systematic


review. Neuropsychiatr Dis Treat. 2017;13:755–66.
[Crossref]

Finley PR. Drug interactions with Lithium: an update. Clin Pharmacokinet.


2016;55(8):925–41.
[Crossref]

Kessing LV, Forman JL, Andersen PK. Does lithium protect against dementia? Bipolar
Disord. 2010;12(1):87–94.
[Crossref]

Murru A, Popovic D, Pacchiarotti I, et al. Management of adverse effects of mood


stabilizers. Curr Psychiatry Rep. 2015;17(8):603.
[Crossref]

Sajatovic M, Strejilevich SA, Gildengers AG, et al. A report on older-age bipolar


disorder from the international society for bipolar disorders task force. Bipolar
Disord. 2015;17(7):689–704.
[Crossref]

Varma S, Sareen H, Trivedi JK. The geriatric population and psychiatric medication.
Mens Sana Monogr. 2010;8(1):30–51.
[Crossref]

Young RC, Mulsant BH, Sajatovic M, et al. GERI-BD: a randomized double-blind


controlled trial of Lithium and divalproex in the treatment of mania in older patients
with bipolar disorder. Am J Psychiatry. 2017;174(11):1086–93.
[Crossref]
© Springer Nature Switzerland AG 2021
R. Tampi et al. (eds.), Absolute Geriatric Psychiatry Review
https://doi.org/10.1007/978-3-030-58663-8_28

28. Antipsychotics
Rakin Hoq1
(1) Department of Child and Adolescent Psychiatry, NYU Grossman
School of Medicine, New York, NY, USA

Rakin Hoq
Email: rakin.hoq@nyulangone.org

Keywords Antipsychotics – Psychosis – Bipolar disorder – Delirium –


Behavioral and psychological symptoms of dementia (BPSD) – QTc
prolongation – Metabolic syndrome – Weight gain – Extrapyramidal
side effects – Neuroleptic malignant syndrome – Long-acting injectable

Questions
1. A 65-year-old man with a diagnosis of bipolar I disorder, obesity,
and type II diabetes is referred to your clinic by his primary care
physician for an evaluation and treatment of depression that has
developed over the past 3 months. He describes signi icant
anhedonia and a feeling of emptiness but denies suicidal thinking.
He is noted to have a history of severe manic episodes but does
not acknowledge any signi icant episodes of depression like this
in the past. He has been clinically stable for the past 35 years on
lithium monotherapy. Which of the following choices would be the
best approach for the pharmacological management of this
patient while avoiding weight gain?
A.
Continue lithium and add mirtazapine
B. Continue lithium and add quetiapine
C.
Continue lithium and add lurasidone
D.
Continue lithium and add valproate
E.
Continue lithium and add escitalopram
2.
A 62-year-old woman with a diagnosis of schizophrenia, obesity,
coronary artery disease, and arthritis presents to your clinic with
concern about weight gain from her antipsychotic medicine. She
has been stable on risperidone for several years and inquires if
there are any alternative medication options that would have less
associated weight gain. Which of the following alternatives would
be the best recommendation for this individual?
A.
Quetiapine
B.
Haloperidol
C.
Olanzapine
D.
Ziprasidone
E.
Clozapine
3. A 68-year-old man with a history of schizophrenia and coronary
artery disease is admitted to the inpatient psychiatric unit with
decompensated psychosis. He was prescribed haloperidol
recently and acknowledges non-adherence to this treatment in
recent weeks. However, a review of outpatient records indicates
that his response to haloperidol was poor. His EKG is also noted to
show a QTc interval of 505 ms. Which of the following
antipsychotic drugs would be the safest recommendation for him
at this time?
A. Risperidone

B.
Ziprasidone
C.
Lurasidone
D.
Quetiapine
E.
Haloperidol
4.
Risk of death from which of the following adverse events has
resulted in a boxed warning by the FDA for the use of
antipsychotics among individuals with dementia?
A.
Cardiovascular events
B.
Falls
C.
Delirium
D.
Seizures
E.
Gastrointestinal bleeds
5. Which of the following antipsychotic medications is most likely to
be tolerated by an individual with dementia with Lewy bodies
(DLB)?
A.
Haloperidol
B.
Risperidone
C.
Aripiprazole
D.
Quetiapine

E. Fluphenazine

6.
Which of the following medications now has an FDA indication for
treating Parkinson’s disease psychosis (PDP)?
A.
Quetiapine
B.
Pimavanserin
C.
Lurasidone
D.
Clozapine
E.
Brexpiprazole
7. Which of the following inds did the CATIE-AD trial demonstrate?
A.
Extrapyramidal symptoms were seen more commonly with
the use of olanzapine and risperidone than with quetiapine
and placebo
B.
Sedation was signi icantly more common in the olanzapine
and quetiapine groups when compared to placebo but not
signi icantly different between the quetiapine and placebo
groups
C.
Olanzapine was associated with signi icantly higher incidence
of stroke and death when compared to risperidone and
quetiapine groups
D.
Quetiapine was associated with signi icantly more
improvement on the Clinical Global Impression of Change
(CGIC) scale when compared to the other groups
E.
Olanzapine was associated with signi icantly more
Olanzapine was associated with signi icantly more
improvement on the Clinical Global Impression of Change
(CGIC) scale when compared to the other groups
8.
A 76-year-old woman with dementia with Lewy bodies (DLB)
presents to the emergency department from her nursing home
due to progressive withdrawal, confusion, muscle rigidity, and a
fever of 102 °F that developed over the last 24 hours. She had
progressively worsening psychosis and agitation in the past
month and was started on a medication for agitation, which was
administered initially last night and again today. What is the most
likely diagnosis for this patient’s condition?
A.
Sepsis
B.
Toxic encephalopathy
C.
Serotonin syndrome
D.
Allergic drug reaction
E.
Neuroleptic malignant syndrome (NMS)
9. A 67-year-old man with a history of severe depression develops
psychotic symptoms. He is started on aripiprazole, which leads to
a resolution in his psychotic symptoms. He remains stable for
3 months before he returns with complaints of a tremor in his
hand. You observe him to exhibit a pill-rolling tremor and
shuf ling gait. What would be the best initial step in management
of his symptoms?
A.
Start oral benztropine
B.
Start levodopa
C.
Reduce the dosage of aripiprazole
D.
Switch from aripiprazole to haloperidol
Switch from aripiprazole to haloperidol
E.
Start ropinirole
10.
Which of the following antipsychotic medications has the FDA
approval for the treatment delirium among older adults?
A.
Haloperidol
B.
Quetiapine
C.
Olanzapine
D.
Risperidone
E.
None of the above
11.
Which of the following statements is false regarding the adverse
effects associated with prescribing antipsychotics to older adults?
A.
The use of atypical antipsychotics in individuals with
dementia is associated with an increased mortality risk
B.
The use of antipsychotic agents in older adults is associated
with an increased risk of falls
C.
Older adults are at greater risk for developing extrapyramidal
side effects from antipsychotic agents
D.
The use of antipsychotics in older adults is associated with
increased risk of gastrointestinal bleeding
E.
The use of antipsychotics in individuals with dementia is
associated with increased risk of cardiovascular events
12. A 66-year-old woman presents to your of ice as a new patient for
follow-up from a psychiatric inpatient admission for a irst-
i d h i Sh d h l id l d h d
episode psychosis. She was started on haloperidol, and the dose
was increased until she had full relief of psychotic symptoms. She
is now complaining of anxiety that has developed since discharge
from the hospital. She says she cannot sleep at night because she
cannot stop getting up and pacing. Which of the following would
be the most appropriate treatment option to give her relief from
her symptoms?
A.
Alprazolam
B.
Propranolol
C.
Buspirone
D.
Diphenhydramine
E.
Pramipexole
13.
A 75-year-old woman with Alzheimer’s dementia who has also
developed psychosis has been started on risperidone. This use of
this medication may increase her risk for developing which of the
following conditions?
A.
Pancreatitis
B.
Gastric ulcer
C.
Osteoporosis
D.
Heart failure
E.
Gout
14. A 66-year-old woman who developed late-onset schizophrenia
has failed two separate trials of a irst- and second-generation
antipsychotics. She is initiated on clozapine, which is beginning to
control her symptoms, but now she is concerned about
experiencing excessive drooling at night. Which of the following
treatments could potentially provide relief of her drooling while
also having the least amount of adverse effects?
A. Oral glycopyrrolate

B.
Oral benztropine
C.
Sublingual atropine
D.
Sublingual ipratropium
E.
Oral trihexyphenidyl
15. A 70-year-old man presents with acute manic symptoms and is
started on oral risperidone. He is also given an additional dose of
intramuscular haloperidol to help calm him. The patient calms
down initially but later develops confusion and muscle stiffness.
He is found to have a fever of 103 °F and a creatinine kinase of
30,000 units/liter. In addition to discontinuing antipsychotics,
giving IV hydration and respiratory support, which of the
following medications would be most helpful for the care if this
patient?
A.
Metoclopramide
B.
Bromocriptine
C.
Lorazepam
D.
Benztropine
E.
Gabapentin
16.
You are evaluating a 66-year-old woman with a diagnosis of
schizophrenia and co-morbid epilepsy for antipsychotic
treatment. Which of the following antipsychotics has the highest
risk for exacerbating her seizure disorder?
A.
Aripiprazole
B.
Quetiapine
C.
Risperidone
D.
Clozapine
E.
Ziprasidone

17.
A 70-year-old woman with schizophrenia is stabilized on
haloperidol decanoate. She feels this is the right treatment for her
and is optimistic about her health as she ages. She inquires what
long-term risks that she may face as she ages. Which of the
following would be considered as a long-term risk with the use of
haloperidol?
A.
Dilated cardiomyopathy
B.
Liver failure
C.
Tardive dyskinesia
D.
Congestive heart failure
E.
Chronic kidney disease
18. A 73-year-old man with history of bipolar disorder, asthma, and
chronic obstructive pulmonary disease presents to the hospital
manic and agitated. He is given an intramuscular dose of
haloperidol to help calm him. However, he begins gasping for air
and holding his neck shortly after the injection. Which of the
following treatment options would be the most appropriate to
quickly relieve his symptoms?
A. Intramuscular benztropine

B.
Intramuscular lorazepam
C.
Intramuscular epinephrine
D.
Inhaled albuterol
E.
Intravenous heparin
19. A 65-year-old man with long history of schizophrenia says that he
is now getting more forgetful and often missing doses of his
prescribed risperidone. He says he can still make it to his
appointments without issue but worries he may decompensate if
he continues to miss doses of his medication. He inquires if there
is a more reliable way to ensure he continues to maintain a
steady-state dose of risperidone. He lives independently and
wonders what strategy could help him without sacri icing his
independence. Which of the following treatment
recommendations would be the most reliable and safe way to
ensure that he gets his prescribed medications?
A.
Recommend pill boxes for his psychiatric medication regimen
B.
Tell him he must have in-home care to help him at his age
C.
Recommend a trial of a long-acting injectable formulation of
risperidone
p
D.
Recommend he change antipsychotic medications to a
different drug with a longer half-life
E.
Recommend he implement phone reminders to help him
remember when to take his medicine
20.
A 70-year-old woman with a diagnosis of schizophrenia and no
previous history of diabetes has been stable on olanzapine for
many years. She now presents to the hospital with diabetic
ketoacidosis. The endocrinologist evaluates her and determines
that she has likely developed diabetes as a result of metabolic
adverse effects caused by olanzapine. The endocrinologist asks
that you change her antipsychotic to a medication that would have
less risk of glucose intolerance and dyslipidemia. Which of the
following agents would be the best choice for this patient?
A.
Ziprasidone
B.
Quetiapine
C.
Risperidone
D.
Perphenazine
E.
Clozapine

Answers
Answer 1C Lurasidone has been shown to be effective in the
treatment of bipolar depressive symptoms with minimal weight gain.
Quetiapine would not be an optimal choice for this patient as it causes
signi icant weight gain. Neither mirtazapine nor escitalopram would be
optimal choices due to the lack of data supporting their use in
treatment of bipolar depression. Valproate is known to have weight
gain as a side effect and may not be the best adjuvant therapy for this
patient if you are trying to avoid weight gain.

Answer 2D Among the antipsychotics listed, ziprasidone is


associated with the least weight gain over time.

Answer 3C Of the antipsychotics listed, lurasidone is the only one


that is not associated with signi icant QT interval prolongation.

Answer 4A Cardiovascular events were the main course for


mortality when antipsychotics were used among individuals with
dementia and resulted in the FDA boxed warning.

Answer 5D Dementia with Lewy bodies is associated with


hypersensitivity to dopamine-blocking agents resulting in higher rates
of extrapyramidal side effects and neuroleptic malignant syndrome
with use of antipsychotic medications. Among the listed antipsychotics,
quetiapine has been shown to be the best tolerated by individuals
diagnosed with Lewy body dementia.

Answer 6B Pimavanserin is the only medication with an FDA


approval to treat Parkinson’s disease psychosis. It is a novel
antipsychotic that works by selective 5-HT2A inverse agonism and
lacks activity at the dopaminergic receptors.

Answer 7A The CATIE-AD trial was a seminal trial in assessing the


ef icacy and tolerability of second-generation antipsychotics in treating
behavioral symptoms associated with Alzheimer’s dementia. The
antipsychotics studied were olanzapine, risperidone, and quetiapine.
The study demonstrated that extrapyramidal symptoms were more
common with the use of olanzapine and risperidone than with
quetiapine. Sedation was signi icantly more common in all three
antipsychotic groups when compared to placebo. There was also no
signi icant difference between groups for serious adverse events
including stroke or death. By the end of the study, none of the treatment
groups were signi icantly different with regard to change on the CGIC
scale.
Answer 8E The patient in this vignette has dementia with Lewy
bodies (DLB), which predisposes her to hypersensitivity to dopamine
blockade and therefore higher risk of developing extrapyramidal side
effects and neuroleptic malignant syndrome (NMS). Antipsychotics are
also the most common agents used to manage agitation and psychosis
associated with dementias but remain risky especially in the older
adult population. The patient’s presenting constellation of symptoms
coupled with the given recent history suggests that she was given an
antipsychotic medication to manage her agitation and is now
developing neuroleptic malignant syndrome.

Answer 9C Due to dopamine blockade of antipsychotic agents like


aripiprazole, drug-induced parkinsonism may occur. Elderly individuals
are more predisposed to developing this adverse effect. This is typically
an insidious issue that develops over weeks to months. The typical
treatment is to reduce the dosage or taper off the offending agent.
Adding dopaminergic agents are not recommended as they can worsen
psychosis or cause delirium. Anticholinergic agents are also not
recommended due to anticholinergic side effects. Switching the
medication to another high potency antipsychotic would not help the
problem and could potentially worsen it. The use of dopamine agonists
such as ropinirole is helpful in treating the parkinsonian symptoms
secondary to Parkinson’s disease but could potentially worsen
psychosis for this patient with antipsychotic-induced parkinsonism;
therefore, it would not be preferable treatment.

Answer 10E Although antipsychotic medications are commonly


used as irst-line pharmacologic agents to manage symptoms of
delirium that threaten the safety or impede the care of the individual
especially when non-pharmacological approaches are insuf icient, there
are no US Food and Drug Administration (FDA)-approved medications
for the treatment of delirium among older adults.

Answer 11D Antipsychotic use in the elderly is associated with


increased risk of falls, increased chance of death and cardiovascular
events in those with dementia, higher rate of extrapyramidal side
effects, and increased risk of osteoporosis related to prolactin changes.
However, it is not associated with an increased risk of gastrointestinal
bleeding.

Answer 12B The patient’s presentation is suggestive of akathisia


caused by haloperidol. The key aspects of the history are that she was
started on an antipsychotic of which the dosage was likely titrated
rapidly in the hospital, and now she presents with a new onset of
restlessness. Akathisia is often described as anxiety by patients
experiencing it. The need for pacing at night is describing the
restlessness and need for movement associated with this adverse effect.
Propranolol is the standard indicated treatment to provide relief for
this symptom and compared to the other options would be best
tolerated. Diphenhydramine has been shown to be effective as an IV
infusion but could cause signi icant sedation and may be risky in an
older individual. Alprazolam would also not be appropriate to start due
to a deleterious side-effect pro ile among older adults and has not been
studied thoroughly for the treatment of akathisia. Treatment with an
antidepressant or serotonin agonist would also not be appropriate for
akathisia; this is listed as a trick option if it is perceived this patient is
developing a primary anxiety disorder. Pramipexole, while an
appropriate treatment for restless leg syndrome, is not an appropriate
treatment for akathisia induced by antipsychotics.

Answer 13C Dopamine-blocking effects of antipsychotics are


associated with elevation of prolactin levels, which can in turn reduce
bone mineral density. When compared to other atypical antipsychotics,
risperidone has been known to cause sustained elevated prolactin
levels. Antipsychotics are not associated with heart failure, pancreatitis,
gout, or gastric ulcers.

Answer 14D Clozapine causes sialorrhea, which can be a distressing


side effect for many individuals. The theory behind clozapine-induced
sialorrhea is its effect as both muscarinic and alpha-adrenergic
antagonistic and these two mechanisms can stimulate saliva
production. All listed treatments have been shown to be potentially
effective treatments for sialorrhea caused by clozapine, but among
them sublingual ipratropium has the least systemic absorption when
given sublingually, which would minimize potential systemic side
effects.

Answer 15B The patient in question has developed neuroleptic


malignant syndrome (NMS) based on the antipsychotics that he was
given. The treatment for NMS largely consists of supportive treatment
in addition to discontinuing the offending agents. But there is
signi icant evidence indicating that certain medications can help relieve
symptoms. Bromocriptine, a dopamine agonist, helps reverse the
antidopaminergic activity associated with NMS. Dantrolene (a muscle
relaxant) and amantadine (an antiviral medication with dopaminergic
properties) have also been found to be helpful. Metoclopramide would
not be helpful and may exacerbate the condition as it also blocks
dopaminergic activity. While IV benzodiazepines may play a role in
treating NMS, diazepam is the benzodiazepine of choice as it is a more
potent muscle relaxer. While anticholinergics have a role in the
treatment of acute dystonic reactions, they do not have a signi icant
role in treatment of NMS. Gabapentin is included as a distractor.

Answer 16D In a review of both irst- and second-generation


antipsychotics, all have been found to increase seizure risk. However,
among the antipsychotics listed in the answer choices, clozapine
confers the highest risk.

Answer 17C The incidence of tardive dyskinesia is much higher in


older age and a higher incidence in females compared to males. The
other answer choices are not known to be long-term adverse effects
associated with antipsychotic medications.

Answer 18A The patient in this question is most likely developing a


laryngospasm after receiving haloperidol. This is a type of dystonic
reaction resulting from administration of antidopaminergic agents and
should be treated with a rapid-acting anticholinergic medication.
Lorazepam does not effectively treat acute dystonic reactions.
Epinephrine would be useful in treating an anaphylactic reaction, but
the patient in question is unlikely to be experiencing this, as there are
currently no known reports of anaphylactic reactions to antipsychotics
causing asphyxiation. Allergic reactions to antipsychotics typically
result in skin changes and at times liver in lammation. Albuterol could
be an appropriate choice if the patient was having an asthma attack, but
the history provided in this vignette is more suggestive of a dystonic
reaction from an antipsychotic. IV heparin would be an effective
treatment for pulmonary embolism but not for an acute dystonic
reaction.

Answer 19C Long-acting injectable formulations of antipsychotics


have been shown to reliably maintain stable drug levels between dose
intervals and are helpful to ensure drug adherence. They have also been
shown not to have a greater side-effect pro ile than their oral
equivalents. While pill boxes and phone reminders can be helpful for
promoting medication adherence, they would not be the most effective
or reliable means to ensure adherence since he is having memory
dif iculties. Enforcing that he gets a home health aide at this time may
go against his desire to maintain independence and could also
negatively affect the therapeutic alliance with him. Since he has been
stable on risperidone for years, changing to a different medication
would run the risk of destabilizing him.

Answer 20A Metabolic insults in terms of weight gain, glucose


intolerance, and dyslipidemia are common side effects caused by
antipsychotic agents; more so with second-generation antipsychotics.
Olanzapine and clozapine are both highly associated with all three of
these long-term adverse effects. Ziprasidone has been widely studied
and shown not to have signi icant increased risk of diabetes and
dyslipidemia long-term relative to the rest of the atypicals. The other
drugs listed have all been shown to have some increased risk of weight
gain, dyslipidemia, and long-term diabetes.

Further Reading
Aronson JK. Neuroleptic drugs. In: Meyler’s side effects of drugs. 16th ed.
Amsterdam: Elsevier Science; 2015. p. 53–119.

Aronson JK. Neuroleptic drugs. In: Meyler’s side effects of drugs. 16th ed.
Amsterdam: Elsevier Science; 2015. p. 2591–5.
Aronson JK. Elderly people. In: Meyler’s side effects of drugs. 16th ed. Amsterdam:
Elsevier Science; 2015. p. 53–119.

Bak M, Fransen A, Janssen J, et al. Almost all antipsychotics result in weight gain: a
meta-analysis. PLoS One. 2014;9(4):e94112.
[Crossref]

Beach SR, Celano CM, Sugrue AM, et al. QT prolongation, torsades de pointes, and
psychotropic medications: a 5-year update. Psychosomatics. 2018;59(2):105–22.
[Crossref]

Bird AM, Smith TL, Walton AE. Current treatment strategies for clozapine-induced
sialorrhea. Ann Pharmacother. 2011;45(5):667–75.
[Crossref]

Freudenreich O, Goff DC, Henderson DC. Antipsychotic drugs. In: Massachusetts


General Hospital comprehensive clinical psychiatry. 2nd ed. London: Elsevier
Science; 2015. p. 475–88.

Gomperts SN. Lewy body dementias: dementia with Lewy bodies and Parkinson
disease dementia. Continuum (Minneap Minn). 2016;22(2 Dementia):435–63.

Guenette MD, Chintoh A, Remington G, et al. Atypical antipsychotic-induced


metabolic disturbances in the elderly. Drugs Aging. 2014;31(3):159–84.
[Crossref]

Kobayashi R, Matsumoto Y, Hayashi H, Suzuki A, Otani K. Neuroleptic malignant


syndrome following quetiapine treatment in a patient with dementia with Lewy
bodies. Asian J Psychiatr. 2017;30:173–4.
[Crossref]

Ló pez-Sendó n JL, Mena MA, de Yé benes JG. Drug-induced Parkinsonism in the elderly:
incidence, management and prevention. Drugs Aging. 2012;29(2):105–18.
[Crossref]

Margolius A, Fernandez HH. Current treatment of tardive dyskinesia. Parkinsonism


Relat Disord. 2019;59:155–60.
[Crossref]

Masand PS, Gupta S. Long-acting injectable antipsychotics in the elderly: guidelines


for effective use. Drugs Aging. 2003;20(15):1099–110.
[Crossref]

Maust DT, Kim HM, Seyfried LS, et al. Antipsychotics, other psychotropics, and the
risk of death in patients with dementia: number needed to harm. JAMA Psychiat.
2015;72(5):438–45.
[Crossref]

Sahli ZT, Tarazi FI. Pimavanserin: novel pharmacotherapy for Parkinson’s disease
psychosis. Expert Opin Drug Discov. 2018;13(1):103–10.
[Crossref]

Sajatovic M, Forester BP, Tsai J, et al. Ef icacy of lurasidone in adults aged 55 years
and older with bipolar depression: post hoc analysis of 2 double-blind, placebo-
controlled studies. J Clin Psychiatry. 2016;77(10):e1324–31.
[Crossref]

Schneider LS, Tariot PN, Dagerman KS, et al. Effectiveness of atypical antipsychotic
drugs in patients with Alzheimer’s disease. N Engl J Med. 2006;355(15):1525–38.
[Crossref]

Thom RP, Levy-Carrick NC, Bui M, Silbersweig D. Delirium. Am J Psychiatry.


2019;176(10):785–93.
[Crossref]

van Rensburg R, Decloedt EH. An approach to the pharmacotherapy of neuroleptic


malignant syndrome. Psychopharmacol Bull. 2019;49(1):84–91.
[PubMed][PubMedCentral]

Wu CS, Wang SC, Yeh IJ, Liu SK. Comparative risk of seizure with use of irst- and
second-generation antipsychotics in patients with schizophrenia and mood
disorders. J Clin Psychiatry. 2016;77(5):e573–9.
[Crossref]
© Springer Nature Switzerland AG 2021
R. Tampi et al. (eds.), Absolute Geriatric Psychiatry Review
https://doi.org/10.1007/978-3-030-58663-8_29

29. Anxiolytics
Mara Storto1 and Meera Balasubramaniam1
(1) Department of Psychiatry, NYU Grossman School of Medicine, New
York, NY, USA

Mara Storto
Email: mara.storto@nyulangone.org

Meera Balasubramaniam (Corresponding author)


Email: meera.balasubramaniam@nyulangone.org

Keywords Anxiolytics – Generalized anxiety disorder – Panic disorder


– Social anxiety – Speci ic phobia – Post-traumatic stress disorder
(PTSD) – Obsessive-compulsive disorder (OCD) – Benzodiazepine –
Selective serotonin reuptake inhibitors (SSRIs) – Serotonin-
norepinephrine reuptake inhibitors (SNRIs) – Tricyclic antidepressants
(TCAs) – Antipsychotics – Anticholinergic effects – Adverse effects –
Treatment augmentation – Medication taper – Off-label use

Questions
1. What class of medications is considered irst line for the
treatment of anxiety disorders in the geriatric population?
A.
Antipsychotics
B.
TCAs
C.
Benzodiazepines
D. SSRIs

E.
Antihistamines
2.
Which of the following has not been reported to be a risk
associated with the long-term use of benzodiazepine among older
adults?
A.
Psychomotor impairment
B.
Increased risk of falls
C.
Cognitive impairment
D.
Increased risk of motor vehicle accidents
E.
Orthostatic hypotension
3. Mr. Jones is a 73-year-old man with a diagnosis of generalized
anxiety disorder and a medical history of hypertension,
osteoarthritis, and obstructive sleep apnea. He presents to his
primary care physician for follow-up of management of his
generalized anxiety disorder. He was started on treatment with
escitalopram 3 months ago. On a therapeutic dose, he has
experienced improvement in some of his symptoms but states he
continues to have persistent worry and anxiety that impair his
daily functioning. What treatment strategy has been shown to be
ef icacious in augmenting the effect of SSRIs for generalized
anxiety disorder among older adults?
A.
Cognitive behavioral therapy (CBT)
B.
Aripiprazole
C.
Gabapentin
D. Benzodiazepines

E.
Dialectical behavioral therapy (DBT)
4.
Mrs. Smith is a 68-year-old woman with a history of major
depressive disorder, hypertension, type 2 diabetes mellitus, and
obesity. She presents to her outpatient psychiatrist with distinct
episodes of impairing anxiety characterized by heart palpitations,
diaphoresis, tremors, and feelings of impending doom that reach
peak severity within minutes of onset. Mrs. Smith explains that
she has been avoiding social situations because she is persistently
worried that she will have another episode. She denies any
current or past use of psychoactive substances. Medical causes for
her presenting symptoms have been excluded. Which of the
following medications would be considered irst line for the
treatment of this patient’s symptoms?
A.
Sertraline 25 mg daily
B.
Bupropion 150 mg daily
C.
Mirtazapine 30 mg daily
D.
Alprazolam 1 mg twice daily as needed
E.
Buspirone 10 mg twice daily
5. Mary is a 73-year-old woman who initially presented to her
primary care doctor 10 months ago with complaints of weight
loss and abnormal bowel movements. Following an extensive
work-up, she was diagnosed with stage III colon cancer. Since that
point in time, she has had worsening anxiety and insomnia,
causing functional impairment and multiple missed
chemotherapy treatments due to the fear of leaving her home. Her
primary care doctor referred her to you for evaluation and
p y y
treatment of her anxiety symptoms. Which of the following
medications would be the most appropriate recommendation for
the treatment of her symptoms of anxiety?
A.
Bupropion
B.
Mirtazapine
C.
Lorazepam
D.
Olanzapine
E.
Gabapentin
6.
Peter is an 82-year-old man with a history of generalized anxiety
disorder managed with paroxetine 30 mg daily. Which of the
following side effects would you be less concerned about
monitoring for in an older adult who is on a stable dose of
paroxetine?
A.
Urinary retention
B.
Dry mouth
C.
Weight loss
D.
Constipation
E.
Increased sedation
7. Mrs. Adams is an 89-year-old woman who has recently
transitioned to an assisted living facility after having lived
independently since the death of her spouse 15 years ago. Since
moving to the facility, she has been noted to be restless and pacing
the hallways throughout the day. When approached by staff, she
has dif iculty verbalizing her needs Her family explains that this
has dif iculty verbalizing her needs. Her family explains that this
behavior is far from her baseline, and they request that she has a

medical evaluation. Her physical examination, brain imaging, and


laboratory results are all normal. Which of the following would be
the best irst-choice medication to help alleviate the patient’s
symptoms?
A.
Imipramine
B.
Escitalopram
C.
Haloperidol
D.
Selegiline
E.
Clonazepam
8. Robert is an 80-year-old man who has come to you to establish
psychiatric care . He was previously followed by a physician who
prescribed him alprazolam 0.5 mg as needed for the past 10 years.
He reports taking alprazolam twice a day and has recently had
worsening anxiety. On review of symptoms, he reports multiple
falls at his home for the past month where he lives with his
daughter. What would be the next best step in management of this
gentleman’s anxiety?
A.
Increase alprazolam dose.
B.
Increase alprazolam frequency.
C.
Increase alprazolam dose and start an SSRI.
D.
Start an SSRI and gradually taper alprazolam.
E.
Transfer to an assisted living facility for placement
9.
What would be the recommended method of tapering alprazolam
in an 80-year-old man who has been prescribed alprazolam 0.5
mg as needed for the past 10 years for his symptoms of anxiety?
A.
Gradual dose reduction
B.
Abrupt discontinuation
C.
Conversion to equivalent dose of diazepam and then gradual
dose reduction
D.
Abrupt discontinuation followed by initiation of gabapentin
E.
A and C

10.
Which of the following medications that are used for the
treatment of anxiety in older adults has been shown to have the
greatest association with development of osteoporosis?
A.
Citalopram
B.
Buspirone
C.
Clonazepam
D.
Hydroxyzine
E.
Mirtazapine
11. Which of the following medications used for the treatment of
anxiety has also been shown to have a role in treatment of alcohol
withdrawal ?
A.
Citalopram
p
B.
Buspirone
C.
Gabapentin

D. Bupropion

E.
Mirtazapine
12.
When prazosin is used to treat a 68-year-old male Vietnam War
veteran with symptoms of PTSD, which of the following symptoms
would this medication improve?
A.
Nightmare
B.
Hyperarousal symptoms
C.
Dream content
D.
Sleep quality
E.
All of the above
13. Mrs. Jones is an 85-year-old woman with late-onset generalized
anxiety disorder who was referred to you by her psychotherapist.
The patient explains that she has had continued symptoms
despite weekly therapy and is interested in medication
management. You decide to trial her on buspirone for her
symptoms. Which of the following is the most commonly reported
adverse effect of buspirone from randomized clinical trials when
compared to placebo?
A.
Weight loss
B.
Hyponatremia
C
C.
Thrombocytopenia
D.
Dizziness
E. Hyperthyroidism

14.
Henry is a 65-year-old man with a diagnosis of obsessive
compulsive disorder who comes to your of ice to discuss options
for medication management. His past medical history includes
hypertension, diabetes, benign prostatic hyperplasia (BPH), and
gastroesophageal re lux disease (GERD). Of the following
medications, which would you not recommend, due to concerns
about worsening the patient’s lower urinary tract symptoms?
A.
Sertraline
B.
Fluoxetine
C.
Paroxetine
D.
Escitalopram
E.
Citalopram
15. Which of the following SSRIs would be considered as a preferred
agent for the treatment of anxiety disorders among older adults?
A.
Paroxetine
B.
Fluoxetine
C.
Fluvoxamine
D.
Sertraline
Sertraline
E.
Choices A and B

Answers
Answer 1D SSRIs are considered the irst line for treatment of
anxiety disorders among older adults. Benzodiazepines and
antihistamines are not considered irst line and are generally avoided in
this patient population due to increased adverse effects.
Benzodiazepines are associated with an increased risk of falls, delirium,
and cognitive impairment. Antihistamines often have anticholinergic
activity and have been associated with increased risk of delirium,
urinary retention, and cognitive impairment.

Answer 2E The Beers Criteria for potentially inappropriate


medication use in older adults recommends against the long-term use
of benzodiazepines in the geriatric population due to a greater risk of
signi icant adverse effects. The most concerning adverse effects include
the risk of cognitive impairment, risk of psychomotor impairment
leading to a higher rate of motor vehicle accidents, and increased risk of
falls. There is evidence suggesting that benzodiazepine use increases
the risk of orthostatic hypotension.

Answer 3A CBT is a recommended and safe augmentation strategy


for generalized anxiety disorder among older adults who are with an
SSRI medication. Aripiprazole has been shown to be an effective
augmentation strategy for patients with major depressive disorder, but
has not been studied for anxiety disorders. Benzodiazepines are not a
recommended treatment for long-term use in the geriatric population
due to a high risk of signi icant adverse effects. Gabapentin has been
used in off-label use for anxiety disorders, but studies have been
inconclusive in its use as an augmentation strategy with SSRIs.

Answer 4A This individual has symptoms consistent with a diagnosis


of panic disorder. First-line medications for the treatment of panic
disorder include SSRI or SNRI antidepressants. Therefore, the most
appropriate initial treatment for this woman would be sertraline at 25
mg daily.

Answer 5B The patient has symptoms of anxiety including fear of


leaving her home, sleep disturbances, and persistent worry causing
signi icant distress and resulting in impairment of her functioning.
Because of the degree of distress and impairment, there is a role for
medication management in this case. Of the listed options, mirtazapine
would be the best choice as it can stimulate appetite and improve sleep
in addition to targeting the core symptoms of anxiety.

Answer 6C Paroxetine is an FDA approved for use in generalized


anxiety disorder and panic disorder, but it has been associated with
more weight gain. Paroxetine is the SSRI with the most anticholinergic
side effects which may lead to dry mouth, constipation, blurry vision,
and urinary retention as well as confusion among older individuals.
Paroxetine is also a potent inhibitor of the cytochrome P450 2D6
hepatic pathway and has potential for drug interaction.

Answer 7B First-line pharmacotherapy for anxiety symptoms among


older adults would be an SSRI antidepressant such as escitalopram.

Answer 8D In this case the most appropriate course of action would


be to gradually taper the alprazolam given the increased frequency of
falls recently and initiate an SSRI antidepressant to target his
worsening anxiety symptoms.

Answer 9E When discontinuing a benzodiazepine after long-term


use, it is important to taper it gradually to avoid withdrawal symptoms
such as rebound anxiety, autonomic instability, perceptual
disturbances, and seizures. The two ways to taper benzodiazepines
include a gradual dose reduction until discontinuation or the
conversion to an equivalent dose of diazepam followed by a gradual
taper. The recommended protocol for gradual taper generally is a 25%
dose reduction every 2 weeks, although it is patient dependent and will
need to be evaluated on a case-by-case basis.
Answer 10A Available evidence indicates a strong association
between SSRIs and the development of osteoporosis, leading to
increased risk of fracture. Hypotheses for this inding include
mechanisms driven by serotonin receptors on osteoblasts and
osteoclasts.

Answer 11C Gabapentin is used off-label for the treatment of anxiety


disorders. It has been shown to have an additional role for off-label use
as an adjunctive agent for the treatment of alcohol withdrawal.

Answer 12E A meta-analysis that evaluated the role of prazosin in


the treatment of post-traumatic stress disorder (PTSD) found that
prazosin improved nightmares (standardized mean difference
[SMD] = 1.01), overall PTSD symptoms (SMD = 0.77), clinical global
improvement (SMD = 0.94, sleep quality (SMD = 0.87), hyperarousal
symptoms (SMD = 1.04), dream content (SMD = 1.33), and total sleep
time (60.98 minutes) when compared to the placebo/control groups.
Prazosin is fairly well tolerated with adverse effects which were similar
between the prazosin and placebo groups.

Answer 13B The more commonly noted adverse effect of buspirone


when compared to placebo from randomized controlled trials is
dizziness (12%) which is followed by nausea, headache, nervousness,
lightheadedness, and excitement.

Answer 14C SSRIs are irst-line pharmacologic treatments for OCD


given their demonstrated ef icacy and tolerability across different trials.
Meta-analyses of placebo-controlled head-to-head trials have failed to
demonstrate meaningful differences in ef icacy between different
agents, indicating that all SSRIs may be equally effective in treating
OCD. However, paroxetine is listed in the Beers Criteria as a potentially
inappropriate medication for use in the geriatric patient population
due to its anticholinergic activity. Anticholinergic side effects include
urinary retention, dry mouth, dry eyes, increased sedation, delirium,
blurry vision, and constipation. In this case, for a patient with a history
of BPH, paroxetine should be avoided as it would increase the risk of
worsening the patient’s lower urinary tract symptoms.
Answer 15D Sertraline, citalopram, and escitalopram would be
considered as preferred agents for the treatment of anxiety disorders
among older adults given their ef icacy and acceptable adverse effect
pro ile. Paroxetine is the SSRI with the most anticholinergic side effects
which may lead to dry mouth, constipation, blurry vision, and urinary
retention as well as confusion in older adults. In addition, paroxetine is
a potent inhibitor of the cytochrome P4502D6 hepatic pathway and
hence has considerable potential for drug interactions among those
individuals taking other medications. Fluoxetine and luvoxamine are
both signi icant inhibitors of select P450 hepatic pathways. For these
reasons, paroxetine, luoxetine, and luvoxamine may not be the ideal
agents for the treatment of anxiety disorders among older adults.

Further Reading
Ahmed S, Bachu R, Kotapati P, et al. Use of gabapentin in the treatment of substance
use and psychiatric disorders: a systematic review. Front Psych. 2019;10:228.
[Crossref]

Airagnes G, Pelissolo A, Lavallé e M, Flament M, Limosin F. Benzodiazepine misuse in


the elderly: risk factors, consequences, and management. Curr Psychiatry Rep.
2016;18(10):89.
[Crossref]

Andreescu C, Lee S. Anxiety disorders in the elderly. Adv Exp Med Biol.
2020;1191:561–76.
[Crossref]

Brown MJ, Mezuk B. Brains, bones, and aging: psychotropic medications and bone
health among older adults. Curr Osteoporos Rep. 2012;10(4):303–11.
[Crossref]

BuSpar. https://www.accessdata.fda.gov/drugsatfda_docs/label/2010/
018731s051lbl.pdf. Accessed 29 May 2020.

By the 2019 American Geriatrics Society Beers Criteria® Update Expert Panel.
American Geriatrics Society 2019 Updated AGS Beers criteria® for potentially
inappropriate medication use in older adults. J Am Geriatr Soc. 2019;67(4):674–94.
[Crossref]

Carvalho AF, Sharma MS, Brunoni AR, et al. The safety, tolerability and risks
associated with the use of newer generation antidepressant drugs: a critical review
of the literature. Psychother Psychosom. 2016;85(5):270–88.
[Crossref]

Crocco EA, Jaramillo S, Cruz-Ortiz C, Cam ield K. Pharmacological management of


anxiety disorders in the elderly. Curr Treat Options Psychiatry. 2017;4(1):33–46.
[Crossref]

Dou C, Rebane J, Bardal S. Interventions to improve benzodiazepine tapering success


in the elderly: a systematic review. Aging Ment Health. 2019;23(4):411–6.
[Crossref]

Gould RL, Coulson MC, Patel N, et al. Interventions for reducing benzodiazepine use
in older people: meta-analysis of randomised controlled trials. Br J Psychiatry.
2014;204(2):98–107.
[Crossref]

Hirschtritt ME, Bloch MH, Mathews CA. Obsessive-compulsive disorder: advances in


diagnosis and treatment. JAMA. 2017;317(13):1358–67.
[Crossref]

Paquin AM, Zimmerman K, Rudolph JL. Risk versus risk: a review of benzodiazepine
reduction in older adults. Expert Opin Drug Saf. 2014;13(7):919–34.
[Crossref]

Ramos K, Stanley MA. Anxiety disorders in late life. Clin Geriatr Med.
2020;36(2):237–46.
[Crossref]

Singh B, Hughes AJ, Mehta G, et al. Ef icacy of prazosin in posttraumatic stress


disorder: a systematic review and meta-analysis. Prim Care Companion CNS Disord.
2016;18(4). https://doi.org/10.4088/PCC.16r01943
© Springer Nature Switzerland AG 2021
R. Tampi et al. (eds.), Absolute Geriatric Psychiatry Review
https://doi.org/10.1007/978-3-030-58663-8_30

30. Hypnotics
Rakin Hoq1
(1) Department of Child and Adolescent Psychiatry, NYU Grossman
School of Medicine, New York, NY, USA

Rakin Hoq
Email: rakin.hoq@nyulangone.org

Keywords Insomnia – Delirium – Z-drugs – Benzodiazepines –


Benzodiazepine receptor – GABA – Melatonin receptor – Orexin
receptor

Questions
1. The spouse of a 72-year-old man with Parkinson’s disease brings
him in for an evaluation of his sleep. She says that at night, he
lails around in his sleep while screaming. This issue is reported to
occur on most nights. The patient does state that he has very vivid
dreams that are often disturbing and feels as if they are real to
him. His spouse has given him “heavy” doses of melatonin which
have not seemed to help. Which of the following treatment
options could be of potential bene it for this patient’s symptoms?
A.
Quetiapine
B.
Diphenhydramine
C.
Zolpidem
D. Clonazepam
E.
Mirtazapine
2.
A 75-year-old woman with a known history of delirium is now
admitted to the hospital with a urinary tract infection. The patient
and her family are concerned about her change in mental status
due to similar presentations in the past when she becomes ill.
Which of the following treatments have the potential to prevent
the development of a delirium in this woman?
A.
Lorazepam
B.
Zolpidem
C.
Ramelteon
D.
Haloperidol
E.
Trazodone
3.
Which of the following benzodiazepines has the longest half-life?
A.
Alprazolam
B.
Lorazepam
C.
Clonazepam
D.
Diazepam
E.
Oxazepam
4. Which of the following hypnotic drugs has the shortest half-life?
A Z l id
A. Zolpidem

B.
Zaleplon
C.
Zopiclone
D.
Eszopiclone
E.
Temazepam
5.
Which of the following is true of how benzodiazepines affect sleep
architecture?
A.
Decrease in REM sleep
B.
Decrease in stages 1 and 2 sleep
C.
Increase in stages 3 and 4 sleep
D.
Increased sleep latency
E.
Increased REM sleep
6. What is the mechanism by which zolpidem and related drugs (Z-
drugs) work?
A.
Inverse agonism at GABAB receptor
B.
Selective agonism at GABAA receptor
C.
NMDA receptor antagonism
D.
Inverse agonism at NMDA receptors
E.
GABA receptor antagonism
7.
Which of the following hypnotic medications exhibits activity at
orexin receptors?
A.
Ramelteon
B.
Chlordiazepoxide
C.
Eszopiclone
D.
Suvorexant
E.
Temazepam

8.
Which of the following medications would be the safest choice for
an 81-year-old man with a diagnosis of mild neurocognitive
disorder and chronic obstructive pulmonary disease who is
complaining of dif iculty falling asleep?
A.
Zopiclone
B.
Zaleplon
C.
Ramelteon
D.
Temazepam
E.
Flurazepam
9. A healthy 70-year-old woman presents with a complaint of
interrupted sleep . She states that she can fall asleep without
dif iculty but wakes up several times in the night causing her to be
sleepy and tired during the day. She cannot think of any physical
sensations disturbing her sleep. Which of the following
se sat o s d stu b g e s eep W c o t e o ow g
medications would be the most suitable choice to help her with
this issue?
A. Eszopiclone

B.
Zaleplon
C.
Ramelteon
D.
Zolpidem
E.
Triazolam
10.
Which of the following benzodiazepines is not approved for the
short-term treatment of insomnia by the US FDA?
A.
Triazolam
B.
Estazolam
C.
Temazepam
D.
Flurazepam
E.
Diazepam

Answers
Answer 1D The patient is experiencing symptoms associated with
REM sleep behavior disorder (RBD) which is highly prevalent in
Parkinson’s disease and dementia with Lewy bodies. Melatonin is often
used to help alleviate RBD symptoms among older adults due to its
favorable tolerability pro ile. However, this has already been tried by
the patient’s spouse without signi icant effect. Moderate dosages of
clonazepam have been demonstrated to provide similar effectiveness in
treating RBD when compared to melatonin and would be an
appropriate medication to trial while also monitoring for adverse
events related to benzodiazepine usage among older individuals.

Answer 2C Ramelteon is a melatonin agonist that has been shown to


reduce the incidence of delirium. Benzodiazepines may increase the
risk of the patient developing delirium and paradoxical agitation due to
its amnestic effect. Zolpidem works similarly to benzodiazepines by
modulating GABA neurotransmission and would also not be an
appropriate choice. Due to the unfavorable safety pro ile associated
with antipsychotic agents (e.g., haloperidol) in the elderly, they are not
given prophylactically.

Answer 3D Of the answers listed, diazepam has the longest half-life


followed by clonazepam, lorazepam, and then alprazolam. When using
these medications among older adults, half-life is an important
consideration as the metabolism of these drugs is slower in older adults
resulting in cumulative toxicity.

Answer 4B Of the non-benzodiazepine sedative hypnotic drugs


listed, zaleplon has the shortest half-life of about 1 hour. This can be an
advantage for older adults who have dif iculty falling asleep. Due to the
very short half-life, it can be advantageous in reducing residual
sedation and cognitive side effects if taken early enough before
waketime.

Answer 5A Benzodiazepines are among the most common


pharmacological agents prescribed for insomnia in the general
population, including older adults. However, they are known to cause
alterations in sleep architecture including decreased sleep latency
(commonly the most desired effect), increase stages 1 and 2 sleep,
decrease stages 3 and 4 sleep, and decrease REM sleep.

Answer 6B All non-benzodiazepine hypnotics (zolpidem, zaleplon,


zopiclone, and eszopiclone) work through agonism at various GABAA
receptor subtypes and thereby potentiate the inhibitory effects of
GABA. Inverse agonism at a GABA receptor would reduce the output at
that receptor theoretically producing the opposite effect of hypnotic
drugs. None of these hypnotic medications affect the NMDA receptor.

Answer 7D Suvorexant is a novel hypnotic medication released in


2014 that works through antagonism at orexin receptor-1 and orexin
receptor-2. Neural projections from orexin receptor sites have been
shown to promote wakefulness, while antagonism at these sites
promotes sleep. This new class of medications has been under
signi icant research and development for the past two decades.
Ramelteon is an agonist at melatonin receptors. Chlordiazepoxide is a
relatively longer-acting benzodiazepine that is sometimes used off-
label as a sleep aid. Eszopiclone is one of the non-benzodiazepine
hypnotic drugs that works through GABAA agonism.

Answer 8C Among the listed choices, ramelteon would be the safest


option. Ramelteon is a melatonin receptor agonist and has not been
shown to be associated with the cognitive impairment, increased risk of
falls, and respiratory depression associated with benzodiazepines
(temazepam) and non-benzodiazepine (zaleplon, zopiclone) hypnotics.

Answer 9A Primary insomnia is often divided into two categories:


sleep onset insomnia and sleep maintenance insomnia. The patient in
this vignette presents with sleep maintenance insomnia. Of the listed
choices, eszopiclone is the only hypnotic agent that has been approved
to speci ically treat sleep maintenance insomnia, while the other
choices primarily treat only sleep onset insomnia.

Answer 10E Benzodiazepines approved by the FDA for the treatment


of short-term insomnia include triazolam, estazolam, temazepam,
lurazepam, and quazepam.

Further Reading
Aronson JK. Benzodiazepines. In: Meyler’s side effects of drugs. 16th ed. Amsterdam:
Elsevier Science; 2015. p. 863–77.

Brenner GM, Stevens CW. Sedative-hypnotic and anxiolytic drugs. In: Brenner and
Stevens’ pharmacology. 5th ed. Philadelphia: Elsevier, Inc; 2017. p. 205–16.

Edmonds C, Swanoski M. A review of suvorexant, doxepin, ramelteon, and


tasimelteon for the treatment of insomnia in geriatric patients. Consult Pharm.
2017;32(3):156–60.
[Crossref]

Inouye SK. Delirium in the older patient. In: Goldman-Cecil medicine. 26th ed.
Philadelphia: Elsevier, Inc; 2019. p. 113–7.

Kilduff TS, Mendelson WB. Hypnotic medications: mechanisms of action and


pharmacologic effects. In: Principles and practice of sleep medicine. 6th ed.
Philadelphia: Elsevier, Inc; 2016. p. 424–31.

Kirkwood C, Breden E. Management of insomnia in elderly patients using


eszopiclone. Nat Sci Sleep. 2010;2:151–8.
[Crossref]

Levy HB. Non-benzodiazepine hypnotics and older adults: what are we learning
about zolpidem? Expert Rev Clin Pharmacol. 2014;7(1):5–8.
[Crossref]

Marcantonio ER. Delirium in hospitalized older adults. N Engl J Med.


2017;377(15):1456–66.
[Crossref]

Posner D. Insomnia. In: Ferri’s clinical advisor 2020. 1st ed. Philadelphia: Elsevier;
2019. p. 796–8.

Roehrs TA, Diederichs C, Roth T. Pharmacology of benzodiazepine receptor agonist


hypnotics. In: Therapy in sleep medicine. 1st ed. Philadelphia: Saunders; 2011. p. 99–
108.

Schroeck JL, Ford J, Conway EL, et al. Review of safety and ef icacy of sleep medicines
in older adults. Clin Ther. 2016;38(11):2340–72.
[Crossref]

St Louis EK, Boeve BF. REM sleep behavior disorder: diagnosis, clinical implications,
and future directions. Mayo Clin Proc. 2017;92(11):1723–36.
[Crossref]

Tampi RR, Manikkara G, Balachandran S, et al. Suvorexant for insomnia in older


adults: a perspective review. Drugs Context. 2018;7:212517.
[Crossref]
Trenkwalder C, Arnulf I, Postuma R. Parkinsonism. In: Principles and practice of
sleep medicine. 6th ed. Philadelphia: Elsevier, Inc; 2016. p. 892–902.
© Springer Nature Switzerland AG 2021
R. Tampi et al. (eds.), Absolute Geriatric Psychiatry Review
https://doi.org/10.1007/978-3-030-58663-8_31

31. Cognitive Enhancers


Rabin Dahal1 and Rajesh Tampi2
(1) Yale New Haven Hospital, New Haven, CT, USA
(2) Department of Medicine, Cleveland Clinic Lerner College of
Medicine of Case Western Reserve University, Cleveland, OH, USA

Rabin Dahal (Corresponding author)


Email: rabin.dahal@yale.edu

Keywords Acetylcholinesterase inhibitors – Donepezil – Rivastigmine


– Galantamine – NMDA antagonist – Memantine

Questions
1.
Which of the following medications is FDA approved for the
treatment of mild neurocognitive disorder?
A.
Donepezil
B.
Galantamine
C.
Rivastigmine
D.
Memantine
E.
None of the above
2. Which of the following statements is true of the ef icacy of
acetylcholinesterase inhibitors for the treatment of Alzheimer’s
y

disease (AD)?
A.
Donepezil > rivastigmine > galantamine
B.
Donepezil = rivastigmine = galantamine
C.
Galantamine > rivastigmine > donepezil
D.
Rivastigmine > galantamine > donepezil
E.
Rivastigmine > donepezil > galantamine
3.
Which of the following is the mechanism of action for
rivastigmine?
A.
Inhibition of acetylcholinesterase
B.
Inhibition of butyrylcholinesterase
C.
Inhibition of monoamine oxidase
D.
Choices A and B
E.
Choices A and C
4. Which of the following is a false statement regarding
galantamine?
A.
It is a reversible competitive inhibitor of acetylcholinesterase
B.
It has strong butyrylcholinesterase inhibitory activity
C.
It is an allosteric modulator at nicotinic cholinergic receptor
site
D. It is FDA approved for the treatment of mild-to-moderate
pp
Alzheimer’s disease

E.

It mainly causes gastrointestinal adverse effects


5.
Which of the following medications is FDA approved for the
treatment of mild, moderate, and severe stages of Alzheimer’s
disease (AD)?
A.
Donepezil
B.
Galantamine
C.
Rivastigmine
D.
Memantine
E.
A and C
6.
Which of the following is the most common adverse effect from
the use of an acetylcholinesterase inhibitor in the treatment of
individuals with Alzheimer’s disease (AD)?
A.
Urinary incontinence
B.
Dizziness
C.
Anorexia
D.
Falls
E.
Nausea
7. The prescription of which of the following medications would be
relatively contraindicated with a prescription of rivastigmine in
relatively contraindicated with a prescription of rivastigmine in
an individual with a diagnosis of Alzheimer’s disease (AD)?
A. Hydrochlorothiazide

B.
Atenolol
C.
Pravastatin
D.
Memantine
E.
Levodopa
8.
Which of the following is the proposed mechanism of action for
memantine?
A.
NMDA receptor inverse agonism
B.
AMPA receptor antagonism
C.
Methylation of calcium in lux channels
D.
Inactivation of overactive NMDA receptors by binding to the
magnesium binding site
E.
All of the above
9. Which of the following is the most common adverse effect from
the use of memantine to treat individuals with Alzheimer’s
disease (AD)?
A.
Urinary incontinence
B.
Dizziness
C.
Anorexia
D.
Falls

E. Diarrhea

10.
A 78-year-old man with a diagnosis of with severe Alzheimer’s
type dementia, hypertension, hyperlipidemia, chronic obstructive
pulmonary disease, and diabetes mellitus is brought to the
hospital for having reduced oral luid intake. His laboratory values
in the emergency department (ED) were remarkable for an
elevated BUN of 45 mg % and high creatinine of 2.5 mg/dl
(baseline 1.0 mg/dl), a calculated creatinine clearance of
21 ml/min, and a blood glucose of 71 mg%. The rest of his
laboratory values are normal. The patient’s medication list is as
follows: aspirin 81 mg orally daily, atorvastatin 40 mg orally at
bedtime, amlodipine 5 mg orally daily, donepezil 10 mg orally
daily, Lantus insulin 20 units as bedtime subcutaneously and
10 units in the morning subcutaneously with sliding scale
correction, memantine 10 mg orally twice daily, and albuterol
inhaler one spray four times daily as needed for shortness of
breath. In addition to adjusting the insulin dose, what other
medication dosing adjustment would be necessary for this
individual at discharge from the hospital?
A.
Lower the dose of donepezil to 5 mg orally daily
B.
Lower the dose of memantine to 5 mg orally twice daily
C.
Discontinue both donepezil and memantine
D.
Discontinue memantine only
E.
Discontinue donepezil only
A 68-year-old male was referred to your clinic for evaluation of
A 68 year old male was referred to your clinic for evaluation of
11. cognitive decline. After a thorough evaluation, he is diagnosed
with mild dementia from Alzheimer’s disease without behavioral
symptoms. Which of the following treatment options is not an
appropriate choice for the initial treatment of this gentleman?

A. Oral galantamine

B.
Oral donepezil
C.
Oral rivastigmine
D.
Oral memantine
E.
Transdermal rivastigmine
12.
A 74-year-old man with Alzheimer’s disease (AD) was referred to
your clinic for a cognitive evaluation. He is currently prescribed
donepezil 10 mg orally daily. On assessment, he was found to have
moderate dementia without any neuropsychiatric symptoms.
Which of the following is the best pharmacological management
for this gentleman?
A.
Discontinue donepezil in order to minimize the
pharmacological burden
B.
Discontinue donepezil and start memantine
C.
Add memantine to the donepezil
D.
Reduce the dose of donepezil to 5 mg orally once daily and
start memantine
E.
Add escitalopram 10 mg orally daily to donepezil
13. Which of the following medications does not increase the plasma
level of memantine?
A.
Cimetidine
B.
Warfarin
C. Quinidine

D.
Nicotine
E.
Quinidine
14.
Which of the following medications is FDA approved for the
treatment of Parkinson’s disease dementia (PDD)?
A.
Donepezil
B.
Galantamine
C.
Rivastigmine
D.
Memantine
E.
Pimavanserin
15. Which of the following medications has been found to be
bene icial among individuals with Lewy body dementias?
A.
Donepezil
B.
Memantine
C.
Levodopa
D.
Melatonin
E.
Clonazepam

Answers
Answer 1E Currently there are no FDA-approved medications for the
treatment of mild neurocognitive disorder.

Answer 2B There is no evidence to suggest superiority or inferiority


of any of the three acetylcholinesterase inhibitors in comparison to
each other in their ef icacy for the treatment of Alzheimer’s disease
(AD).

Answer 3D Rivastigmine is a unique cholinesterase inhibitor with


both acetylcholinesterase (AchE) and butyrylcholinesterase (BuChE)
inhibitory activity.

Answer 4B Galantamine is a reversible, competitive inhibitor of


acetylcholinesterase with very little butyrylcholinesterase inhibitory
activity. It is also an allosteric modulator at nicotinic cholinergic
receptor sites. Galantamine is FDA approved for use in the treatment of
mild-to-moderate Alzheimer’s disease (AD).

Answer 5E Donepezil and rivastigmine are FDA approved for the


treatment of all three stages of Alzheimer’s disease (AD).

Answer 6E The most common adverse effects from the use of an


acetylcholinesterase inhibitor for treating individuals with AD are
gastrointestinal symptoms (nausea and vomiting, diarrhea, and
abdominal pain), anorexia, and muscle cramp followed by weight loss
and urinary incontinence.

Answer 7B The combination of rivastigmine and atenolol can


increase the risk for bradycardia and syncope. Rivastigmine can
exacerbate the effect of beta blockers by its cholinergic effect.

Answer 8D Memantine acts as a noncompetitive antagonist at the


NMDA receptor site. It binds to the magnesium binding site, and
magnesium limits calcium in lux and thereby limiting calcium-induced
excitotoxicity and cell death.

Answer 9B The most common adverse effects from the use of


memantine for treating individuals with AD are dizziness, headache,
confusion, and constipation.

Answer 10B Approximately 75% of memantine is excreted in the


urine. In renal failure, this process is impaired and results in the toxic
accumulation of memantine. The recommendation is to renally dose
memantine once the patient’s creatinine clearance is less than
30 ml/minute. Donepezil does not need a renal dosing. Neither
medication should be stopped as this will result in a cognitive and
functional decline in the individual.

Answer 11D The use of memantine should be reserved for the


treatment of moderate-to-severe Alzheimer’s disease.

Answer 12C Memantine is approved for moderate-to-severe


Alzheimer’s dementia and is often used in combination with
acetylcholinesterase inhibitors. As this gentleman does not have any
neuropsychiatric symptoms including depression, escitalopram is not
indicated in his case.

Answer 13B Increased plasma levels of memantine can occur when


coadministered with amantadine, cimetidine, ranitidine, procainamide,
quinidine, quinine, and nicotine. Serum hydrochlorothiazide levels may
be reduced when coadministration with memantine. Memantine can
increase the level of warfarin when coadministered.

Answer 14C Only rivastigmine is approved by the US Food and Drug


Administration (FDA) for the treatment of PDD.

Answer 15A Donepezil and rivastigmine have shown some bene it in


the treatment of individuals with dementia with Lewy bodies.

Further Reading
Aricept-FDA. https://www.accessdata.fda.gov/drugsatfda_docs/label/2018/
020690s042,021720s014,022568s011lbl.pdf. Accessed 1 Apr 2020.

Birks J. Cholinesterase inhibitors for Alzheimer’s disease. Cochrane Database Syst


Rev. 2006;(1):CD005593.

Birks JS, Harvey RJ. Donepezil for dementia due to Alzheimer’s disease. Cochrane
Database Syst Rev. 2018;6:CD001190.
[PubMed]

EXELON® PATCH (rivastigmine transdermal system). https://www.accessdata.fda.


gov/drugsatfda_docs/label/2015/022083s020lbl.pdf. Accessed 2 Apr 2020.

Folch J, Busquets O, Ettcheto M, et al. Memantine for the treatment of dementia: a


review on its current and future applications. J Alzheimers Dis. 2018;62(3):1223–40.
[Crossref]

Gauthier S, Molinuevo JL. Bene its of combined cholinesterase inhibitor and


memantine treatment in moderate-severe Alzheimer’s disease. Alzheimers Dement.
2013;9(3):326–31.
[Crossref]

Goldman JG, Sieg E. Cognitive impairment and dementia in Parkinson disease. Clin
Geriatr Med. 2020;36(2):365–77.
[Crossref]

Johnson JW, Kotermanski SE. Mechanism of action of memantine. Curr Opin


Pharmacol. 2006;6(1):61–7.
[Crossref]

Kandiah N, Pai MC, Senanarong V, et al. Rivastigmine: the advantages of dual


inhibition of acetylcholinesterase and butyrylcholinesterase and its role in
subcortical vascular dementia and Parkinson’s disease dementia. Clin Interv Aging.
2017;12:697–707.
[Crossref]

Loy C, Schneider L. Galantamine for Alzheimer’s disease. Cochrane Database Syst Rev.
2004;(4):CD001747.

Namenda – FDA. https://www.accessdata.fda.gov/drugsatfda_docs/label/2013/


021487s010s012s014,021627s008lbl.pdf. Accessed 1 Apr 2020.

Pasqualetti G, Tognini S, Calsolaro V, et al. Potential drug-drug interactions in


Alzheimer patients with behavioral symptoms. Clin Interv Aging. 2015;10:1457–66.
[PubMed][PubMedCentral]
Paulison B, Lé os CL. Potential cardiotoxic reaction involving rivastigmine and beta-
blockers: a case report and review of the literature. Cardiovasc Toxicol.
2010;10(4):306–10.
[Crossref]

RAZADYNE (galantamine hydrobromide) – FDA. https://www.accessdata.fda.gov/


drugsatfda_docs/label/2017/021169Orig1s032,021224Orig1s030,021615Orig1s023l
bl.pdf. Accessed 2 Apr 2020.

Vega JN, Newhouse PA. Mild cognitive impairment: diagnosis, longitudinal course,
and emerging treatments. Curr Psychiatry Rep. 2014;16(10):490.
[Crossref]

Walker Z, Possin KL, Boeve BF, et al. Lewy body dementias. Lancet.
2015;386(10004):1683–97.
[Crossref]

Wong CW. Pharmacotherapy for dementia: a practical approach to the use of


cholinesterase inhibitors and memantine. Drugs Aging. 2016;33(7):451–60.
[Crossref]
© Springer Nature Switzerland AG 2021
R. Tampi et al. (eds.), Absolute Geriatric Psychiatry Review
https://doi.org/10.1007/978-3-030-58663-8_32

32. Stimulants
Rakin Hoq1
(1) Department of Child and Adolescent Psychiatry, NYU Grossman
School of Medicine, New York, NY, USA

Rakin Hoq
Email: rakin.hoq@nyulangone.org

Keywords Stimulants – Methylphenidate – Amphetamine – Moda inil –


Fatigue – Depression

Questions
1. A 71-year-old man in your clinic who was diagnosed with major
neurocognitive disorder due to Alzheimer’s disease and severe
depression presents for a follow-up appointment after initiating
treatment with citalopram which has been titrated to 20 mg daily
for the past 3 months. He reports he is feeling better and says he
has the motivation to paint again, but poor energy prevents him
from doing so. His family also notes that he is no longer having
crying spells and no longer speaks about death, but they remain
very concerned about his lethargic disposition stating he still lacks
the will to do anything. Which of the following treatment strategies
would be most likely to help with the concerns of this patient and
his family?
A.
Increase citalopram to 40 mg daily
B. Continue escitalopram at the current dose and add
methylphenidate
C.
Discontinue escitalopram and start aripiprazole

D.
Discontinue escitalopram and start bupropion
E.
Discontinue escitalopram and start duloxetine
2.
You are consulted to evaluate an 80-year-old woman in hospice
care due to metastatic breast cancer who has recently stopped
engaging in any of her interests or hobbies. Her family is concerned
she is depressed. Although she does not have crying spells or talk
about hastening her death but appears withdrawn and spends
most of the day in bed despite not having any physical disabilities
preventing her from normal physical functioning and reports that
she has no initiative to do anything. Which of the following
medications would be the most suitable to rapidly alleviate this
patient’s symptoms?
A.
Escitalopram
B.
Mirtazapine
C.
Duloxetine
D.
Dextroamphetamine
E.
Aripiprazole
3. Which of the following psychostimulants has been shown to have
the lowest potential for abuse?
A.
Methylphenidate
B.
Dextroamphetamine
C.
Methamphetamine
Methamphetamine
D. Moda inil

E.
Lisdexamfetamine

Answers
Answer 1B Psychostimulants such as methylphenidate could be used
as adjunctive treatment for depression in the elderly, particularly to
target symptoms of fatigue and apathy. In this vignette, reports from
the patient and his family suggest his depression has improved, but he
continues to experience signi icant fatigue and apathy that continue to
be the major concerns to both him and his family. Thus, a carefully
monitored trial of adjunctive therapy utilizing methylphenidate could
be a reasonable choice to target these symptoms. As citalopram is
currently at the maximum FDA-approved dose for adults ≥60 years, it is
unlikely that any further increase in the dosage will improve his
symptoms without increasing the risk of adverse events or side effects.
While aripiprazole may be an effective augmenting agent for treating
depression, it may not be effective in treating this man’s depression as a
monotherapy. It would not be recommended to discontinue the
citalopram as it appears to have contributed to a substantial
improvement in his depressive symptoms.

Answer 2D The patient has signi icant symptoms of depression


including anhedonia and fatigue, likely related to her cancer diagnosis.
This vignette speci ically questions which psychotropic agent would be
most effective in providing the most rapid relief for her current
symptomatology. While an antidepressant would be bene icial for her
depressive symptoms, an antidepressant would not be the most
optimal option for rapid relief of her symptoms due to the delayed
ef icacy of the irst-line antidepressant agents such as serotonin
reuptake inhibitors. A psychostimulant such as dextroamphetamine
would more likely produce a rapid therapeutic effect in targeting the
patient’s predominant symptoms of lethargy and avolition.
Antipsychotics would unlikely provide immediate relief for her
depressive symptoms.

Answer 3D Moda inil has been shown to have less association with
abuse and dependence when compared to the other psychostimulants
listed which have all been associated with signi icant abuse potential.
This is theorized to be related to moda inil producing less euphoric
effect as well as having a long half-life.

Further Reading
Aronson JK. Moda inil. In: Meyler’s side effects of drugs. 16th ed: Elsevier Science:
Waltham, MA, USA. 2015. p. 1081–3.

Cremens MC, Wilkins JM, Wiechers IR. Care of the geriatric patient. In:
Massachusetts General Hospital handbook of general hospital psychiatry. 7th ed:
Elsevier: Philadelphia, PA, USA. 2017. p. 539–45.

Fernandez-Robles CG, Irwin KE, Pirl WF, Greenberg DB. Patients with cancer. In:
Massachusetts General Hospital handbook of general hospital psychiatry. 7th ed:
Elsevier: Philadelphia, PA, USA. 2017. p. 349–58.

Greene WM, Gold MS. Drug abuse. In: Conn’s current therapy 2020. 1st ed: Elsevier:
Philadelphia, PA, USA. 2019. p. 773–8.
© Springer Nature Switzerland AG 2021
R. Tampi et al. (eds.), Absolute Geriatric Psychiatry Review
https://doi.org/10.1007/978-3-030-58663-8_33

33. Drugs for Tardive Dyskinesia


Rakin Hoq1
(1) Department of Child and Adolescent Psychiatry, NYU Grossman
School of Medicine, New York, NY, USA

Rakin Hoq
Email: rakin.hoq@nyulangone.org

Keywords Tardive dyskinesia – Valbenazine – Deutetrabenazine –


Vesicular monoamine transporter

Questions
The following two questions are related to the following prompt:
A retired 65-year-old male biochemistry professor with bipolar
disorder that has been stable on olanzapine for the last 7 years
presents to your clinic with concerns about constant movements he is
doing with his mouth. He says he did not notice them until his spouse
and children have started remarking about it for the past 6 months and
now feels very self-conscious about them. He recalls that this issue
could be related to the medication he takes for his bipolar disorder. He
inquires if there is any additional treatment that he can take to help
alleviate these symptoms.
1. Which of the following adjunctive treatments can you offer to him?
A.
Benztropine
B.
Diphenhydramine
C. Valbenazine
D.
Propranolol
E.
Lorazepam
2.
The patient inquires as to how this drug you are offering him
works. Which of the following would be the most appropriate
explanation for the mechanism of action for the drug that you
prescribe for his symptoms?
A.
It is an inhibitor of vesicular monoamine transporters
B.
It is an antagonist of acetylcholine receptors
C.
It is an inverse agonist of histamine receptors
D.
It is an antagonist of beta-adrenergic receptors
E.
It is a selective dopamine agonist

Answers
Answer 1 C The patient has developed tardive dyskinesia from the
long-term treatment with olanzapine. Of the treatments listed,
valbenazine is the only choice shown to be effective in treating tardive
dyskinesia. Thus far, there are two main pharmacologic treatments that
have been shown to be signi icantly effective for treating tardive
dyskinesia: valbenazine and deutetrabenazine. The other medication
options are not known to be as effective for treating tardive dyskinesia.

Answer 2 A Valbenazine is a selective inhibitor of the vesicular


monoamine transporter 2 (VMAT2). Benztropine produces its effect by
antagonism of acetylcholine neurotransmission. Inverse agonism of
histamine receptors is accomplished by antihistamines such as
diphenhydramine. Propranolol is a known beta-adrenergic receptor
antagonist. The choice “dopamine agonist” is included as a distractor
since dopamine agonists are utilized to treat movement disorders such
as Parkinson’s disease and are more likely to worsen tardive dyskinesia
symptoms.

Further Reading
Gray JA. Tardive dyskinesia. In: Ferri’s clinical advisor 2020. 1st ed: Elsevier: Mosbi,
Philadelphia, PA. 2019. p. 1347–7.

Margolius A, Fernandez HH. Current treatment of tardive dyskinesia. Parkinsonism


Relat Disord. 2019;59:155–60.
[Crossref]
© Springer Nature Switzerland AG 2021
R. Tampi et al. (eds.), Absolute Geriatric Psychiatry Review
https://doi.org/10.1007/978-3-030-58663-8_34

34. Adverse Effects of Drugs


Kristen Cannon1
(1) Department of Psychiatry, Ohio State University, Columbus, OH,
USA

Kristen Cannon
Email: kristen.cannon@osumc.edu

Keywords Adverse effects – Side effects – Medication management –


Drug reactions – Drug interactions – Confusion – Bleeding

Questions
1. A 72-year-old man with a history of osteoarthritis undergoes a
total hip arthroplasty. On the second postoperative day, he
develops hyperactive delirium and attempts to rip out his IV. A
second-generation antipsychotic is initiated for control of his
agitation. However, 24 hours after the initiation of this drug, the
patient develops severe urinary retention. Which of the following
is the most likely antipsychotic that was initiated in this patient?
A.
Olanzapine
B.
Quetiapine
C.
Risperidone
D.
Aripiprazole
E. Ziprasidone
2.
An 81-year-old woman with a history of major depressive
disorder, generalized anxiety, and Alzheimer’s dementia is started
on warfarin following an ischemic stroke. Her previous
medication regimen consisted of buspirone, luoxetine,
mirtazapine, hydroxyzine, and rivastigmine. Which of these
medications may increase the risk for bleeding due to an
interaction with warfarin?
A.
Buspirone
B.
Fluoxetine
C.
Mirtazapine
D.
Hydroxyzine
E.
Rivastigmine
3.
Which of the following does not increase the risk for acute lithium
toxicity ?
A.
Older age
B.
A diagnosis of nephrogenic diabetes insipidus
C.
The concomitant use of angiotensin-converting enzyme (ACE)
inhibitors
D.
The concomitant use of hydrochlorothiazide
E.
The concomitant use of acetaminophen
4. Which of the following increases the risk psychotropic-associated
hyponatremia among older adults?
A. Female gender

B.
Concomitant use of angiotensin-converting enzyme inhibitors
C.
The presence of heart failure
D.
The presence of adrenal insuf iciency
E.
All of the above
5.
An 82-year-old Vietnam War veteran is diagnosed with post-
traumatic stress disorder (PTSD) after his daughter encourages
him to see an outpatient psychiatrist. In addition to
psychotherapy, the psychiatrist recommends starting the patient
on prazosin to alleviate his frequent nightmares. The patient is
hesitant to start a new medication and asks about potential side
effects. Which of the following is not a potential adverse effect of
prazosin?
A.
Dizziness
B.
Headache
C.
Increased appetite
D.
Drowsiness
E.
Lack of energy
6. Which potential adverse effect of antipsychotic therapy is known
to be less common among older adults when compared to
younger adults?
A.
Neuroleptic malignant syndrome
Neuroleptic malignant syndrome
B. Akathisia

C.
Tardive dyskinesia
D.
Orthostatic hypotension
E.
Metabolic syndrome
7.
Which of the following tricyclic antidepressants is least likely to
cause sedation and orthostatic hypotension when used among
older adults?
A.
Nortriptyline
B.
Amitriptyline
C.
Imipramine
D.
Doxepin
E.
Clomipramine
8. A 69-year-old woman presents to her outpatient sleep specialist
for a follow-up appointment. After a failed trial of cognitive
behavioral therapy for insomnia, the patient’s physician agreed to
prescribe her a non-benzodiazepine hypnotic. However, today, the
patient tells her physician, “That medicine works great but I had
to stop taking it. I just couldn’t stand the bitter taste it left in my
mouth. Isn’t there something else I can try?” Which hypnotic was
the patient most likely prescribed?
A.
Zolpidem
B
B.
Zaleplon
C. Eszopiclone

D.
Trazodone
E.
Doxepin
9.
A 75-year-old man with a diagnosis of bipolar disorder and a
recent diagnosis of seizure disorder is seen in the emergency
department of a local hospital for sudden onset of severe right
lank pain. He is diagnosed with right nephrolithiasis. Which of
the following medications used to treat bipolar disorder and/or
seizure disorder could increase the risk for nephrolithiasis in this
individual?
A.
Divalproex sodium
B.
Carbamazepine
C.
Oxcarbazepine
D.
Lamotrigine
E.
Topiramate
10. A 71-year-old woman with no previous psychiatric history
presents to her physician requesting “something for my anxiety.”
She explains that her husband is currently in hospice care and she
is experiencing constant racing thoughts about what life will be
like without him. She states, “Sometimes it gets so bad I feel like I
can’t even breathe. I just don’t know how to get through these
next few weeks, watching him get sicker and trying to igure out
how to live on my own again.” Her physician agrees to prescribe a
short course of a benzodiazepine for her acute anxiety On routine
short course of a benzodiazepine for her acute anxiety. On routine
laboratory testing, ALT is 90 IU/L and AST is 75 IU/L. The patient
denies any physical complaints. Which benzodiazepine is the
safest choice for this patient to treat her acute anxiety symptoms?
A. Diazepam

B.
Chlordiazepoxide
C.
Oxazepam
D.
Clonazepam
E.
Lorazepam
11.
A 68-year-old man with a past medical history of schizophrenia
has been stable on a second-generation antipsychotic for much of
his life. He has also been a one-pack-per-day (ppd) smoker for the
past 40 years. However, the individual decides to cut back on his
smoking habits after the birth of his irst grandchild. His
outpatient psychiatrist decides to reduce his antipsychotic dose,
explaining that he may be more susceptible to adverse medication
effects now that he is smoking less. Which antipsychotic was this
individual most likely taking?
A.
Olanzapine
B.
Aripiprazole
C.
Quetiapine
D.
Paliperidone
E.
Risperidone
12. When prescribing an antidepressant to a 68-year-old woman with
a history of major depressive disorder and morbid obesity which
a history of major depressive disorder and morbid obesity, which
of the following antidepressants would you be cautious in using to
avoid further weight gain?
A. Mirtazapine

B.
Amitriptyline
C.
Paroxetine
D.
Bupropion
E.
Choices A, B, and C
13.
Carbidopa is known to decrease the risk of adverse effects in
patients taking levodopa for Parkinson’s disease (PD). Which of
the following adverse effects of levodopa is not alleviated by
adjunctive treatment with carbidopa?
A.
Psychosis
B.
Nausea
C.
Hypotension
D.
Choices B and C
E.
All of the above
14. A 70-year-old woman with a diagnosis of schizophrenia who is
being treated with clozapine 300 mg a day develops a seizure
disorder. Which of the following anticonvulsants should be
avoided in this individual?
A.
V l t
Valproate
B.
Carbamazepine
C.
Lamotrigine

D. Gabapentin

E.
Topiramate
15. A 77-year-old woman with a history of hyperlipidemia, diabetes
mellitus, and major depressive disorder with psychotic features
presents to her outpatient physician. Her medication regimen
currently consists of atorvastatin and metformin. She is not
currently taking any psychotropic medications and reports that
she had not experienced any signi icant psychiatric symptoms for
the past 10 years, until approximately 2 months ago. She tells her
physician, “I really thought the depression was a thing of my past.
But for the past few months I’ve just been sad all the time, and I
feel like a burden to my family. I’ve even stopped going to church
with them. And then last week I started hearing voices again. They
tell me I’m worthless and don’t deserve to live.” The physician
agrees to initiate a trial of a psychotropic medication. Her routine
lab work, including serum glucose and lipid studies, are within
normal limits. The patient’s HbA1c is 6.4%. Given this patient’s
presentation, which antidepressant would be best avoided in the
treatment of her depression?
A.
Citalopram
B.
Duloxetine
C.
Sertraline
D.
Bupropion
E.
Escitalopram
Escitalopram
16.
Which medication class is associated with increased risk for death
when used among older adults with dementia resulting in an FDA
boxed warning?
A.
Typical antipsychotics
B.
Atypical antipsychotics
C.
Antidepressants
D.
Choices A and B
E.
Choices A, B, and C

17.
Which of the following mechanisms is not associated with
antipsychotic-associated metabolic side effects including weight
gain?
A.
H1 receptor antagonism
B.
M3 receptor antagonism
C.
5-HT 1A receptor partial agonism
D.
D2 receptor antagonism
E.
Choices A and B
18. Which of the following ocular effects has NOT been associated
with antipsychotic use among older adults?
A.
Glaucoma
B
B.
Cataracts
C.
Lenticular pigmentation
D.
Optic atrophy
E. Choices A and B

19.
You want to start a 75-year-old individual with a diagnosis of
major depressive disorder on citalopram, but are worried about
the development torsades de pointes (TdP). Which of the
following would not be a risk factor for the development of TdP
with the use of citalopram?
A.
Older age
B.
Female sex
C.
Treatment with diuretics
D.
Hypercalcemia
E.
Hypomagnesemia
20. A 72-year-old woman is started on risperidone for the treatment
of paranoid delusions that her neighbors are trying to kill her and
steal her money. Approximately 6 weeks later, she presents to
your of ice with complaints of restlessness and inability to sit still
throughout the day. You diagnose the woman with having
developed akathisia from the risperidone. If you choose to
continue her on risperidone instead of switching to another
antipsychotic medication, then which of the following drugs
would not be considered a irst-line agent for the management of
her symptoms of akathisia?
A.
Mirtazapine
B.
Propranolol
C.
Benztropine
D. Lorazepam

E.
Choices B and C

Answers
Answer 1A While antipsychotics may be effective in controlling acute
agitation associated with delirium, some antipsychotics have a
signi icant af inity for muscarinic receptors, thus causing
anticholinergic side effects that may be especially detrimental among
older adults. In a study that measured the anticholinergic activity (AA)
of medications commonly used by older adults using a radioreceptor
assay, typically administered doses of amitriptyline, atropine, clozapine,
dicyclomine, doxepin, L-hyoscyamine, thioridazine, and tolterodine
demonstrated AA exceeding 15 pmol/mL. Chlorpromazine,
diphenhydramine, nortriptyline, olanzapine, oxybutynin, and
paroxetine had AA values of 5–15 pmol/mL. Citalopram, escitalopram,
luoxetine, lithium, mirtazapine, quetiapine, ranitidine, and temazepam
had values less than 5 pmol/mL. Amoxicillin, celecoxib, cephalexin,
diazepam, digoxin, diphenoxylate, donepezil, duloxetine, fentanyl,
furosemide, hydrocodone, lansoprazole, levo loxacin, metformin,
phenytoin, propoxyphene, and topiramate demonstrated AA only at the
highest concentrations tested. The remainder of the medications
investigated did not demonstrate any AA at the concentrations
examined. Psychotropic medications were particularly likely to
demonstrate AA. In another study of in vitro anticholinergic activity
measured by a radioreceptor assay, therapeutic doses of clozapine,
olanzapine, and to a lesser extent quetiapine were associated with
clinically signi icant anticholinergic effects, while risperidone,
aripiprazole, and ziprasidone were not.

Answer 2B Selective serotonin reuptake inhibitors (SSRIs) such as


luoxetine have a synergistic effect with warfarin. The anticoagulant
effects of warfarin are augmented by the inhibition of platelet serotonin
uptake. SSRIs are also known to inhibit cytochrome p-450, which is
involved in the elimination of warfarin.

Answer 3E The symptoms of lithium toxicity tend to occur when the


lithium level is higher than 1.5 mEq/L. Aging increases the risk for
toxicity due to a decrease in GFR. Nephrogenic diabetes insipidus
increases the toxicity risk, possibly through a feedback loop of elevated
lithium levels leading to further diuresis and lithium level elevations.
Other risk factors for lithium toxicity include the concomitant
medications including diuretics, loop diuretics, angiotensin-converting
enzyme inhibitors, angiotensin II receptor blockers, and nonsteroidal
anti-in lammatory drugs, which can increase lithium levels up to 50%.

Answer 4E Important factors that contribute to the development of


psychotropic-associated hyponatremia among older adults include the
following: female gender; low body mass index; low baseline serum
sodium; concomitant use of medications including diuretics, NSAIDs,
proton-pump inhibitors, and angiotensin-converting enzyme inhibitors;
and the presence of heart failure, malignancy, liver disease, or adrenal
insuf iciency.

Answer 5C For prazosin, the most important adverse effect is the


“ irst-dose effect” syncope with a sudden loss of consciousness which
occurs in 1% of individuals given an initial dose of at least 2 mg. Hence,
prazosin should always be started at a 1 mg dosing. The most common
adverse effects of prazosin are as follows: dizziness (10%), headache
(8%), drowsiness (8%), lack of energy (7%), weakness (7%),
palpitations (5%), and nausea (5%). Approximately 1–4% of
individuals taking prazosin report the following adverse effects:
vomiting, diarrhea, constipation, edema, orthostatic hypotension,
dyspnea, syncope, vertigo, depression, nervousness, rash, urinary
frequency, and nasal congestion. A <1% of individuals taking prazosin
report the following: abdominal discomfort/pain, tachycardia,
paresthesias, hallucinations, pruritus, incontinence, impotence, and
priapism.

Answer 6A Although NMS can occur among individuals of both sexes


and all ages, young adult male predominance in the cases of NMS tends
to occur. Individuals with organic brain syndromes and those with
intellectual disability are thought to be at a high risk for developing
NMS.

Answer 7A Tertiary tricyclics, such as amitriptyline, imipramine, and


doxepin, are not considered safe for use among older adults due to their
adverse effect pro ile. These medications are more likely to cause
sedation, anticholinergic effects, cardiac effects, and orthostatic
hypotension. Secondary tricyclics such as nortriptyline and
desipramine are considered safer for use among older adults although
they should still be avoided among individuals with ischemic heart
disease or cardiac conduction disturbance due to their anti-arrhythmic
effects.

Answer 8C Evidence indicates that individuals taking eszopiclone


often report unpleasant (bitter) taste, dry mouth, somnolence, and
dizziness.

Answer 9E Topiramate (TPM) is a carbonic anhydrase inhibitor. TPM


reduces the excretion of urinary citrate that results in a rise in urinary
pH. TPM has also been found to inhibit the activity of speci ic carbonic
anhydrase enzymes in the kidney resulting in the development of
metabolic acidosis, hypocitraturia, hypercalciuria, and elevated urinary
pH, all of which may lead to an increased risk of nephrolithiasis.

Answer 10E Lorazepam, temazepam, and oxazepam are 3-hydroxy-


benzodiazepines. The presence of the hydroxyl group in the third
position results in these benzodiazepines not requiring oxidative drug
metabolism for clearance. They are conjugated via glucuronidation
(phase 2 metabolism only). For this reason, the half-lives of these drugs
are not affected by age, liver disease, or concomitant use with
cytochrome (CYP) enzyme inducers or inhibitors. Lorazepam is the
most potent drug of this group. As the 3-hydroxy-benzodiazepines are
not susceptible to age, liver disease, and CYP enzyme-mediated drug-
drug interactions, they are, especially lorazepam, the choice
benzodiazepine for use among individuals who have decreased liver
function.

Answer 11A Cytochrome P450 isoenzyme (CYP1A4) is involved in


the metabolism of olanzapine and clozapine. Cigarette smoking has
been shown to induce CYP1A4 isoenzyme and thus inducing the
metabolism of its substrates: olanzapine and clozapine. Quitting
smoking may therefore result in higher serum concentrations of these
drugs and a more signi icant side effect pro ile. It is suggested that the
doses of olanzapine and clozapine be reduced by 30% and 50%,
respectively, among non-smokers when compared with smokers in
order to obtain an equivalent olanzapine or clozapine concentration.

Answer 12E In a meta-analysis by Serretti and Mandelli, the


investigators found that that amitriptyline, mirtazapine, and paroxetine
were associated with a greater risk of weight gain. In contrast, they
found that some weight loss occurs with the use of luoxetine and
bupropion. However, the effect of luoxetine appears to be limited to the
acute phase of treatment. Other antidepressants were found to have no
transient or negligible effect on body weight in the short term. The
effect of each antidepressant may vary greatly depending on an
individual’s characteristics and generally becomes more evident in the
long term to a variable degree across compounds.

Answer 13A Carbidopa is a common adjunct therapy among


individuals with PD taking levodopa due to its inhibition of peripheral
conversion of levodopa to dopamine. When the peripheral conversion
is minimized, peripheral side effects such as nausea and hypotension
are minimized as well. Carbidopa cannot cross the blood-brain barrier,
so it has no effect on the central conversion of levodopa to dopamine.
Psychosis is a side effect of this central conversion, so it is not affected
by carbidopa.
Answer 14E Seizures are a known adverse effect of clozapine
therapy. Tonic-clonic seizures comprise the majority of seizures due to
clozapine use. The exact etiology of clozapine-induced seizure remains
unknown as there are con licting reports regarding total oral dose,
serum concentration, dose titration, and concomitant medication use
that make it dif icult to identify a single etiology for the seizure risk. If it
is in the best interest of the patient to continue clozapine treatment,
then the use of an antiepileptic drug (AED) for seizure prophylaxis is
often required. The AED of choice for clozapine-associated seizures
appears to be valproate, but reports also support the use of lamotrigine,
gabapentin, and topiramate. Both clozapine and carbamazepine uses
are associated with the risk for agranulocytosis, and they should not be
combined together in treatment.

Answer 15D Although rare, the dopaminergic effects of bupropion


have been shown to induce psychosis in vulnerable populations. Older
adults are more susceptible to the adverse effects of bupropion due to
the greater plasma accumulation of both bupropion and bupropion
metabolites. Patients with a history of psychosis are also much more
likely to experience worsening or returning psychotic symptoms with
bupropion. Because this older woman is reporting auditory
hallucinations in addition to depressive symptoms, bupropion should
be avoided.

Answer 16D Antipsychotics are associated with an increased risk of


death among older adults with dementia. The possible mechanisms for
this association include cerebrovascular events, arrhythmias and
sudden cardiac death, venous thromboembolism, pulmonary embolism,
and aspiration pneumonia. In 2005, the FDA released a warning that
atypical antipsychotics increase the risk of death when prescribed to
older adults for the treatment of dementia-related psychosis. In 2008,
the FDA concluded that irst-generation antipsychotics carry a similar
risk. There is no such risk associated with the use of antidepressants.

Answer 17C Researchers have suggested that the 5-HT 1A receptor


partial agonist properties of some antipsychotics may actually decrease
carbohydrate craving. The other listed answer choices have all been
shown to contribute to weight gain or metabolic effects of
antipsychotics. H1receptor antagonism causes increased hypothalamic
AMPK activity, which is often implicated in weight gain. D2 receptor
antagonism increases reward-seeking behaviors such as food intake
and disinhibits prolactin from the posterior hypothalamus, leading to
weight gain. M3 receptor antagonism impairs glucose tolerance and
reduces insulin secretion from pancreatic beta cells, often leading to
diabetes.

Answer 18D Glaucoma, cataracts, and lenticular pigmentation are all


potential ocular adverse effects of antipsychotic use among older
adults. The irst-generation antipsychotics thioridazine and
chlorpromazine are especially likely to cause cataracts or lenticular
pigmentation. Optic atrophy is not one of the ocular complications
noted with antipsychotic use.

Answer 19D The risk factors for drug-induced TdP are as follows: a
QTc interval >500 ms and/or prolongation of the QTc interval ≥60 ms,
female sex, individuals >65 years of age, acute myocardial infarction,
hypokalemia, hypomagnesemia and hypocalcemia, therapy with
diuretics, concomitant administration of ≥2 QTc interval-prolonging
drugs, rapid infusion of intravenously administered QTc interval-
prolonging medications, and congenital long QT syndrome (LQTS).

Answer 20D Propranolol, mirtazapine, and anticholinergics


(benztropine , biperiden, and trihexyphenidyl) are considered as the
irst-line agents for the management of antipsychotic-induced
akathisia. Amantadine, clonidine, benzodiazepines (lorazepam,
clonazepam, and diazepam), and 5HT2A antagonists (mianserin and
cyproheptadine) are considered the second-line agents for the
management of akathisia.

Further Reading
Balt SL, Galloway GP, Baggott MJ, et al. Mechanisms and genetics of antipsychotic-
associated weight gain. Clin Pharmacol Ther. 2011;90(1):179–83.
[Crossref]
Chew ML, Mulsant BH, Pollock BG, et al. A model of anticholinergic activity of
atypical antipsychotic medications. Schizophr Res. 2006;88(1–3):63–72.
[Crossref]

Chew ML, Mulsant BH, Pollock BG, et al. Anticholinergic activity of 107 medications
commonly used by older adults. J Am Geriatr Soc. 2008;56(7):1333–41.
[Crossref]

Cohen R, Wilkins KM, Ostroff R, et al. Olanzapine and acute urinary retention in two
geriatric patients. Am J Geriatr Pharmacother. 2007;5(3):241–6.
[Crossref]

de Leon J, Santoro V, D’Arrigo C, Spina E. Interactions between antiepileptics and


second-generation antipsychotics. Expert Opin Drug Metab Toxicol. 2012;8(3):311–
34.
[Crossref]

Filippatos TD, Makri A, Elisaf MS, Liamis G. Hyponatremia in the elderly: challenges
and solutions. Clin Interv Aging. 2017;12:1957–65.
[Crossref]

Francis Lam YW. Warfarin and SSRIs. The Brown University Psychopharmacology
Update. 2017;28(12):2–3.
[Crossref]

Gareri P, Segura-Garcı́a C, Manfredi VG, et al. Use of atypical antipsychotics in the


elderly: a clinical review. Clin Interv Aging. 2014;9:1363–73.
[PubMed][PubMedCentral]

Gillman PK. Tricyclic antidepressant pharmacology and therapeutic drug


interactions updated. Br J Pharmacol. 2007;151(6):737–48.
[Crossref]

Henry G, Williamson D, Tampi RR. Ef icacy and tolerability of antidepressants in the


treatment of behavioral and psychological symptoms of dementia, a literature
review of evidence. Am J Alzheimers Dis Other Dement. 2011;26(3):169–83.
[Crossref]

Kheirbek RE, Fokar A, Little JT, et al. Association between antipsychotics and all-
cause mortality among community-dwelling older adults. J Gerontol A Biol Sci Med
Sci. 2019;74(12):1916–21.
[Crossref]

Koola MM, Varghese SP, Fawcett JA. High-dose prazosin for the treatment of post-
traumatic stress disorder. Ther Adv Psychopharmacol. 2014;4(1):43–7.
[Crossref]

Kumar S, Kodela S, Detweiler JG, et al. Bupropion-induced psychosis: folklore or a


fact? A systematic review of the literature. Gen Hosp Psychiatry. 2011;33(6):612–7.
[Crossref]

Kurdyak PA, Juurlink DN, Kopp A, Herrmann N, Mamdani MM. Antidepressants,


warfarin, and the risk of hemorrhage. J Clin Psychopharmacol. 2005;25(6):561–4.
[Crossref]

Laliberté V, Yu C, Rej S. Acute renal and neurotoxicity in older lithium users: how can
we manage and prevent these events in patients with late-life mood disorders? J
Psychiatry Neurosci. 2015;40(4):E29–30.
[Crossref]

McCrae CS, Ross A, Stripling A, Dautovich ND. Eszopiclone for late-life insomnia. Clin
Interv Aging. 2007;2(3):313–26.
[PubMed][PubMedCentral]

Patel K, Allen S, Haque MN, et al. Bupropion: a systematic review and meta-analysis
of effectiveness as an antidepressant. Ther Adv Psychopharmacol. 2016;6(2):99–144.
[Crossref]

Preskorn SH. A way of conceptualizing benzodiazepines to guide clinical use. J


Psychiatr Pract. 2015;21(6):436–41.
[Crossref]

Richa S, Yazbek JC. Ocular adverse effects of common psychotropic agents: a review.
CNS Drugs. 2010;24(6):501–26.
[Crossref]

Salem H, Nagpal C, Pigott T, Teixeira AL. Revisiting antipsychotic-induced akathisia:


current issues and prospective challenges. Curr Neuropharmacol. 2017;15(5):789–
98.
[Crossref]

Serretti A, Mandelli L. Antidepressants and body weight: a comprehensive review


and meta-analysis. J Clin Psychiatry. 2010;71(10):1259–72.
[Crossref]

Sommer BR, Fenn HH. Review of topiramate for the treatment of epilepsy in elderly
patients. Clin Interv Aging. 2010;5:89–99.
[Crossref]

Sommer BR, Fenn H, Pompei P, et al. Safety of antidepressants in the elderly. Expert
Opin Drug Saf. 2003;2(4):367–83.
[Crossref]

Tambasco N, Romoli M, Calabresi P. Levodopa in Parkinson's disease: current status


and future developments. Curr Neuropharmacol. 2018;16(8):1239–52.
[Crossref]

Tampi RR, Tampi DJ, Balachandran S, et al. Antipsychotic use in dementia: a


systematic review of bene its and risks from meta-analyses. Ther Adv Chronic Dis.
2016;7(5):229–45.
[Crossref]

Teles JS, Fukuda EY, Feder D. Warfarin: pharmacological pro ile and drug interactions
with antidepressants. Einstein (Sao Paulo). 2012;10(1):110–5.
[Crossref]

Tisdale JE. Drug-induced QT interval prolongation and torsades de pointes: role of


the pharmacist in risk assessment, prevention and management. Can Pharm J (Ott).
2016;149(3):139–52.
[Crossref]

Tsuda Y, Saruwatari J, Yasui-Furukori N. Meta-analysis: the effects of smoking on the


disposition of two commonly used antipsychotic agents, olanzapine and clozapine.
BMJ Open. 2014;4(3):e004216.
[Crossref]

Ware MR, Feller DB, Hall KL. Neuroleptic malignant syndrome: diagnosis and
management. Prim Care Companion CNS Disord. 2018;20(1):pii: 17r02185.
[Crossref]

Williams AM, Park SH. Seizure associated with clozapine: incidence, etiology, and
management. CNS Drugs. 2015;29(2):101–11.
[Crossref]
© Springer Nature Switzerland AG 2021
R. Tampi et al. (eds.), Absolute Geriatric Psychiatry Review
https://doi.org/10.1007/978-3-030-58663-8_35

35. Interventional Psychiatry


Edward V. Singh1
(1) Geriatric Psychiatry, Yale-New Haven Hospital, New Haven, CT, USA

Edward V. Singh
Email: Edward.Singh@yale.edu

Keywords Electroconvulsive therapy (ECT) – Transcranial magnetic


stimulation (TMS) – Vagus nerve stimulation (VNS) – Deep brain
stimulation (DBS) – Depression – Melancholic – Catatonia

Questions
1. An 84-year-old woman with multiple medical comorbidities
(heart failure, diabetes mellitus (DM), chronic obstructive
pulmonary disease (COPD), chronic kidney disease (CKD)) with
catatonia, who failed to respond to pharmacotherapy, is being
considered for electroconvulsive therapy (ECT). Which of the
following is true of the use of ECT among older adults?
A.
ECT is contraindicated in patients with a history of heart
failure
B.
ECT is contraindicated in patients with a history of diabetes
mellitus
C.
ECT is contraindicated in patients with a history of COPD
D.
ECT is contraindicated with patients with a history of CKD
E. There are no medical comorbidities that are absolute
contraindications for ECT
2.
In which one of the following conditions is ECT considered a
second-line intervention instead of a irst-line treatment
according to all major guidelines?
A.
Severe major depressive episodes unresponsive to
psychotherapy and/or pharmacotherapy
B.
Previous positive response to ECT
C.
Patient preference for the use of ECT
3.
Which one of the following factors does not determine the ECT
stimulus dose to be administered?
A.
Electrode placement
B.
Sex
C.
Age
D.
Anesthetic dosage
E.
Comorbid medical conditions
4. Which of the following is true with regard to electrode placement
when comparing the rapidity of the response of major depressive
episodes to ECT?
A.
Bitemporal (BT) = bifrontal (BF) = right unilateral (RUL)
B.
Bitemporal (BT) > bifrontal (BF) = right unilateral (RUL)
C.
Bitemporal (BT) = bifrontal (BF) > right unilateral (RUL)
D Bif t l (BF) i ht il t l (RUL) bit l (BT)
D. Bifrontal (BF) > right unilateral (RUL) > bitemporal (BT)

E.
Bitemporal (BT) = right unilateral (RUL) > bifrontal (BF)
5.
Which of the following is true of the anterograde amnesia and
retrograde amnesia with ECT?
A.
Anterograde amnesia is permanent and retrograde resolves
slowly
B.
Anterograde amnesia resolves slowly and retrograde amnesia
is permanent
C.
Both anterograde and retrograde amnesias resolve slowly
D.
Both anterograde and retrograde amnesias are permanent
E.
Both anterograde and retrograde amnesias are not side
effects of ECT
6.
Which of the following statement is true about maintenance ECT
among older adults?
A.
Maintenance ECT is inferior to pharmacotherapy
B.
Maintenance ECT is superior to pharmacotherapy
C.
Maintenance ECT is as effect of pharmacotherapy
D.
The risk of side effects outweighs the bene it of maintenance
ECT
E.
There is no therapeutic bene it with maintenance ECT
7. Which of the following is a true statement regarding the response
of older adults with major depressive disorder to ECT?

A. Response rates to ECT decreases with age based on the


severity of the depressive episode
B.
Response rates to ECT decreases with age based on the
presence of psychosis in the depressive episode
C.
Response rates to ECT decreases with age based on the
number episodes of depression
D.
Response rates to ECT remain unchanged with age based on
the severity of depressive episode, the presence of psychosis,
or the number of episodes of depression
E.
Response rates to ECT improve with age irrespective of the
severity of depressive episode, the presence of psychosis, or
the number of episodes of depression
8.
A patient with an implantable cardioverter-de ibrillator (ICD) is
preparing for ECT. What additional steps must be taken prior to
treatment with ECT?
A.
Turn off the ICD prior to treatment, and then turn it back on
when the seizure ends
B.
ECT is contraindicated in patients with ICD
C.
Leave the ICD on during treatment and turn it off after
treatment
D.
No modi ications to the ICD are needed prior to treatment
E.
Remove ICD prior to treatment
9. An 80-year-old man with a history of bipolar 1 disorder, who is
currently depressed and reports suicidal ideation, is being
cu e t y dep essed a d epo ts su c da deat o , s be g
considered for ECT. He has a history of spinal surgery with
implanted spinal rods. He was cleared by neurosurgery for

treatment. What steps during treatment can alleviate spinal


complications?
A.
Decrease the total charge of stimulus delivered
B.
Plan to have the seizure duration be less than 10 seconds in
duration
C.
Patient with a history of spinal surgery is contraindicated to
receive ECT
D.
Use unilateral lead placement instead of bilateral placement
E.
Ensure an appropriate dose of neuromuscular agent is given
to the patient prior to treatment
10.
An 88-year-old man who is being treated with ECT has a history
of benign prostatic hypertrophy (BPH). He is also being given
atropine to minimize asystole following stimulation. Which of the
following precautions would be advised in this patient?
A.
Straight catheterization of the patient immediately after
treatment
B.
Increase the dose of alpha blockers prior to treatment
C.
No added precautions are necessary if the BPH is well
controlled
D.
Observe the patient until he voids urine
E.
Properly hydrate the patient prior to treatment
Wh i i f d ld d l
11. What maintenance treatment is favored among older adults
following a successful treatment of ECT?
A.
Maintenance ultrabrief pulse right unilateral ECT alone

B. Ultrabrief pulse right unilateral ECT combined with


medication
C.
Medication alone
D.
No maintenance treatment
E.
Maintenance ultrabrief pulse bilateral ECT alone
12.
A 75-year-old man with a diagnosis of schizophrenia is being
treated for catatonia. He has had a partial response to
pharmacotherapy. Which of the following is true of the use of ECT
among individuals with catatonia?
A.
ECT is ineffective for the management of catatonia
B.
ECT is contraindicated in the management of catatonia
C.
ECT is an effective management option for catatonia
D.
No evidence supports the use of ECT among individuals with
catatonia
E.
The use of ECT will exacerbate the symptoms of catatonia
13. Which of the following statements is true for the treatment of
MDD among individuals with Parkinson’s disease (PD) using ECT?
A.
ECT is contraindicated for the treatment of MDD among
individuals with PD
B.
ECT is not bene icial for the treatment of MDD among
g
individuals with PD
C.
ECT worsens the motor symptoms when used for the
treatment of MDD among individuals with PD
D. ECT worsens the cognitive symptoms when used for the

E. treatment of MDD among individuals with PD

ECT improves the motor and depressive symptoms without


worsening cognition when used for the treatment of MDD
among individuals with PD

14.
A 69-year-old woman with catatonia has immobility and chronic
kidney disease. In addition, her potassium level is at the upper
limit of normal. She receives hemodialysis prior to treatment with
ECT for the catatonia. The administration of succinylcholine as a
muscle relaxant during ECT increases the risk of which of the
following conditions?
A.
Worsening of catatonia
B.
Increased urinary retention
C.
Increased seizure threshold
D.
Ventricular tachycardia
E.
Respiratory distress
15. A 75-year-old man who suffered a stroke 3 years ago develops
symptoms of depression. Several trials of antidepressant have
been ineffective for the treatment of his symptoms of depression.
A decision is made to initiate treatment with ECT for this
individual’s symptoms of depression. Which of the following is
true of the use of ECT for the treatment of depression among
k i ?
poststroke patients?
A.
ECT is ineffective for the treatment of depression among
poststroke patients
B. ECT will exacerbate neurological de icits among poststroke
patients with depression

C.

ECT is effective and well-tolerated treatment for depression


among poststroke patients
D.
ECT is contraindicated for the treatment of depression among
poststroke patients
E.
ECT increases the risk of stroke among poststroke patients
when used for the treatment for depression
16.
Which of the following is the most recent estimate of mortality
rate for ECT from a systematic review?
A.
2.1 per 100 treatments
B.
2.1 per 1000 treatments
C.
2.1 per 10,000 treatments
D.
2.1 per 100,000 treatments
E.
2.1 per 1,000,000 treatments
17. Transcranial magnetic stimulation (TMS) is FDA approved for the
treatment of which of the following psychiatric disorders?
A.
Chronic pain
B.
Unipolar major depression
p j p
C.
Bipolar depression
D.
Smoking cessation
E.
All of the above
18.
Which of the following does not moderate the ef icacy of TMS
when used among older adults?
A.
Cortical atrophy
B.
Decreased functional hemispheric asymmetry
C.
Increased white matter disease burden
D.
Age
E.
Choices A and C

19. Which of the following is false regarding vagal nerve stimulation


(VNS)?
A.
The VNS bipolar electrode is surgically attached to the left
vagal nerve
B.
The VNS stimulator is FDA approved as an adjunctive long-
term treatment for chronic or recurrent depression
C.
The VNS is FDA approved for use among individuals
≥18 years of age
D.
The VNS is FDA approved for use among individuals with a
major depressive episode not showing adequate response to
≥2 adequate antidepressant trials
E.
Choices B and C
Choices B and C
20.
Which of the following is an FDA-approved indication for deep
brain stimulation?
A.
Treatment-refractory obsessive compulsive disorders (OCD)
B.
Treatment-refractory major depressive disorder (MDD)
C.
Treatment-refractory posttraumatic stress disorder (PTSD)
D.
Treatment-refractory schizophrenia
E.
All of the above

Answers
Answer 1E There are no absolute contraindications for ECT among
older adults.

Answer 2A Most ECT guidelines considered ECT as a irst-line


treatment for severe depressive episodes that are life-threatening and
where a rapid response is required especially in the presence of
psychotic features, catatonia, high suicide risk and/or food, or luid
refusal. Additionally, a previous positive response to ECT and patient
preference are also considered as indications for the use of ECT as a
irst-line treatment. ECT is considered a second-line intervention for
individuals with severe major depressive episodes that are not
responsive to psychotherapy and/or pharmacotherapy. Guidelines also
indicate that ECT is considered a second-line intervention for those
individuals with mania or schizophrenia who have not responded to
pharmacotherapy.

Answer 3E Electrode placement, sex, age, anesthetic dosage, and


concomitant medications determine the ECT stimulus dose to be
administered.
Answer 4B The three electrode placements commonly used in ECT
are bitemporal (BT), bifrontal (BF), and right unilateral (RUL). Available
evidence indicates that each electrode placement is associated with a
comparable improvement in depressive symptoms with a more rapid
effect seen with BT placement when compared to BF and RUL electrode
placements.

Answer 5C Anterograde amnesia typically resolves within a few


weeks after the treatment course, whereas retrograde amnesia trends
to resolve more slowly.

Answer 6C Studies have shown that maintenance ECT is probably as


effective as continuation medication in severely depressed elderly
patients after a successful course of ECT and is generally well tolerated.

Answer 7E Older age is a reliable predictor of improved response


among individuals with MDD to ECT irrespective of the severity of
depressive episode, the presence of psychosis, or the number of
episodes of depression.

Answer 8A ICD may ire during transient changes in heart rate


during ECT treatment. This can be avoided by turning off the device
prior to treatment. The ICD can safely be restarted after the seizure has
remitted.

Answer 9E An appropriate dose of a neuromuscular blocking agent


can decrease the risk spinal rod dislodgement during the ECT
treatment. Decreasing the charge or duration of seizure is not
necessary to safely use ECT among individuals with implanted spinal
rods.

Answer 10D Atropine and glycopyrronium have strong


anticholinergic properties which may exacerbate urinary retention.
Hence it is advised to observe a patient with BPH until they can
successfully urinate.

Answer 11B The PRIDE study displayed that the odds of relapsing
with ultrabrief pulse right unilateral ECT with medication was less than
the medication alone in the maintenance of depressive symptoms
following remission.

Answer 12C ECT is an effective management option among


individuals with catatonia especially for those individuals who have
severe or malignant symptoms.

Answer 13E A systematic review that evaluated data from 43 articles


indicated that among individuals with PD and MDD, depression
improved in 93.1% of the individuals, motor symptoms’ severity
improved in 83% of the individuals, and cognition did not worsen in
94% of the individuals, indicating that ECT can bene it individuals with
PD and depression.

Answer 14D The administration of succinylcholine causes increases


in the serum potassium level by 0.5–1 mEq/L. However prolonged
immobilization can potentiate hyperkalemia induced by
succinylcholine. The hyperkalemia can increase the risk of ventricular
arrhythmias.

Answer 15C ECT is an effective and safe treatment for poststroke


depression.

Answer 16D A systematic review that included data from 15 studies


and 32 countries which reported on a total of 766,180 ECT treatments
identi ied a total of 16 cases of ECT-related death resulting in an ECT-
related mortality rate of 2.1 per 100,000 treatments. The mortality of
general anesthesia in relation to surgical procedures is 3.4 per 100,000
indicating that the death caused by ECT is an extremely rare event.

Answer 17B Prefrontal transcranial magnetic stimulation (TMS)


therapy that is repeated daily over 4–6 weeks (20–30 sessions) is US
Food and Drug Administration (FDA) approved for treating unipolar
major depressive disorder among adults who have not responded to
prior antidepressant medication treatments.

Answer 18D Cortical atrophy, decreased functional hemispheric


asymmetry, and increased white matter disease burden can moderate
the ef icacy of TMS among older adults by affecting the depth and
spread of stimulation.

Answer 19D The VNS stimulator was approved in 2005 by the FDA
as an adjunctive, long-term treatment for chronic or recurrent
depression among individuals ≥18 years of age who are experiencing a
major depressive episode that has not responded adequately to four or
more adequate antidepressant treatments. The VNS requires surgery to
attach a bipolar electrode on the left vagus nerve which is then
connected to a stimulator that is implanted in the chest wall. The
electrical impulses are sent via the stimulator along the nucleus tractus
solitarius to other brain regions to exert its antidepressant effect.

Answer 20A The FDA approved DBS with the electrode placement in
the ventral capsule/ventral striatum for the management of treatment-
refractory OCD in 2009.

Further Reading
Borisovskaya A, Bryson WC, Buchholz J, et al. Electroconvulsive therapy for
depression in Parkinson's disease: systematic review of evidence and
recommendations. Neurodegener Dis Manag. 2016;6(2):161–76.
[Crossref]

Dougherty DD. Deep brain stimulation: clinical applications. Psychiatr Clin North
Am. 2018;41(3):385–94.
[Crossref]

Fink M. Electroshock therapy and catatonia: a productive synergism. J ECT.


2019;35(4):219–21.
[Crossref]

Fink M, Kellner CH, McCall WV. Optimizing ECT technique in treating catatonia. J
ECT. 2016;32(3):149–50.
[Crossref]

Geduldig ET, Kellner CH. Electroconvulsive therapy in the elderly: new indings in
geriatric depression. Curr Psychiatry Rep. 2016;18(4):40.
[Crossref]

Hudcova J, Schumann R. Electroconvulsive therapy complicated by life-threatening


hyperkalemia in a catatonic patient. Gen Hosp Psychiatry. 2006;28(5):440–2.
[Crossref]

Iriarte IG, George MS. Transcranial magnetic stimulation (TMS) in the elderly. Curr
Psychiatry Rep. 2018;20(1):6.
[Crossref]

Kellner CH, Husain MM, Knapp RG, et al. A novel strategy for continuation ECT in
geriatric depression: phase 2 of the PRIDE study. Am J Psychiatry.
2016;173(11):1110–8.
[Crossref]

Kellner CH, Obbels J, Sienaert P. When to consider electroconvulsive therapy (ECT).


Acta Psychiatr Scand. 2020;141(4):304–15.
[Crossref]

Kerner N, Prudic J. Current electroconvulsive therapy practice and research in the


geriatric population. Neuropsychiatry (London). 2014;4(1):33–54.
[Crossref]

McDonald WM. Neuromodulation treatments for geriatric mood and cognitive


disorders. Am J Geriatr Psychiatry. 2016;24(12):1130–41.
[Crossref]

Meyer JP, Swetter SK, Kellner CH. Electroconvulsive therapy in geriatric psychiatry:
a selective review. Psychiatr Clin North Am. 2018;41(1):79–93.
[Crossref]

Ostroff RB, Katz RB. The use of electroconvulsive therapy in late-life psychiatric
disorders. Curr Treat Option Psych. 2018;5:228–41.
[Crossref]

Perera T, George MS, Grammer G, et al. The clinical TMS Society consensus review
and treatment recommendations for TMS therapy for major depressive disorder.
Brain Stimul. 2016;9(3):336–46.
[Crossref]

Peterchev AV, Rosa MA, Deng ZD, et al. Electroconvulsive therapy stimulus
parameters: rethinking dosage. J ECT. 2010;26(3):159–74.
[Crossref]

Pinna M, Manchia M, Oppo R, et al. Clinical and biological predictors of response to


electroconvulsive therapy (ECT): a review. Neurosci Lett. 2018;669:32–42.
[Crossref]
van Schaik AM, Comijs HC, Sonnenberg CM, et al. Ef icacy and safety of continuation
and maintenance electroconvulsive therapy in depressed elderly patients: a
systematic review. Am J Geriatr Psychiatry. 2012;20(1):5–17.
[Crossref]
© Springer Nature Switzerland AG 2021
R. Tampi et al. (eds.), Absolute Geriatric Psychiatry Review
https://doi.org/10.1007/978-3-030-58663-8_36

36. Psychotherapy
Laurel J. Bessey1
(1) Department of Psychiatry, University of Wisconsin School of
Medicine and Public Health, Madison, WI, USA

Laurel J. Bessey
Email: bessey@wisc.edu

Keywords Psychotherapy – Late-life depression – Late-life anxiety –


Non-pharmacological management – Group therapy – Problem-solving
therapy – Interpersonal psychotherapy

Questions
1. A 90-year-old man forgets to take his medications regularly
despite not having signi icant cognitive de icit. Which of the
following statements is most consistent with treatment modality
of problem-solving therapy (PST) that can be used to help him
take his medications regularly?
A.
The therapist validates how frustrating it must be to forget
his morning medications.
B.
The patient and therapist work together to brainstorm ideas
to reduce the stress of this issue. The patient creates an action
plan involving putting a post-it-note on his bathroom mirror
reminding him to take his medications.
C. The therapist works with the patient to identify automatic
thoughts associated with this forgetting. They discuss
evidence for and against these automatic thoughts and
g g
reframe to come up with a more rational thought.
D.
The therapist and patient create an interpersonal inventory to
identify social supports available to the patient.
E.
The patient and therapist discuss how past experiences and
relationships contributed to the current problem in order to
better understand the patterns of behavior.
2.
Which of the following is not a valid reason for why older adults
receive less psychotherapy for the treatment of their depression?
A.
Older adults favor treatment in primary care settings.
B.
Access to care is dif icult for older adults who are homebound
or in other hard-to-reach settings such as rural areas.
C.
Older adults prefer pharmacotherapy to psychotherapy.
D.
Low availability of clinicians trained to provide evidence-
based psychotherapy interventions in older adults.
E.
Stigma of seeking care for mental health diagnoses among
older adults.
3. Which of the following is not an example of a core problem area
that can be resolved using interpersonal psychotherapy (IPT)?
A.
Unresolved grief
B.
Interpersonal de icit
C.
Role dispute
D.
Role transition
E.
Ineffective communication
4.
Which of the following statements is true regarding the treatment
of late-life depression?
A.
Both psychopharmacology and psychotherapy are effective
for treating depressed older adults.
B.
Only psychopharmacology has been shown to be effective for
treating late-life depression.
C.
Only psychotherapy has been shown to be effective in treating
late-life depression.
D.
Neither psychopharmacology nor psychotherapy is effective
for treating depressed older adults.
E.
There are no studies regarding the effectiveness of
psychotherapy for treating late-life depression.
5.
Which theorist’s model of psychosocial development changed the
assumption that psychotherapy is ineffective among older adults?
A.
Sigmund Freud
B.
Erik Erikson
C.
Jean Piaget
D.
John Bowlby
E.
B. F. Skinner
6. Which of the following psychotherapies has evidence for
effectiveness in treating late-life depression?
A.
Cognitive behavior therapy
B
B.
Problem-solving therapy

C. Brief psychodynamic therapy

D.
Reminiscence therapy
E.
All of the above
7.
Which of the following is a common example of transference in
psychodynamic psychotherapy in older adults that results from
the difference in age between the therapist and the older adult
patient?
A.
The patient may relate to the therapist as a son or daughter.
B.
The patient has romantic feelings toward the therapist.
C.
The therapist relates to the patient based on memories of
parents or grandparents.
D.
The therapist believes the patient cannot change due to older
age.
E.
The patient believes visiting a therapist means they are
“crazy.”
8. Which of the following is considered the irst-line treatment for
insomnia among older adults?
A.
Interpersonal therapy
B.
Benzodiazepines
C.
Sleep hygiene and a non-benzodiazepine sleep agent
D.
Antipsychotic medications and sleep hygiene
Antipsychotic medications and sleep hygiene
E.
Cognitive behavioral therapy and sleep hygiene
9.
Which of the following psychotherapy modalities has the most
evidence for the treatment of late-life anxiety disorders?
A.
Reminiscence therapy
B.
Interpersonal therapy
C.
Brief psychodynamic therapy
D.
Cognitive behavior therapy
E.
Problem-solving therapy
10.
Which of the following is not a treatment indication for traditional
forms of psychotherapy?
A.
Late-life depression
B.
Late-life insomnia
C.
Dementia
D.
Late-life anxiety disorders
E.
Bereavement
11. Which one of the following therapies encourages individuals to
look over their life with photographs, music, and videos of the
past in order to achieve ego integrity described by Erik Erikson?
A.
Reminiscence therapy
B
B.
Cognitive behavioral therapy
C.
Psychodynamic therapy
D. Supportive therapy

E.
Interpersonal therapy
12.
Which of the following psychotherapy modalities has evidence
supporting their effective use in primary care settings for older
adults with depression?
A.
Problem solving therapy
B.
Interpersonal therapy
C.
Cognitive behavior therapy
D.
A and B
E.
A, B, and C
13. According to research evidence, which of the following is most
important to consider among older adults when choosing among
evidence-based psychotherapies for treating late-life depression?
A.
Patient preference
B.
Therapist age
C.
Severity of patient’s depression
D.
Frequency of psychotherapy sessions
E.
P i d id i
Patient gender identity
14.
Which of the following psychotherapies has been shown to be
effective in reducing disability and suicidal ideation among older
adults with major depression and executive dysfunction?
A.
Reminiscence therapy
B.
Interpersonal therapy
C.
Brief psychodynamic therapy
D.
Problem-solving therapy
E.
Supportive psychotherapy

15.
Which of the following is true regarding the treatment of
complicated grief among older adults?
A.
Interpersonal psychotherapy is the most effective treatment
for complicated grief in older adults.
B.
Medication management with antidepressants is most
effective for treating complicated grief in older adults.
C.
Complicated grief therapy is shown to be most effective for
treating complicated grief in older adults.
D.
Complicated grief therapy has not been studied speci ically in
older adults.
E.
Complicated grief resolves on its own without treatment in
older adults.
16. Which of the following is true regarding the effectiveness of
various formats of psychotherapy used to treat late-life
d i ?
depression?
A.
Individual therapies are more effective than other forms of
psychotherapy.
B.
Group therapy formats are more effective than other forms of
psychotherapy.
C. Bibliotherapy formats are less effective than individual
psychotherapy and group psychotherapy.

D.
Bibliotherapy formats are more effective than other forms of
psychotherapy.
E.
There is no difference noted whether the therapy is delivered
in the individual, group, or bibliotherapy format.
17.
Which of the following is not associated with poorer treatment
outcomes for psychotherapy among older adults with depression?
A.
Greater baseline anxiety
B.
Greater baseline depression
C.
Lower self-rated health
D.
Presence of personality disorder
E.
Endogenous depression (vs. non-endogenous depression)
18. Which of the following is true regarding the study of telehealth
psychotherapy interventions for the treatment of depression in
older adults?
A.
In a study of depressed low-income homebound older adults,
almost all patients had extremely positive attitudes toward
telehealth problem-solving therapy.
B.
A study of older adult veterans showed that telehealth-
delivered psychotherapy for depressed older adults was not
inferior to in person treatment.
C.
Telehealth problem-solving therapy reduced suicidal ideation
among depressed low-income homebound older adults.
D. A, B, and C.
E.
A and B only.

19.
Which of the following therapies that has been shown to slow the
rate of cognitive decline more than the administration of
donepezil alone in Alzheimer’s dementia involves the guided
practice on a set of tasks designed to re lect certain cognitive
functions?
A.
Cognitive stimulation therapy
B.
Supportive psychotherapy
C.
Validation therapy
D.
Reality orientation
E.
Art therapy
20. Which of the following psychotherapies has evidence for treating
bipolar disorder among older adults?
A.
Interpersonal and social rhythm therapy (IPSRT).
B.
Cognitive behavior therapy.
C
C.
Problem solving therapy.
D.
A and B.
E.
No psychotherapy modality has been studied speci ically in
the treatment of older adult with bipolar disorder.
21.
Which of the following is not an effective accommodation strategy
to address the challenges of cognitive impairment among older
individuals participating in group psychotherapy?
A.
Reorienting and redirecting group members
B.
Presenting group material in shorter segments
C.
Attempting to create a group with individuals of similar
cognitive ability
D.
Creating a group with members who change frequently
E.
Emphasizing structure and cuing in sessions

22. Which of the following psychotherapies has been shown to be


effective in treating older adults with comorbid major depressive
disorder and personality disorders ?
A.
Reminiscence therapy
B.
Problem solving therapy
C.
Mental health support group
D.
Dialectical Behavior therapy
E.
Moral reconation therapy
23.
Which of the following is a true statement regarding the 12-step
support group model for substance use disorder treatment among
older adults?
A.
The 12-step models are equally effective for younger, middle-
aged, and older adults.
B.
There is strong evidence that age-speci ic groups in the 12-
step models improve outcomes for older adults more than
mixed age groups.
C.
The 12-step models are understudied in older adults.
D.
The 12-step models are not effective for substance use
treatment in older adults.
E.
Older adults with substance use concerns are largely unable
to participate in 12-step programs as they frequently present
with cognitive impairment.

24. Which of the following is not a bene it of group therapy for older
adults in the nursing home setting?
A.
Group therapists can let other nursing home staff know about
frustrations individual nursing home patients have with them
to give staff help in addressing the problems.
B.
Group therapy can improve life satisfaction for patients.
C.
Group therapists can promote the patient’s efforts at
independence, assertiveness, and autonomy.
D.
Group therapy can help patients successfully adjust to living
in a facility.
E.
Patients can work together and support each other to
Patients can work together and support each other to
reconcile their similar late-life challenges.
25.
Which of the following is an incorrect statement regarding marital
therapy for older adults?
A.
Retirement can cause permanent changes to the marital
relationship causing discord among couples.
B.
Physical health issues in one or both partners can create
increased dissatisfaction in marriages.
C.
Financial insecurity remains an issue for older married
couples that can be addressed with marital therapy.
D.
Changes in sexual activity and gender roles can lead to
marital issues and may be addressed by education and work
with the couple on acceptance.
E.
Marital therapy is less relevant for older couples as most
long-term marriages have worked out strategies to cope with
problems often addressed by this therapy.

Answers
Answer 1 B PST involves training participants in a ive-step problem-
solving model followed by subsequent sessions to enhance skills. It
helps patients identify problems in their lives and create a method for
selecting solutions and making concrete plans for problem resolution,
which is what is described in option B. Option A is an example of
supportive psychotherapy. Option C is most consistent with cognitive
behavior therapy. Option D describes interpersonal therapy. Option E
describes psychodynamic therapy.

Answer 2 C Studies show that older adults prefer psychotherapy to


pharmacologic interventions for the treatment of depression. All other
options are genuine barriers to older adults receiving psychotherapy
for the treatment of their depression.

Answer 3 E IPT begins by capturing the essence of the patient’s


problems in one of the four broad categories, which include unresolved
grief, role transition, role dispute, or interpersonal de icits. Retirement,
relocation, and transition to the patient role are all examples of role
transitions, which involve dif iculty coping with changes in life
circumstances. Unresolved grief involves the death of a person close to
the patient. Interpersonal de icit involves a history of few or
contentious relationships with others. Role disputes arise from
interpersonal relationships that have nonreciprocal role expectations
between the two parties. Ineffective communication is not a core
problem area de ined in IPT.

Answer 4 A A 2006 meta-analysis that compared pharmacotherapy


and psychotherapy among depressed older adults indicated that both
were effective with moderate-to-large effect sizes. Authors cautioned
against reading too much into comparison of effect sizes between
psychotherapy and psychopharmacology given that
psychopharmacology studies more effectively produce a placebo effect
than therapy studies.

Answer 5 B Erik Erikson included phases of older adulthood in his


model of psychosocial development, speci ically the phase of ego
integrity vs. despair. This changed many assumptions by identifying
that older adults still have life tasks, opportunities for growth, and
possibilities for maladaptation, which are appropriate for
psychotherapy. The remaining theorists focused on childhood
development.

Answer 6 E A 2016 meta-analysis of randomized controlled trials for


psychotherapy treatment of depression in late-life indicates that all
bona ide psychotherapies studied were effective. In addition to the
above listed modalities, interpersonal therapy and life review therapy
were also shown to be effective.
Answer 7 A Psychodynamic therapy has evidence for treating many
issues in frail older adults. Option A is an example of transference
speci ic to the age difference between a therapist and the patient.
Transference involves redirecting emotions or ideas toward the
therapist, which serves as a substitute for people in the patient’s life.
Option B is an example of transference that can occur in older adults
but does not relate to the age difference between the patient and
therapist. Option C is an example of countertransference. Options D and
E describe the barriers to older adults receiving psychotherapy.

Answer 8 E Based on the evidence of safety and ef icacy, cognitive


behavior therapy and sleep hygiene should always be the irst-line
treatment for insomnia in older adults. Interpersonal therapy does not
have evidence in treating insomnia. Benzodiazepines, antipsychotics,
and non-benzodiazepine sleep agents are all effective in treating
insomnia but should not be used as irst line in older adults due to the
safety concerns with the medications.

Answer 9 D Data from systematic reviews and meta-analyses


indicate that cognitive behavior therapy is the most studied therapy
modality for the treatment of anxiety disorders among older adults. All
other therapies listed are also effective psychotherapies among older
adults with anxiety disorders, but they have been studied mainly for the
treatment of late-life depression.

Answer 10 C While many psychotherapy studies for depressed or


anxious older adults included individuals with cognitive impairment,
there is no evidence for traditional psychotherapy forms such as CBT,
IPT, and psychodynamic therapy for treating dementias. However, some
studies show modest evidence for behavioral interventions such as
music or art therapy in managing behavioral and psychological
symptoms of dementia (BPSD).

Answer 11 A Reminiscence therapy is frequently done with older


adults with depression or dementia and may be done in a group or as
one on one. This involves looking back over one’s life with photographs,
objects, or themes. The aim is to help patients feel satis ied with one’s
life, resolving Erikson’s con lict in older age of ego integrity vs. despair.
Answer 12 D This question derives from two important randomized
control trials studying interventions in primary care settings for the
treatment of late-life depression. This is important, as older adults
prefer care in primary care settings. The irst study used problem-
solving therapy (PST) as the therapy modality in the intervention called
improving mood-promoting access to collaborative treatment
(IMPACT) for late-life depression. The second study used interpersonal
therapy (IPT) in an intervention which reduced depression and suicidal
thoughts in older adults as part of the Prevention of Suicide in Primary
Care Elderly: Collaborative Trial (PROSPECT).

Answer 13 A Evidence suggests that patient preference is an


important consideration in choosing a treatment approach for
depressed older adults [1]. One psychotherapy study in older adults
suggests that patient preference is an important consideration in
treatment selection [2], and another argues for a focus to personalized
psychotherapy interventions [3].

Answer 14 D Studies speci ic to older adults with executive


dysfunction and major depression show that problem-solving therapy
is effective in reducing disability and suicidal ideation in this
population. The other therapies do not have evidence in this speci ic
population.

Answer 15 C Complicated grief therapy incorporates strategies from


exposure therapy, motivational interviewing, and interpersonal therapy
and has been found to be most effective in reducing symptoms of
complicated grief. Among older adults, complicated grief therapy was
more effective in treating complicated grief when compared to
interpersonal therapy. There is little evidence that antidepressant
medications are a useful strategy in treating complicated grief unless
associated with comorbid with major depressive disorder.

Answer 16 E A 2006 meta-analysis of randomized controlled trials of


psychological treatments for late-life depression showed that there was
no difference in effectiveness between individual, group, or
bibliotherapy formats of psychotherapy.
Answer 17 B A 2011 review of moderators and predictors of
treatment outcomes for psychotherapy showed that results among
studies were inconsistent regarding predicting effects of baseline
depression severity to treatment outcomes. All other options were
associated with worse treatment outcomes for psychotherapy in older
adults. Endogenous depression is a type of major depression that tends
to have a higher genetic component and is related to poor affective
control, anhedonia, and agitation.

Answer 18 D Preliminary studies of telehealth psychotherapy


interventions showed that almost all depressed low-income
homebound older adults had extremely positive attitudes toward
telehealth problem-solving therapy. In addition, it was found in a study
of older adult veterans that telehealth-delivered psychotherapy for
depressed older adults was not inferior to in person treatment.
Furthermore, telehealth problem-solving therapy was found to reduce
suicidal ideation among depressed low-income homebound older
adults.

Answer 19 A Cognitive stimulation therapy has been shown to slow


the rate of cognitive decline more than the administration of donepezil
alone in Alzheimer’s dementia and involves the guided practice on a set
of tasks designed to re lect certain cognitive functions. The other
options are therapies that can bene it patients with dementia but do
not have evidence for slowing cognitive decline.

Answer 20 E IPSRT and CBT are both evidence-based


psychotherapies for bipolar disorder, however neither have been
studied speci ically in older adults with bipolar disorder. Studies are
lacking regarding any speci ic psychotherapy and the treatment of
older adult bipolar disorder.

Answer 21 D Cognitive impairment is a speci ic challenge for


creating psychotherapy groups in older adults. Having some degree of
homogeneity in cognitive ability, using reorientation and redirecting
techniques, presenting material in shorter segments, using cues to
stimulate memory recall, and structuring sessions are all
accommodations which may make group psychotherapy more effective
for the cognitively impaired. Changing group members frequently is not
a strategy for addressing cognitive impairment among older adults
receiving group therapy. While group members may not know other
participant’s names, they will recognize each other over time, which
allows for more meaningful participation.

Answer 22 D Marsha Linehan’s dialectical behavior therapy is the


only speci ic type of psychotherapy studied in the treatment of
personality disorders in older adults. Dialectical behavior therapy is a
group therapy where patients work on skills to improve distress
tolerance, emotion regulation, and interpersonal effectiveness. Moral
reconation therapy is a group therapy used to treat antisocial
personality disorder but is not studied in older adults. The remaining
therapies have not been studied with respect to personality disorders
in older adults.

Answer 23 C The 12-step models are largely understudied in older


adults. In 1998, the Center for Substance Abuse Treatment made
recommendations that age-speci ic groups for 12-step programs may
be more helpful for older adults; however there have been no studies
con irming this. Although older adults are more likely to have cognitive
impairment, the 12-step models have been shown to be effective in
reducing shame, stigma, and isolation in older adults who abuse
substances, though no systematic studies have been done.

Answer 24 A Group therapists in nursing homes should stay in the


therapist role maintaining the con identiality of the group and instead
promote patient efforts to assert themselves to address problems on
their own. The exception to this would be mandated reporting events.
All other options have shown the bene its of group therapy in the
nursing home setting.

Answer 25 E Role changes including retirement, physical health


issues, and sexual and gender roles are all issues that can arise in older
adult marriages and may be helped by marital therapy. Financial
insecurities cause marital con lict at any age. Although older adults may
be less likely to change ingrained habits, marital therapy is still relevant
as older adult couples go through many of the changes described above.
Further Reading
Agronin M. Group therapy in older adults. Curr Psychiatry Rep. 2009;11:27–32.
[PubMed]

Alexopoulos GS, Raue PJ, Kiosses DK, et al. Problem solving therapy and supportive
therapy in older adults with major depression and executive dysfunction: effect on
disability. Arch Gen Psychiatry. 2011;68:33–41.
[PubMed][PubMedCentral]

Arean PA, Hegel MT, Reynolds CF. Treatment of depression in older medical patients
with psychotherapy. J Clin Geropsychol. 2001;7:93–104.

Boylin W, Gordon SK, Nehrke MF. Reminiscing and ego integrity in institutionalized
elderly males. The Gerontologist. 1976;16(2):118–24.
[PubMed]

Bruce ML, Ten Have TR, Reynolds CF 3rd, Katz II, Schulberg HC, Mulsant BH, et al.
Reducing suicidal ideation and depressive symptoms in depressed older primary
care patients: a randomized controlled trial. JAMA. 2004;291(9):1081–91.
[PubMed]

Bruer RA, Spitznagel E, Cloninger CR. The temporal limits of cognitive change from
music therapy in elderly persons with dementia or dementia-like cognitive
impairment: a randomized controlled trial. J Music Ther. 2007;44(4):308–28.
[PubMed]

Center for Substance Abuse Treatment. Substance Abuse among Older Adults;
Treatment Improvement Protocol (TIP) Series 26. Rockville, MD: Substance Abuse
and Mental Health Services Administration, U.S. Department of Health and Human
Services; 1998.

Chapman SB, Weiner MF, Rackley A, Hynan LS, Zientz J. Effects of cognitive-
communication stimulation for Alzheimer’s disease patients treated with donepezil.
J Speech Language Hearing Res. 2004;47:1149–63.

Choi NG, Hegel MT, Marti N, et al. Telehealth problem-solving therapy for depressed
low-income homebound older adults. Am J Geriatr Psychiatry. 2014;22(3):263–71.
[PubMed][PubMedCentral]

Choi NG, Marti CN, Conwell Y. Effect of problem-solving therapy on depressed low-
income homebound older adults’ death/suicidal ideation and hopelessness. Suicide
Life Threat Behav. 2016;46(3):323–36.
[PubMed]
Cuijpers P, van Straten A, Smit F. Psychological treatment of late-life depression: a
meta-analysis of randomized controlled trials. Int J Geriatr Psychiatry.
2006;21(12):1139–49.
[PubMed]

Dakin EK, Areá n P. Patient perspectives on the bene its of psychotherapy for late-life
depression. Am J Geriatr Psychiatry. 2013;21(2):155–63. 3
[PubMed]

Dols A, Beekman A. Older age bipolar disorder. Psychiatr Clin North Am.
2018;41(1):95–110.
[PubMed]

Egede LE, Acierno R, Knapp RG, et al. Psychotherapy for depression in older veterans
via telemedicine: a randomized, open-label, non-inferiority trial. Lancet Psychiatry.
2015;2(8):693–701.
[PubMed]

Goldstein S, Preston J. Marital therapy for the elderly. Can Med Assoc J.
1984;130(12):1551–3.
[PubMed][PubMedCentral]

Goncalves DC, Byrne GJ. Interventions for generalized anxiety disorder in older
adults: systematic review and meta-analysis. J Anxiety Disord. 2012;26(1):1–11.
[PubMed]

Gum A, Areá n PA. Current status of psychotherapy for mental disorders in the
elderly. Curr Psychiatry Rep. 2004;6(1):32–8.
[PubMed]

Gum AM, Areá n PA, Hunkeler E, Tang L, Katon W, Hitchcock P, et al. Depression
treatment preferences in older primary care patients. The Gerontologist.
2006;46(1):14–22.
[PubMed]

Gustavson KA, Alexopoulos GS, Niu GC, McCulloch C, Meade T, Arean PA. Problem-
solving therapy reduces suicidal ideation in depressed older adults with executive
dysfunction. Am J Geriatr Psychiatr. 2016;24(1):11–7.

Hendriks GJ, Oude Voshaar RC, Keijsers GP, Hoogduin CA, van Balkom AJ. Cognitive
behavioural therapy for late-life anxiety disorders: a systematic review and meta-
analysis. Acta Psychiatr Scand. 2008;117(6):403–11.
[PubMed]
Huang AX, Delucchi K, Dunn LB, Nelson JC. A systematic review and meta-analysis of
psychotherapy for late-life depression. Am J Geriatr Psychiatry. 2016;23(3):261–73.

Kiosses DN, Alexopoulos GS. Problem-solving therapy in the elderly. Curr Treat
Options Psychiatry. 2014;1(1):15–26.
[PubMed][PubMedCentral]

Kiosses DN, Leon AC, Arean PA. Psychosocial interventions for late-life major
depression: evidence-based treatments, predictors of treatment outcomes, and
moderators of treatment effects. Psychiatr Clin North Am. 2011;34(2):377–401. viii
[PubMed][PubMedCentral]

Knight ZG. A proposed model of psychodynamic psychotherapy linked to Erik


Erikson’s eight stages of psychosocial development. Clin Psychol Psychother.
2017;24(5):1047–58.
[PubMed]

Kuerbis A, Sacco P, Blazer DG, Moore SS. Substance abuse among older adults. Clin
Geriatr Med. 2014;30:629–54.
[PubMed][PubMedCentral]

Lynch TR, Cheavens JS, Cukrowicz KC, Thorp SR, Bronner L, Beyer J. Treatment of
older adults with co-morbid personality disorder and depression: a dialectical
behavior therapy approach. Int J Geriatr Psychiatry. 2007;22(2):131–43.
[PubMed]

Miklowitz DJ. A review of evidence-based psychosocial interventions for bipolar


disorder. J Clin Psychiatry. 2006;67(Suppl 11):28–33.
[PubMed]

Miller MD. Using interpersonal therapy (IPT) with older adults today and tomorrow:
a review of the literature and new developments. Curr Psychiatry Rep. 2008;10:16–
22.
[PubMed]

Moran JA, Gatz M. Group therapies for nursing home adults: an evaluation of two
treatment approaches. Gerontologist. 1987;27:588–91.
[PubMed]

Morgan AC. Psychodynamic psychotherapy with older adults. Psychiatr Serv.


2003;54:1592–4.
[PubMed]

Peisah C, Lawrence G, Reutens S. Creative solutions for severe dementia with BPSD: a
case of art therapy used in an inpatient and residential care setting. Int
Psychogeriatr. 2011;23(6):1011–3.
[PubMed]

Pinquart M, Duberstein PR, Lyness JM. Treatments for later-life depressive


conditions: a meta-analytic comparison of pharmacotherapy and psychotherapy. Am
J Psychiatry. 2006;163(9):1493–501.
[PubMed]

Raue PJ, Schulberg HC, Heo M, Klimstra S, Bruce ML. Patients’ depression treatment
preferences and initiation, adherence, and outcome: a randomized primary care
study. Psychiatr Serv. 2009;60(3):337–43. 2
[PubMed][PubMedCentral]

Raue PJ, McGovern AR, Kiosses DN, Sirey JA. Advances in psychotherapy for
depressed older adults. Curr Psychiatry Rep. 2017;19(9):57.
[PubMed][PubMedCentral]

Schroeck JL, Ford J, Conway EL, Kurtzhalts KE, Gee ME, Vollmer KA, Mergenhagen
KA. Review of safety and ef icacy of sleep medicines in older adults. Clin Ther.
2016;38(11):2340–72.
[PubMed]

Schwartz K. Remembering the forgotten: psychotherapy groups for the nursing home
resident. Int J Group Psychother. 2007;57:491–514.

Shear K, Wang Y, Skritskaya N, Duan N, Mauro C, Ghesquiere A. Treatment of


complicated grief in elderly persons: a randomized clinical trial. JAMA Psychiat.
2014;71:1287–95.

Simon NM. Treating complicated grief. JAMA. 2013;310:416–23.


[PubMed][PubMedCentral]

Tripathi RK, Tiwari SC. Psychotherapeutic approaches in the management of


elderlies with dementia: an overview. Delhi Psychiatry J. 2009;12(1):31–41.

Unü tzer J, Katon W, Callahan CM, et al. Collaborative care management of late-life
depression in the primary care setting: a randomized controlled trial. JAMA.
2002;288(22):2836–45.
[PubMed]

Woods B, Spector A, Jones C, Orrell M, Davies S. Reminiscence therapy for dementia.


Cochrane Database Syst Rev. 2005;2:CD001120.

Wu LT, Blazer DG. Illicit and nonmedical drug use among older adults: a review. J
Aging Health. 2011;23(3):481–504.
[PubMed]
© Springer Nature Switzerland AG 2021
R. Tampi et al. (eds.), Absolute Geriatric Psychiatry Review
https://doi.org/10.1007/978-3-030-58663-8_37

37. Interdisciplinary Approaches


Rajesh Tampi1 and Deena Tampi2
(1) Department of Medicine, Cleveland Clinic Lerner College of
Medicine of Case Western Reserve University, Cleveland, OH, USA
(2) Behavioral Health Advisory Group, Princeton, NJ, USA

Keywords Interdisciplinary approaches – Multidisciplinary teams –


Multicomponent interventions – Non-pharmacological management –
Behavioral and psychological symptoms of dementia (BPSD) –
Potentially inappropriate prescribing (PIP)

Questions
1. A 78-year-old widowed woman with a history of bipolar disorder
presents to the emergency room for the ifth time in 6 months for
worsening symptoms of depression and suicidal ideation. She has
had two psychiatric hospitalizations in this period. The patient did
well during these hospitalizations and was discharged home with
outpatient psychiatric follow-up. The patient failed to attend all her
outpatient psychiatric follow-up appointments despite reminders
by the clinic and has run out of medications on two occasions.
Which of the following would be the most useful and supportive
way to prevent emergency room visits and rehospitalizations for
this woman?
A.
Prescribe a long-acting injectable mood stabilizer
B.
Prescribe electroconvulsive therapy (ECT)
C. Admit to a state hospital for long-term psychiatric hospital stay
D.
Assign a psychiatric nurse who can follow-up with the patient
at her home
E.
Admit the patient to a skilled nursing facility (SNF)
2.
Multicomponent interventions (MI), which involves a series of non-
pharmacological strategies handled by nursing staff, signi icantly
reduces which of the following outcomes among older adults with
delirium?
A.
Incident delirium
B.
Duration of delirium
C.
Length of hospital stay
D.
Mortality rates
E.
Rates of dementia
3. Which of the following is true for the non-pharmacological
management of behavioral and psychological symptoms of
dementia (BPSD)?
A.
Specialized dementia units show consistent bene its in the
management of BPSD
B.
Education of caregivers and residential care staff show bene its
in the management of BPSD
C.
Severity, but not frequency, of BPSD was reduced by the non-
pharmacological management strategies
D.
Caregiver burden due to BPSD is not reduced by non-
h l i l i
pharmacological management strategies
E. 20–25 sessions delivered over 2–3 years are required for the
non-pharmacological management strategies to be effective for
the management of BPSD
4.
Which of the following is not an outcome of potentially
inappropriate prescribing (PIP) among older adults?
A.
Mortality
B.
Emergency room visits
C.
Adverse drug events
D.
Functional decline
E.
Hospitalizations
5.
Which of the following is not a predictor of long-term care (LTC)
placement among individuals with dementia?
A.
White race
B.
Older age
C.
Being married
D.
Greater dementia severity
E.
Caregiver burden

Answers
Answer 1 D In this situation, assigning a psychiatric nurse who can
follow-up with the patient at home to ensure that she takes her
medications, attends her outpatient appointments, and does not run
out of medications is the most useful and supportive way to prevent
emergency room visits and rehospitalizations for this woman. The
prescription of an injectable mood stabilizer and ECT should improve
the patient’s symptoms of bipolar disorder and prevent the relapse of
symptoms, but it will not ensure that the woman follows up with her
outpatient appointments, does not run out of medications, or miss
treatments. Admissions to a state hospital or to a SNF are reserved for
chronically ill individuals or for those individuals who have
impairments in activities of daily living.

Answer 2 A A recent systematic review that evaluated


multicomponent interventions for delirium among older adults found
that MI signi icantly reduced incident delirium and accidental falls
during hospitalization, but was not effective in reducing the rates of
dementia. MI also produced nonsigni icant reductions in the duration
of delirium, length of hospital stay, and mortality.

Answer 3 B A systematic review by Livingston et al. found that


educating caregivers and residential care staff, and possibly cognitive
stimulation therapy, appears to be effective interventions in the
management of BPSD. However, specialized dementia units did not
show consistent bene its in the management of BPSD. Wandering was
reduced by visually changing the environment and unlocking doors. A
meta-analysis by Brodaty and Arasaratnam found that the frequency
and severity of BPSD and caregiver burdens were both reduced by non-
pharmacological interventions provided by the family caregivers. They
investigators also found that these interventions had effect sizes similar
to those associated with pharmacotherapy. Approximately 9–12
individual sessions designed to meet the needs of the individuals with
dementia and their caregivers and delivered to their homes over a 3–6-
month time period with follow-ups were the most successful.

Answer 4 A A recent meta-analysis that included data from eight


articles found that PIP among older adults did not affect mortality (risk
ratio [RR] 0.98; 95% CI, 0.93–1.05) but was signi icantly associated
with emergency room visits (RR 1.63; 95% CI, 1.32–2.00), adverse drug
events (RR 1.34; 95% CI, 1.09–1.66), functional decline (RR 1.53; 95%
CI, 1.08–2.18), health-related quality of life (standardized mean
difference − 0.26; 95% CI, −0.36 to −0.16), and hospitalizations (RR
1.25; 95% CI, 1.09–1.44).

Answer 5 C A meta-analysis that included data from 37 studies found


that white race, greater dementia severity, older age, behavioral and
psychological symptoms of dementia, greater degree of functional
impairment, and caregiver burden increased the risk of LTC placement
among individuals with dementia. Married individuals with dementia
and living with their caregiver reduced the risk for LTC placement.

Further Reading
Brodaty H, Arasaratnam C. Meta-analysis of nonpharmacological interventions for
neuropsychiatric symptoms of dementia. Am J Psychiatry. 2012;169(9):946–53.
[Crossref]

Cepoiu-Martin M, Tam-Tham H, Patten S, Maxwell CJ, Hogan DB. Predictors of long-


term care placement in persons with dementia: a systematic review and meta-
analysis. Int J Geriatr Psychiatry. 2016;31(11):1151–71.
[Crossref]

Liew TM, Lee CS, Goh Shawn KL, Chang ZY. Potentially inappropriate prescribing
among older persons: a meta-analysis of observational studies. Ann Fam Med.
2019;17(3):257–66.
[Crossref]

Livingston G, Johnston K, Katona C, et al. Old Age Task Force of the World Federation
of Biological Psychiatry. Systematic review of psychological approaches to the
management of neuropsychiatric symptoms of dementia. Am J Psychiatry.
2005;162(11):1996–2021.
[Crossref]

Martinez F, Tobar C, Hill N. Preventing delirium: should non-pharmacological,


multicomponent interventions be used? A systematic review and meta-analysis of
the literature. Age Ageing. 2015;44(2):196–204.
[Crossref]

Sorrell JM. Caring for older adults with bipolar disorder. J Psychosoc Nurs Ment
Health Serv. 2011;49(7):21–5.
[Crossref]
© Springer Nature Switzerland AG 2021
R. Tampi et al. (eds.), Absolute Geriatric Psychiatry Review
https://doi.org/10.1007/978-3-030-58663-8_38

38. Setting-Speci ic Treatment Issues


Arushi Kapoor1
(1) Department of Psychiatry, Perelman School of Medicine at the
University of Pennsylvania, Philadelphia, PA, USA

Keywords Dementia – Goals of care – Minimum data set (MDS) –


Long-term care facilities – Special care units (SCUs)

Questions
1. A 92-year-old woman with advanced Alzheimer’s disease who is
nonverbal, bedbound, and resides at a nursing home presents to
the emergency department (ED) with fever and mild respiratory
distress. The patient is found to have a right lobar pneumonia on
chest X-ray. Her daughter (who is also her healthcare proxy)
informs the physician that the patient was recently discharged
from the ICU after being intubated for aspiration pneumonia
2 weeks earlier. Which one of the following options is the most
appropriate next step in her treatment?
A.
Intubate the patient immediately to prevent further respiratory
decompensation
B.
Admit to the hospital for further monitoring
C.
Admit to the hospital for the administration of IV antibiotics
and luids
D.
Discuss with the daughter the goals of care before proceeding
E. Discharge the patient back to the nursing home, as further
medical treatment is futile
2.
Which one of the following factors is not associated with nursing
home placement?
A.
Increased age
B.
Low self-rated health status
C.
Race
D.
Functional impairment
E.
Prior nursing home placement
3.
Which of the following is true for the treatment of chronic pain
among older adults living at nursing homes?
A.
System modi ication interventions are better in bene it than
educational interventions
B.
System modi ication interventions are better in bene it than
analgesic interventions
C.
Analgesic interventions are better in bene it than educational
interventions
D.
Analgesic interventions are equal in bene it than system
modi ication interventions
E.
Analgesic interventions are equal in bene it than educational
interventions
4. When a resident is admitted to a long-term care setting, several
initial assessments and evaluations are performed to establish a
data set. Which of the following statements is false regarding the
minimum data set (MDS)?
minimum data set (MDS)?

A. It is only applicable to residents in skilled nursing facilities


funded by Medicare which mainly cover short-term stay
rehabilitation and skilled nursing care in adults following
hospitalization
B.
It is part of a federally mandated process for clinical
assessment of residents in Medicare- or Medicaid-certi ied
nursing homes
C.
It includes assessments on behavioral symptoms, psychosocial
functions, mobility, cognitive status, restraints, and
medications
D.
It is updated every 3 months to assess for needs associated
with functioning and disability in the vulnerable older adults
E.
MDS plays a role in the reimbursement system and in
monitoring the quality of care provided by nursing facility
residents
5. Special care units (SCUs) are an aspect of some nursing homes.
Which of the following statements is not true about SCUs?
A.
Research on the effectiveness of SCUs is dif icult to interpret
and generalize because of the heterogeneity of these facilities,
lack of adjustment for differences between comparator groups,
and absence of any randomized controlled trials
B.
The rate of decline in activities of daily living is not
signi icantly slower for SCU residents when compared to non-
SCU residents
C.
The SCUs are signi icantly more effective in bringing about
positive outcomes as opposed to traditional units and
recommended for dementia care
D.
SCUs include increased family involvement and specialized
training of staff members in Alzheimer’s dementia
training of staff members in Alzheimer s dementia
E. SCUs commonly refer to specially designed residential care
settings catering only for people with dementia or more
speci ically for individuals with Alzheimer’s disease (AD)

Answers
Answer 1D The evidence indicates that only 1% of residents
admitted to nursing homes with advanced neurocognitive disorder
were perceived to have a life expectancy of less than 6 months.
However more than 70% of these individuals died within that period.
Given the high level of mortality especially in the light of the recent
hospitalization, the physician should have a detailed discussion with
the daughter (healthcare proxy) before proceeding with any
interventions for the patient. This discussion will determine how well
the daughter appreciates the patient’s prognosis. Any plans to intubate,
hospitalize, or discharge the patient should be irst discussed with the
daughter.

Answer 2C Predictors with strong evidence for nursing home


placement include increased age, low self-rated health status,
functional and cognitive impairment, dementia, prior nursing home
placement, and a high number of prescriptions. Predictors with
inconsistent results were male gender, low education status, low
income, stroke, hypertension, incontinence, depression, and prior
hospital use.

Answer 3C In a meta-analysis that evaluated the ef icacy of


interventions aimed at reducing chronic pain among residents of
nursing home who were >65 years in age, the investigators found that
residents receiving analgesic interventions bene ited most followed by
those receiving educational interventions and those receiving system
modi ication interventions. This data indicates that analgesics are the
most effective pain intervention and should be considered the irst-line
treatment among these individuals.

Answer 4A Minimum data set (MDS) is a standardized assessment


tool that measures health status (performed and recorded by nursing
home staff and includes information on a number of aging-relevant
domains including functional and cognitive status, psychosocial
functioning, geriatric syndromes, and life care wishes) in nursing home
residents. Interim assessments are also required for patients who have
had a recent major change in health status. MDS assessments occur
more frequently during periods of care covered by the Medicare skilled
nursing facility (SNF) bene it. MDS assessments are completed every
3 months (or more often depending on circumstances) on nearly all
residents of nursing homes in the United States. As such, MDS is an
extremely valuable resource for studying function and disability on a
large scale in vulnerable older adults. MDS data are collected and made
available as one of the many data products of the Centers for Medicare
and Medicaid Services (CMS).

Answer 5B Special care units (SCUs) have lourished since the 1980s
with the aim of taking care of dementia patients, usually those with
Alzheimer’s disease (AD) and in particular for those individuals with
behavioral problems. Although lacking a standard de inition, SCUs are
usually situated within nursing homes and commonly include the
features of trained staf ing, special programming, a modi ied physical
environment, and family involvement. The costs of SCUs are commonly
higher than for “standard” nursing home care. However, evaluations of
the outcomes of SCUs have yielded con licting results. There is limited
evidence to support the assumption that the care of people with
dementia in special care units is superior to care in traditional nursing
units. Despite the interest in and proliferation of SCUs in long-term care
settings, the outcomes of SCUs are not without controversies. Some
studies show associations between SCU environments and
improvements or slowed decline in residents’ self-care skills, social
function, mobility, and affective responses, while others reveal no
particular bene its when compared with traditional units.

Further Reading
Cadigan RO, Grabowski DC, Givens JL, Mitchell SL. The quality of advanced dementia
care in the nursing home: the role of special care units. Med Care. 2012;50(10):856–
62.
[Crossref]

Doupe MB, Poss J, Norton PG, et al. How well does the minimum data set measure
healthcare use? A validation study. BMC Health Serv Res. 2018;18(1):279. Published
2018 Apr 11
[Crossref]

Joyce NR, McGuire TG, Bartels SJ, Mitchell SL, Grabowski DC. The impact of dementia
special care units on quality of care: an instrumental variables analysis. Health Serv
Res. 2018;53(5):3657–79.
[Crossref]

Knopp-Sihota JA, Patel P, Estabrooks CA. Interventions for the treatment of pain in
nursing home residents: a systematic review and meta-analysis. J Am Med Dir Assoc.
2016;17(12):1163.e19–28.
[Crossref]

Kok JS, Berg IJ, Scherder EJ. Special care units and traditional care in dementia:
relationship with behavior, cognition, functional status and quality of life – a review.
Dement Geriatr Cogn Dis Extra. 2013;3(1):360–75.
[Crossref]

Lai CK, Yeung JH, Mok V, Chi I. Special care units for dementia individuals with
behavioural problems. Cochrane Database Syst Rev. 2009;4:CD006470. Published
2009 Oct 7

Lix LM, Yan L, Blackburn D, et al. Agreement between administrative data and the
resident assessment instrument minimum dataset (RAI-MDS) for medication use in
long-term care facilities: a population-based study. BMC Geriatr. 2015;15:24.
Published 2015 Mar 11
[Crossref]

Luppa M, Luck T, Weyerer S, et al. Prediction of institutionalization in the elderly. A


systematic review. Age Ageing. 2010;39(1):31–8.
[Crossref]

Mitchell SL, Kiely DK, Hamel MB. Dying with advanced dementia in the nursing
home. Arch Intern Med. 2004;164:321–6.
[Crossref]
© Springer Nature Switzerland AG 2021
R. Tampi et al. (eds.), Absolute Geriatric Psychiatry Review
https://doi.org/10.1007/978-3-030-58663-8_39

39. Special Management Considerations


Kyle Resnick1
(1) Department of Psychiatry & Behavioral Sciences, Cleveland Clinic
Akron General, Akron, OH, USA

Kyle Resnick
Email: kresnick@neomed.edu

Keywords Special management considerations – Wandering –


Restraints – Aggression – Behavioral disturbance – Behavioral and
psychological symptoms of dementia (BPSD)

Questions
1. Which of the following is a false statement regarding wandering
among individuals with dementia?
A.
Individuals with Alzheimer’s disease (AD) are more likely to be
wanderers than those with a vascular dementia (VaD)
B.
Wandering correlates with the severity of cognitive
impairment
C.
Functional impairment is closely associated with wandering
D.
Mild depression is found more frequently in wanderers with
dementia
E.
Wandering is associated with double the risk of fractures
2.
Which of the following would be considered a irst-line
intervention to manage behavioral and psychological symptoms of
dementia (BPSD)?
A.
Behavioral therapy
B.
Communication training for caregivers
C.
Music therapy
D.
Aromatherapy
E.
All of the above

3.
Which of the following statements are true about the use of
physical restraints among older individuals with dementia?
A.
The use of restraints has been associated with decreased risk
of disruptive behavior
B.
The use of restraints has been associated with increased
morbidity and mortality
C.
The use of restraints has been associated with decreased risk
of fall or injury
D.
The use of restraints has been associated with decreased risk
of cognitive impairment
E.
All of the above
4. Which of the following medications has shown bene it in the
management of BPSD among older adults?
A.
Donepezil
B.
Citalopram
C. Risperidone

D.
Carbamazepine
E.
All of the above
5.
In which of the following circumstances is it not reasonable to use
pharmacotherapy to manage BPSD?
A.
An individual with dementia does not do as requested by their
caregiver
B.
Non-pharmacological management techniques have been tried
with limited bene it
C.
The behaviors exhibited by the individual with dementia are
severe and persistent
D.
The behaviors exhibited by the individual with dementia are
dangerous to their well-being
E.
All of the above

Answers
Answer 1 D Wandering behaviors, typi ied by aimless or disoriented
ambulation, are commonly observed in more severe cases of dementia.
Individuals with AD are more likely to be wanderers than those with a
VaD. Wandering correlates with the severity of cognitive impairment.
Conversely, functional impairment, especially as it re lects the ability to
walk, limits who can wander and relatively preserved activities of daily
living is closely associated with wandering behavior. Moderate-to-
severe depression is found more frequently in wanderers with
dementia. Individuals with dementia who wander have double the risk
of fractures.

Answer 2 E Non-pharmacological interventions have provided


positive results in reducing symptoms of BPSD. Most studies show that
these interventions have important and signi icant ef icacy. It is
recommended that non-pharmacologic interventions be attempted irst
to manage BPSD.

Answer 3 B The use of physical restraint has been associated with


increased risk of morbidity and mortality when used on older
individuals with dementia. These outcomes have included physical
injuries due to entrapment, decreased mobility, or worsening
psychological symptoms. The use of restraints has not been associated
with any improvements to the risk of falls, injury, disruptive behavior,
or cognitive impairment and may in increase the risk of these events.

Answer 4 E Most guidelines support non-pharmacological


interventions and optimization of an individual’s current medication
regimen as the irst step in managing BPSD. However, there is some
evidence for the use of different medication classes to alleviate agitated
and aggressive behaviors among older individuals with dementia.
Cholinesterase inhibitors like donepezil are generally safe and well-
tolerated, and they are recommended for individuals with mild-to-
moderate disease. The use of cognitive enhancers may also delay the
onset of BPSD. Antidepressants such as selective serotonin reuptake
inhibitors may decrease agitation among patients who also exhibit
symptoms of depressed mood or anxiety. Special care should be taken
to titrate these medications slowly due to the increased risk of falls
associated with their use. Antipsychotics may help with severe
agitation and aggression, especially if there is a psychotic component to
the presentation. Due to a signi icant adverse effect pro ile, special
consideration should be made when prescribing antipsychotics, such as
using the lowest possible dose and discontinuing the drug as soon as
the symptoms resolve. Antiepileptics like carbamazepine may help
decrease agitation behaviors, but they will require careful monitoring
due to the elevated risk of toxicity in older adults.
Answer 5 D It is generally accepted that non-pharmacologic
interventions should be pursued before utilizing pharmacotherapy to
manage agitated and aggressive behaviors among individuals with
dementia. However, in certain circumstances, it is reasonable to use
pharmacotherapy. These circumstances are as follows: non-
pharmacological management techniques have been tried with limited
bene it, the behaviors exhibited by the individual with dementia are
severe and persistent, and the behaviors exhibited by the individual
with dementia are dangerous to their well-being and the well-being of
others.

Further Reading
Cipriani G, Lucetti C, Nuti A, Danti S. Wandering and dementia. Psychogeriatrics.
2014;14(2):135–42.
[Crossref]

de Oliveira AM, Radanovic M, de Mello PC. Nonpharmacological interventions to


reduce behavioral and psychological symptoms of dementia: a systematic review.
Biomed Res Int. 2015;2015:218980.
[PubMed][PubMedCentral]

Gerlach LB, Kales HC. Managing behavioral and psychological symptoms of dementia.
Psychiatr Clin North Am. 2018;41(1):127–39.
[Crossref]

McDermott CL, Gruenewald DA. Pharmacologic Management of Agitation in patients


with dementia. Curr Geriatr Rep. 2019;8(1):1–11.
[Crossref]

Mö hler R, Richter T, Kö pke S, Meyer G. Interventions for preventing and reducing the
use of physical restraints in long-term geriatric care. Cochrane Database Syst Rev.
2011;2:CD007546.
Part VI
Policy, Ethical and Legal Issues
In this part, we review various aspects of caring for older adults with
psychiatric disorders including policy, ethical and legal issues. We also
review the role of geriatric psychiatrists in the health care systems and
treatment-setting regulations.
Policy · Ethical issues · Legal issues · Geriatric psychiatrists · Health
care systems · Treatment-setting regulations
Rajesh R. Tampi
Deena J. Tampi
Nery A. Diaz
Arushi Kapoor
© Springer Nature Switzerland AG 2021
R. Tampi et al. (eds.), Absolute Geriatric Psychiatry Review
https://doi.org/10.1007/978-3-030-58663-8_40

40. Role of Geriatric Psychiatrist in


Healthcare Systems
Rajesh Tampi1 and Deena Tampi2
(1) Department of Medicine, Cleveland Clinic Lerner College of
Medicine of Case Western Reserve University, Cleveland, OH, USA
(2) Behavioral Health Advisory Group, Princeton, NJ, USA

Keywords Geriatric psychiatrist – Falls – Prescription drug misuse –


Benzodiazepines – Dementia

Questions
1. A 67-year-old woman who has been taking 4 mg of lorazepam for
the last 40 years for anxiety symptoms is referred to you by her
primary care physician as she had two recent falls. The patient is
insisting on continuing on the same dose of lorazepam , but the
physician is uncomfortable prescribing the lorazepam to this
patient. The physician wants you to see the patient for a
consultation in your geriatric psychiatry clinic. What would you
recommend to the physician do irst, before you see the patient in
your clinic?
A.
Refuse to prescribe any lorazepam to the patient
B.
Meet with the patient, review the reason for the falls, and
explain and document the concerns with lorazepam
prescription
C. Ask the physician to switch the patient to sertraline
D.
Ask the physician to switch the patient to gabapentin

E.
Refuse to see the patient back in clinic
2.
While rounding with the psychiatric consultation team at your local
academic medical center, you start discussing the risk of falls
among older adults who are treated with medications. The trainees
in the team want to know which among the following medication
classes was found to be associated with the highest risk for falls
among older adults?
A.
Diuretics
B.
Benzodiazepines
C.
Narcotics
D.
Antidepressants
E.
Neuroleptics and antipsychotics
3. You are giving a lecture to the Emergency Medicine (EM) residents
regarding the need to check urine drug screens on all older adults
who present to the emergency room with altered mental status as
older adults have higher rates of prescription drug misuse. One of
the EM residents asks whether any of the opioids cannot be
detected on a routine urine drug screen. Of the following, which
opioid would you say is not detectable on a routine urine drug
screen?
A.
Fentanyl
B.
Morphine
C.
Codeine
Codeine
D. Methadone

E.
Hydromorphone
4.
You are discussing the risk of developing dementia among users of
benzodiazepines with your internist colleague. What would you tell
your colleague about this risk?
A.
The odds ratio (OR) of developing dementia with ever use of
benzodiazepines is approximately 1.39
B.
The odds ratio (OR) of developing dementia with ever use of
benzodiazepines is approximately 0.75
C.
The odds ratio (OR) of developing dementia with ever use of
benzodiazepines is approximately 0.39
D.
The odds ratio (OR) of developing dementia with ever use of
benzodiazepines is approximately 2.30
E.
The odds ratio (OR) of developing dementia with ever use of
benzodiazepines is approximately 5.00

Answers
Answer 1 B This is a fairly common scenario in geriatric psychiatry –
an individual is referred for evaluation when there is concern regarding
the use of controlled medications when the individual is having
possible adverse effects. In this scenario, the best option available to
the primary care physician is to meet with the patient, review the
reason for the falls, and explain and document their concerns with
lorazepam prescription. Refusing to prescribe lorazepam or refusing to
see the patient may be construed as patient abandonment. Additionally,
the patient may have withdrawal symptoms without proper taper from
the lorazepam. Switching medications may be done after proper
education and discussion with the patient.

Answer 2 B An important meta-analysis that was completed on nine


unique drug classes found that benzodiazepines, odds ratio (OR) of
1.41, was found to be associated with the highest risk for falls among
older adults. This was followed by neuroleptics and antipsychotics (OR,
1.39) and antidepressants (OR, 1.36). The risk of falls with diuretics
(OR, 0.99) and narcotics (OR, 0.96) was lower that the psychotropic
medication classes.

Answer 3 A Urinalysis testing for opiates detects the metabolite of


heroin and codeine, namely, morphine. Fentanyl is usually not detected
on routine urine drug screens due to a lack of metabolites, and
oxycodone is not usually detected because it is derived from thebaine
(not detected in the urine).

Answer 4 A A meta-analysis that included data from 15 studies found


that ever use of benzodiazepines was associated with a signi icantly
increased risk of dementia [odds ratio (OR) 1.39, 95% con idence
interval (CI) 1.21–1.59]. Long-acting benzodiazepines were associated
with a marginally higher magnitude risk (OR 1.21, 95% CI 0.99–1.49)
than short-acting benzodiazepines (OR 1.13, 95% CI 1.02–1.26), but the
former failed to reach statistical signi icance (p = 0.059).

Further Reading
Aguiluz J, Alvarez M, Pimentel E, Abarca C, Moore P. How to face a patient with
benzodiazepine dependence in primary health care? Strategies for withdrawal.
Medwave. 2018;18(1):e7159.
[Crossref]

Moeller KE, Lee KC, Kissack JC. Urine drug screening: practical guide for clinicians.
Mayo Clin Proc. 2008;83(1):66–76.
[Crossref]

Page AT, Potter K, Clifford R, Etherton-Beer C. Deprescribing in older people.


Maturitas. 2016;91:115–34.
[Crossref]
Penninkilampi R, Eslick GD. A systematic review and meta-analysis of the risk of
dementia associated with benzodiazepine use, after controlling for protopathic bias.
CNS Drugs. 2018;32(6):485–97.
[Crossref]

Woolcott JC, Richardson KJ, Wiens MO, Patel B, Marin J, Khan KM, Marra CA. Meta-
analysis of the impact of 9 medication classes on falls in elderly persons. Arch Intern
Med. 2009;169(21):1952–60.
[Crossref]
© Springer Nature Switzerland AG 2021
R. Tampi et al. (eds.), Absolute Geriatric Psychiatry Review
https://doi.org/10.1007/978-3-030-58663-8_41

41. Medicolegal
Rajesh Tampi1 and Deena Tampi2
(1) Department of Medicine, Cleveland Clinic Lerner College of
Medicine of Case Western Reserve University, Cleveland, OH, USA
(2) Behavioral Health Advisory Group, Princeton, NJ, USA

Keywords Medical malpractice – Civil commitment – Legal standard of


proof – HIPAA Privacy Rule – Capacity

Questions
1.
Which one of the following in not part of the four Ds of medical
malpractice?
A.
Denial
B.
Dereliction
C.
Duty
D.
Direct
E.
Damages
2. Which of the following is not a criterion for the civil commitment of
an individual to a psychiatric hospital in most states in the United
States?
A.
Presence of a psychiatric illness
Presence of a psychiatric illness
B. Grave disability due to the psychiatric illness

C.
Imminent danger to self or to others due to the psychiatric
illness
D.
The need for treatment of psychiatric illness
E.
An individual’s refusal to take psychotropic medications
3.
Which one of the following is the legal standard of proof for
criminal cases in the United States?
A.
Preponderance of the evidence
B.
Beyond a reasonable doubt
C.
Clear and convincing evidence
D.
Some credible evidence
E.
Awareness of the crime
4. Which one of the following is the legal standard of proof for civil
suits in the United States?
A.
Preponderance of the evidence
B.
Beyond a reasonable doubt
C.
Clear and convincing evidence
D.
Some credible evidence
E.
Awareness of the problem
5.

Which one of the following is the highest legal standard of proof in


the United States?
A.
Preponderance of the evidence
B.
Beyond a reasonable doubt
C.
Clear and convincing evidence
D.
Clarity of consciousness
E.
Awareness of the crime or problem

6.
Which of the following is not required for an older adult, who lives
at a skilled nursing facility, to have in order to consent to sexual
activity?
A.
A legally appointed guardian
B.
Awareness of the relationship
C.
Capacity to avoid exploitation
D.
Awareness of potential risks of being in the relationship
E.
All of the above
7. Which of the following is an exception to the Health Insurance
Portability and Accountability Act of 1996 (HIPAA) Privacy Rule?
A.
Coordination or management of healthcare services
B.
Consultation between providers
p
C.
Referral of individuals for treatment
D.
Provision of information to a public health authority
E. All of the above

8.
An 89-year-old man living at a skilled nursing facility wants to vote
in the upcoming general elections. Based on the Doe Voting
Capacity Standard, which of the following is not required for the
individual to exercise his right to vote?
A.
Order from the local probate court on his competence to vote
B.
Understand the nature of voting
C.
Understand the effect of voting
D.
The ability to make a choice
E.
All of the above

Answers
Answer 1 A The four Ds of medical malpractice are Dereliction from
Duty that Directly causes Damages to the patient. Denial is not one of
the requirements for medical malpractice.

Answer 2 E The civil commitment criteria in most states in the


United States require that a person, by way of a psychiatric disorder,
poses an imminent danger to the safety of self or others or be gravely
disabled due to the psychiatric disorder, and in need of treatment in
order to be involuntarily admitted to a psychiatric hospital. Refusal of
medications is not a criterion for civil commitment.
Answer 3 B “Beyond a reasonable doubt,” which requires that the
reviewer of facts be convinced of their decision without any
reservations that would be expected of a reasonable person or group, is
the required legal standard of proof for criminal cases in the United
States.

Answer 4 A “Preponderance of the evidence,” where the reviewer of


facts is certain that their decision is more likely to be correct than
incorrect, is the legal standard of proof for civil suits in the United
States.

Answer 5 B “Beyond a reasonable doubt” is the highest legal


standard of proof in the United States.

Answer 6 A The capacity of an older individual to consent to sexual


activity usually involves the individual’s awareness of the relationship,
capacity to avoid exploitation, and the awareness of potential risks of
being in the relationship. A legally appointed guardian is not required
for an older individual to consent to sexual activity.

Answer 7 E Under the Privacy Rule, covered entities may disclose


without an individual’s authorization protected health information
(PHI) about the individual as necessary to treat the individual or to
treat another individual. Treatment includes the coordination or
management of healthcare and related services by one or more health
care providers, consultation between healthcare providers, and the
referral of individuals for treatment. The HIPAA Privacy Rule also
recognizes the legitimate need for public health and related authorities
who are responsible for ensuring public health and safety to have
access to protected health information that is necessary to carry out
their public health mission. The Privacy Rule permits covered entities
to disclose needed protected health information without individual
authorization.

Answer 8 A According to the 2001 Doe Voting Capacity Standard,


which was approved for determining voting competence among older
adults, any individual who has an understanding of the nature and
effect of voting and has the ability to make a choice can vote.
Further Reading
Civil Commitment and the Mental Health Care Continuum: Historical Trends and
Principles for Law and Practice. https://www.samhsa.gov/sites/default/ iles/civil-
commitment-continuum-of-care.pdf, Accessed November 28, 2019.

https://www.hhs.gov/sites/default/ iles/ocr/privacy/hipaa/understanding/special/
emergency/hipaa-privacy-emergency-situations.pdf. Accessed November 28, 2019.

Karlawish JH, Bonnie RJ, Appelbaum PS, et al. Addressing the ethical, legal, and social
issues raised by voting by persons with dementia. JAMA. 2004;292(11):1345–50.
[Crossref]

Lichtenberg PA, Strzepek DM. Assessments of institutionalized dementia patients'


competencies to participate in intimate relationships. Gerontologist.
1990;30(1):117–20.
[Crossref]

Rodgers C. Special topics in psychiatric malpractice. Am J Foren Psychiatry.


2009;30:5–16.

Testa M, West SG. Civil commitment in the United States. Psychiatry (Edgmont).
2010;7(10):30–40.
© Springer Nature Switzerland AG 2021
R. Tampi et al. (eds.), Absolute Geriatric Psychiatry Review
https://doi.org/10.1007/978-3-030-58663-8_42

42. Ethics
Nery A. Diaz1
(1) Columbia University Irving Medical Center, New York, NY, USA

Nery A. Diaz
Email: NAD2149@cumc.columbia.edu

Keywords Decision-making capacity – Autonomy – Bene icence –


Justice – Informed consent – Surrogate decision-maker – Psychiatric
advance directives

Questions
1. A 68-year-old woman presents to the emergency department (ED)
with nausea and vomiting. She reported that earlier in the day she
experienced an episode of bright red bloody stool. The
gastrointestinal (GI) service wants to admit the patient for further
workup and treatment, including a colonoscopy. The woman states
that she does not want to be admitted to the hospital and is
refusing a colonoscopy. The ED team requests a decision-making
capacity assessment, and the woman is found to lack decision-
making capacity. The assessing physician suggests contacting
family members to seek surrogate consent. What ethical principal
is the physician protecting by seeking a surrogate decision-maker
for the woman?
A.
Autonomy
B.
Bene icence
C. Nonmale icence

D.
Social justice
E.
Informed consent
2.
A 69-year-old man is unable to consent to a routine elective
surgery due to severe depression and associated apathy affecting
his decisional capacity. What is the next best step in the care of this
patient?
A.
Petition the hospital’s ethics committee to make a decision on
the patient’s behalf
B.
Advise the surgical service to proceed with the routine elective
surgery
C.
Advise the patient that he should proceed with the routine
elective surgery
D.
Discharge the patient home with appropriate follow-up
E.
Attempt to restore capacity by treating the depression
3. A 75-year-old woman is unable to consent to a routine elective
surgery due to severe manic symptoms that is affecting her
decisional capacity. Administering the appropriate treatment for
her manic symptoms and subsequently reassessing the woman’s
decisional capacity protects which of the following ethical
principles?
A.
Autonomy
B.
Bene icence
C.
Nonmale icence
D. Justice

E.
Nonalignment
4.
Which of the following ethical principles is the act of self-
determination?
A.
Autonomy
B.
Bene icence
C.
Nonmale icence
D.
Justice
E.
Nondiscrimination
5.
Which of the following ethical principles is the act of the
appropriate use of scarce health resources?
A.
Autonomy
B.
Bene icence
C.
Nonmale icence
D.
Justice
E.
Nondiscrimination
6. Which of the following statements is false regarding psychiatric
advance directives (PAD) in the United States?
A.
A PAD is a legal document that contains a person’s preferences
A PAD is a legal document that contains a person s preferences
for future health treatment
B. A PAD is a legal document that allows the appointment of a
health proxy

C.
A PAD is a legal document that allows a health proxy to
interpret preferences only during a crisis
D.
PADs can be used to plan for the possibility that someone may
lose capacity to give or withhold informed consent to
treatment during acute episodes of psychiatric illness
E.
All states permit PADs for healthcare, which can be used to
direct at least some form of psychiatric treatment
7. A 68-year-old man with schizophrenia, anxiety, cerebral palsy, and
an IQ of 51 presents to your private practice for treatment. He is
accompanied by his healthcare aide who has known the patient for
2 weeks. The man is divorced and has no children. His father is
alive and is involved with man’s care. The patient is requesting
medications for this anxiety symptoms as his psychiatrist of many
years has recently retired. After your assessment, it is determined
that the patient lacks the capacity to provide valid informed
consent. The patient does not have an advance directive. What is
the next best step in the management of this patient?
A.
A substituted decision-maker should be sought as soon as
possible
B.
Every effort should be made to identify the causes of the
impairment and to remedy them
C.
Initiate intensive efforts at education aimed at the patient’s
understanding of the relevant information
D.
Enlist the help of the healthcare aide in the consent process
E.
Contact the patient’s father to act as a surrogate decision
Contact the patient s father to act as a surrogate decision-
maker
8.
Which of the following ethical principles is the basis for the famous
Hippocratic aphorism?
A.
Autonomy
B.
Bene icence
C.
Nonmale icence
D.
Justice
E.
Nondiscrimination

Answers
Answer 1 A The process of seeking a surrogate decision-maker for an
individual who lacks capacity for decision-making is designed to help
protect the ethical principal of autonomy.

Answer 2 E Attempting to restore the capacity of the individual by


treating the depression is the next best step in the care of this patient as
this is a routine elective surgery and not a life-threatening condition
that needs emergency surgery.

Answer 3 C Repeating capacity assessment after the individual


receives appropriate treatment for her manic symptoms protects the
ethical principle of nonmale icence. Nonalignment is not an ethical
principle.

Answer 4 A Autonomy is the ethical principle de ined as the act of


self-determination.
Answer 5 D Justice is the ethical principle that is de ined as the act of
the appropriate use of scarce health resources.

Answer 6 E Psychiatric advance directives (PADs) are relatively new


legal instruments that may be used to document a competent person’s
speci ic instructions or preferences regarding future mental health
treatment, in preparation for the possibility they may lose capacity to
give or withhold informed consent during acute episodes of psychiatric
illness. Almost all states permit advance directives for healthcare,
which can be used to direct at least some forms of psychiatric
treatment. In the past decade, 25 states have adopted speci ic
psychiatric advance directives statutes.

Answer 7 E The next best step in the management of this man’s care
is to contact a family member to act as surrogate decision-maker.

Answer 8 C The ethical principle of nonmale icence refers to the


requirement to avoid harming patients and forms the basis for the
famous Hippocratic aphorism – “First do no harm!”

Further Reading
ABIM Foundation. American Board of Internal Medicine. Medical professionalism in
the new millennium: a physician charter. Ann Intern Med. 2002;136(3):243.
[Crossref]

Appelbaum PS. Clinical practice. Assessment of patients’ competence to consent to


treatment. N Engl J Med. 2007;357(18):1834–40.
[Crossref]

Gupta A, Balasubramaniam M. Policy, ethical, and legal issues. Psychiatric disorders


late in life. New York: Springer; 2018. p. 347–72.
[Crossref]

McDevitt D. Psychiatric advance directives: navigating the regulatory landscape.


Nursing. 2019;49(12):14–7.
[PubMed]

Murray H, Wortzel HS. Psychiatric advance directives: origins, bene its, challenges,
and future directions. J Psychiatr Pract. 2019;25(4):303–7.
[Crossref]

Tampi RR, Young J, Balachandran S, et al. Ethical, legal and forensic issues in geriatric
psychiatry. Curr Psychiatry Rep. 2018;20(1):1.
[Crossref]

Taylor RM. Ethical principles and concepts in medicine. Handb Clin Neurol.
2013;118:1–9.
[Crossref]

Walaszek A. Clinical ethics issues in geriatric psychiatry. Psychiatr Clin North Am.
2009;32(2):343–59.
[Crossref]
© Springer Nature Switzerland AG 2021
R. Tampi et al. (eds.), Absolute Geriatric Psychiatry Review
https://doi.org/10.1007/978-3-030-58663-8_43

43. Treatment-Setting Regulations


Arushi Kapoor1
(1) Department of Psychiatry, Perelman School of Medicine at the
University of Pennsylvania, Philadelphia, PA, USA

Keywords Automobile driving – Telepsychiatry – Elder care abuse –


Restraints and seclusions

Questions
1. A 69-year-old retired lawyer whom you previously treated for mild
depression has just had a small stroke leading to mild right-sided
weakness which is mostly resolved by now. You notice that his
mobility is reduced and are concerned about the issue of safe
driving, given the recent stroke. Based on the results of the
Assessment of Driving-Related Skills (ADReS), the patient has
signi icant de icits that are unlikely to respond to medication
adjustments. In your judgment it is no longer safe for the patient to
drive a car. Which one of the following steps should come next in
the care of this patient with regard to his driving?
A.
Refer him to a driver rehabilitation specialist (DRS) who will
directly assess the patient’s ability to drive a car and provide
recommendations
B.
Since you are not liable for his driving safety, recommend he
may choose to stop driving and follow-up in 1 year
C.
Explain your indings to the patient and ask him to follow up
with the Department of Motor Vehicles (DMV)
D. Send the patient to the emergency room until his family arrives
and lock away his keys in a secure place
E.
Report the patient to the Department of Motor Vehicles (DMV)

2.
Which of the following statements is accurate regarding statutes
and reporting laws for elder abuse in all states in the United States?
A.
Reporting is voluntary in most states, and there are state-to-
state variations about who is a mandated reporter of elder
abuse cases
B.
Many of the elder abuse statutes clearly de ine what elder
abuse is, and this is a universal de inition
C.
They are modeled after child abuse laws; therefore,
practitioners can use their state’s child abuse statutes to guide
them in elder abuse cases
D.
Reporting is mandatory in most states, and Adult Protective
Services (APS) has been designated in all states to investigate
all cases of elder abuse
E.
Mandatory reporting laws require certain groups to tell
designated authorities about con irmed cases of elder abuse
3. A 69-year-old white woman comes to the clinic with her adult son.
The son is constantly hovering over his mother and looking
anxious. The patient appears unkempt, tired, and frail. You ask your
intern to see the patient alone, but her son initially refuses to leave
his mother’s side. He then reluctantly agrees to wait outside the
examination room. The intern proceeds to ask the woman some
questions to assess whether she has been abused. Which of the
following statements would you tell the intern is false regarding
ethical issues of autonomy, informed consent, and con identiality?
A. Autonomy is the ability to control one’s own life based on
independence, while self-determination is characterized by the
ability to make informed decisions and plans to ful ill personal
goals
B.
An older adult’s level of autonomy is not static as it may change
due to altering medical conditions and level of functioning
C.
Self-determination and an individual’s mental capacity are
components of an informed consent
D.
If an elder abuse victim does not take action against the
perpetrator to end the abusive relationship, it is assumed the
victim lacks decision-making capacity
E.
The practitioner’s duty to respect a patient’s privacy of
information is con identiality
4. A 65-year-old woman is brought to the emergency room by the
emergency medical services (EMS) from a group home. Upon
examination she is verbally aggressive toward staff and throwing
items on the loor and attempting to put her hand into the “Sharp
Objects” container in the examination room. EMS staff state that
the woman was secluded in her room for the last couple days. Due
to limited staff personnel, the group home staff are unable to
provide a detailed history. Which of the following interventions is
LEAST indicated at this time for the patient’s care?
A.
Monitor the patient in an examination room and have a 1:1
sitter placed for safety
B.
Isolate and place patient in a seclusion room
C.
Isolate and place patient in four-point restraint for safety of
staff and patient
D.
Obtain collateral information including allergies and medical
history from previous admissions
E. Consider bloodwork and offer voluntary oral medication to
address agitation
g

5.
Which of the following statements is false regarding telepsychiatry?
A.
Telepsychiatry improves access to care, reduces wait times for
appointments, and reduces travel time and costs
B.
Concerns about establishing rapport, privacy, safety, and
technology limitations have slowed acceptance of
telepsychiatry
C.
Clinicians are concerned about reimbursement and regulatory
issues due to limited education and learning surrounding
telepsychiatry
D.
Patients generally report increasing comfort and satisfaction
once they have used telepsychiatry after initial apprehension
and discomfort
E.
There is an enhanced therapeutic rapport established during
telepsychiatry visits when compared to in-person visits

Answers
Answer 1A Referral to a driver rehabilitation specialist (DRS) is the
irst step in this patient care with regard to his driving, as the of ice
assessment is not the same as a real-world test of the patient’s driving
ability. The DRS will conduct an assessment to move the process to the
next step and provide an intermediary buffer for helping you preserve
your treatment alliance with the patient. If the patient is unwilling or
unable to afford the assessment, you may then need to report directly
to the DMV, which may then conduct a driving assessment. Relying on
patients to contact the DMV themselves is unrealistic, as it often very
dif icult for elderly persons to give up driving. Taking away the patient’s
car keys may be disruptive to the treatment alliance, unnecessary, and
also poor risk management, as it may involve issues of assault and
battery, wrongful taking, or invasion of privacy, exposing the physician
to legal liabilities of one sort or another.

Answer 2A All 50 states have enacted legislation dealing with elder


abuse, and they share many features with child abuse statutes. Elder
abuse laws are also called Adult Protective Services (APS) laws because
many states combine the protection of elders and disabled adults under
one law. They provide legal de initions of elder abuse, establish
administrative channels for the investigation of and intervention in
elder abuse, de ine who is mandated to report, and designate penalties
for violations. However, the speci ics of state laws vary and elder abuse
laws may be ambiguous. Individual states are responsible for their own
interpretations of the laws. Mandatory reporting laws (in all states
except New York) require certain groups to tell designated authorities
about reasonable suspicions of elder abuse. Elder abuse laws differ
from state to state, and de initions, procedures, and training can
likewise vary. Unlike child abuse statutes that mandate professionals in
all states to report incidences of child abuse, there is less consistency
among the states regarding mandatory reporting for elder abuse. For
example, practitioners may be mandated to report only in cases in
which the elder is residing in institutional settings. The de inition of
elderly may also differ. It is important to remember that unlike the child
abuse, older adults can refuse services or accept only partial services,
even if APS substantiates an incident of abuse. Those people who are
designated as mandated reporters vary from state to state. All states
except New York, the District of Columbia, and Puerto Rico require
healthcare professionals to report, and failure to do so can result in
penalties. As with child abuse statutes, the practitioner does not have to
prove that the abuse occurs before reporting; the practitioner must
report even if he/she only suspects abuse. Only 75% of the state laws
on elder abuse include a criminal penalty for failure to report. To locate
the number to call for speci ics about your state’s elder abuse laws or to
report elder abuse, visit the National Center on Elder Abuse (NCEA’s)
website at https://ncea.acl.gov/Resources/State.aspx.

Answer 3D Both autonomy and self-determination are important


aspects of care and should be protected. The state may intercede when
an individual cannot protect him-/herself from harm. Referred to as
parens patriae , the state’s intervention allows agencies such as APS to
provide voluntary and involuntary services for at-risk elders. The
issues of self-determination and an individual’s mental capacity are
components of assuring informed consent. Although protecting a
patient’s self-determination is important, there will be times when a
person cannot give informed consent. Effective collaboration involving
sharing of information across various agencies to coordinate services to
protect the older adult is vital, as is the protection of the individual’s
right to con identiality. The promotion of con identiality should not
breed inaction. It is unethical to assume that all older adults do or do
not have the capacity to make decisions. If an elder abuse victim does
not take action against the perpetrator to end the abusive relationship,
this does not indicate a lack of capacity. It is often assumed that with
old age comes diminished intellectual ability, which is an ageist
attitude. Normal aspects of aging, such as hearing loss, may make
communication dif icult, which may in turn be misinterpreted as
diminished capacity. Therefore, each patient’s physical limitations
should be taken into account and communication modi ied as
necessary.

Answer 4C A physical restraint is a direct application of physical


force with or without the patient’s permission to restrict freedom of
movement. The physical force may be human, mechanical devices, or a
combination thereof. Seclusion is an involuntary con inement of a
person alone in a locked room. The behavioral healthcare reasons for
the use of restraint and seclusion are primarily to protect the patient
against injury to self or others because of an emotional or behavioral
disorder. If and when prior interventions including redirection, verbal
intervention, behavioral, and environmental interventions, i.e., less
restrictive interventions, have been ineffective, it could be necessary to
use the most coercive approach as a inal response to a person whose
behavior is not otherwise manageable. Behavioral and environmental
interventions include avoiding confrontation; using neutral facial
expressions and voice tone, calm environments without intense
stimulations or triggers, and open room allowing for exit if needed; and
avoiding turning your back to the patient.
Answer 5E The most frequent barrier for telepsychiatry was the view
that telepsychiatry was less personal and that it was more dif icult to
establish rapport between the patient and the provider. Both users and
particularly nonusers of telemedicine reported disliking the loss of
personal patient contact. Decreased ability to detect nonverbal cues
during videoconferencing may limit rapport building with clinicians
noting dif iculty picking up nuances and emotions. For patients,
telepsychiatry improves access to care, reduces wait times for
appointments, and reduces travel time and costs. Patients generally
report increasing comfort and satisfaction once they have used
telepsychiatry after initial apprehension. Clinicians, who often come to
the profession because they desire contact with patients, may share
these concerns. Like patients, clinicians also report improved attitudes
toward telepsychiatry after trying it, suggesting that increased
exposure for clinicians may be important to alleviate their concerns
about rapport.

Further Reading
Betz ME, Scott K, Jones J, Diguiseppi C. “Are you still driving?” Metasynthesis of
patient preferences for communication with health care providers. Traf ic Inj Prev.
2016;17(4):367–73.
[Crossref]

Betz ME, Villavicencio L, Kandasamy D, et al. Physician and family discussions about
driving safety: indings from the LongROAD study. J Am Board Fam Med.
2019;32(4):607–13.
[Crossref]

Cowan KE, McKean AJ, Gentry MT, et al. Barriers to use of Telepsychiatry: clinicians
as gatekeepers. Mayo Clin Proc. 2019;94(12):2510–23.
[Crossref]

Daly JM, Merchant ML, Jogerst GJ. Elder abuse research: a systematic review. J Elder
Abuse Negl. 2011;23(4):348–65.
[Crossref]

Hallett N, Dickens GL. De-escalation of aggressive behaviour in healthcare settings:


concept analysis. Int J Nurs Stud. 2017;75:10–20.
[Crossref]
Hilty DM, Ferrer DC, Parish MB, et al. The effectiveness of telemental health: a 2013
review. Telemed J E Health. 2013;19(6):444–54.
[Crossref]

Hoover RM, Polson M. Detecting elder abuse and neglect: assessment and
intervention. Am Fam Physician. 2014;89(6):453–60.
[PubMed]

Kontio R, Vä limä ki M, Putkonen H, Kuosmanen L, Scott A, Joffe G. Patient restrictions:


are there ethical alternatives to seclusion and restraint? Nurs Ethics. 2010;17(1):65–
76.
[Crossref]

Quinn K. Cultural and ethical considerations in late-life polyvictimization. J Elder


Abuse Negl. 2017;29(5):327–38.
[Crossref]

Rabinowitz T, Murphy KM, Amour JL, et al. Bene its of a telepsychiatry consultation
service for rural nursing home residents. Telemed J E Health. 2010;16(1):34–40.
[Crossref]

Ramos-Rı́os R, Mateos R, Lojo D, Conn DK, Patterson T. Telepsychogeriatrics: a new


horizon in the care of mental health problems in the elderly. Int Psychogeriatr.
2012;24(11):1708–24.
[Crossref]

Rees J, King L, Schmitz K. Nurses’ perceptions of ethical issues in the care of older
people. Nurs Ethics. 2009;16(4):436–52.
[Crossref]

Stiegel L. Recommendations for the elder abuse, health, and justice ields about
medical forensic issues related to elder abuse and neglect. J Elder Abuse Negl.
2006;18(4):41–81.
[Crossref]

US Preventive Services Task Force, Curry SJ, Krist AH, et al. Screening for intimate
partner violence, elder abuse, and abuse of vulnerable adults: US Preventive Services
Task Force inal recommendation statement. JAMA. 2018;320(16):1678–87.
[Crossref]

Voskes Y, Kemper M, Landeweer EG, Widdershoven GA. Preventing seclusion in


psychiatry: a care ethics perspective on the irst ive minutes at admission. Nurs
Ethics. 2014;21(7):766–73.
[Crossref]
Part VII
Medical and Neurologic Aspects of Geriatric
Psychiatry
In this part, we will discuss medical and neurological aspects of
geriatric psychiatry, including geriatric medical syndromes, psychiatric
disorders due to a general medical condition, psychiatric complications
of medical treatment and psychological factors affecting medical
conditions.
Geriatric medical syndromes · Psychiatric complications ·
Psychological factors · Medical complications
Paroma Mitra
Ankit Jain
© Springer Nature Switzerland AG 2021
R. Tampi et al. (eds.), Absolute Geriatric Psychiatry Review
https://doi.org/10.1007/978-3-030-58663-8_44

44. Geriatric Medical Syndromes


Paroma Mitra1
(1) Department of Psychiatry, New York University Grossman School
of Medicine, New York, NY, USA

Keywords Falls – Pain – Frailty – Urinary incontinence – Fecal


incontinence and constipation

Questions
1.
What is the fraction of the older adult population (≥ 65 years) that
is likely to experience a fall in their life?
A.
1/2
B.
1/3
C.
1/5
D.
1/10
E.
1/15
2. Which of the following is the most common type of pain reported
among older adults?
A.
Lower back or neck pain
B.
Musculoskeletal pain
Musculoskeletal pain
C. Peripheral neuropathic pain

D.
Chronic joint pain
E.
Dental pain
3.
The American Geriatrics Society (AGS) recommends which of the
following regarding the pharmacological management of persistent
pain among older adults?
A.
Opioids are absolutely contraindicated among older adults due
to their serious adverse effects
B.
Begin using pain medications at higher doses to obtain bene it
and then taper appropriately
C.
Acetaminophen is recommended as a irst-line therapy for pain
among older adults
D.
The effect of pain medications is enhanced when used in
combination with other pain analgesics and nondrug strategies
E.
Choices C and D
4. Which of the following is not considered a part of the most
commonly accepted phenotype model de inition of frailty?
A.
Unintentional weight loss
B.
Self-reported chronic pain
C.
Self-reported exhaustion
D.
Poor grip strength
E.
Sl lki d
Slow walking speed
5.
Which of the following is part of prompted voiding to prevent
urinary incontinence among older adults?
A.
Letting nursing home residents determine when they use the
restroom
B.
If assistance is requested, the individual is encouraged to use
the toilet independently
C.
Residents are prompted up to seven times to request for
assistance to use the restroom
D.
Residents in nursing homes are approached every 2 hours and
asked if they are “wet” or “dry”
E.
Discouraging the use of assistance entirely so that residents
become aware of the need to void

6.
Which of the following is the most common correlate to fecal
incontinence (FI) among older adults living in care facilities?
A.
Cognitive impairment/dementia
B.
Ethnicity
C.
Age
D.
Constipation and diarrhea
E.
Stroke
7. Which of the following is not part of the Rome IV criteria for the
diagnosis of chronic constipation, a condition commonly seen
among older adults?
A
A.
Straining at defecation
B. Lumpy or hard stools

C.
Sensation of incomplete evacuation
D.
Manual maneuvers to facilitate defecations
E.
Fewer than two spontaneous bowel movements per week

Answers
Answer 1B Falls are reported in one-third of the older adult
population in their life and is the leading cause of physical trauma in
this age group.

Answer 2A The most common types of pain among older adults are
lower back or neck pain (65%), musculoskeletal pain (40%), peripheral
neuropathic pain (40%), and chronic joint pain (20%).

Answer 3E Among properly selected and monitored individuals,


opioid analgesics are a potentially effective and sometimes an
indispensable part of the multimodal strategy in the management of
various types of persistent cancer and noncancer pain. Pain treatment
with medications should begin with the lowest possible dose and
increase slowly based on response and side effects. Acetaminophen is
an effective agent for the management of symptoms of osteoarthritis
and lower back pain, and it is not associated with signi icant
gastrointestinal bleeding, adverse renal effects, or cardiovascular
toxicity. Due to its greater safety when compared to traditional
nonsteroidal anti-in lammatory drugs (NSAIDs), acetaminophen is
recommended as the irst-line therapy for pain. The effect of pain
medications is enhanced when used in combination with other pain
analgesics and nondrug strategies.
Answer 4B The phenotype model de inition of frailty includes
unintentional weight loss, self-reported exhaustion, poor grip strength,
slow walking speed, and low physical activity. Individuals are usually
considered as being frail if they meet at least three of the ive criteria
and as pre-frail if they meet one or two of these criteria.

Answer 5D Prompted voiding is used to teach people with or without


cognitive impairment to initiate their own toileting through requests
for help and positive reinforcement from caregivers when they do this.
This is distinctive from other therapies such as habit training, timed
toileting, and bladder training because of the participation of the
individual in the process. Habit training attempts to determine the
urination pattern for an individual which is then used to achieve
continence. Timed toileting is ixed by time or event which is caregiver
led and is not individualized. Bladder training actively includes the
individual in attempting to increase the interval between the desire to
void and the actually voiding. Prompted voiding requires the education
of both the person with incontinence and the staff using a program of
scheduled voids that requires prompting from the caregiver.

Answer 6A Cognitive impairment/dementia is the most consistently


reported correlate of FI among care home residents. Care home
residents with dementia experience increased FI, and over time they
experience the highest increases in care dependency when compared
with the residents without dementia.

Answer 7E The Rome IV criteria for chronic constipation include the


following: symptom onset should be at least 6 months before the
diagnosis, symptoms should be present during the last 3 months, and
more than one-fourth of defecation must include two or more of the
following: straining, lumpy or hard stools, sensation of incomplete
evacuation, sensation of anorectal obstruction/blockage, manual
maneuvers to facilitate defecations, and fewer than three spontaneous
bowel movements per week.

Further Reading
American Geriatrics Society Panel on Pharmacological Management of Persistent
Pain in Older Persons. Pharmacological management of persistent pain in older
persons. J Am Geriatr Soc. 2009;57(8):1331–46.
[Crossref]

Bergen G, Stevens MR, Burns ER. Falls and fall injuries among adults aged ≥65 years –
United States, 2014. MMWR Morb Mortal Wkly Rep. 2016;65(37):993–8.
[Crossref]

Eustice S, Roe B, Patterson J. Prompted voiding for the management of urinary


incontinence in adults. Cochrane Database Syst Rev. 2000;2:CD002113.

Jones MR, Ehrhardt KP, Ripoll JG, et al. Pain in the elderly. Curr Pain Headache Rep.
2016;20(4):23.
[Crossref]

Musa MK, Saga S, Blekken LE, et al. The prevalence, incidence, and correlates of fecal
incontinence among older people residing in care homes: a systematic review. J Am
Med Dir Assoc. 2019;20(8):956–962.e8.
[Crossref]

Rohrmann S. Epidemiology of frailty in older people. Adv Exp Med Biol.


2020;1216:21–7.
[Crossref]

Zheng S, Yao J. For the Chinese Geriatric Society, Editorial Board of Chinese Journal of
Geriatrics. Expert consensus on the assessment and treatment of chronic
constipation in the elderly. Aging Med. 2018;1:8–17.
[Crossref]
© Springer Nature Switzerland AG 2021
R. Tampi et al. (eds.), Absolute Geriatric Psychiatry Review
https://doi.org/10.1007/978-3-030-58663-8_45

45. Care of Patients with Neurologic


Disease
Ankit Jain1
(1) Penn State Health Milton S. Hershey Medical Center, Hershey, PA,
USA

Keywords Seizures – Epilepsy – Cerebrovascular diseases –


Pheochromocytoma – Headache – Tremors

Questions
1.
Mortality among older adults with epilepsy is how many times
greater than the mortality in general population?
A.
2 to 3 times
B.
5 to 10 times
C.
10 to 20 times
D.
20 to 30 times
E.
35 to 45 times
2. What is the most common cause of new-onset epilepsy among
adults over the age of 65 years?
A.
Stroke and other cerebrovascular diseases
B. Primary neurodegenerative disorders

C.
Head trauma
D.
Brain tumors
E.
Unknown
3.
Which of the following comorbid psychiatric disorders is most
common among individuals with pheochromocytoma?
A.
Major depressive disorder
B.
Delusional disorder
C.
Schizoaffective disorder
D.
Anxiety disorder
E.
Conversion disorder
4. A 66-year-old woman presents to you with complaints of shaking
in both of her hands when she picks up objects. These shakes have
been gradually worsening over the last 10 years. Her son tells you
that she spills coffee and food frequently. The woman’s father had a
similar condition. She has a history of hypothyroidism which is
currently well controlled using levothyroxine. On physical
examination you observe bilateral postural tremor in her hands,
feet, and head. Her tremor does not alleviate with distraction. The
remainder of neurological examination is unremarkable. Which of
the following is true of this condition?
A.
This condition has an X-linked recessive mode of inheritance
B.
This condition is associated with Parkinson’s disease
This condition is associated with Parkinson s disease

C. Alcohol consumption suppresses this type of tremor

D.
Dopamine agonists are drugs of choice for this condition
E.
Primidone will worsen this tremor
5.
A 72-year-old man with a history of depression and migraine
headaches presents to your of ice with complaint of unilateral
headaches that are exacerbated by touch. He notes that these
recent episodes are different from other headache episodes that he
experiences usually. He also reports discomfort while chewing
foods, which has led to anorexia, weight loss, and fatigue. What
would be the next best step in the management of this patient’s
condition?
A.
Give sumatriptan orally immediately for pain relief
B.
Obtain a magnetic resonance imaging (MRI) brain scan to look
for mass lesions
C.
Obtain a retinal artery biopsy and lumbar puncture
D.
Give thrombolytic therapy for embolic stroke
E.
Give high-dose steroids

Answers
Answer 1A The mortality rates among older adults with epilepsy is
about two to three times higher when compared to the general
population.
Answer 2A Stroke and other cerebrovascular diseases are the most
common risk factors for the development of new-onset epilepsy among
older adults and account for 30% to 50% of all identi ied causes.
Epilepsy can develop at the time of or after a stroke or can even be an
early clinical manifestation of cerebrovascular diseases. Studies have
reported that the risk of developing epilepsy in the irst year after a
stroke increases by 20 times. Less common risk factors for epilepsy
among older adults include primary neurodegenerative disorders
(account for ~10–20%), head trauma (accounts for 10–20%), and brain
tumors (account for nearly 10–30%). About one-third of new-onset
epilepsies in the older adult population have undetected causes.

Answer 3D Among individuals with pheochromocytoma , anxiety is


the fourth most common symptom after the classic triad of headache,
sweating, and palpitations. It is estimated to occur in about 30% of
patients with pheochromocytoma. It could be hypothesized that the
very high levels of catecholamines may contribute to increased anxiety,
although this correlation has not been clearly established.

Answer 4C This patient’s symptoms are most consistent with


essential tremor. Among older adults with tremor, the differential
diagnosis is usually between essential tremor (ET) and Parkinson’s
disease (PD). The hallmark of essential tremor is that it is commonly
suppressed with the use of alcohol and benzodiazepine intake.
Essential tremors typically have an autosomal-dominant inheritance
pattern. The American Academy of Neurology gave A-level
recommendations to primidone, propranolol, and propranolol LA. B-
level recommendations were given to alprazolam, atenolol, gabapentin,
sotalol and topiramate. C-level recommendations were given for
clonazepam, clozapine, nadolol, nimodipine, and botulinum toxin for
tremor of the upper limb, head, and voice and DBS of the thalamus and
thalamotomy. It is also important to know that selective serotonin
reuptake inhibitors, lithium, and valproate can induce a postural
tremor that may present similarly to essential tremor, so it is important
to make note of the timeline of the development of symptoms and any
associated medication changes.
Answer 5E This is a case of temporal arteritis, also known as giant
cell arteritis. This condition is typically seen among individuals above
50 years of age. Individuals with this condition often present with
intermittent jaw claudication, unilateral headaches, and in severe cases
loss of vision. Some other symptoms commonly seen with temporal
arteritis include anorexia, weight loss, and fatigue. Erythrocyte
sediment rate (ESR) is a good indicator of in lammation and is typically
elevated in active cases of temporal arteritis. Increased C-reactive
protein is also common. De initive diagnosis requires temporal artery
biopsy which reveals skip lesions. Temporal arteritis is treated with
high-dose steroids, which should be given even prior to clinical workup
if the diagnosis is suspected, to prevent visual loss or stroke.

Further Reading
Alguire C, Chbat J, Forest I, et al. Unusual presentation of pheochromocytoma:
thirteen years of anxiety requiring psychiatric treatment. Endocrinol Diabetes Metab
Case Rep. 2018;2018:17–0176.
[PubMedCentral]

Brodie MJ, Kwan P. Epilepsy in elderly people. BMJ. 2005;331(7528):1317–22.


[Crossref]

Liu S, Yu W, Lü Y. The causes of new-onset epilepsy and seizures in the elderly.
Neuropsychiatr Dis Treat. 2016;12:1425–34.
[Crossref]

Morgan JC, Kurek JA, Davis JL, Sethi KD. Insights into pathophysiology from
medication-induced tremor. Tremor Other Hyperkinet Mov (N Y). 2017;7:442.
[Crossref]

Reich SG. Does this patient have Parkinson disease or essential tremor? Clin Geriatr
Med. 2020;36(1):25–34.
[Crossref]

Serling-Boyd N, Stone JH. Recent advances in the diagnosis and management of giant
cell arteritis. Curr Opin Rheumatol. 2020;32(3):201–7.
[Crossref]
© Springer Nature Switzerland AG 2021
R. Tampi et al. (eds.), Absolute Geriatric Psychiatry Review
https://doi.org/10.1007/978-3-030-58663-8_46

46. Psychiatric Disorders Due to


General Medical Condition
Paroma Mitra1
(1) Department of Psychiatry, New York University Grossman School
of Medicine, New York, NY, USA

Keywords Thyroid disease – Human immunode iciency virus – HIV-


associated mild neurocognitive disorder – HIV-associated dementia –
Multiple sclerosis – Central nervous system tumors

Questions
1.
Which of the following is considered a distinguishing for
hyperthyroidism among older adults when compared to younger
adults?
A.
Memory changes
B.
Emotional lability
C.
Apathy
D.
Psychosis
E.
Agitation
2. A 68-year-old man with no previous psychiatric history and with a
past medical history of human immunode iciency virus (HIV)
infection was seen in the clinic with new-onset attentional issues.
He later revealed that he was struggling to balance his check book
and frequently losing his keys. He endorsed compliance with his
HIV medication. When a detailed cognitive assessment was
completed, he showed de icits in attention testing and recall tasks
which were one standard deviation below normal. He was given a
diagnosis of HIV-associated neurocognitive disorder. Based on the
patient’s clinical presentation, which stage of HIV-associated
neurocognitive disorder is he presenting with at this time?
A.
Asymptomatic neurocognitive impairment
B.
Symptomatic neurocognitive impairment
C.
Mild neurocognitive disorder
D.
HIV-associated leukopathy
E.
HIV-associated encephalopathy
3.
Which of the following is not a known risk factor for the
development of cognitive decline among individuals with multiple
sclerosis (MS)?
A.
Female gender
B.
Gray matter atrophy
C.
Early-age-onset of symptoms
D.
Secondary progressive course
E.
Low baseline cognitive reserve
4. Which of the following would be considered a neuropsychiatric
i di id l i h HIV i d
symptom seen among individuals with HIV-associated

neurocognitive disorders?
A.
Psychomotor retardation
B.
Myoclonic jerks
C.
Hyperre lexia
D.
Neuropathy
E.
Tremors
5.
Which of the following central nervous system (CNS) cancers in
adults is directly associated with psychiatric indings among older
adults?
A.
Diffuse gliomas
B.
Chordomas
C.
Schwannomas
D.
Non-Hodgkin lymphomas
E.
Sarcomas

Answers
Answer 1C Apathy has been de ined as a lack of motivation or
emotion not attributed to impaired consciousness, cognitive
impairment, or mood disorder. Apathy in younger adults is usually seen
with hypothyroidism versus older adults where it is associated with
hyperthyroidism. In addition, hyperthyroidism in older adults is
associated with weight loss, anorexia, memory changes, and apathy.

Answer 2C The patient has de icits in both neuropsychological


testings (attention and memory recall) and notable de icits in
functioning. He meets the criteria for mild neurocognitive impairment
secondary to HIV disease. The working research criteria for HIV-related
neurocognitive complications recognize the following three conditions:
asymptomatic neurocognitive impairment (ANI), HIV-associated mild
neurocognitive disorder (MND), and HIV-associated dementia (HAD).
ANI includes impairment in at least two cognitive domains, at least one
SD below the mean in testing, and no impairment with daily
functioning. MND includes the same neuropsychological test results as
ANI and at least mild interference with functioning. The third category
is the HAD which includes impairment in at least two cognitive
domains and two standard deviations below testing.

Answer 3A Cognitive impairment is observed in approximately 65%


of individuals with MS. Cognitive impairment has been detected across
all stages of MS, tends to progress with time, and is most severe during
the secondary progressive phase. The predictors of cognitive decline
include male gender, gray matter atrophy, early age onset, secondary
progressive course, and low baseline cognitive reserve.

Answer 4A Psychomotor retardation is a neuropsychiatric symptom,


whereas the others are neurological and motor signs. Earliest
symptoms and signs in persons with neurocognitive disorder
secondary to HIV disease are impairment in concentration, memory,
and executive functioning. As the disease progresses, psychomotor
retardation, depressive symptoms, irritability, and at times subclinical
motor signs are observed. These motor signs include tremors,
hyperre lexia, and myoclonic jerks among others. Later clinical features
evolve to overall dementia, myelopathy, neuropathy, and even
Parkinson-like features.

Answer 5A CNS lymphomas and gliomas all point toward an


intracerebral origin or pathology and have been directly associated
with psychiatric symptoms. Diffuse gliomas have a median age of onset
of 63 years and often present with acute mood changes and fatigue as a
primary symptom. The other options all emerge outside the cerebral
cortex and may have psychiatric symptoms that can present later due to
metastasis or secondary to treatment modalities.

Further Reading
Antinori A, Arendt G, Becker JT, et al. Updated research nosology for HIV-associated
neurocognitive disorders. Neurology. 2007;69(18):1789–99.
[Crossref]

Bataille B, Delwail V, Menet E, et al. Primary intracerebral malignant lymphoma:


report of 248 cases. J Neurosurg. 2000;92(2):261–6.
[Crossref]

Feinstein A, DeLuca J, Baune BT, et al. Cognitive and neuropsychiatric disease


manifestations in MS. Mult Scler Relat Disord. 2013;2(1):4–12.
[Crossref]

Gá llego Pé rez-Larraya J, Delattre JY. Management of elderly patients with gliomas.
Oncologist. 2014;19(12):1258–67.
[Crossref]

McArthur JC, Steiner J, Sacktor N, Nath A. Human immunode iciency virus-


associated neurocognitive disorders: mind the gap. Ann Neurol. 2010;67(6):699–714.
https://doi.org/10.1002/ana.22053.
[Crossref][PubMed]

Stern RA, Robinson B, Thorner AR, et al. A survey study of neuropsychiatric


complaints in patients with Graves’ disease. J Neuropsychiatry Clin Neurosci.
1996;8(2):181–5.
[Crossref]
© Springer Nature Switzerland AG 2021
R. Tampi et al. (eds.), Absolute Geriatric Psychiatry Review
https://doi.org/10.1007/978-3-030-58663-8_47

47. Psychiatric Complications of


Medical Treatment
Paroma Mitra1
(1) Department of Psychiatry, New York University Grossman School
of Medicine, New York, NY, USA

Keywords Interferon – Glucocorticoids – Vinca alkaloids – Highly


active antiretroviral therapy (HAART) – Non-nucleoside reverse
transcriptase inhibitor (NNRTI) – Prednisone – Vincristine

Questions
1.
Which of the following mechanisms is a possible explanation for
interferon to cause depression when used to treat hepatitis C virus
(HCV) infection among older adults?
A.
Inhibition of the CYP1D2 enzyme system
B.
Induction of a key enzyme of tryptophan’s catabolism
C.
An autoinduction system
D.
Inhibition of the norepinephrine system
E.
Inhibition of serotoninergic receptors
2. Which of the following is the most commonly observed psychiatric
sequelae when glucocorticoids are taken by older adults?
A. Delirium

B.
Psychosis
C.
Depression
D.
Personality changes
E.
Anxiety
3.
An 80-year-old man with severe chronic obstructive pulmonary
disease (COPD) is started on prednisone for acute onset of severe
respiratory symptoms. Which of the following is the most
signi icant risk factor for the development of neuropsychiatric
manifestations among individuals treated with glucocorticoids?
A.
Age of the individual
B.
Gender of the individual
C.
Past psychiatric history of the individual
D.
The dosage of steroid used
E.
Choices A and C
4. A 67-year-old Caucasian man presents with fever, weight loss, and
painless lymph nodes in the neck and axilla. He is diagnosed with
non-Hodgkin’s lymphoma and is prescribed chemotherapy. He
discusses the treatment with his oncologist, and they decided to
start a vinca alkaloid given the good response rate among the older
adult population. These drugs are associated with mood disorders,
especially depression. Which of the following mechanisms is
associated with the development of depression with the use of this
drug?
drug?

A. Enhancement of serotonin reuptake

B.
Increases of dopaminergic activity
C.
Inhibition of the release of dopamine B-hydroxylase
D.
Decrease in dopamine transmission
E.
Enhancement of norepinephrine reuptake
5.
A 66-year-old man with HIV presents with feelings of anxiety and
panic attacks for the last 3 months. He has been referred to a
psychiatrist by his primary care provider (PCP) for evaluation. The
patient reveals that he is compliant with highly active antiretroviral
therapy (HAART) since starting it decades ago, but notes the dose
of his medication was recently increased. Which older non-
nucleoside reverse transcriptase inhibitor (NNRTI) has been
associated with neuropsychiatric side effects?
A.
Saquinavir
B.
Ritonavir
C.
Efavirenz
D.
Fortovase
E.
Interferon alpha

Answers
Answer 1B Approximately 30–70% of individuals treated with
interferon develop depression. Many mechanisms are suggested to
underlie interferon-induced depression. One possible mechanism for
the development of interferon-induced depression is interferon’s
induction of a key enzyme of tryptophan catabolism, indoleamine 2,3-
dioxygenase (IDO).

Answer 2C The most commonly observed psychiatric sequelae when


glucocorticoids are taken by older adults are depression and mania
followed by psychosis and delirium.

Answer 3D Among the risk factors for the development


neuropsychiatric symptoms from glucocorticoid use, dosage appears to
be the most signi icant. Higher doses have more risk. It is also thought
that female gender may be a risk factor for the development of
neuropsychiatric complications from steroids. There is no clear
evidence that a history of psychiatric disorders and age of the
individual are risk factors for the development of glucocorticoid-
associated neuropsychiatric symptoms.

Answer 4C Vinca alkaloids such as vincristine inhibit the release of


dopamine beta-hydroxylase, preventing the conversion of dopamine to
norepinephrine. This is associated with the development of depression
and anxiety.

Answer 5C Efavirenz is an old NNRTI that causes neuropsychiatric


side effects such as depression and anxiety symptoms. Studies show
that this effect is dose-related, and polymorphism of the CYP2D6 gene
has been associated with increased plasma concentrations of the drug.

Further Reading
Cerullo MA. Expect psychiatric side effects from corticosteroid use in the elderly.
Geriatrics. 2008;63(1):15–8.
[PubMed]

Dubovsky AN, Arvikar S, Stern TA, Axelrod L. The neuropsychiatric complications of


glucocorticoid use: steroid psychosis revisited. Psychosomatics. 2012;53(2):103–15.
[Crossref]

Muñ oz-Moreno JA, Fumaz CR, Ferrer MJ, et al. Neuropsychiatric symptoms
associated with efavirenz: prevalence, correlates, and management. A
neurobehavioral review. AIDS Rev. 2009;11(2):103–9.
[PubMed]

Pinto EF, Andrade C. Interferon-related depression: a primer on mechanisms,


treatment, and prevention of a common clinical problem. Curr Neuropharmacol.
2016;14(7):743–8.
[Crossref]

Yoffe G, Rice L, Alfrey CP Jr, Hattig RA. Depressive reaction to vincristine overdose.
Clin Lab Haematol. 1986;8(1):80–1.
[PubMed]
© Springer Nature Switzerland AG 2021
R. Tampi et al. (eds.), Absolute Geriatric Psychiatry Review
https://doi.org/10.1007/978-3-030-58663-8_48

48. Psychological Factors Affecting


Medical Conditions
Paroma Mitra1
(1) Department of Psychiatry, New York University Grossman School
of Medicine, New York, NY, USA

Keywords Psychological factors – Stress – Stigma – Shame –


Nonadherence – Mental stress-induced myocardial ischemia

Questions
1.
Which of the following is considered a direct psychological cause of
nonadherence to psychotropic medications among older adults?
A.
Cost of medications
B.
Complex treatment regime
C.
Adverse effects
D.
Stigma and shame of illness
E.
Inadequate education of mental health needs
2. Psychological factors can have an impact on the physiological
functions of various organ systems, an effect which can be even
more pronounced in the elderly. Which of the following is an
example of this phenomenon?
A. Smoking has been associated with lung cancer

B.
Stress has been experimentally shown to cause myocardial
ischemia
C.
Alcohol consumption has been associated with sleep
disruption
D.
Neurocognitive disorder has been associated with higher risk
of delirium
E.
Traumatic brain injury has been associated with cognitive
disorder
3.
Which of the following would be considered a psychological factor
causing a medical condition?
A.
A 68-year-old man with marijuana use disorder experiencing
chest pain after shouting at an assistant
B.
A 63-year-old woman with Lewy body dementia complaining
of increased stiffness when walking
C.
A 70-year-old man with alcohol use disorder complaining of
insomnia
D.
A 78-year-old woman with Parkinson’s disease falling when
making sharp turns
E.
A 65-year-old woman, a nursing home resident, with major
neurocognitive disorder with dif iculty swallowing

Answers
Answer 1D Psychological factors affecting other medical conditions
(PFAOMC) is a disorder that is diagnosed when a general medical
condition is adversely affected by psychological or behavioral factors.
The factors may precipitate or exacerbate the medical condition,
interfere with treatment, or contribute to morbidity and mortality.
Choices A, B, C, and E are social and biological causes, while the answer
D is a psychological cause.

Answer 2B Stress is psychological, while the others are either


external causes or organic causes that have led to a medical condition.
With aging, the incidence of chronic cardiovascular conditions is
increased.

Answer 3A The other four answers are preexisting neurological or


psychiatric conditions that interfere with functioning and are directly
associated or causative of the result described. Marijuana use disorder
is not directly related to chest pain, so it is possible that psychological
stress led to the observed result.

Further Reading
Groves MS, Muskin PR. Psychological responses to illness. In: Levenson JL, editor. The
American Psychiatric Publishing textbook of psychosomatic medicine and
consultation-liaison psychiatry. 3rd ed. Washington, DC: American Psychiatric
Publishing; 2019. p. 53.

Krantz DS, Burg MM. Current perspective on mental stress-induced myocardial


ischemia. Psychosom Med. 2014;76(3):168–70.
[Crossref]

Martino G, Langher V, Cazzato V, Vicario CM. Editorial: psychological factors as


determinants of medical conditions. Front Psychol. 2019;10:2502.
[Crossref]

Sirri L, Fabbri S, Fava GA, Sonino N. New strategies in the assessment of


psychological factors affecting medical conditions. J Pers Assess. 2007;89(3):216–28.
[Crossref]

Zhang L, Bao Y, Wang X, et al. A meta-analysis on the prevalence, associated factors


and diagnostic methods of mental stress induced myocardial ischemia. J Transl Med.
2020;18(1):218.
[Crossref]
Index
A
Absorption
Abuse
Acamprosate
Accountable care organizations (ACOs)
Acetaminophen
Acetylcholinesterase inhibitor
Acute lithium toxicity, risk for
Adult Protective Services (APS) laws
Adverse effects of drugs
acute lithium toxicity
antipsychotic therapy
carbidopa
cognitive behavioral therapy
major depressive disorder and morbid obesity
mechanisms
medication class
osteoarthritis
posttraumatic stress disorder
schizophrenia
seizure disorder
Affective disorders
Affordable Care Act (ACA)
African American ethnicity
Aging
demographics of
sleep changes
Aging brain
thalamus in
Akathisia
Alcohol
Alcohol consumption
Alcohol use disorder
co-morbid with
lifetime symptoms
sign or symptom of
Alcohol withdrawal, treatment of
Alcohol withdrawal syndrome
Alcohol, Smoking, and Substance Involvement Screening Test (ASSIST)
Alkaline phosphatase (AKP)
Alprazolam
Alzheimer’s dementia
Alzheimer’s disease (AD)
adverse effect
diagnosis of
development of
diagnosis of
genetic risk factor for
mild dementia from
psychosis of
Amantadine
Amino acid neurotransmitter
Amitriptyline
Anatomy
Angiotensin-converting enzyme (ACE) inhibitors
Anticholinergic activity (AA)
Anticholinergic agents
Anticholinergics-benztropine
Antidepressants
bupropion
health effects of
ocular complications
plasma concentration
severe depression
shortest half-life
Antiepileptics
Antihypertensive
Antipsychotics
acute manic symptoms
adverse effects
Alzheimer’s dementia
CATIE-AD trial
delirium
dementia with Lewy bodies
depression
long-acting injectable formulations of
Parkinson’s disease psychosis
pharmacokinetics of
schizophrenia
schizophrenia and coronary artery disease
Anxiety
assessment and clinical characteristics of
cognitive symptoms of
irst-line pharmacotherapy for
prevalence of
prospective study of
treatment of
Anxiolytics
alcohol withdrawal
anxiety disorders, treatment of
benzodiazepine
generalized anxiety disorder
psychiatric care
PTSD
Apathy
Aphasia
APOE ε4 allele carriage
Appetite, increased
Aripiprazole
Assessment of Driving-Related Skills (ADReS)
Asymptomatic neurocognitive impairment (ANI)
Atenolol
Atorvastatin
Atrial ibrillation
Atrophy
Atropine
Atypical antidepressants
Atypical antipsychotics
Atypical parkinsonian disorders (APDs)
Autonomy
Autonomy versus bene icence
B
Basic Activities of Daily Living (BADLs)
Beers criteria
Behavioral and environmental interventions
Behavioral and psychological symptoms of dementia (BPSD)
Benign prostatic hypertrophy (BPH)
Benzodiazepines
Bereavement
Beyond a reasonable doubt
Binswanger’s disease
Biochemistry
Biperiden
Bipolar depression
Bipolar disorder
anti-suicidal
initial presentation
psychotherapy
side-effect of lithium
symptoms of
treatment of
type I and type II
Bipolar I disorder
Bipolar II disorder
Bipolar symptoms
Bitemporal hemianopsia
Brain Fag
Breast cancer
Broca’s aphasia
Bromocriptine
Bupropion
Buspirone
C
Caffeine
Carbamazepine
Carbidopa
Cardiac implantable devices
Cardiovascular events
Caregiver burden
Cataracts
Catatonia
Catecholamines, synthesis of
CATIE-AD trial
Caudate nucleus
Centers for Medicare and Medicaid Services (CMS)
Central nervous system (CNS)
Central nervous system lymphomas
Cerebellar lesion
Cerebrovascular accident (CVA)
Cerebrovascular diseases
Charles Bonnet syndrome (CBS)
Cholinergic agents
Cholinergic neurons
Cholinesterase inhibitors
Chronic back pain
Chronic constipation, diagnosis of
Chronic kidney disease (CKD)
Chronic obstructive pulmonary disease (COPD)
Chronic pain, treatment of
Chronic pain syndrome
Cigarette
Circadian rhythms
among older adults
loss of
Cirrhosis
Citalopram
Civil commitment criteria
Civil suits, legal standard, proof for
Clinical laboratory testing
Clonazepam
Clozapine
Codeine
Cognition, retirement on
Cognitive behavioral therapy (CBT)
Cognitive de icit
Cognitive enhancers
Cognitive functions
Cognitive impairment
Cognitive performance
Cognitive stimulation therapy
Community-based mental health outreach services
Comorbid psychiatric disorders
Complete Blood Count (CBC)
Complicated grief therapy
Conduction aphasia
Confusion Assessment Method (CAM)
Copper
Copper de iciency
Coronary artery disease
Cortical white matter
Corticobasal degeneration (CBD)
cognitive features of
diagnosis of
Cranial nerves
Creutzfeldt-Jakob disease (CJD)
Culture bound syndrome (CBS)
CYP2D6 gene
CYP2D6 inhibitors
CYP3A4 inhibition
CYP3A4 isoenzymes
Cytochrome (CYP) enzyme
Cytochrome (CYP)-450
Cytochrome P450 (CYP450) 3A4
Cytochrome P450 isoenzyme (CYP1A4)
D
D2 blockade
Dantrolene
Deep brain stimulation, indication for
Delirium
multicomponent interventions for
precipitating factor for
prevalent subtype of
risk factor for
Delusional disorder
Delusions
Dementia
development of
rates of
sundowning in
TBI and
Dementia with Lewy Bodies (DLB)
Dementia-related psychosis
Department of Motor Vehicles (DMV)
Depressed mood
Depression
assessment and management of
development of
higher rates of
in late-life
outcomes for
psychotherapy treatment of
signi icant symptoms of
treatment
Depressive episode
Depressive symptoms
Developmental and transitional events
Dextroamphetamine
Diabetes mellitus (DM)
Dialectical behavior therapy
Diazepam
Diffuse gliomas
Digoxin
Diphenhydramine
Divalproex
Dizziness
Doe Voting Capacity Standard
Donepezil
Dopamine
Dopamine agonist
Dopamine blockade of antipsychotic agents
Doxepin
Driver rehabilitation specialist (DRS)
Drug pharmacokinetics
absorption stage of
metabolism stage of
Drug use disorder, criteria for
Drug-drug interactions
Drug-induced TdP
Drug-metabolizing enzymes
DSM-5 criteria
Duloxetine
E
Early onset Alzheimer’s disease (EOAD)
Early onset schizophrenia (EOS)
Economics
Efavirenz
Electroconvulsive therapy (ECT)
anterograde amnesia and retrograde amnesia with
Emergency Medical Services (EMS)
Enterocytes
Epilepsy
Erectile dysfunction (ED)
Erikson, Erik
Erythrocyte sediment rate (ESR)
Escitalopram
Eszopiclone
Ethics
Ethnic minorities
Excitatory neurotransmitter
Executive dysfunction
Executive functioning
F
Falls
FAST scale
Fatigue
Fecal incontinence (FI)
Female sex
Fentanyl
Flibanserin
Fluctuating cognition
Fluoxetine
Fluvoxamine
Frailty
phenotype model de inition of
Frontotemporal dementia (FTD)
Functional assessment
Functional Assessment Staging (FAST) scale
G
Gabapentin
Gait apraxia
Galantamine
γ-aminobutyric acid transferase (GABA-T)
γ-glutamyltransferase (GGT)
Gastrointestinal (GI) service
General medical condition, psychiatric disorders due to
Generalized anxiety
Generalized anxiety disorder (GAD)
diagnosis of
neurobiology
physical symptoms
recognition of
symptoms of
Geriatric Medical Syndromes
Geriatric psychiatrist, in health care systems
Geriatric psychiatry
Giant cell arteritis
Glaucoma
Gliomas
Glomerular iltration rate
Glucocorticoids
Glutamate
Glutamate, gamma aminobutyric acid (GABA)
Glycopyrronium
H
Haloperidol
Haloperidol decanoate
Hamilton Rating Scale for Depression
Head injury
Health care systems, geriatric psychiatrist in
Health disparities
Health Insurance Portability and Accountability Act of 1996 (HIPAA)
Privacy rule
Health policy
Hemispatial neglect
Hepatitis C virus (HCV) infection
HgA1c
Highly active antiretroviral therapy (HAART)
HIPAA Privacy Rule
Hippocratic aphorism
HIV-associated neurocognitive disorder (HAND)
Horner’s syndrome
5-HT 1A receptor
5-HT 1A receptor partial agonism
Human immunode iciency virus (HIV) infection
Hydrochlorothiazide
3-Hydroxy-benzodiazepines
5-Hydroxytryptophan
Hyperammonemia
Hyperammonemia encephalopathy
Hypercalcemia
Hyperlipidemia
Hypertension
Hyperthyroidism
Hypnotic drugs
Hypnotics
benzodiazepines
delirium
hypnotic drugs
interrupted sleep
neurocognitive disorder and chronic obstructive pulmonary disease
Parkinson’s disease
zolpidem and related drugs
Hypoactive deep tendon re lexes
Hypoactive delirium
Hypoactive sexual desire disorder
Hypomanic
Hypomanic episodes
Hyponatremia
Hypothyroidism
I
Implantable cardioverter-de ibrillator (ICD)
Improving Mood-promoting access to collaborative treatment
(IMPACT)
Impulses
Inattention
Indoleamine 2,3-dioxygenase (IDO)
Ineffective communication
In lammatory arthritis
Inhibitory neurotransmitter
Insomnia
irst line treatment for
management of
treatment of
Instrumental Activities of Daily Living (IADLs)
Interdisciplinary approaches
International Late-Onset Schizophrenia Group
Interpersonal psychotherapy (IPT)
Interpersonal and social rhythm therapy (IPSRT)
Interrupted sleep
Interventional psychiatry
Intramuscular benztropine
Ipsilateral loss
Iron
K
Korsakoff’s syndrome
L
Labor force
Lamotrigine
Language functions
Laryngospasm
Late life anxiety disorders, treatment for
Late-life bipolar disorder, clinical course of
Late-life depression (LLD)
outcomes in
psychological treatments for
treatment of
Late-onset GAD, risk factor for
Late-onset schizophrenia (LOS)
Lateral medullary syndrome
Later-stage dementias
Legal system, older prisoners in
Legally-appointed guardian
Lenticular pigmentation
Lesbian, gay and bisexual (LGB)
Lesbian, gay, bisexual, transgender, or queer (LGBTQ)
Lewy bodies (DLD)
Lewy body dementias
Light therapy
Limbic system
Lipid panel
Lipid soluble psychotropics, pharmacokinetics of
Lithium
dosing of
Lithium toxicity, symptoms of
Lorazepam
Lurasidone
M
Magnetic gait
Magnetic resonance imaging (MRI)
Major depression
Major depressive disorder (MDD)
Major depressive episode
clinical course of
transformation from
Major frontotemporal neurocognitive disorder
genetics of
Major neurocognitive disorder
diagnosis of
risk factor for
risk factors for
Male gender
Malignant catatonia
Mandatory reporting laws
Manic episode
Marijuana use
Marijuana use disorder
Marital therapy, psychotherapy
Marsha Linehan’s Dialectical Behavior Therapy
Medical conditions, psychological factors affecting
Medical malpractice, 4 Ds of
Medical treatment, psychiatric complications of
Medicare Part A
Medicare Part B
Medico-legal
Melatonin receptors
Memantine
mechanism of action for
plasma level of
Menopause
Mental health, engagement in
Mental health disorders
Mental health/substance abuse clinic
Mental illness
category of
Mental status examination
Metformin
Methylphenidate
Middle cerebral artery stroke 2
Migraine headaches
Mild cognitive impairment
Mild neurocognitive disorder
Mini Mental State Examination (MMSE)
Minimum Data Set (MDS)
Mirtazapine
Misuse prescription drugs
Moda inil
Monoamine oxidase inhibitors (MAOI)
Montreal Cognitive Assessment (MoCA)
Mood disorders
Mood stabilizers
Mood symptoms
Mortality rates
Motor neuron disease (MND)
Motor symptoms
Motor vehicle accident (MVA)
Multicomponent interventions (MI)
Multiple sclerosis (MS)
Multiple system atrophy
N
Naltrexone
National Center on Elder Abuse (NCEA's) website
Nausea
Nervous system
Neuroanatomy
Neurocognitive disorder
annual rate, progression
etiology for
factors
progression
risk factor
screening tool
Neurocognitive pro ile
Neuroleptic malignant syndrome (NMS)
Neurologic disease
Neurological examination
Neurological/psychiatric conditions
Neurons, type of
Neuropathology
Neuropharmacology
Neuropsychiatric evaluation, referral for
Neuropsychological evaluation
Neuropsychological test
Neuroticism
Nicotinic receptors
Nihilistic delusions
NMDA receptor
NMDA receptor antagonism
Nocturnal erections
Non-contrast head CT
Non-male icence, ethical principle of
Non-nucleoside reverse transcriptase inhibitor (NNRTI)
Non-pharmacological interventions
Non-rapid eye movement (NREM)
Nonsteroidal anti-in lammatory drugs (NSAIDs)
Noradrenaline
Nortriptyline
Nursing home
O
Obesity
Obsessive compulsive disorder (OCD)
Obstructive sleep apnea
Odds ratio (OR)
Off-label use
Olanzapine
Older adults
caregivers of
circadian rhythms among
community care for
delirium
ECT among
family caregivers of
medical decision-making capacity in
with mental health disorders
mental illness
obesity rates
psychiatric disorder
stand trial among
substance among
subsyndromal depression
subsyndromal depression among
Older African Americans
Older age bipolar disorder
Older women
Opioid analgesics
Oral memantine
Orexin receptors
Organ systems, physiological functions of
Orthostatic hypotension
Osteoarthritis
Osteoporosis
Oxazepam
Oxycodone
P
Pacemakers
Pain
type of
types of
Pain sensation
Pancreatic cancer
Panic attack
Panic disorder
parens patriae
Parkinson plus syndromes
Parkinson’s disease (PD)
Parkinson’s disease dementia (PDD)
Parkinsonism
Paroxetine
Patient Protection
Patient-centered medical homes (PCMHs)
Persistent depressive disorder
criteria for
diagnosis of
Persistent depressive symptoms
Persistent pain, pharmacological management of
Personality disorders
Personality scales
Personality traits
P-glycoprotein
Pharmacodynamics
Pharmacokinetics
Pheochromocytoma
Phobia
Physical restraint
Physiology
Pimavanserin
Pindolol
Polypharmacy
Pontine white matter
Positron emission tomography (PET)
Post-menopausal women
Posttraumatic stress disorder (PTSD)
Postulated neuropathologic process
Potentially inappropriate prescribing (PIP), outcome of
Pramipexole
Prazosin
Preponderance of the evidence
Prescription opioids
Primary care provider (PCP)
Primary caregivers
Primary insomnia
Privacy Rule
Problem solving therapy (PST)
Progressive neurocognitive disorder
Progressive supranuclear palsy (PSP)
Prolonged QTc interval
Prompted voiding
Propranolol
Prostitution
Protected health information (PHI)
Psychiatric advance directives (PADs)
Psychiatric care
Psychiatric complications, medical treatment
Psychiatric disorder
due to general medical condition
Psychiatric interview
biases in
component of
Psychiatric sequalae
Psychiatric symptoms
Psychoactive medication
Psychodynamic psychotherapy, transference in
Psychodynamic therapy
Psychological and neuropsychological testing
Psychological distress
Psychological factor
Psychological factors affecting other medical conditions (PFAOMC)
Psychomotor agitation
Psychomotor disturbances
Psychomotor retardation
Psychosis
Psychostimulants
Psychotherapy
bipolar disorder
cognitive de icit
depression
depression, treatment
formats, effectiveness of
group therapy
insomnia, irst line treatment for
interpersonal psychotherapy
late life anxiety disorders, treatment for
late-life depression
major depressive disorder and personality disorders
marital therapy
pharmacologic interventions
psychodynamic psychotherapy, transference in
psychosocial development, assumption
telehealth psychotherapy interventions for
traditional forms of
treatment outcomes for
Psychotic disorder
Psychotic major depression
Psychotic symptoms
Psychotropic associated hyponatremia
Psychotropic medications
pharmacokinetics and pharmacodynamics of
Psychotropics
Putamen
Q
Quetiapine
R
Ramelteon
Rapid eye movement (REM) sleep
REM sleep behavior disorder (RBD)
Remelteon
Reminiscence therapy
Resting tremor
Restless leg syndrome (RLS)
Riluzole
Risk Evaluation and Mitigation Strategy (REMS) program
Rivastigmine
Romberg sign
Romberg Test
Routine elective surgery
S
Scarce health resources
Schizoaffective disorder
Schizophrenia
diagnosis
diagnosis of
history of
lifetime prevalence of
proportion of individuals with
Secondary amines
Secondary tricyclics
Seizures
Seizure disorder
Selective agonism at GABAA receptor
Selective serotonin reuptake inhibitors (SSRIs)
Self-determination
Sertraline
Setting-speci ic treatment issues
Severe depression
Sexual activity
Sexual dysfunction
Sexual response
Sexual response cycle
Shopping
Shuf ling gait
Sialorrhea
Single-photon emission computerized tomography (SPECT)
Sleep disturbance
Sleep ef iciency
Sleep episode
Sleep hygiene
Sleep-wake disorders
Smoke cigarettes
SNRIs
Social functioning
Sodium
Special Care Units (SCUs)
Special management considerations
Speci ic phobia
diagnosis of
Spousal loss
State laws
Stimulants
Stimulus control
Stress
Stroke
history of
management of
Structural and functional imaging
Subcortical ischemic vascular dementia
Subjective cognitive changes, psychiatric appointment for assessment
of
Sublingual ipratropium
Substance, among older adults
Substance abuse services
Substance related and addictive disorders
alcohol use disorder
major depressive disorder and anxiety
risk factor
Subsyndromal anxiety
Subsyndromal depression
diagnosing
warning sign
Subsyndromal depressive symptoms
Succinylcholine
Suicidal ideation
Suicide
risk of
assessment
Suicide risk assessment
Sundowning
irst-line treatment for
Suprachiasmatic nucleus
atrophy of
Suvorexant
Symbol Digit Modalities Test
T
Tardive dyskinesia
Telehealth psychotherapy interventions
Telepsychiatry
Temazepam
Temporal arteritis
Tertiary tricyclics
Thalamus
aging brain
Thiazide diuretics
Thrombocytopenia
Tobacco
Token Test
Tolerance
Tonic-clonic seizures
Topiramate (TPM)
Transcranial Magnetic Stimulation (TMS) therapy
Transgender
Trans-woman
Traumatic brain injury (TBI)
cognitive impairments after
cognitive impairment and timing
and dementia
psychiatric complication of
Trazodone
Treatment-setting regulations
Tremor
Tricyclic antidepressant (TCA)
Trihexyphenidyl
Tryptophan catabolism
Type 2 diabetes
U
Unipolar major depression
Upper gastrointestinal bleeding
V
Vagal nerve stimulation (VNS)
Vaginal dryness
Valbenazine
Valproate
Vascular dementia
Vascular disease
Venlafaxine
Ventricular dilation
Ventricular system
Verbal and physical stimuli
Very late onset schizophrenia (VLOS)
Vesicular monoamine transporter 2 (VMAT2)
Vinca alkaloids
Vincristine
Visual hallucinations (VHs)
Vitamin B12 de iciency
W
Wallenberg’s syndrome
Wandering behaviors
Warfarin
Weakness
Weight gain
Weight loss
Wernicke’s aphasia
Wisconsin Card-Sorting Test
Y
Yale-Brown Obsessive Compulsive Scale
Z
Zaleplon
Z-drugs
Ziprasidone
Zolpidem

You might also like